You are on page 1of 332

NAVIGATION

AND NAUTICAL
ASTRONOMY,

IN

THEORY AND
PRACTICE...
John Radford Young

HI

NAVIGATION
AMD

NAUTICAL ASTRONOMY,
IN

THEOBY AND PRACTICE.

ATTEMPTS TO FACILITATE THE FINDING OF THE


TIME AND THE LONGITUDE AT SEA.

By

E.

J.

YOUNG,

FORMERLY PROFESSOR OF KATHKUATICS IK BF.LFAET COLLEGE.

LONDON

JOHN WEALE,

69,

HIGH HOLBOEN.

1668.

Digitized By

Google

MUDBCltS AHD

PRI5IEH5,

WIHWHAM.

Oigiiizod 0/

Google

PREFACE.
The following work is an attempt to exhibit, in a moderate
compass, the theory and practice of Navigation and Nau-

Astronomy ; to supply an elementary manual

tical

able for educational purposes at home, and which the

avail-

young

the exercise of his

those already acquainted with

the subject, even a

navigator

may

profitably

consult

professional duties at sea.

To

cursory examination of the following pages will suffice to

show, that I have ventured to depart from the plan adopted


several important particulars.

in existing treatises in
have, for instance, been

ment

of

usually

logarithms,

much more

by the

sparing in the employ-

aid of

recommended that every

which numbers

it

is

nautical calculation should

be performed.

But having long

entertained the conviction that the indis-

criminate uae of logarithms in the simpler operations of

since in such operations they save

trigonometryis injudicious
neither time nor trouble

with them in

all

I have

and

it

is

resolved here to dispense

those computations of navigation, in which

the right-angled triangle only

enters into

consideration,

with the right-angled triangle almost exclusively

that the practical business of navigation has to do.

In the introductory chapter, I have

sufficiently

prepared

PREFACE.

IV

the learner for this innovation

if

innovation it be considered;

and I am not without hopes that persona whose practical


experience in these matters qualifies them to form a correct
judgment, will assent to the change thus introduced.
multiplications

more

and

satisfactorily,

Petty

can be more expeditiously, and

divisions

performed, without the aid of logarithms

than with it ; and that sort of assistance which rather retards


than expedites the end in view,

With the

is

in fact no assistance at

exception of this change in the

ducting the numerical operations,

the*

mode

all.

of con-

be found

will

little

of peculiarity or novelty in the treatment of the navigation


proper, unless indeed

be in the uniform blending together

it

The custom of making

of theory and practice.

a book

on

navigation to consist of only a collection of authoritative


rules, either

without any theory at

or with the investi-

all,

gations thrown together in the form of a supplement or

appendix, to be studied or not, as the learner pleases,

which I think should now be abandoned.

More

is

one

attention

is

being paid to professional training, better provision for

is

supplied, and a higher standard of qualification demanded,

than was the case

fifty

And

or sixty years ago.

it

our elemen-

tary scientific text-books must harmonise with this improved


Btate of our educational

system

it is

not enough now that

knows what

a candidate for professional distinction

book

why

tells

him; he must know what

as well as the how.

But

it

is

it

proves to him

more

especially

part devoted to Nautical Astronomy


of a favourable reception of

vanced and more

difficult

my

that

book.

all

I indulge hopes

In

cases

this

more ad-

where verbal

precepts and directions would be long and tedious

mapped

the

portion of the subject, I have

dispensed with formal " rules" in

instead, have

his

the

from an

impartial examination of the second part of this treatise

out, as it were, a

Hank form of

and
the

Oigiiizod By

Google

PRE ACE.
If

route which the

calculation

By

translation.

Mathematical

take.

to

ia

formula of any complexity are but

adapted to verbal

ill

a person even hut slightly acquainted with

algebraical notation, the formula itself willj in general,

be

preferred for a working model, to the rule derived from

it

but a blank form

preferable even to the symbolical expres-

is

inasmuch as

sion,

this,

though indicating

all

the numerical

most convenient way

operations, suggests nothing as to the

of ordering those operations. Blank forms have, in particular

problems, been recommended, and even partially adopted


before

but, I believe, not

replace rules

till

now

systematically given to

and I have no doubt that they

acceptable in actual practice at sea.*


attention to the
latitude

from a

discussed,

page 147.
this

manner

in which the

single altitude of the

more

off the

especially to the practical

Less consideration than

will prove

would

also invite

problem of finding the

sun

it

meridian

is

inferences at

deserves

given to

is

problem in former treatises, on account of an afErmed

ambiguity in the calculated

I think

result.

that the ambiguity complained of

it is

here Bhown

more imaginary than

iB

real.

The chapter " On Finding the Time


has also, I think, some claim to notice

at Sea," page 183,


as I believe I have

introduced a practical improvement in the working of this

important problem.

what

ia

'I

would more particularly

chapter.

To the

Bubject of the sixth chapter, "

the Longitude at Sea," I have also

much

refer

to

contained between page 192 and the end of the

careful consideration

which every person

* Steps 1,

2,

in the

critically

course be the article on


form at

as

it

On

deserves

Finding

devoted

the part of this chapter to


disposed will turn, will of

Clearing the
p. 148,

Lunar

Distance.

ahonld stand side-by-side

the

narrowness of the page here lenders this arrangement impracticable.

Digitized By

Google

EEEFAOK.

here

Besides the well-known logarithmic process of Borda

modified

little

may be

logarithms

have also given a method in which


altogether dispensed with;

in which

subsidiary tables, and auxiliary arcs are not needed

common

in which (besides a little

operation
table

and

arithmetic) the whole

performed with the aid of only a single small

is

a table of natural cosines.

Should, however, the computer prefer to use logarithms

where I have employed common arithmetic,


optional with

him

to do so

it

of course

is

I have exhibited both modea of

proceeding, and if any one takes the trouble to count the

number of
he

figures brought into operation in each method,

will find that,

on the average, the arithmetical process

will

not require above half-a-dozen more than the logarithmic

and one advantage of the former


readily revised.

This work, in

is,

that the work

all its details, it is

preserve rather than to record a mere abstract

after the lapse of several hours, if a recurrence to

lead to the detection

be too late to put

There

is

all

of any numerical error,

should

it

it will

may be

dozens of them

examined and compared

all

all

in favor of that here proposed, I have

most in esteem

it

for

easily

I have

those which different

authors have selected, and steadily resisting

the conclusion that

not

to rights.

educed from the same fundamental expression.

ment

more

and even

no merit in devising formula and rules

clearing the lunar distance

carefully

is

better to

bias of judg-

been forced to

The method

has a claim to adoption.

at present, is that first given

by Krafft of

which requires a table of versed and suversed


and another special table of " Auxiliary Arcs." This

St. Petersburg,
sines,

latter

table

is

somewhat

complicated, and will seldom

furnish the arc required to within a second or

truth

but the method

is

two of the

nevertheless the most simple and

Digitized By

Google

PREFACE.
convenient hitherto proposed
all

whether the dispensing with

such special tables, and thus securing accuracy to the

nearest second, will entitle the process I have recommended

to a favorable comparison with that just mentioned, others

must of course determine.

The

accompany

tables which are to

this work, in con-

junction with the logarithmic tables already published in

the present series of Kudimeutary Treatises, will compre-

hend

all

those which are indispensably necessary in Naviga-

tion and Nautical


necessary.

The

Astronomy

will

and

will

be so arranged that the

always appear at the_ top of the page, bo

that the extract to be


the eye

no more than are

table of natural cosines will give degrees,

minutes, and seconds

" Argument "

and

down the page

made will always be found by running


:

there will never be any necessity

to proceed upwards, a plan which will of course facilitate

the references.

The logarithmic
with those

them

now in

distinct.

tables just adverted to

preparation, but

very

little

ifc

may be bound up

will be better to

keep

them

will

familiarity with

enable the computer at once to put his hand on that one of

the two collections which contains the particular table he


wants, which table

it will

volume than in a large one

be more easy to find in a small


:

the two volumes of tables will

be distinguished one from the other by difference of colour


in the covers.
I have only further to add, that most of the astronomical

examples in this book are accommodated to the Nautical

Almanac

of the current year, 1858

in advance of the time

they

when they were

all refer

to dateB

framed, and are

therefore of course all hypothetical,


J.

K.

YOUNG.

May, 1858.

Oigiiizod By

Google

The Index, or Table

of Contents,

will be found at the

end of the

Volume.

Ca- The

NAVIGATION TABLES,

to

accompany

be published-: Price,

this

Is. 6rf..

Work,

will shortly,

NAVIGATION AND NAUTICAL ASTRONOMY.


INTBODUCTION".
As

rudimentary treatise

this

is

intended principally for

tlie

instruction of persons having only a very moderate acquaintance'

with mathematics, we

shall devote

a few introductory pages to

practical commutations of the sides

and angles

tin;

of plane triangles,

a portion of the general doctrine of Trigonometry that

is

indis-

pensably necessary to the thorough understanding of the rules and


operations of Navigation.

Although the path of a ship at sea is always traced upon a


curved surface, and is usually a line of a complicated form, yet it
fortunately happens that all the essential particulars respecting
this curved line

essential, that

is,

to tho purposes of

Navigation

are derivable from the consideration of straight lines only, all


drawn upon a piano surfaco ; and tha most complicated figure;
with which wo have to deal, in Navigation proper, is merely the

plane triangle.

And

it

may

be as well here, at the outset, to apprise Uic

beginner that he

is

not to suppose that tho substitution of tho

straight line on a plane surface, for the spiral curve on a spherical


surface, in the various computations of navigation, is a contriv-

ance forced upon us on account of tho

diflicullies

tho discussion of the more intricate form of the


simplicity

attendant upon
latter,

and that

attained at the expense of accuracy, by substituting

is

a straight line on a plane, for a curve


shown in the proper place (Chapter

line on a sphere,
II.),

It will bo

that this substitution

that with the curvilinear form of a ship's

involves no error at all

path we have in reality nothing

curve

may

to

do

that,

in imagination, the

be straightened out, and that everything connected

Digitized 0/

Google

INTHODUCTIOS,

with the course, the distance sailed, and the difference of latitude

made,

may

be accurately embodied in a plane triangle.

The learner will thus

readily perceive that a familiarity with

the rules for calculating the sides and angles of plane triangles

is

preliminary indispensable to the attainment of a sound practical

knowledge of navigation
attention

to

what

and we therefore earnestly

delivered

is

invite his

following introductory

the

in

chapter, written with an especial view to the declared object of

the present rudimentary treatise.

PRELIMINARY CHAPTER.
On iee Computations of the

Every
the three

Tbiaxgi.es.

Sij>es akb Angles of Tlaxe


part* as they are
and the three angles. As lines and angles
we cannot directly combine

triangle consists of six

sides,

called

are magnitudes quite distinct in kind,

a line and an angle in calculation, any more than we can combine


a mile and a ton. To obviate this difficulty, and to convert all
the magnitudes with which trigonometry deals into linear magni-

tudes only, employed in connection with abstract numbers, certain


trigonometrical tines, or numbers having reference to the angles,
are always used in the computations of trigonometry, instead of

the angles themselves.

to

concerning the

conduct investigations

triangles,

be shown presently what these


and how completely they enable us
sides and angles of

It will

trigonometrical quantities are,

without the latter kind of magnitudes ever directly

entering the inquiry

however,

how

it will

previously to this,

be necessary to explain

angles themselves are measured.

About the vertex A, of any


'

B A C, as

a centre,

described:

the

let

circle

intercepted

vary as the angle

11

AC

that if the angle change to

angle

B C D E F be
B C will

arc

that
If

is

to say,

A C, whether

greater or smaller than the former, then will the intercepted arc

change from

E C to B
EAC

angle

C,

so as to give the proportion

angle

BA

are

BC

arc

B C,

Oigiiizod By

Google

IISTBODUCTIOS.
nLvi'jH-

fnvm prop, xxxiii.

ris

:s

is

true wliatever be

the radius

tlie

o!'

sixth

.Km.'iid'.-

magnitude of the

And

W>k.

circle, or tlie

this

length of

AB.

The circumference

of the circle is conceived to he divided into

360 equal parts, called degrees; so that, from the above pro-

an angle at the centre, subtended by an arc

portion,
is

of

40 degrees,

double the angle at the centre subtended by an arc of 20 degrees,

by an arc of 10 degrees, and so


whatever be the radius of the circle described

three times the angle subtended

on

and

this is true

about A.

Ihe degrees of one

circle differ of course in

length from the

when the two circles have different


a degree being the 360th part of the circumference,
whether the circle be small or great yet it is plain, that if a
whether larger or smaller than that before us, were
degrees of another circle,
radii

circle,

described about A, the are of

it,

intercepted

by the

sides

A E, A C

of the angle, w ould be the same part of the whole circumference

which it belongs, that the arc li C of the circle above is of the


whole circumference to which it belongs
in other words, the
angle at the centre would subtend the same number of degrees,
whatever be the length of the radius of the circle on which those
degrees aro measured
the degrees themselves would be unequal
in magnitude, but the number of them would he the same.
By viewing an angle in reference to the number of degrees in
the circular are which subtends it, as here explained, we arrive at
a simple and effective method of estimating angular magnitude
the circular degree suggests the angular degree, which we may
regard as the unit of angular measurement the angular degree
being that angle the sides of which intercept one degree of the
circle.
Angles are thus measured by degrees, and fractions of a
degree the measures applied being the same in kind as the quanto

tities

measured, just as in

mure

i'or the
is

all

other cases of measurement.

coiivcuis.-nl e\]iie.-,ion ui i'rat lwn.il pitrts,

a decree

conceived to he divided into >i\ty equal portions, called minutes,

and each

of these into sixty equal parts, called seconds; further

subdivisions are usually regarded as unnecessary, so that

when-

ever it is thought requisite .to express an angle with such minute


accuracy as to take note of the fraction of a second, that fraction
is actually

The

written as such.

notation for degrees, minutes, and seconds, will be readily

b 2

Digitized by

Google

IKTBODffCTIOTT.

perceived from an instance or two of

use

its

thus, 24 degrees

16 minutes 2S seconds would be expressed, in the received nota24 a 16' 28" ; and 4 degrees 9 minutes 1 2 seconds
and three quarters of a second, would he written 4 9' 12"J.
If we were required actually to construct an angle from having
its measurement in this way given, and were precluded from the
use of any peculiar mechanical contrivance for this purpose, we
tion, as follows

should first draw a straight line, as A B, in the preceding diagram;


A as centre, and with any radius that
might be convenient, we should describe a circle BDP, &c. the
we should divide into 360 equal parts,
or the half of it, B E F, into 180 equal parts, or the fourth of it
then, with the extremity

circumference of this circle


(the quadrant),

T>,

into 90 equal parts

from B as many of these

parts,

measure of the angle, adding

we should then count

or degrees, as there are in the

to the aro,

made up

of these degrees,

whatever fractional part of the nest degree, in advance, was


expressed by the minutes and seconds: the whole extent (B C) of
arc, subtending the angle to be constructed, would thus be discovered and by drawing A C, the required angle B A C would
;

be exhibited. But the practical difficulties of all this would be


very considerable, if not insurmountable they need not, however,
be encountered, as instruments for constructing angles, and for
measuring those already constructed, are easily procurable: the
common protractor, with which all eases of mathematical instru;

ments

is

sufficient

furnished, .enables us speedily to effect the business with

accuracy for

all

ordinary purposes.

a semi-

It is simply

circular arc divided into degrees, as above described, with the

marked on the diameter connecting its extremities.


But the construction, or measurement, of angles upon paper, is
a mechanical operation with which we have nothing to do in
calculations respecting triangles; and we have adverted to it
solely for tiio purpose of giving greater clearness and precision to
the student's conception of angular measurement to satisfy him,
iu fact, that the numerical expression for the value of any angle
centre

does really convey an accurate

using the notation explained above

idea of the amount of opening it refers

datum for the


mechanical

to,

and furnishes a sufficient


merely

actual construction of the angle, supposing no

difficulties to

to the diagram, at page 2,

must be added

to

any

stand in the way.

we have

Referring again

further to remark, that

arc, or subtracted

from

it,

to

make

it

what

become

Digitized 0/

Google

ISTBODUCTTON.
an arc of 90, is called the complement of that aro:
C D is the complement of the arc BC; and DE, taken subjectively, is the complement of the arc B E.
In like manner,
what must he applied to an arc to make it a semicircum fere nee, or
a quadrant, or
thus,

180, is called the supplement of that arc

thus,

CDF

is

the

B C, and E F the supplement of the arc


The same terms apply to the angles' subtended thus,
is the complement of the angle B A C ; and the
taken subtractively, is the complement of the angle
B A E. In like manner, the angle C A F is the supplement of
the angle BAG, and the angle E A F the supplement of the
supplement of the arc

B C

E,

the angle

CAD

angle

DAE,

angle

BAB.

For example, the complement of 2i"


are or angle,

Gj j

is

48',

and

its

whether we refer to

12',

supplement

is

155 48'.

The Trigonometrical Sines, Cosines, e.


It has already been observed, that as an angle
line
is

and a

straight

cannot possibly be combined in any numerical calculation,

it

necessary to employ either lines or abstract numbers instead of

angles in

all

the rules

and

investigations of trigonometry, the

quantities thus employed being, of course, such as to always suggest


or indicate the angles themselves

The Sise,
the angle

From

BAC

circle, let a

AB

the sine of the arc

an

aro

is,

Cm

bo drawn

this perpendicular

C.

The

sine of

therefore, a line which,

bo thus defined

we deduce them as followB :


B C, subtending

extremity C, of the arc

at the centre of the

perpendicular

to the radius
is

tiie

It

is

may

the perpendi-

from the end of the arc, to the


radius drawn to the beginning of that
cular,

arc: all the arcs considered are supposed to have their origin at B.

Just as from the measure of an arc

angle

it

we

derive the measure of the

subtends, so from the sine of an arc

we deduce

the sino of

It would not do to regard, without any modification,


the sine of an arc as the sine of the angle it subtends, because,
though the angle remain unchanged, the subtending arc and

the angle.

consequently the sine of it may be of any length whatever, in


the absence of

all limitation as to

the length of the radius.

Ju

Digitized by

Google

INTRODUCTION.

may have

order, therefore, that every angle

nate sine, the radius

is

a fixed and determi-

always regarded as the linear representa-

tion of the numerical unit or 1,

upon which hypothesis

is

it

plain

that the sine of an angle will always he the same fraction, since
is

always eoual to

alluded to
radius.

(Eue.

no other than the

is

1.

so that

yi.),

the fraction

ratio of the sine of the arc to the

It is this ratio or fraction that is called the trigonometrical

; it is an abstract number : the sino of


arch called the linear or geometrical sine: it is a straight line.
The Cosine. Tho cosine of the arc B C is the portion Am of

sine, or sine of the angle

the

the radius intercepted between the centre and the foot of the sine

The trigonometrical

of the arc.

cosine, or the cosine of the anglo

B A C, which the aro subtends, is the numerical representation of


Am conformably to the scale A B = 1. In other words, it is the
ratio or fraction

if*

or

AC

^
AC

The T Ay g est .The tangent

BC

of the are

the straight line

is

B T, touching the arc at its commencement B,


^ and terminating in T, where the prolonged
radius through the end Cot the arc meets it. The

,(

trigonometrical tangent, or tangent of the angle

A C,

is

the numerical value of tho

n the hypothesis that A B


15

The Cotangent.

f- *

BT

1.

AB

same

is

the line Dt,

touching the complement of that are at D, and terminating in

BA

AC

The trigonometrical cotangent, or cotangent of the


numerical value of the same line, on the hypo-

prolonged.

angle

line

In other words,

the trig,
s tangent.
6

The cotangent of the arc B C

C, is the

AB = 1.

thesis that

AD

D(

The Secant.

Tho

Jn other words,

it

is

i^, the trigonometrical

AD

secant of the are

BC

is

the ratio

for

cotangent.

the line

A T from

the centre up to the tangent its numerical value on the hypois the trigonometrical secant, or seeant of
thesis of
:

AB=1,

the angle

BA

C.

This numerical value

is

the

ratio

for

INTBOBUCTION.
The Cosecant.The Cosecant of the
from the centre up

to the cotangent

or cosecant of the angle

BAC

the hypothesis that

AD

ADiA(::

The learner

1:

BC

arc

the line

the numerical valnc of

At

At on

this value is the ratio

for

will perceive that cosine, cotan-

and cosecant are nothing more than the

gent,

complemental arc being considered as at D.

and
commencement of

sine, tangent,

secant of the complement of the arc or angle, the


.the

is

the trigonometrical cosecant,

is

It is also further

ohvious that any geometric:!.! sine, cosine, &o,, if divided by the


radius of the are with which
cosine, &e., of the angle

connected will give the sine,

it is

which that

arc

these trigonometrical quantities, though

suhtends at the centre

pure numbers, may,

all

as already explained, ha represented by lines-- the same lines as


those employed in connection with the arc, provided only

we

agree to regard the radius of that are as the linear representation


of the unit

unit

that

ia

1.

The advantage

we can

of thus regarding the radius as

investigate the relations

among

the trigono-

metrical quantities defined above without introducing the radius

may always

as a divisor, since a unit-divisor

may

bo suppressed, and

avail ourselves of the aid of geometry for this purpose.

Thus, referring to the right-angled triangles in the preceding

diagram we have from Euclid, Prop. 47, Book I.


Cm2 + Aw!=AC ; AT;=AB- + BT-, AfcADHD;'
,

that

is,

the radius being regarded as

by A,
+ cos=A=l, se^A^l +

1,

and the angle being

represented
Bia-

tan2 A, cosee-

A=l

cot 1 A. ..(1)

Again, because the sides about the equal angles of equiangular


triangles are proportional, the triangles ACm, ATB, A(D furnish

the following proportions, namely

A
sin A
cos A
tan A
sin A
cos

A
oos A

sin

A
cot A

tan

see

cot

cosec

TSTUODU Olios'.

From

wo

the relations (1)

sin

cob" A), cos

(1

see that

A=z V (1 sin' A),

cosecA=:V(l + cot3 A)

And from
that

sec

kz= /

(1

+ tans A)

(4)

with

these, in conjunction

when

(2)

either the sine or the cosine of

may

the other trigonometrical values

and (3), we further see


an angle is known, all
it. More-

he computed from

over, from (3) it appears that the following pairs of values are the

namely
See A, cos A ; tan A, cot A; sin A, cosec A.
would be out of place here to discuss the relations among
for a more com-

reciprocals of each other,

But

it

the trigonometrical values at any greater length

prehensive view of the general theory of these quantities, the


learner

is

referred to the rudimentary treatise on Trigonometry.

On

(lie

The numerical values

Trigonometrical Tables.
of all the trigonometrical lines, conform-

= are
= 0 up to A=90 and

ably to the hypothesis, that radius


for all angles

Such a table

from
is

1,

carefully computed,

arranged in a

table.

&c, to dis&c, to he here-

called a table of natural sines, cosines,

tinguish it from a fable of logarithmic sines, cosines,


after adverted to.

In the construction of such a table

sary to compute for angles above 90",

it is

unneces-

for, as a little reflection,

on

reference to the diagrams in which the trigonometrical lines are


exhibited, is sufficient to show, the sine, cosine, &c. of an are or

angle above 90,

is

a line of the same length as the

of an arc or angle as

much below

of an are or angle has the

sine, cosine,

&c.

90, so that the sine, cosine, &c.,

same linear and numerical value as the

p sine,

cosine, &c. of the tupp!ementa that

arc or angle, and this


learner

is

a truth that the

must always keep

remem-

in

brance: thus Bin 120=sin CO 3 sin 135"


,

=sin
-

4j,

and

so on.

Similarly of the

B cosines, tangents, &c, only here

it is

to

be noticed that the cosine of a supple-

mental are

lies in a.uojipr,sitn ih'rectionto

the cosine of the aro

itself,

remembering

that all arcshere considered are supposed

N3"

the origin
stance, if

DE

li

have one common origin or comin the preceding diagrams.


For in-

to

bo made equal to

DC,

the arc

BE

will he the

Digitized By

Google

INTRODUCTION.
supplement of the arc EC, both arcs commencing at

BE

this arc

that

An, a

therefore

The

we have moans

cos

120=<s

60,

to

to

we

cos

60".

line

BC is drawn from B
marking the prolongation of the
BE

meet the dotted line marking the prolonga-

E A agreeably

The two tangents though equal


tion,

120=

cos

but the tangent of the supplemental arc

drawn downwards

tion of the radius

is directly

This opposition

so that instead of writing

the tangent of

meet the dotted

radius through
is

Am of the arc BC.

we shonM write

Similarly for the tangents

upwards

it is

BC is

of indicating algebraically: the opposite

+ and furnish these means,

signs

and

cosine of the supplement of

though the same in length,

line which,

opposed in situation to the cosine


of direction

Ii,

are to deal with as the supplement of BC, and

we

not the equivalent arc FC.

accordingly write tan

to the definition of the tangent.

being opposite in direc-

in length,

120"=

tan C0.

It is sufficient,

however, that we know whether the cosine be plus or minus, in


order to enable us to pronounce upon the algebraic sign of any other
of the trigonometrical quantities belonging to an are or angle

between 0 J and 180 3

thus the equations

tangent and cotangent, and the

others

cosines.

&c, are computed

as

may

(2),

page 7, give us
and cosecant.

secant

how

It is not our business to explain here

is

the natural sines,

be easily imagined, the work

of a very laborious character, but tables having been constructed

once for

As

all,

there

to the use of

is

no occasion

for a repetition of the labour.

such a tablo in facilitating calculations respect-

ing the sides and angles of plane triangles,


explanations

we

oiler the following

Iteferring to the

diagram at page

0,

wc

see that

Am C, A B T

are two similar right-angled triangles.

In the

first

the numerical value or representative of

in the seoond, 1 is

tha numerical representative of

mated according

of these, 1 is

B, the other sides being esti-

the numerical values of theso sides,


0 up to A =: 90=, the
for all values of the angle A, from
to this scale

values increasing minute by minute, aro arranged in the table,


in C, Am, under the heads of sine and cosine of tho
and the values of B T, A T under the heads of tangent

the values of

angle

and

secant of the same angle.

It follows, therefore, whatever right-angled triangle we may


have to consider ia actual practice, that the numerical values of

lyTEODUCTIOK.

10

the sides of two right-angled triangles, similar to

will always
For instance,

it,

he found already computed for us in the table.

we were dueling with a right-angled triangle of which


is 34 27'
we turn to the table for 34 27',
this particular value of the angle A, and against it we find, under
the head Sine, the number for mC, and under the head Cosine,
the number for Am and we know already that the number for
A C is 1. Thus we know completely the numerical values of all
the sides of a triangle AtC similar to that proposed for consideration these values, as furnished by the table, are
suppose

the angle at the base

mC='565G9,
or,

Am =-82462,

and

AC = 1;

using the trigonometrical names by which these are called,


sin 34"

How

27'=-5G569, cos 34 27'='82462, and rad

although these tabular numbers are

yet there

we take

or yards, or miles, provided only

them

as so

1.

numbers,

all abstract

to our regarding

no hindrance

is

many

feet,

care to regard the radius

as 1 foot, yard, or mile.

Again referring

to the table for the particulars connected

BT ='68600,
or,

with

A li T, we find

the other similar triangle

AT=1'2126S, and

AB=1

using the language of the table,

tan 34 2T=-6S600, see 34 27'=1'21268, rad=l.


Suppose the hypotenuse of the triangle proposed to us

is

then comparing our triangle with the tabular triangle

which the hypotenuse

we know

is 1 foot,

times

Am

A C,

56 sin 34 27'= the required


and 50 eos 34 27'= the required

The work

is

sin

of

perpendicular,
base.

as below
34 27'='56569

cos Si"

27'='824G2
SG

58
339414

404772

282S45
The perpendicular 31
sec

feet,

tho per-

Cm, and the base 56

hence
3

"We

50

AC,

that, as the hypotenuse

of the proposed triangle is 56 times the hypotenuse

pendicular of the former must be 56 times

by

'f)7.

'M

412310
t'ect

TIiv

this illustration, that of the

t?i

so= 4

1;
-

TS7 i

foot.

two tabular triangles

11

IHTBODUO T IOTT

A B T, we

AmC,

do not take either, at random, to compare with

the triangle under consideration

which the radius


Such a selection

(1)
is

we

two in

select that of the

corresponds to the side whose length is given.

always to be made.

If,

for instance, the lase

say equal

of a right-angled triangle be given

47

to

feet,

and

it

be required, from this and the angle at the base aay 34" 2.7', as
compute the* perpendicular and hypotenuse, wc then,

.before, to

refer to the table for the triangle

base that

is 1

wc

ABT

because in this

it is

the

thus have

47 tan 34 3 27 the required perpendicular,


and 47 sec 34' 27 the required hypotenuse,
the work being as follows
47 sec 345 27'=47-^-cos 34* 27'

tan 34= 27'=r6SG

47

-8246)47

2744

(57

ft.

the hypotenuse.

57700

The perpendicular 32-242

57 22
'

feet.

These illustrations will, we think,

suffice

convey to the

to

learner a clear idea of the use of a table of natural sines, cosines,

&c,

we may,

in the solution of right-angled triangles; and

there-

fore, proceed at once to discuss the several cases that occur in

To oblique-angled

practice.
article
all

but

may

it

triangles

we

shall devote a distinct

be well to apprise the learner, that nearly

the calculations concerning the course and distance sailed of a

ship at sea, involve the consideration of right-angled triangles


only.
Solution of Right-angled Triangles.

Of

the six parts of which every triangle consists

the sides and

any three, except the three angles, being given, the


remaining three may be found by calculation. In a right-angled
triangle one angle is always known, namely, the right-angle, so
the angles

that

it is sufficient for

the solution that any two of the other five

parts (except the two acute angles) be given.


triangle, therefore, the given parts
1.

or 2.

A side and
Two

must be

In a right-angled

either

one of the acute angles;

of the sides.

The reason why a knowledge

of the three angles of a triangle

DigiiizM By

Google

12

IXTHODTTCTIOH'.

will not enable us to find the sides, is that

similar to one another, however their Bides

aU triangles that are


may differ, have the

three angles in any one respectively equal to the three angles in

any other;

so that

of triangles

may

with the same three angles, an

infinite variety

he constructed.

It follows, therefore, that in every practical example that can


ae to place the example

must be such

occur, the given quantities

under one or other of the following four cases


I. The hypotenuse and one of the acute angles given.
II. The base or perpendicular, and an acute angle given.

The hypotenuse and one of the other


IV. The base and perpendicular given.

sides given.

III.

We

shall consider these four eases in order.

The hypotenuse and one of

la

the acute angles given,

the right-angled triangle in the margin, let the hypotenuse

AB

and the angle A at the base be given,


and let the numerical values of the
three sides be denoted by a, b, c, these

,g
j

$S

small letters corresponding to the large


4

denoting

letters

"

IVe are to compare

the opposite

angles.

this triangle

with the

tabular triangle having tho same base angle A, and of which the

hypotenuse

is 1

A, and

its

sin

tho perpendicular of this tabular triangle will be

base eos

given hypotenuse

is

(sec

diagram, p.

times that of the

And

o).

since

oar

tabular triangle, the

perpendicular of our triangle must be c times that in the table,

and the

base, c times the tabular base

required perpendicular and base

a
If,

= c sin A,

we

and

however, the vertical angle

that

shall
b

is

to soy, for the

have

= c cos A.

were given instead of the base

angle A, then, since these angles are the complements of each


other,

we should have

sin

a=c

A= oos B,

cos B,

and

Hence we deduce the following rule


.

Role

Fur the Perpendicular.

1.

by the sine of the angle

and cos

b=c

A= sin B,

so that

sin B.

Multiply the given hypotenuse

at the base, or

by

the cosine of the ver-

tical angle.
2.

For

the Base.

Multiply the given hypotenuse by the cosine

of the angle at the base, or

by the sine of the vertical angle.

DigiiizM By

Google

IKTRODlTCTIOir.

EXAMPLES.
1.

a,

Ia the right-angled triangle All C are given the hypotenuse


37 28', to find tho perpendicular
48 feet, and the angle A

and the base

b,

as also the angle B.


cos 37

sin 87 28'=-<J0S3

A + B=90
A^37

Given

2.

2S'=-7937

48

48

63490

48664

28'

a=29-1984

c= 63

0=33-0976

feet.

and A=2419';

yards,

to

find a

and

feet.

i,

as

also B.
bin 24

10'=-4118

cos 21 I9'=-flll3

03

C3

12354

27339

A +B=90
A=24

ID'

54678

24708

B^GoMl"'

o=26-9434
3.

6=57-4119

yards.

yards.

Given the vertical angle B=C3" 12', and the hypotenuse


foot, to find the remaining parts of the triangle.

e=ys

sin 33"

12'= -5476

492S4

a=82-00S4

6=53-0048

feet

feet.

33 12'=56 48'.
Also A=90
In the foregoing operations only/ur decimal places have been
taken from tho table a number of places amply sufficient for all

the purposes of Navigation.

Sote.

The

learner- will not forget that

of a right-angled triangle

is

and an acute angle are

when one acute angle

given, the other is virtually given,

being the complement of the former


given,

whenever therefore a

we may always regard

adjacent to the given side as given.

Now

it will

side

tho angle

save the neces-

sity of all reference to diagrams

and formula; or rules, if with the


vertex of this adjacent angle as centre, and the given side as
radius, wc conceive an arc to be described, and notice whether

Digitized by

Google

14

UTTBODUCTIOK.

the required side becomes a sine, a cosine, a tangent, or a secant

whichever of these

for,

it

name

that is the

is,

of the trigono-

metrical quantity to be taken from the table, for a multiplier of

the given side, in order to produce the required side.

The

whose vertex

table

is

thus

a right-angled triangle is 38 feet,

and

of course, to be entered with that angle

is,

taken for centre.


4.

The hypotenuse of

the angle at the base 27 42'

required the other sides, and the

vertical angle.

Ans. a=17-CC2 feet, S=33-645 feet. B=62 18'.


The hypotenuse is 76 feet, and the vertical angle 43 IS'
required the perpendicular and base.
Ans. 0=55-313 feet, 6=52-121 feet,
(i.
The hypotenuse is 521 feet, and the vertical angle 30" 0'
5.

required the other sides.

Ans. 0=420-97, 6=306-97


II.

Base

and one of

or perpendicular

Let the base

AC

feet.

the acute angles given.

A be given:

and the angle

then,

we have

to

com-

pare our triangle with that one of the two similar tahular triangles,

whose base

(the radius) is

triangle will be tan A,


p. C).

And

The perpendicular

1.

and

its

of this tabular

hypotenuse see A.

[See diagram

since our given base is b times that of the tabular

triangle, our required perpendicular

must be

b times that of the

tabular one, and our required hypotenuse also 5 times that of

the tabular one, lienee tho required perpendicular and hypotenuse


will be

a=b

tan A, and

If it be the vertical angle

that

e=b
is

see A.

given instead of tho base

angle A, then since


cot

we

shall

]!=tan A, and eosee B=sec A,

have

a=b

cot B,

and cb cosec B.

In the Tables, tho secants and cosecants are frequently omitted,


because from the fact that secant is 1 divided by cosine, and cose1 divided by sine, they may be dispensed with. {See p. 7.)
Making, therefore, these substitutions for secant and cosecant

cant

above, and remembering also that cotangent


gent, the values of a

is 1

divided by tan-

and c may be expressed thus

Digitized by

Google

JSTEODUCTrON.
(t

= b tan

tan

and c

A, and

a=

15

~
Bin

IS

and those expressions furnish the following


Rule

1.

For

or divide it

rules

Multiply the given base by the

the perpendicular.

tangent of the base angle

by the tangent

of tho ver-

tical angle.
2.

For

the base angle

Note. As
if

Divide the given base by the cosine of

the hypotenuse.
:

or

by the

sine of the vertical angle.

either of the

two

sides

the perpendicular bo given, namely,

may be considered as base,


CB instead of AC, we have

only to conceive tho triangle to be turned about

till

base and per-

pendicular change positions, and then to apply the rule.


also the

Note

at p. 13, tho directions in

(See

which will enable the

learner to dispense with formal rules).

1.

At the

distance of 85 feet from tho bottom of a tower, the

angle of elevation

A of

the top

is

found to be 52 a 30'

required

the height of the tower.

Here the base and the base angle


Hence

dicular, as in the margin.

the height of the tower

concluded to bo 110|

may

feet.

be

are given to find tho perpen-

tan 52" 3O'=l*3032

85
65160
104256

Height= 11O-7T20

feet.

If tho angle of elevation A, bo takon not from the horizontal

plane of tho base of the tower but from the eye, by means of a

quadrant or other instrument, then, of course, the height of the


eye above that plane must be added. If in tho present case the
height of the eye be 5^ feet, then the height of tho tower will be

116
2,

feet.

Required the length of a ladder that will reach from the

point of observation to the top of the tower in the last example.

Digiiized by

Google

16

IHTBODUCTIOH.

Here the base and base angle

are given to find the hypotenuse,

30-= -6 0,8,8) 8 5

as in the margin.

We eon-

elude,

that the

009

length of the ladder must

241

be 140 feet nearly.

243

therefore,

cos 52

(140

The division iu the margin is what is called contracted division,


which saves figures, and which may always bo employed for this
(See the
purpose whenever the divisor has several decimals.
Rudimentary Arithmetic.)
The learner may easily prove the
correctness of the two results in this and the former example by
applying the principle of the 47th proposition of Euclid's

fifth

book, namely, that in a right-angled triangle the hypotenuse

Bum

equal to the square root of the

of the squares of the other

is

two

thus

sides:

85

110'S

Square of pcrp.=12277

85

110'S

Square oflmo

7225

SSBi

425
CS0

10502(140

12188

.*,

Hypotenuse

= 140

nearly.

ft.

Minute decimals arc of course disregarded

in all practical opera-

tions of this kind.


3.

From

the top of a ship's mast, 80 feet above the water, the

angle of depression of another Bhip's hull was found to be 4 J

re-

quired the distance between the ships.

Tho angle of depression


lino from, the mast-head,

the angle

and

ship,

taken.

B
I!

the angle between the horizontal

is

and the slant

The complement,

to the distant ship.

of the triangle

ABC

lino

from the same point

therefore, of this angle is

(p. 12),

where

the value

work

is

therefore, is 90 a

b=a tan B=80 tan

$G,

the distant

From the edge

is

1144

of a ditch

it

we have

for

AC

tan 8G=14-3007

SO

cp-

feet.

is

4=8G, and the perpen-

and the

as in the margin, from which

pears that the distance


4.

is

tho mast-head whence the angle of depression

The angle B,

dicular or side, BC, adjacent to this being given,

U44-t)o(i

8 feet wide, and which surrounded

a fort, the angle of elevation of the top of the wall

was found

to

Oigiinod By

Google

17

ISTEOBTTCTIOK.
be 62 40'

required the height of the wall, and the length of a

ladder necessary to scale

A=62

Here

it.

40'

and 6

tan 62 40'= 1-9347

=18

cos 62"

18

to find a and c.
40'=-4 5,i),2)18 (30-2=e.
1378
:

"422

154776
19347

413

<j=34-8246 feet

Hence the height

34-8

is

feet,

and the length of the ladder 39-2

feet.
5,

A flagstaff, known

to

be 24 feet in length,

subtend an angle of 38' at a ship at


of the eliff on the edge of which the
to he 14.

What

The distance

is

is

observed to

and the angle of elevation

sea,

staff is planted is also

observed

the distance of the ship from the cliff P

or base of the triangle being b, it is plain that,

t tan 14 38'=the height of the top of the staff,

and

h tan

14=

6 (tan 14
.-.

,,

cliff,

,,

38'-tan 14)=Iength of staff=24

feet.

24
feet.
tan 14 38' tan 14"
tan 14 38-26110
=-24933
tan 14

b=

01^77)24 (2039
2354

feet,

the distance.

11
11
6.

nd

Given the base 73

feet,

and the angle

at the base 52' 34', to

the perpendicular and hypotenuse.

Ans. perp.=95-365
7.

Given the base 327

feet,

and the

ft.,

8.

From

hyp.=120-097 ft.
anglc=35 43'

vertical

required the perpendicular and hypotenuse.


Ans. perp. 454-8

ft., hyp. 560 feet.


the top of a castle 60 feet high, standing on the edge


INTRODUCTION.

18
of a

cliff,

the aogle of depression of a ship at anchor was observed

From the bottom of the castle, or top of the cliff",


Required the horizontal disthe angle of depression was 4 2'.
to he 4 52'.

tance of the ship, and the height of the

Ans.

dist. of

cliff.*

ship 4100 feet, height of

The base of a right-angled

9.

opposite angle 54" 36';

triangle

346

is

289

cliff,

feet,

ft.

and the

required the perpendicular and hypo-

Perpendicular 246-2

tenuse.

hyp. 425-1

ft,,

ft.

The hypotenuse and one of the other sides given. Representing the perpendicular, base, and hypotenuse by a, b, and e, as
III.

before,

we have

seen (p. 12) that

and b=c

\a=o

sin A,

sin

A=

.-.

and

cos

cos A.

A=A

and these expressions *ivo the following rule


Rule.
Divide the perpendicular by the hypotenuse, the
:

quotient will bo the sine of the angle at the base.

by the hypotenuse, the quotient

A reference to

the haso.

angle

An

itself.

Divide the base

will be the cosine of tho angle at

the table will, in either case, give the

angle being thus found, tho remaining side

"of

the triangle becomes determinable by cither of the foregoing rules.


Or, without
triangle

first

may

an

angle, the remaining side of the

be computed from Euclid 47, I

proposition, "-J-o ;
*,

or,

finding

which isthesame

=c

for since

by that

!
.

<s= V {t? 6 and 1= </(<*")


a=V (+&) (e b) |andi=^j
s

).

(c+a)(e)

In a right-angled triangle are given the perpendicular a=


192 feet, and the hypotenuse e=240 : to find the angles A, B, and
1.

the base

b.

The work

is

For

as follows

the angle

A.

240)192(-8=sin 53

..B=90 Q

* The learner will


depression of a point
elevation of

For
8'

the base b.

cob 53

8'=-6

and 240X -6=144=0.

192
,-.A=53 3 8'.

recollect,

A from

53 S'=36=52', and &=144


from Euclid,

an elevated point

29,

B,

feet.

L, that the angle o


is

equal to the angle of

from A.

Digiiizod by

Google

19-

INTEODUCTIOTT.

To

find the base 6, without first

computing the

20736(144

angle A, we have

6= V

(o

The

=V

+ a) (ea)

24 ) 10

= v"2073S

432 x 48

operation for this square root

284)1136

in the

is

margin.

1136

Given the hypotenuse c=d4-6S foet, and the base 6=35-5,.


A, B, and the perpendicular a.

2.

to find the angles

For

the angle

A.

For

the perpendicular a.

31'= -7606

sin 49 s

-(6492^003 49 31'

5,4/6,S)35-5

32808

54-68

2692

-7606

2187

3280S
3280S
38276

505
492

f.'=-ll-UH!)))l.)fj

.-.

A=49

31'.

..

B=90

40= 31'=4029', and <c=41-6

In computing a, as above, it is plain that several


more decimals are calculated than are at all necessary
the contracted method, as exhibited in the margin,
;

dispenses with these superfluous figures {see the llu-

To

find a independently of the angle

a= V'

=y

(c-t-b)(cb)
j

The

in

the

=s */1729-6524.

90-18 + 19'ia
J

And

margin.
is

is

1729 '6524(41-589'

the

a sufficient

confirmation of the accuracy of all the


operations.

3281
32

A, we have

extraction of this squaro root

exhibited

agreement of the two results

feet.

54'68-

6067
38276

16
81)

129
SI

825)4863

4125

Digitized 0/

Google

20

I5TRODUCTI0H".
Given the hypotenuse c=200 feet, and the base 6=118
and the perpendicular a.

3.

feet,

to find the angles A, B,

Ans. A=53 J 51', B=36 9', b=161'6 feet.


Given the hypotenuse c=645 feet, and the perpendicular o=
407-4 feet, to find tho angles A, B, and the base 6.
4.

A=39

Ans.

(p. 14)

B=

10',

IV. The lose and perpendicular


the sides and angles being as

i/iven.

6=500

50 50',

The

letters

we have

hefore,

feet.

denoting

already seen

that

a~ b tan A

,-,

tan

A]soJ

The

= tttanB

tan

.'.

and

c=

Divide

the perpendicular

by tho perpendicular
angle at the vertex.

by

the base

c,

or,

the quotient

divide the base

the quotient will be the tangent of the

An

angle being found, the hypotenuse

be computed as already taught;


for

rule therefore is as follows

Btjle.

will be the tangent of the angle at the base

may

from the general expression

or,

above, without the aid of an angle.

Given the base i=3$-5, and the perpendicular a=:41-6,


and e.

1,

to

find A, B,

For

the

angUi

35;5)41-6(1-I718

A and

For

B.

the hypotenuse.

tan 19 31'

c=6-j-

90

3BS

cos.

355

(54-68

32460

3040

255
2485
Go

sos A.

A=-6,4,3,2)3S-5

.-.13= 40-29'

61

2597
413
.'.A=49

31',

B=40

20',

c=54'G3

ft.

339

355

54

295

51

2340

The same value

of c will be given

by the formula

*/

{a"

b")

Given o=75fi4 yards, and 6=3987 yards, to find A, D,


and c.
Ans., A=6212', B=2748' c=8550 yards.
2.

ISTBODUCTION.
3.

Given tho base of an

21

isosceles triangle equal to 71 feet,

the altitude equal to 41*6 feet

58',

The preceding
all

each of the equal sides 54'68

and

rules

=80

feet.

practical illustrations exhibit, with

we would
by navigators and surveyors

necessary fulness, the arithmetical operations which

recommend always

to bo adopted

in tho solution of right-angled triangles.

Persons engaged in

calculations of this kind, almost invariably use logarithms

work

certainly thus

is

shorter,
is

and

required the other parts.

Ans., base angles each =4931', vertical angle

but a

made

trouble,

and

this

the

somewhat

to appear, in general,

experience will prove that this greater brevity

little

attended with an increased consumption of time.

of logarithms is not bo

much

Tho

object

to save figures as to save timo

and

they signally effect in

the

latter object

all

computations of trigonometry, except in those confined to rightangled triangles

and as before remarked, it is with right-angled


seamen have to do in calcu-

triangles, almost exclusively, that

lating the course, distance,

&c,

of a ship at sea.

Keeping, therefore, the special purposes of the present rudi-

mentary

treatise in view,

we

shall discuss the subject of oblique-

angled triangles with less amplification.


on logarithms must, however,

The following

article

be previously studied, not only

on account of the use of these numbers in the solution of obliqueangled triangles, but also because a familiarity with logarithms
is indispensable in the operations of nautical astronomy.

On
Logarithms are a

set of

Logarithms.

numbers contrived

for the purpose of

reducing the labour of the ordinary operations of multiplication,


division,

and the extraction of

roots,

and they are

of especial service

in most of the practical inquiries of trigonometry and astronomy.

In what has been delivered

in

the foregoing article,

the

arithmetical operations referred to, multiplication and division,


have entered in so trifling a degree, that no irksomeness can
have been experienced in the performance of them, and, therefore,
the want of any facilitating principle cannot have been felt.
But the learner will readily perceive that if the work of any
of the examples just given had involved tho mult i plication together of two or three sines or cosines, or successivo divisions,

22
by

INTBODUCTIOTT.
these, the calculations

would have become tedious, aud the

risk of error, in dealing with so

as

is

it

many

figures, increased.

Now

main object of logarithms to convert multiplication


and division into subtraction, the value of these

the

inte addition,

numbers in computations such

We

as those just

shall in this articlo briefly

mentioned

is

obvious,

show how the conversion alluded

to is effected.

Two
1.

algebra will have to bo


Rudimentary Algebra).
any number, and x and y any expoit, agreeably to the valuation for powers and

principles fully established in

admitted.

(See

That

if

represent

nents placed over


roots, then,

H
2.

N*=S , +*

and N'-j-N^N"-*

That S being any positive number greater than unity, and


any positive number ehosen at pleasure, wo can always

also

determine the exponent x so as to satisfy the condition,


This last truth being admitted,

it

K I =n.

follows that every positive

number () can be expressed by means of a single invariable


number (N) with a certain suitable exponent {x) over it. For
let 10 be chosen for the invariable number N, and let
any number, say the number 58G2, be chosen for w. Algebra
teaches that the value of x that satisfies the condition 10'=o8G2

example,

is

^=3-768046,

so that
!

10*

so that if the

""=6862,

power of

10,

that

is

10'

OOU

=5a62

denoted by the numerator of the

exponent, were taken, and then the root of that power, denoted

by the denominator, were extracted, the result would be the


number 58G2. It is the exponent of 10 just exhibited, namely,
3- 7GS046, that is called the logarithm of the number 5862.
In like manner, if any other value be chosen for u, algebra
always enables us to find the proper exponent to be placed over
the base 10 to satisfy the condition

10*=n

thus

10'" S= *=73540
10"'""'"=327, I0 i '"""' s
=i761,

so that

log 327

= 2-514348,

log -17G1

= 3-G77G9S,

Iog73j40

4- 86G524.

A table of the logarithms of numbers is nothing more than a table


of the exponents of 10 placed against the several

numbers them-

Digilizcd 0/

Google

INTEODUCTIOy.

Any number

selves.

above unity, other than 10, might serve


but there are peculiar

for the base of a system of logarithms,

advantages connected with the base 10 which have recommended


it to

general adoption.

The

actual construction of a table of logarithms, notwithstand-

ing the appliances of modern algebra,


able labour

managed by

be easily

Adverting to tbe

know

is

a work of very consider-

but this labour onee performed, arithmetical com-

that would otherwise be nearly impraotieable,

putations,

first

can
see.

we

aid thus afforded, as

we

now

of the above algebraical propositions,

tlie

shall

that

10*x 10=KK+i-, and 10"-f-10r=10*-.

Tbe first equation shows that the logarithm of the product of


two numbers is the sum of the logarithms of the factors or numbers
themselves, and the second shows that the logarithm of tho
quotient of two numbers is tho difference of the logarithms of the
numbers themselves.
Hence if we have to multiply two numbers together, we look
in tbe table for tbe logarithms of those numbers, take them out
and add them ; the sum we know must he the logarithm of the
product sought, which product we find in the table against the
logarithm. If we have to divide one number by another, we
subtract tbe logarithm of the latter from that of the former, the

remainder
table

we

is

the logarithm of the quotient, against which in the

find inserted the quotient itself.

If several factors arc to be multiplied together, then the loga-

rithms of

all

are to be added together to obtain the logarithm of

and in the case of successive divisions, the logbe added together, and the sum

the product;

all the divisors arc to

arithms of

subtracted from the logarithm

remainder

is.

of

the proposed dividend; the

the logarithm of the final quotient. The logarithmic

operation for finding a product, at once suggests that for finding

a power which
'

is

only a product raised from equal factors.

the power arises from p factors each equal to n, then

it is

that p log n must be the logarithm of that power, that

log h.

root,

If instead of a

is

power of a number wc have

to

tbe ;>th root of n, then representing this root by

log

we have

it

r''

,*,

log

= r log n.
'

;i

log r

,*,

log

i*

n*=

that

putting,

n'

If

plain

compute
r,

is

24

IKTHODUCTIOTT.

Even
'',

if

the root were

still

more complicated,

then, as before, representing

=r

.'.

= r'

.*,

it

by

r,

aa for instance,

we have

m log n = p log r

.-,

log r

jjlos

"We thus derive the following practical rules for performing


more troublcsonio operations of arithmetic by logarithms.
Multiplication. Take the log of each factor from the table and
add them all together the sum will be the log of the product.
Refer to tho table for this new log and against it will be found
the number which is the product.
Division.
Subtract the log of the divisor from that of the
dividend the remainder is a log against which in the table will
be found the quotient.
Powers and Moots. Multiply the log of the number whose power
or root is to be found, by the exponent denoting the power or root,
whether it be integral or fractional ; the product will be a log
against which in the table will be found the power or root sought.
It may be proper to mention here that the decimal part only of
the logarithm of a number is inserted in the table ; there is no
occasion to encumber the table with the preceding integer when
the log has one, as this may always be prefixed without any such
aid ; and this is the principal advantage of making the number 10
the

the base of the table

for since

I0'=10, 10100, KfclOOO, 10^=10000, &c,

we

see at once, 1st, that the log of a

number

consisting of but

a single integer, however many decimals may follow it, being


less than 10, cannot have its log so great as 1 ; hence the integral
part of the log of such a

number must bo

0.

2nd, that the log of

a number consisting of two integral places, with decimals or not,


is, a number between 10 and 100, must lie between 1 and 2
hence the integral part of the log of such a number must be 1.
number having threo integral places,

that

3rd, that the log of a

or lying between 100 and 1000, must have 2 for

Hence when any number

many

is

places,

teristic as it is

minus

sometimes

and the proper decimals

I, will

it

integral part.
to

count how

the figure espressiog the

be the integral part or charac-

called, of the log of the

may

its

we have only

proposed,

integer places there are in

number of

then be united

proposed number,

to it

from the table.

Digitized by

Google

25

ISTEODUCTIOH".

Thus, as 235'6 consists of three integer places, the integral part


of

its log is 2

its

log

is 3,

as 4368 consists of four places, the integer part of

and

we

decimal parts

By

so on.

referring to the table for the proper

find

log 235'6=2-372t75, and log 43G8=3'G40283.

The tables here referred to arc those published in the Rudimentary Series, under the title of "Mathematical Tables," to
which is prefixed a much more comprehensive account of logarithms and their construction than is suitable for this place, and
to

which, therefore, the learner

is

referred for all additional infor-

mation necessary.

Hules and Formula for

The present

the sohdiun

of oblique-angled Triangles.

The space which

article will be entirely practical.

tlds introductory chapter has already occupied forbids that further

extension of

it

which a

full discussion of the theory of oblique-

angled triangles would demand, and which, in


accessible to the learner

ter

iii.).

~v7c

faot, is

already

in the Rudimentary Trigonometry (chap-

might, indeed, have been justified in omitting the

preliminaries on which we have been dwelling altogether,

and have

contented ourselves with a general reference, on these points, to


the work just mentioned.

any

But books on Trigonometry not having

special practical object in view

as

we have

in the present

treatise are generally deficient in that amount of mere arithmetical illustration which he
so

muoh

requires.

And

in

who

is

in training for actual practice

such examples as are given in these

hooks the writers usually consider each case as one in which the

utmost attainable accuracy of result

is

to

be secured, and they

accordingly calculate their angles to seconds.

Such refinements

are worse than] useless in Navigation; they tend to mislead the


calculator,

and to beget a

false confidence in his conclusions

there

are always errors in the data, practically unavoidable, which

render the results of the computations founded upon them at best


but approximations to the truth. It is the business of Nautical

Astronomy

to

supply the short-comings of Navigation, and to

rectify its inaccuracies.

Digitized by

Google

IKTB0DUCTI01T.

Utiles

and Formula! for Oblique-angled

Triangles.

In the solution of oblique-angled triangles there are three cases,

and only three cases, to be considered. The data or given parts


must he either,
1. Two angles and an opposite Bide, or two sides and an opposite
angle.
2.

Two

3.

Tho three

I.

Given either two angles and an opposite

sides and the included angle.

Rule.
angle

is

sides

to find the other parts.

side,

or two sides and

angle.

tijqiosite

,vii

If two sides are given, then the sido opposite the given
to the other side as tho sine of that given angle to the

sine of the angle opposite tho latter side.

If

two angles

then the sine of the angle opposite the given side

is

are given,

to the sine of

the other givon angle as the given side to the side opposite the
latter angle.

This

moro

is

"briefly

expressed by the precept that the sides of

triangles are to one another as the sines of the angles opposite to

them

by the formula

or

a, b

sin

stand for any two sides, and A,

where

sin

for the angles oppo-

them (Rudimentary Trig. p. 52). The first and third


terms of this proportion are always the two given parts opposite to
site to

each other
proportion

Bote.

when an angle is to be found, the first term of the


is a side
when a side is to be found, the first term is a
:

It is proper to apprise the learner before he proceeds to

logarithmic operations, that the log sines, log cosines,


all

are

See.,

computed on tho hypothesis that the numerical value of tho


is not 1 but 10 "; so that the log of this

trigonometrical radius

radius
cosine,

is

&c,

increased

by

In consequence of this change, every log

10,
is

the logarithm of the natural

sine,

sine,

cosine,

log

&c,

10.

Example. Given two sides of an oblique-angled triangle, 336


and 355 feet, and the angle opposite the former 49 26': required the remaining angles?

feet

Digitized 0/

Google

ENTKODUCTIOS.

27

Here are given =336, 6=355, and A=49

As a=336,
6=355,

which the log

of

sin A, 49 26'

53 23'

is

B.

26', to find

2-52634 to be subtracted.
2-55023
9-88061

12-43084
Bin

"

In

of the

1!,

9-90450

He mainder.

this operation the log at the top is subtracted

two

from the sum

logs underneath, since, in logarithms, addition supplies

the place of multiplication,

and subtraction that

of

division.

But, by a simple contrivance, the subtractive operation


dispensed with, and the whole reduced to addition.
is

the plan adopted to bring this about;

may

bo

The following

the subtractive log

2-52634, being before us in the table, instead of copying

it out,

figure by figure, we put down what each figure wants of 9, until


we arrive at the last figure (in the present case 4), when we put
down what it wants of 10. Thus, commencing at the 2, we write
down 7 passing to the 5, we write down 4, to the 2, we write 7
and arriving at the 4,
to the 6, we write 3
to the 3, we write 6
wc write 6 so that, instead of the logarithm 2-52634, wo write
down 7-47366, which is evidently what the logarithm itself wants
of 10; in fact, in proceeding as just directed, we have been
;

merely subtracting, in a peculiar way, 2'52634 from 10, the remainder being 7-47366 ; this remainder is called the arithmetical
complement of 2-52634

and a

little

practice will render it quite

as easy, by looting at the successive figures of any log, to write

down

the arithmetical complement of that log as to copy out the

Now, if, in the foregoing work, we had omitted to


log itself.
introduce the subtractive log 2-52634, our result 12-43084 would
have been erroneous in excess by 2'52G34; and if, in addition to
suppressing this subtraction, we had actually added the complement 7'473G6, the result would obviously have erred in excess by
10, an error very easily allowed for by the dismissal of 10 from the
total amount.

And

always adopted, so that instead

this is the plan

of the above, the work would stand as below


As a=336, arith. comp. 7-47366
:

6=355

2-55023

sinA,492G'

9-88061

sin B, 53 23'

9-90450
c 2

28

U-'TBODCCTION.

The result, 10 being suppressed, is tho same as before, and a


row of figures is dispensed with.
We shall give another example, worked out in this way.
2.

Given one

adjacent to

22 37' and 114 40'

it

117 yards, and the angles

side of a plane triangle

required the other parts ?

The sum of the two given angleB being 137


is

18(P

137"

23=42

Hence we have A=42

As
:

the

37',

37',

sin A, 42 37'
sinB, 22 37'

37',

....
....
....

6=66-45

the third angle

and <i=117.

comp. 0-16935
9-58497

arith.

a=117

23',

angle opposite the given side.

B=22

2-06819
1-82251

now to find the third side c, for which


A=42 37', 0=114 46', and =117.

It remains

have given

As

sin A, 42 37' arith. comp.

we

purpose

0-16935

sinC, 114=46' (supplemont=65 14 ) 9-95810


1

a=117

2~19564

c=156-9

Bote.

2-06819

The

case in

which the given parts are two

sides

and an

angle opposite to one of them is, in certain circum stances, a oasc


may be two different triangles
having the same two sides and opposite angle in common, and the
remaining three parts in each different, so that we may be in
doubt as to which of the two triangles is that to which the given
of avih'ijuity ; in other words, there

parts exclusively refer,


A'

Thus
\

sides

let

A B

C he a

BUOn *^ a*
8X0 A
centre,
and with C

/y\

A,s

/
b
two

B A

A',

prolonged in A'.

triangle,

and C

as

described from
as radius,

may

11,

tbe two sides

A',

The angle

and the

side

CA

or

C A'

opposite to it being-

given, the rule would determine the sine of tho angle

A' opposite the other given


sine in the table

which

ia

its

cut

It is plain that the

A B C, A' B C, will have the


and the angle B in the one, the same as
C B, and the angle B in the other.

C two triangles,

C A, C

is

acute,

or

side.
The angle connected with a
but we know that the obtuse angle

supplement has the same

sine, so

that in the

am-

29

INTEODUCTIOS.

biguous case the acute angle has no more claim to selection

than the obtuse angle.

It is plain that in the

above diagram the

B AC and A' are supplements of one another, inasmuch


B A C and C A A' are, and C A A', C A' A are equal.
If, however, the given angle B be obtuse, then there can be no

angles
as

ambiguity, since both the remaining angles must be acute.

Neither can there be any ambiguity

if,

B being

opposite to it is greater than the ether given side

acute, the side


for the greater

side being opposite to the greater angle, the angle whose sine is
determined by the rule must be also acute, and leBs than the

given one.
It thus appears that the ambiguity can have place only

the given angle

the sought angle

that

we

is

either that furnished

But in actual

supplement.

when

and the side opposite to it less than the


In these circumstances all wo can say is, that

is acute,

other given side.

practice

it

by the

tables o its

can but seldom happen

are so unacquainted with tho form of our triangle, as to

be in doubt as to whether the angle in question

is

aeute or

obtuse.

H. Given ttoo sides and the included angle.


EtJxe. As the sum of the two given sides

Is to their difference,

So

is

the tangent of half the

To the tangent

sum

of the opposite angles

of half their difference.

Or, expressed in Algebraic symbols instead of words, the rule

tan h

(A

B)

tan 4 (A

~ B)

the given parts being a, b, and C, and consequently


-f B, since
90'
180"
B)
The last term
C, or i (A
C.
of the proportion being found, a reference to tho table gives us

A+B

B), which added to \ (A + B) gives the greater of the


\ (A
two angles A, B aud subtracted from \ (A + B) gives the less.
Example. Given two sides of a triangle equal to 47 and 85
and the angle between them 52 40', required the
remaining parts
;

respectively,

= 85, b = 47, C = 52 40'

Here a
.

J (A

B)

= 90 26' 20' = 63 40'

Oigiiizod By

Google

INTBODUCTIOH.

8Q

+b=

Also a

As a
a

b = 38.

and a

antli, coinp.

(A

II),

B)

The greater angle


The less angle

sin

Vi,

1-57978

63 40'

1 0-30543

30 11'

9-76404

= 93 51'
B = 33 29'

"We have now to determine the side

As

7-87943

.....

tan

32,

= 132

= 38

tan \ (A

33" 29' arith.

as follows

e,

comp

-25830

sin C, 52" 40'

9-90043

= 47

1-67210

= 67-74

1-83083

HI. Given

the three sides.

For the solution of

this ease it is better to

than from any rule expressed in words.

work from a formula

There are two formuhe

adapted to logarithmic computation, and these very readily furnish a third. It is generally matter of indifference which of the
three be employed, at least as respects accuracy of result

second of them
brevity.

is,

however, a

little

the

preferable on the score of

In certain extreme and therefore unusual instances,

however, one form

is to

be preferred to another to secure greater

precision, as will bo noticed presently.

sum

of the three sides, that

And

dividing the

is, let s

Let

= (a

riniA=v'^Ul^
cos

sine divided

by

v"

first

gives

cosine

formula

taniA

jjl^lg)

of these

=V

by

stand for half the

then

(1)

(2)

the second, recollecting that

tangent,

*~^ ~
8

4- 5 4- c)

we have

for

a third

C)
(3)

If the anglo A, to he determined, is foreseen to bo so small as


to amount to only a few minutes, then A had better bo derived
from the first formula or the third instead of from the second,

because the cosines of angles differing but

little

from O 3 differ
,

themselves by so small a quantity that the fiye or six leading

Digitized By

Google

INTRODUCTION.
decimals

may

81

equally belong to several consecutive cosines, so

we

that if our table be limited to this extent of decimals,


find a succession of small angles with the

shall

same cosine against

each, so that if wo enter the table with this cosine, we shall be at


a loss which of these small angles to select.
If the angle A bo very near 180, and therefore i A very near
90, then the second formula will be preferable to the first, because
very near 90* the sines differ from one another only in their

remote decimals.

These niceties, however, are only worth attention in oases


where the minutest oceuracy is desirable in Navigation any one
:

of the above formula?

Example.

is

just as good as another.

The three sides of a triangle are


a

required the angle

By Formula
a=19fi

= 195, b = 216, c = 291,

A?
(1).

6=210

arith.

comp.

7-CG555

ap.

7-66555

c=-291

arith.

coup.

7"5361l

7-53611

2)702

2)19-94009
.

-,

A =42

S-97005

i'

In eaoh of the foregoing operations two arithmetical complements are introduced, consequently the result of the addition is
;
but after the division by 2 the final
but 10 in excess, which additional 10 is necessary to

in each ease too great by 20


result

is

complete the logaiilhniia tine, and cosine of the table (see

p. 26).

Oigiiizod By

Google

PBIKCTPLES OP KAYIGATIOS.

32

THE PRINCIPLES OF NAVIGATION.


The business of navigation

known place on the surface


may be intended to reach

is

to conduct a ship from any-

of the globe, to any other she


as also to determine her posi-

tion at any period of the voyage.


into two distinct branches

Astronomy.

The

subject

divided

is

Navigation proper, and Nautical

It is with the former branch only that

we

are

comprehends all those operawhich are independent


of an appeal to the heavenly bodies, and which are matters
of daily routine on shipboard.
If the direction in which a ship is sailing at any time, and
the rate of her progress through the water, could always be
measured with accuracy, there would be comparatively but
little need for astronomical observations in navigating a
veBsel from one port to another but impelled by the wind
and the waves forces proverbially fickle and inconstant
the practical difficulties in the way of such accuracy of
measurement are insuperable and therefore, as already obat present to

be occupied:

it

tions, tributary to the ultimate object,

served, the mariner

must content himself with approxima-

tions only to the truth.

But, fortunately for him, the tur-

bulence of the ocean can never disturb the tranquillity of


the nines

his own unavoidable


moon and the sun have

and he knows that during all

aberrations from his proper path, the

never for an instant deviated from theirs.


positions of these

How the relative

two bodies, or the position of the former in

reference to the stars

among which she moves, can enable

the navigator to correct his own position, and thus, with


renewed confidence, to start afresh, is an inquiry to be
answered only by Nautical Astronomy.

DigiuzM by

Google

CHAPTEE
DEFIK1TI0HS.

I.

IFSTEUMENTS.

In Geography and Navigation the earth is regarded as a


It is known from actual measurements at various

sphere.

parts of its surface to slightly differ from this


flattened at the poles, as a

axis

may be

sphericity

is

it

is

little

body constantly rotating on an


But the departure from
no practical error of any

expected to be.
so

trifling,

that

moment can arise from our treating the earth as a globe,


in laying down geographical positions, and in framing
directions for sailing over its surface.
With a view to
these objects, certain lines are imagined to be traced on the
surface of the earth

and on the

artificial globes,

on which

the prominent features of this surface are depicted, the

imaginary lines alluded to are actually drawn.


nitions, with those of certain

follows

Axis.

The

is

defi-

axis of the earth is the diameter about which

performed

its daily rotation is

tion

Their

remarkable points, are as

from west to east;

it

is

the direction of this rota-

completed in twenty-four

hours.

Poles.

The two

poles of the earth

extremities of the axis are called the

that to which we, in these countries, are

the North Pole, the other

is the South Pole


as
they are the extremities of a diameter, they are 180

nearest,

is

apart.

Equatob.

The

equator

is

a great circle

on the earth

equally distant from the poles, dividing the globe into two
o 3

DigiiizM By

Google

DEFIBTHOirS.
equal parts, or hemispheres,

the northern hemisphere and

the southern hemisphere. The poles of the earth are the


poles of the equator, every point in this latter circle being

90 (of a great

circle) distant

observed, that

by a great

from either

It

pole.

meant a

circle is

must be

upon

surface;

its

the

circle of

sphere, having for its centre the centre of the sphere

greater circle can be traced

all

no

other

circles are called small circles.

Mekidiaks
the other

Every semicircle drawn

is called

through which

it

from one pole to

the meridian of every place on the earth

Of

passes.

all

the innumerable meridians

may be imagined on the globe of the earth, one is


always selected by every civilised kingdom as a principal, or
meridian; it is usually that which passes through the
that

first

national observatory

in this

kingdom the

first

meridian

is

that of the Greenwich observatory, in France it is that of

the Paris observatory.

Latitude. The latitude of any place on the surface of


the earth is the distance of that place from the equator,
measured in degrees and minutes on the meridian of that
place.
The latitude is north when the place is situated in
the northern hemisphere, and south when it is situated in

The latitude of each pole is 90,


must be less than 90.
Parallels of Latitude. Every small circle on the

the southern hemisphere.


that of any other spot

globe, parallel to the equator, is called a parallel of latitude

every point on

its

circumference, being equally distant from

the equator, has the same latitude.

Difference of Latitude. The


any two places

is

difference of latitude

of

the arc of a meridian contained between

the two parallels of latitude passing through those places.


If the places are both on the same side of the equator, their
difference of latitude is found

tude from the greater

if

by subtracting the

less lati-

the places are one on each side of

the equator, their difference of latitude

is

found by adding

the two latitudes together.

OigiiLzed 0/

Google

DF,FINIT IONS

Longitude.

The longitude

35

of any place on the earth

the arc of the equator, intercepted between the

and the meridian of the


of the

first

meridian

If the place

place.

has east longitude,

it

first

is

meridian

to the east

lie

if it lie

to the

west it has west longitude. Wo place thorefore can exceed


180 in longitude, whether east or west.

Difference of Longitude. The


of two places

is

difference of longitude

the arc of the equator intercepted between

the meridians of those places

;
if the places lie both east, or
both west, of the first meridian, the difference of longitude
found by subtraction ; but if one have east longitude and

is

the other west, the difference

Hoeizon.

is

found by addition.

A plane conceived to touch the surface of the

earth at any place, and to be extended to the heavens,

And

called the sensible horizon of that place.


parallel to this, but passing
is

called the rational horizon of that place.

whether sensible or

mote bounding

is

a plane

through the centre of the earth,

The

horizon,

a plane; but the rewhich, to an eye elevated above the

rational, is thus

circle

surface of the ocean, appears to unite sea

and

sky, is that

which mariners more commonly regard as the horizon, and


the sea-horizon, or

call it

offing.

The plane

of this circle

obviously dips below the planes of the sensible and rational


horizons,

and the amount of

called the dip

this depression is that

which

is

of the horizon.

The Compass.

The

straight line in which the plane of

the meridian of any place cuts the sensible horizon of that


place, is called the horizontal meridian, or north

is

and south

and the horizontal straight line, perpendicular to this,


the east and west line of the horizon.
The sensible

line

horizon

under

is artificially

Bide of

which

represented by a circular card, on the


is fixed

a magnetised bar or

Needle,

in the direction of the north and south line, or horizontal

meridian.

The

horizontal,

and to turn

card being so suspended as to always remain,


freely about ita centre, the tendency

of the needle to point north and south causes the meridian

DigiiizM By

Google

THE cour-Ass.
line,

on the upper surface of the card, to settle in the proper


and the intervals between the four points E. W.

direction

N.

S.

the four cardinal points

subdivided,

as they are called

being

the annexed figure, the instrument

aB in

is

placed securely in a brass circular bos or bowl with a glass


cover,

and hung upon brass hoops (gimbals), so that the

horizontal position of the card

motion of the

ship.

may not be

This instrument

disturbed by the

is

Tee Mariner's Compass.

The

four quadrants into which the meridian line If.

S.,

and

the east and west line E.W., divides the rim of the card, are

each subdivided into eight equal parts called points, so that


each point

is

an arc of 11

into quarter points.

into

360 degrees

15',

and

The outer rim

this is farther divided

of the card

is

divided

the thirty-two points of the compass,

and the angles at the centre which the corresponding

OlgilizM by

Google


37

THE COMPASS.

make with

lines

(neglecting

the meridian

minutes) are exhibited in the following Table

of

quarters

POTHTS.

2' 49'
5

N.

6.

N. N.E,

N.

N.W.

N.

b.

W.

lj

N.E.

b.

N.W.

N.

b.

19 41'
22 30'

23

30 56'

z\

N.W.

4$
N.E.

b.

E.

E. N.E.

N.W.

W.

b.

W. N.W.

n
6
8i
6J
8?

E.

6.

N.

W.

b.

N.

n
n

W.

The compass

is

S. S.E.

3*
31

N.E.

37'J
8 26'
11 15'
14 4'

lj

S.E.

36" 34'
39 22'i
42 11'
45 0'

53
56
59
61
64
67
70
73
75
78
81
84
87
90

b. S.

3.S.W.

S.W.

6. S.

S.W.

S.E.

26'
15'

8.B.

E.

b.

S.W.

W.

6.

4'

52'
41'
30'
19'

E. S.E.

W. S.W.

56'

45'
34'

E.

b,

S.

w.

b.

22'i
11'
0'

w.

E.

placed near the helm, and the line from

the centre, in the direction of tho ship's head, denotes the


angle which its track is making with the meridian, or north

and south

line

N.

S.

It

is

noticed, however, that the

zontal meridian

this line, deviates

called a

N. S.

rhumb

line is

line.

It must he

not truly the hori-

the needle, which settles the position of

from the true direction

it

does not point

accurately to the north, and the angle between the true

meridian and that in which the needle

settles, called

the

Digitized 0/

Google

COUESES, LEEWAY, ETC.


magnetic meridian,

amount of

the variation of the compass.

is

this variation at

site

The

any place may be discovered by

Nautical Astronomy. Another correction

is

in general requi-

the iron in the ship necessarily influences the needle,

the disturbance thus occasioned

ia called

the demotion of the

compass, the amount of which can be ascertained only


special experiments.

by

Since the introduction of iron vessels,

engaged a good deal of attention


more fully in a future chapter.
So long as a ship sails on the same rhumb-

this local attraction has

we

Bhall advert to the subject

Coueses.
her track makes the eame angle with the successive
this angle, indicated by the compass, is called
If the course be not corrected for varia-

line,

meridians

the Bhip's course.

tion, it is the compass-course;

The compass

course.

course,

when

corrected,

be

observed, supposes the

it

it is

the true

needle to be previously freed from the effects of the local

The contrivances for this purpose


The variation, whether to the
right or left, when known, is easily allowed for in what
follows in the next cliapter we shall suppose the allowance
attraction of the ship.
will

be noticed hereafter.

to be made, and the courses mentioned- to be the true courses.

Leeway.

The course may

also

be

affected

by the leeway,
by the action

or the oblique motion of the vessel occasioned

of the wind sideways, impelling the ship along a track


oblique to the fore-and-aft line this angle of deviation from
is the leeway
it is to
be estimated and allowed for, according to circumstances,
from, the navigator's observation and experience of the
;

the direction shown by the compass

behaviour of his ship.

Bate op

Sailing.

on any course

is

or the log-skip,

which

The
rim

is

is

The rate

at

which a ship

is

sailing-

measured by an instrument called the

and a

log,

line attached to it called the log-line,

about 120 fathoms in length.

log itself

is

wooden quadrant, of which the

loaded with lead, so that

into the water,

it

settles in

when

it is hove,

an upright

or

circular

thrown

position, with its

Digiiized by

Google

THE LOO.

35)

centre just above the surface, ant! the log-line


to

that the face of the log

it

order that

may

it

offer

dragged along, as tho line

is

is

being

the advancing motion of the vessel.

unwound
sailing

The

is

so fastened

kept towards the ship, in

the greatest resistance to being

unwound from a reel by


The length of line thus

in half a minute gives the distance run, or rate of

per

liour,

on the following principle

log-line is divided into equal parts, each part being

Now

the 120th of a nautical mile.

a nautical mile that

is,

the 60th part of a degree of the equator, or of a meridian,

about 6080

is

log-line i3

By

50

feet, so

feet

that each of the equal parts of the

8 inches.

The

several divisions are

marked

pieces of string passed through the strands of the log-

line

and knotted, the number of knots in any string indinumber of parts between it and the end of the
that is, how many parts have run off the reel. If,

cating the
log-line

therefore,

we

take note of the

half a minute,

we

shall learn

number of knots reached

how many

many

in

120ths of a mile

the Bhip has sailed in the 120th of an hour, which


course, be at the rate of so

miles per hour.

will,

As

of

the

knots thus give the miles per hour, sailors are in the habit
of calling the miles sailed per hour so

stray-line,

knots.

log-line

a portion of line
run out before the marking begins.
the

many

do not commence at the log


about 10 or 12 fathoms, is suffered to

The marks on the

This portion

which allows the log to

settle in

is

called

the -water

commences the
marked by a piece of red

clear of the ship, before the half-minute

termination of the stray-line


cloth,

and

is

at the instant this passes from the reel the half-

minute sand glass

is

the sand is run out.


The above is the

turned, and the reel stopped as soon as

common

log

but there

is

an improved

instrument, called Mtmey'a log, constructed on a different


principle,

and which

is

generally preferred.

The course and rate of sailing at any time being measured


by the instruments now described, a record is kept of the

L.'l

Z-J-3

Lv

PLANE SAILING.
progress and position of the ship from day to day.
actual distance run,

gitude

and the

made from noon

to

difference of latitude and


noon being deduced from

The
lonthis

record, without the correcting aid of astronomical observations,

we have the

ship's account

CHAPTER
PLANE SAILING.

Plane

SINGLE

by dead reckoning.

II.

COTJB8ES.

sailing is usually defined to

COMPOUND

COUBSES.

be the art of navigating

a ship on the supposition that the earth is a plane. This


definition is erroneous in the extreme
in all sailings the
:

earth

is

regarded as what

a sphere.

it really is

Every case

of sailing, from which the consideration of longitude

is

a name
which merely implies, that although the path of the ship is
on a spherical surface, yet we may represent the length of
this path by a straight line on a plane surface, and may
embody all the particulars necessary to be considered, longitude excepted, in a plane triangle.* This will sufficiently
appear from the following investigation of the theoretical
Let A, F,
principles upon which plane sailing is founded.
represent two places on the spherical surface of the ocean,
excluded, involves the principles of plane Bailing

the lines drawn from the pole

being meridians equidistant

from one another, and so close together that the intercepted


*

Even when longitude enters into consideration,

plane triangle only that wc have to deal


in

which longitude

sailing

is

concerned

it is still

with the

and the reason that the sailings


sailing and Merea tor's

mid-latitude

are not comprehended

distinctly refer to,

sailed over

under plane sailing, is that those sailings


and are founded upon, the spherical figure of the surface

but, as the investigation hero given in the text shows, the

rales for plane sailing would equally bold good though the surface were a
plane.

Notwithstanding this truth, however,

it

is still

incorrect to say

that these rules are founded on the supposition that the earth

no such supposition

is

is

a plane;

made.

OigiiizM By

Google

PLASB SAILIKO.

A B, EC, CD, Ac,

portions

from

The

may each

to F,

we may

portions of such trifling length, that


to estimate

41

of the ship's track, in Bailing

be regarded as a atraight

learner will at once see that

them other than

it

would be impossible

as straight lines

conceived, for instance, as only a yard or


also

UZ

line.

conceive these

they may be

two

long.

Let

be an arc of the

equator, and draw the parallels

of latitude aB exhibited by the

dark lines in the figure.


series

A H B,

triangles,

of

BIC, CKD, Ac,

will thus be
formed on the surface of the

sphere,

so

may be

that

small,

each

regarded

practically

as a plane triangle, without

any

sensible

angular;

for

H,

Ac., are

I,

K,

angles,

These

error.

plane triangles are

all

the

equi-

angles at
all

right

and the ship's track

cuts every meridian which

it

crosses, while preserving the

same course

at the

YL, we have

the continued proportion

same angle.

Consequently, by Euclid

4.

AB:AH::BO:BI::CD:CK, Ac;
and
its

Bince, in

a continued proportion, one antecedent

is to

consequent as the sum of the antecedents to the sum of

the consequents (Euc.

5. V.),

we have

AB AH AB+BC+CD+Ac AH+BI+CK+&C.
Now AB+BC + CD+Ac, is the distance sailed from A
:

to

::

on the course

HAB and AH+BI+CK:+ Ac, is the


AO between A, the place left, and F,
;

difference of latitude

the place arrived

at.

Let now a right-angled plane


right-angled triangle

AHB,

triangle, similar to the little

be constructed

that

is,

a right-

PLATTE BAILING.

angled triangle in which

A is

the aDgle of the course, and

AB

_ let the hypotenuse

distance

AT

/
/

sailed,

on the glohe

that

represent the
the length of

is,

then

it is

obvious that

the perpendicular .40 will represent the

AO

difference of latitude

OS the

while the base

side opposite to the

will represent

sum

the

KD,

departures, -HB, IC,

course

of all the small

&c, from the

meridians which

successive

crosses.

it

For since

AB HE BC IC CD KD &c.
AB + BC + CD+&C. HB+IC+KD + &C.
:

.;.AB

HB

: :

: :

But the plane

triangle

ABC is constructed so that


AS OB
AS = AB+BC + CD+&C. on

AB HB
:

and, moreover, so that

the

globe, consequently

OS =HB+IO+KD+&c.
OB is colled the Departure made by the ship
in sailing from A to F
there is no line corresponding to it
This length

on the globe

it

merely expresses the sum of

made by the

finitely small departures

all

the inde-

ship in passing over

the small intervals between the innumerable meridians conceived to be interposed between

Z, the meridian arrived


It

is

BU,

the meridian

left,

and

at.

thus fully established that the distance sailed on any

oblique course, the difference of latitude made, and the


departure,

may

all

be accurately represented by the sides of

a right-angled plane triangle, the angle opposite to the


departure being the angle of the course.
just mentioned

Of the

four things

Distance, Difference of Latitude,

namely

Departure, and Course, any two being given, the remaining

two may,

therefore,

be determined by the solution of a


and so far as these particulars

right-angled plane triangle


PLANE SAILING.

43

are concerned, the results are obviously just the same as

they would be

if

the ship were to

instead of on a spherical surface

sail

on a plane surface

the curve meridians being

replaced by parallel straight lines, and the perpendiculars

"We do not

to these regarded as the parallels of latitude.

make
is

the supposition that the surface actually sailed

a plane, with the meridians parallel straight lines

taking the surface as

it really is

far as the particulars

may

spherical,

we

upon
;

but

find that, so

mentioned above are concerned, we

by such a plane surface. And this is the


only justification of the name, Plane Sailing.
In the examples in plane sailing which follow, the learner
is

replace

it

recommended

to sketch the right-angled triangle in each

be the north and the


bottom the south, so that the east will be on the right hand
and the -west on the left. Having drawn a north and south
case, regarding the top of the paper to

line,

representing the portion of meridian due to the differ-

he should draw from the latitude arrived


towards the
it be
the hypotenuse will then represent the distance
and the angle it makes with the difference of latitude,
be the course.
The vertex of this angle is to be
regarded as the centre of the compass, or of the sensible
ence of latitude
at,

the base of the triangle for the departure,

right if the departure be east, and towards the left if

west;

sailed,

will

horizon at commencing the course


right or

left,

the angle will

according as the Bailing

is

lie

to the

towards the easterly

or westerly side of the

meridian started from.

collections of tables for

the use

In

all

of navigators, there

is

inserted a Difference of Latitude and

Departure Table,
usually called a Traverse Table; by entering which, with the

measured course and

distance,

we can

get the corresponding

and departure by inspection.


The
table usually extends up to distances of 300 miles, and may
be nsed for greater distances by cutting up the greater
difference of latitude

distance into parts that will come within the limits of the

table/
* See the Navigation Tables which accompany this Rudimentary Treatise.

L'i j

SINGLE COUBSES.

Examples in Single Courses.


1.

ship from latitude 40 30'

distance of 103 miles

N.

sails

N.W. by

N-, a

required the latitude in, and the

departure made ?

The course being 3 points, is 33 45', the angle contained


between the given hypotenuse 103 miles, and the required
diff. lat.
hence by right-angled triangles, we have
;

For

For

the diff. lat.

= bos coarse x

Diff. lat.

Dep.

(list.

cos 3 points*

-8315

dist.

103

the departure.

= sin course x dist.


= "5556

sin 3 points

dist

Diff. lat.

N.

103
16068

24915
8315

5556

85-6445 miles.

Dep.

= 57 2268 miles.

W.

Sy Inspection. Eeferring to that page of the Traverse


Table headed " 3 Points," we find against the distance 103,
in the column marked "Lat." the number 85 6, and in the
column marked "Dep." the number 57-2; we infer, therefore, that the difference of latitude is 85-6 miles, and the
-

departure 57'2 miles.


Since 60 miles

is

a degree, a nautical mile being a minute

of the meridian, 85-6 miles=l 25'-6, which added to 49 a


30', the latitude left, gives
2.

50 55'-6 N. the

lat. in.

A ship sails from lat. 37 3' N., S.W. by S.

\ S.

a distance

of 14S miles required her latitude in and the departure


;

For
Diff. lat.

Out

For

diff. lat.

= cos course x dist.

cos 3

Dep.

coarse x diet,

pointa=

"773

sin 31 points

148

(list.

dist.

6184
3092
773
Diff. lat. S.

Lat. left

=
=

"0343

143

50744

= 114-404
=

lat. in.

made?

the departure.'

= sin

1"54'
37

3'

35'

9 'N.

* The author has prepared a small table for the

sines, cosines,

of

SINGLE COITJiSO.

By Inspection. "With

the course 31 points, and distance

148, the Traverse Table gives

lat.=114'4, and dep.

diff.

=93-9.

In the foregoing computations we


decimals
the
are
contracted method of multiplication
Note.

in

superfluous.

results

as at

which

is fully

see that several

By

using the

page 19, and

explained in the " ltudimentary Arithmetic,"

may be

unnecessary decimals

the

dispensed with;

thus,

the four multiplications in the above examples, become con-

by reversing the

tracted into the following


8315

301

841

5556

778
309

301

8315
249

167

85'64

57-23

multipliers

6343

773

5556.

841

3537
507

62

93-S7

We have

only to notice under what decimal place of the

multiplicand the units figure of the multiplier stands, iu

order to determine the number of decimals in. the product


thus, in the first operation the units figure is under the

second decimal, therefore two decimals are to be marked


in the product. In like manner, two are to be pointed
in the second operation

one in the third

and two

off
off

in the

fourth.

3.
till

ship sails from lat. 15 55' S.

she finds herself in

lat.

on a S.E. ^E. course

18 49' 8.

required the dis-

tance ruu and the departure made ?

Latitude lea 15" 55' S.


Latitude in. 18 49' S.
Diff. lat.

courses

it will

be ionnd at

tlie

to accompany thia work, and


more extensive tables.

2 54' = 174 miles.

commencement of the NaTigatioa Tables

will save the trouble of searching in the

L'.'i

Lv

46

SINGLE C0DB9EB.
For
Dist.

For

the distance.

kt.H-eos cou

iliff.

Dep.

4points= '6,3,4,4)174

the departure.

= tan course x

= 174,

diff, Jat.

= 1*2185

tau 4J points
ist,

reversed

471

121S5

Dist.=274'3

By

254

milea.

Inspection.

In

that page of the traverse table de-

voted to the course 4^ points, and in the lat. column, is


is the nearest to the given diff. lat., 174.

found 173"8, which

And
dist.

4.

against this number, in the proper columns, are found

=274, and

= 211-8.

dep.

Yesterday at noon we were in lat. 38" 32'


and tliis
lat. 36 56' N.
"We have run on a
between S. and E-, at the rate of 5 knots an
required the course steered and the departure made ?

day at noon we are in


single course

hour

lat. from 33 32'


Lat. in

N.

= uumLcr ofkours.

24

36" 56' H.

Dist.= 132 miles.

For
Cos course

96

diff. lat.

~- dist.

Dep.

the departure.

= sin course x dist.


20'=

.0862

dist. 132, reversed

231

sin 43"

The departure may

be found tlm?,

V {132 + 96) (13296)


= ,/ 228x36 = V8208
j

=90-58

Dep

= 90,58
'

miIoa -

Hence the course


nearly,

steered is 9. 43 20' E., or S.E. by S. E.


and the departure is 90'58 miles Easterly.
may he solved by the traverse table, but

This example

Digitized by

Google

SINGLE COURSES.
not without some trouble

47

we should have

to examine the

we

several pages in which the distance

132

came to that page in which, against

this 132, stands 96, or

is

inserted,

till

a number near to this (m. 96-5), in the lat. eolumn. At


the top of this page will be found tho course nearly, namely,

column the number 90.


Bails N.E. by N. 296 miles
and the departure made ?
52 46' N. dep. E. 164-4 miles.
N. has sailed 8.W. by S. a
distance of 93 miles required her lat. in, and the departure
made ?
Ans. lat. in, 46 9' N. dep. W. 54'45 miles.
7. A ship has sailed from lat. 37 30' N. to lat. 46 8' N.,
on a S.E. by S. course required the distance run and the
departure made P
Ans. dist. 98-6 miles dep. E. 54'S miles.
8. A ship from lat. 3= 16' N. sails S.W. by W.fW.,
until she has made 356 miles of departure required her Int.
-43,

and

in the dep.

A Bhip

5.

required the

from lat 48 40' N.

lat. in,

Ans.

6.

lat. in,

ship from lat. 47 30"


:

in,

and the distance

sailed ?

Ans.
9.
S.

A ship

and "W.

from

till

lat.

lat. in,

36 12' N.

she arrives in

miles of departure

ship in

lafc.

N."W. by "W. "W., in

11.
miles,

A ship
till

and the

S.,

35

1'

46 57' "W.

3 52' S.
lat 4 30'

course must the ship

departure will she

sails in

lat.

dist.

415

miles.

a direction between
N., having

made 76

required her course, and distance sailed

Ans. course
10.

0 17' S.

sail

dist.

bound for a port bearing


N. what distance on that
the port, and what

is

to reach

have made during tho voyage

Ans.
from

dist.
lat.

104 miles.

1065 miles dep. "W. 939 miles.


50 13' sails between S. and E. 98

her departure

is

82 miles

required her course,

latitude arrived at ?

Ans. course S., 56 47' E. lat. in, 49 19'


12. If a ship take her departure at six o'clock in the
evening from Cape Verde, in lat. 14 45' N., and sail
;

"W.S.'W. i"W. at the rate of Beven miles an hour until the

Digitized By

Google

COMPOUND COUnSES.

48

next day at noon: what will be her distance run, her


departure, and the latitude in ?
diet. 126 miles ; dep. W.,
120-6 miles; lat.in 14 8'N.

Ans.

Compound
"When a ship

sails

Courses.

on different courses, as she usually

does in a voyage of any length, the zig-zag track she describes is called a

compound course

or a traverse, and the

determination of the single course and distance from the


place left to that arrived at

In order

is called

resolving the traverse.

to do this, the difference of latitude and departure

for each distinct course

must be found, and the aggregate

of the several differences and departures taken for the Bingle


difference

and departure which would be made by sailing


left to that reached on a single course.
The

irom the place

determination of this course, and the corresponding distance,


is

then to be effected as in the preceding article.


In resolving a traverse it is usual to take the

diff.

lat.

and dep. due to each of the component courses from the


and having prepared six columns, with the
annexed example, to insert each
diff. of lat., and departure, in its proper column.
This done, we have only to add up all the differences of
latitude marked N., and all marked S., and to take the difference of the two sums, and then to do the same with the
departures marked E. and "W., to obtain the diff. lat. aud
dep. due to the equivalent single course.
traverse table

suitable headings, as in the


course, dist.,

Examples in Compound Courses.


1.

A ship from lat. 51 24'

N. during the

last

twenty-four

hours has run the following courses, namely


1st.

S.E.,

40

miles.

4th.

2nd, H.E., 28 miles.


3rd.

8.W. by. W. 52

N.W. byW., 30

5th. S.S.E.,
miles.

6th. S.B.

by

miles.

36 miles.
,.,

58 miles.

Digitized by

Google

COMPOUND COUKBZH.
Eequired

fclie lafc.

arrive at

in,

and the

49

and distance

direct course

to

it

Trayirsb Table.*

N.
40

S.E.

N.B,
S.W. by
S.S.B.
S.E. by

Direct course,

Direct distance

E.

28 '3
IB "8
43
24

16-7
33-3
32'2

25 3 59'E.

S.

W.

s.

28-9

52
30
36
58

IT.

28-3
19-8

23

Vr.

N.W. by

122-7
36-5

36 5

10-1

G8-1

es-i

80*2

95 87 mile

'2

'9

13-8
48 -2

42

The results of the above table show that the whole difl*.
made ia 86-2 miles S., and the departure 42 miles E.,

lat.

and from these we compute the direct course and distance


as follows

For the
Tan course
8 l6\2)42

Far

direct ceurte.

= dep.
= tan 25 SO*

the data-nee,

= dep.

Pist

-f-diff. lat.

('4872

-7- sin

conr

(05-87

4,3,8,1,1)42

8448
752
Lat. left
Diff.

lat.

61 21' N.

1 26' S.

8C-2 m.

" Before referring In the general traverse table,

extracting the several particulars to be entered in

for the

tills, it

jraruoKe of

will be n security

against putting any extract in the wrong column, if against each course
and distance we put a small mark, as a cross, in each column where an
entry connected with that course and distance is to be made, the mark

being put sufficiently near the margin of the column to leave room for the
entry to be placed against

a mark
S.

is to

occurs,

it.

Thus

wherever N. occurs

be placed opposite to that course in the

a mark in the

3.

column.

When

E.

IT.

in

the course-,

column: whererer

occurs,

mark

in like

OigilizM By

Google

COMPOUND COURSES.

50

Hence the course


95-87 miles, and the

The

Note.

is

S.S.E. ^ E. nearly, the distance'

lat. in,

made

balance of the departures,


is

is

49 58' N.

learner should be here

apprised that the

in a succession of courses,

not in strictness the same as the single departure made

from the place left to that ultimately


sailing.
Suppose a ship in any

in the single course

reached by the traverse

due west or due east

latitude to sail

then her entire

dis-

But if another ship were


on the same meridian to the
would exceed

tance will be also her departure.


to sail from a lower latitude

same

place, it is obvious that her departure

and if she sailed from a higher


latitude her departure would be less.
Id a single day's run the inaccuracy of taking the balance
that of the former ship

of a set of departures as the departure due to the single


equivalent course,

too small to lead to any practical error

is

We

of consequence.

shall advert to this

matter

again, at

the close of the neit chapter.


2.

ship from

S. S. E. i E.,

2nd.

51" 25' N. has sailed on the following

namely

courses,
1st

lat.

& S. EL,

16 miles.

23 miles.

it

4th.

miles.

a N., 12 miles.

miles.

and the direct course and distance

in,

Ans. direct course


3.

W.

by E, J E. 41

S. E.

5th.

Required the latitude


to reach

W. by W. i W., 3S

3rd. S.

S.

18 12' E.

disfc.

62|

miles.

ship from lat. 1 12' S. has sailed the following

courses and distances, namely


1st. E.

by N.

N., BS miles.

4th. IT. J E., 68 miles.


5th. E. S. E., 40 miles.

2nd. N. i E., 80 mileB.


3rd. S. by E. 4 E., 96 miles.

7th. E.

manner the

E. column,

the traverse table

6th.

by

and when

may be

S.,

W.

N. H. W. { W., 86 milea.

66 miles.

oceuru, the

W. column.

This done,

referred to for the proper entries to be placed

against the marks.

OigiiizM By

Google

51

COMPOUSD COUBSUS.

inquired the lot. in, and the course and distance made
good ?
Ana. Jat. in, 0 48'N: course, N. 51 47' E,
193'8 miiea.

dist.

noon the following courses and distances


have been run, namely
4. Since laBt

1st. S.

W.

2nd. S.
3rd.

W.

3 W-, 82 miles.

by W., 16

miles.

40 miles.

I 8.,

4th. B.

W.

5th. S.

by

8.

tith.

W., 20 miles.

.,

30

miles..

., 14 miles.

Bequired the difference of latitude made, and the course and


distance

made good
Ans.

diff. lat.

Int.

from

1 55' S.

24 32' N.

lat.

5th. S.

W.

lat. in,

the following courses

sails

S.W. by W., 45miles.

S.W., 30

3rd.

2nd. E. S. E., 50 miles.

Kequired her

course, S. 43 14'

158 miles.

dist.

A ship

5.

W., 63

S. [

miles.

by E

4th. 8. E.

W. by

GO miles.

miles.

her departure, and the direct course

and' distance P

Ans.

lat. in,

dist.

Yesterday noon

6.

then

we have run

lat.

N. N.

E.,

22

we were

miles.

by

4th. E. S. E.,

5th. 8. S.

E.,

40

W., IS

made good

N.

dep. 0
3

lat.

W.

course, S.

18'

S.,

.V

42 miles.
W., 45 miles.

S.,

20 miles.

10th. K. by N. 4 E.

and

and sinee

by H. i W., 50 miles.

W. by S.
W. by

9th. 8.

miles.

lat.

W.

7th. N. E. J B.,
8th.

miles.

25 miles.

Eequired our present


tance

in

6t-h.

2nd. N. by W., 30 miles.


3rd. N. E.

3'

149 miles.

the foHowing courses, namely

22

t>2

miles.

dep., with the course

and

dis-

Ans.

lat. iu,

1 39' S.

course,

N. 30

nearly

dist.

dep. 5S'4 miles E.

32' E. or

N.N.E. } E.

115 miles.

b 2
DigiiizM By

Google

TAIlALLiL 8A1LIKG.

CHAPTER

III.

PiaiLtBI SAIXIHG. MID-LATITUDE SAILING.


Parallel Sailing.

Wheh

a ship

due east or due west, her track

sails

on

a parallel of latitude, and the case


her distance run

is one of parallel sailing


then the same as her departure, her

is

.-

and her difference of longi-

difference of latitude is nothing,

tude

determined upon the following principles.

is

In the annexed

figure, let

then

'

jX""""""-.

31 represent the equator,

Q,

B I) A any parallel of latitude

and

I will be the radius of the

equator, and e

the radius of the

Let B D be the distance


this parallel, then the
made will be
measured by the are I Q of the
equator and since similar arcs are
parallel.

-^L~-^

difference of longitude

on

sailed

to each other as the radii of the

which they belong, we have the proportion,

circles to

But

I,

that

B
is

is,

diet.

BD

diff.

long. I Q.

the cosine of the latitude I


c

is

X
,",

cos lat.

cos

djff.

to the radius

I times the trigonometrical cosine of

the latitude, so that the above proportion

lat.

long.

I
:

dist.

dist.

is

diff.

diff.

long.

long.

(1)

dist. sailed
:

cos latitude

d between any two meridians be measured


whose latitude is f, and the distance d' between

If the distance

on a

parallel

OigiiizM by

Google

PARALLEL SAILING.

53

the same meridians be measured on another parallel whose


latitude is

I',

then, calling the difference of longitude of the

two meridians L, we have from


cos
COS

.. COS

that

on

COS

::

(1),

by

alternation

I'

d' ::

did'

the intervals between any two meridians, measured

is,

different parallels, are as the cosines of the latitudes of

so that if we know the length of a degree


on the equator, or on any given parallel, we may thus readily
find the length of a degree on any other given parallel.

those parallels

The proportion

(1) or the equation (2) suffices for the solu-

tion of every example' in parallel sailing

plane sailing,

and, just as in

we may embody the necessary


Thus,

right-angled triangle.

let

particulars in a

the base represent the

distance sailed, the hypotenuse the difference of longitude,

in linear measure, and the angle between the two the

tude of the parallel


hyp.

then, by right-angled triangles

bM8

that

is,

long.

diff.

cos base angle

which

table

therefore, solve

problem in plane
in order to this,

= g^igg,
cos latitude

tho equation (2).

is

We may,
like a

any problem in

sailing,

parallel sailing

by inspection of the traverse

we have ouly

to regard the latitude

of the parallel as course, and the distance sailed on


Int.;

will

the corresponding distance,

be

lati-

diff.

long.

in

the

it

traverse

as

diff".

table,

The perpendicular of our right-angled

triangle has no significance;

it

serves merely to connect

the other parts 'together.


.Note.

If logarithms be used in working any example

parallel sailing, then,

in

on account of the change in the radius

of the table, the 1 in the proportion (1) must be changed

Oigiiized by

Google

PABALLBIi BAILING.

54,

into 10 10, this being the numerical value of the logarithmic

The proportion may be written thus

radius.

cos lat.

where radius

radius

dist.

diff.

so that,
log

diff.

long,

1 for the table of natural sines and cosines,

= 10 for the table of log


by logarithms, we should have

and log radius

long.

10

+ log dist.

We think, however, that

and cosines

sines

log cos

lat.

(4).

in general logarithms should be

dispensed with, whenever the work by natural sines or


cosines requires only one reference to the table.

Examples in Parallel

1.

due

~W".

diff.

ship in

dist

Sailing.

49 32' N., and long. 10

lat.

118 miles

long.

W W.,

sails

required the longitude arrived at ?

-r cos

cos 49 32'

lat.

-0,4,90

118

182

miles.

649
Long,
Diff. long.

10 16'

left

182 milea

Inspection.

fliT"

2' "W.

13" 18'

Long, in

Sy

course,

W.

019

W.

"l2

Taking the latitude

and 118 as

(or rather 49) as a

the corresponding distance in


but if we take 50 lat. as a course,

diff. lat.,

the traverse table ia 180


and the same 118 as diff. lat. the corresponding distance in
the table will be 184; half the sum of these, namely, 182, is
;

therefore about the true

diff.

long.

2. A ship in lat. 36 58' N., and long. 20 25' W., is


bound to St. Mary's, one of the Western Ialanda, in the
same latitude, and in long. 25 13' W. What distance must

she run to arrive at her destination


(list

cos lat.

x diff.

4 48'

long, reversed'

2025'W.

W.

Diff. long.

cos 36 58' = -799

long.
diff.

Long, of ship

Long, of St. Mary's 25 13'

882

1598
639
288 miles.
Dist.

_J^_
230

miles.

OigiiizM 0/

Google

PABiLLMi BAILINR.

By

Inspection.

Taking

!5

37 as a course, and 288 as a

distance, the corresponding

diff. lat.

in the traverse table

is

230, the distance required.

Prom two

3.

on the

parallel

both in

ports,

apart, measured

they arrive at

on the

lat.

lat.

parallel,

44 Stf N.

32 20'

N".,

two ships

and 256 miles

sail

How many

due N.,

till

miles measured

reached are they apart P

is to be worked by the proportion or


and as there are two trigonometrical quan-

This example

formula

(3),

concerned, namely, cos

tities

I,

and cos

I',

we

use

shall

logarithms.

As

cos
cos

44 30'

.id,

256

2161

Hence, measured on the


216 miles apart.

The work

-0732

32 20' arith. comp.

I,
I',

.... 9-8532
.... 2-4082
.... 2-3346

parallel arrived at, the ahips are

of this example, -without logarithms, as indi-

cated by the formula

d'

= d cos

I'

~- cos

figures than that above, but probably not

learner is

recommended

I,

occupies

more time.

more
The
an

to solve it in this latter way, as

additional exercise.
4.

due

A
W.

ship in

lat.

286 miles

ship in

lat.

due E. 125 miles

57 20' N., and long. 1" 47' "W.,

sails

required the longitude in P


Aiis. long, in 2 6'

G.

sails

required the longitude arrived at P


Ans. long, in 3 40' E.

5.

53 86' N., and long. 10 18' E.,

A ship in lat. 32 N.

is

E.

bound to a port in the same

but lying 6 24' of longitude to the E.


what
distance has she to run ?
Ans. dist. 3256 miles.

latitude,

7.

of

On

diff.

a certain parallel 384 miles answers to 500 miles

long.

required the latitude of the parallel ?


Aiib. lat. 39 49'.

8.

A ship from

long. 81 3C' "W. sails

W.

310

miles,

and

HID- LATITUDE SAILING.


then finds by observation that her longitude

what

If a ship sail due E. 126 miles from the

9.

N.

in Lapland, and then due

is

how

far

must she

sail

till

she arrives at

Ans.

In what latitude

times the distance she

Worth Cape
73 26'

lat.

due "W. to reach the meridian of

the North Cape ?


10.

91 50' "W.

the latitude of the parallel on which she has


Ans. lat. 59 41'.

is

sailed ?

will

a ship's

sails

on the

111-3 miles.

lat.

be three

long,

parallel

Ans.

latitude ?

dist.
diif.

having that

70 32' nearly.

Mid-Latitude Sailing.

"We have aeon

in the preceding article

how the

difference

may be determined when


latitude we are now to consider

of longitude which a ship makes,

she

sails

on a

parallel of

the more general problem, namely, to find the difference of

upon an

longitude

made when the

For the

solution of this problem,

ship sails

oblique course.

without astronomical

two distinct methods the


one to be here explained, called

observations, Navigation offers

middle latitude sailing;

and

the other, to be discussed in

next

chapter,

called

Merca-

tor's sailing.

M id- latitude

a
combination of plane sailing

and
\

sailing

parallel sailing

is

pro-

it

coeds on the supposition that

what in plane
the

sailing

is

called

departure, namely,

Hr5

+ IC+KD + LE+ MP,


made by
the

a ship, in sailing on

oblique

rhumb

equal to the distance

the meridians of

of latitude between

A F,
T

S,

is

of

and F, measured on the middle parallel


and 3?, or between A and O.

57

MID-i-ATITDDB SAILING.

Assuming then

TS

that

by a ship in sailing from

equal to the departure

is

A to

made

F, the rule for finding the

between

difference of longitude

and F, may be deduced

as follows

It has
latitude

been seen in plane sailing that the difference of

A O,

may

course

triangle

Now

A F, and

the distance run

all

AB

A of

the angle

the

be correctly represented in a right-angled

C, as in the margin.

the side of the triangle marked departure

is,

in the

same as the
mid-latitude distance between the meridians sailed from and arrived at, so
present hypothesis^ the

that the difference of longitude

by the ship
sailed

is

made

the same as

the distance

latitude parallel.

if it had
on the mid-

CB

"We have now there-

fore a ease of parallel sailing, the line

representing the distance

as in that sailing,

if

so that,

we make C B

the

'

^
'~

base of a right-angled triangle, and the


angle at the base the latitude of the
parallel, that is the mid-latitude, it is

plain that the hypotenuse

BD,

will bo the difference of

longitude.

We

thus have two connected right-angled triangles ; one,


the lower in tbe above diagram, constructed conformably
to the principles of plane sailing, the upper agreeably to
'

tho principles of parallel sailing; and what


in the lower triangle,

is

superfluous except

for

is

departure

regarded as distance on the mid-

latitude parallel in the upper.

the

The perpendicular C D

purpose

completing

of

is

the

triangle.

Now,

by right-angled

triangles,

we have from the upper

triangle,

departure
D^^g co-mid-lat.
,

D 3

Digitized By

Google

MID-LATITUDE SAILING.

58

But from the principles of plane


triangle, we have
departure

x rin course

[list

from the lower

sailing, op

x tan eonrae.

lt.

(lift*.

Consequently,

Diff long

_
~~

departure
cos mid-lat!

And these

cos mid.

lat.

rad. (1)

sin course

tan course

3.

cos raid.

lat.

lat,

x tan course.

cos mid-lat.

embody the whole theory

of parallel

stated as proportions thus

2. cos mid. lat.

1.

cliff,

cos mid-lat.

expressions

They may be

sailing.

course

diaj^*

dep.

: ([iff.

dint.

long.

diff. lat.

If logarithms be used, then in the

first

long.

diff.
:

diff.

long.

proportion log rad.

=10.
Examples in Mid-Latitude
1.

sails

A ship from latitude 52

6'

TT.W. by "W. 224 miles

armed

Sailing.

Singh Courses.

N., and longitude 35


required the

lat.

6' "W".

and long,

at P

For
Diff. lat.

For

ike diff. lat,

=dist, x oos course,

cos 5 points=

the mid-lot.

60)124

-55B6

52

N=diff.

i'

6" If

lat

E4 10- N-lat,

222
Diff. lat.

C3

124-45 miles.

For

As

the

2)106'

68

5 points

dist.

long.

8',

.224
.

in.

S'=jroro, or mid-lat.

(By logarithms).

cos mid-lat.
sin course

diff.

lat.
left.

of latitudes.

long.

diff".

16'=sum

310 4

arith.

comp, 0-2219
9'9198

....
....
....

2 3502
2 4919

Or, using proportion 3 instead of 2, the work will be,

Oigiiized 0/

Google


MID-LATITUDE SilLlSB.

As

tan course

mid-lat. 53

coa

diff.

gives 40 16 r

W.

By

310-4

l, as diff.

lat.,

155, the double of which

is

The above method

Note.

of longitude

is

2-4018

W.,
N.
and distance 224,
124 4.
53 as course, and the half of

For course 5

Impaction.

namely 03

2 0948

for the long, in, the lat. hi being 54 10'

Again, for the mid.-lat.

diBt.

long, is 5 10' "W., which, added to 35 6'

points,

the traverse table gives dep. 1S6'2, and

186'2,

10-1751

124-4

long.

diff.

aritb.comp. 0-2210

5 points

diff. Int.

Hence the

8',

not

diff. lat.

the traverse table gives for


310, the

diff.

long.

of determining the difference

strictly accurate, since

the departure

i8

not exactly equal to the mid-latitude distance between the

meridian

left

and the meridian reached. For a single day's


is of no practical consequence, and
more especially if the angle of the course

run, however, the error


in

low

latitudes,

be large, that is, if the track of the ship be nearly due east
or due west, the method may be depended upon, even for
several days' run.
But by applying to the mid-latitude the
correction given in the table*, the method may always
be employed with safety the table is used thus
Take out
the correction under the given difference of latitude, and
:

against the given mid-latitude.

mid-latitude
it,

call

the

Add

sum the true

this correction to the

mid-latitude,

and employ

instead of the uncorrected mid-latitude in the calcu-

lation.

If the difference of latitude be not more than 1, no correction will be necessary

The

principle

when it
But

some such

Since

* See

table is necessary

diff.

long.

is 2,

on which the table

explained at a future page.

= dep.

" Navigation

it is

is

and under

Tables

3,

add 1'.

constructed will be

easy to show here that

thus

-H cosmid-lat. =dep. x

sec mid-Iat.

the table for correcting mid-latitnde.

60
it

SlID-LATITUDE SAILIBC1.
follows that

parture,

it'

thsifc

is,

there be any error in estimating the de-

regarding the mid-latitude distance

in

between the meridians as equal to


still

greater in the resulting

diff.

there will be an error

it,

long, because secant always

exceeds unity, so that in high latitudes the error in longi-

tude may be

seriously wide of the truth.

In the next example, where the diff. lafc. is large, we


work, for the diff, long, with the true mid -latitude.
2.

S.

33

ship from

W.

8'

shall

51 18' N., long. 9 50' W.,

lat.

a distance of 1024 miles: required the

sails

lat.

and

long, in ?

For

jr. tat.

tke true miil-hi'.

G,0)85.7

tlist.

37*

1'

lat.

44

8:17-4 miles.

Correction

= lat. in.
= sum lata.

N.

2)SS"10'
diff.

mid-la t.

S'i
27'

= true

44" Se'J

For

As

tJte diff.

sin course 33

dist.
diff.

01476
9'7377

8'

1024

8-0103

78G-3=13 3

long.

mid-lat.

long.

cos truemid-lafc. 44 36'*, arith. comp.

G'

2-895G

work the correction had been omitted, the diff.


would have been 780-1, which is six miles in error.

If in this
long,

9 50"
3.

A ship

N/W- by

from

+ 13 C ~ 22 56' W.
lat.

"W. 229 miles

52
:

6'

long. in.

N., and long. 35

required the

lat.

6' W., sails


and long, arrived

at?
Ana.

lafc.

54 13' N.

long.

40 c

23' TV.

61

MlD-JiATlTTTUE BAILlXli.

A Bbip from

4.

tween

49 57' N., long. 5 11' W., sails beshe arrives in lat. 38 27'N. t when she

lat.

and W.,

S.

till

what was the


course steered, the distance run, and the long, arrived at?
Ana. course, S. 32 32' W. dist. 818 miles long,

finds she lias

made 440 miles of departure

15 28'

in,

5.

N. 33

ship

from

W.
37 N. long., 22 56' W., steers

lat.

19' E., till Bhe finds herself in lat. 51 18'

longitude

is

she then in

N.

Ana. 9 45'

A ship from

6.

a place in

for

lat.

what

lat.

37 48' N., long. 25 10> "W\,

50 a

13' N.,

and long. 3 38'

W.

is

W.
bound

required

her course and distance?

Ans. course, N. 51

A ship

7.

from

iat.

7'

E.

dist.,

1187

miles.

38 42'i N., long. 9 8'1 W., sails on


:
required the lat.

a W.S.AV. course, a distance of 700 miles

and

long, arrived at ?
long. 12 33

Ans. lat.
W.
8. A ship from lat. 40 41' N., long. 16 37' "W\, sails
between N. and E. till she arrives at lat. 43 57' N. f and
finds that she has made 248 miles of departure: required
the course, distance, and long, in ?
Ans. course, 51 41' E. dist. 31G miles
37 54' N.

long, in, 11 "W.

Note.

Prom

the principles discussed iu the foregoing

article, it is evident, as

was observed at

p. 50, that

the de-

termination of the direct course and distance from the


balance of the departures on a compound course must involve

some amount
it

is

found

the end of a series of courses,

of error.

If, at

that the

departures cast

departures west, the custom

returned to the meridian

left

is
;

just

balance

the

to conclude that the ship has

but

it is

plain from the prin-

ciples of mid-latitude sailing, that if a ship in

N.

lat. sail

obliquely towards the N.E. quarter, and thee, altering her

mehcatob's bailing.
course, sail towards the N."W\ quarter,

meridian

it is

till she reach the same


between
must exceed that on the
somewhat

plain that the mid-latitude distance

the meridians, on the

first

course,

second, and the correction tnken from the tahle

It follows, therefore, that the result-

increases this excess.

ant of the departures in a traverse will not be the correct


departure for the equivalent single course and distance,

although in a day or two's run, the inaccuracy

may be

of no

practical consequence.

CHAPTER
MSBCATOB'S SAILING.

The

IV.

TBAVEBSJJ8

BY BOTH SAILINGS.

principal object of mid-latitude sailing, as

just seen,

is

we have

to render the results of ordinary plane sailing

available for the purpose of discovering difference of longi-

tude.

The determination of longitude may,

indeed,

be

considered as the master problem of Navigation, and ac-

more than any

other,

of scientific and nautical men.

But

cordingly

it has,

engaged the attention


of all the methods of

excepting those dependent upon


observations that which we are about to

solution hitherto proposed

astronomical
explain

is

the most ingenious and satisfactory.

invented by Gerrard Mercator, a Pleming,


lished a chart, constructed

upon

who

It was
in

first

1556 pub-

peculiar principles, from

which differences of longitude could be deduced.

The mathematical theory

of this construction, however,

and the tables necessary for bringing Meroatob's Sailing


under the dominion of numerical computation, is due to an
Englishman, Edward "Wright, who formed Mb table of Meridional Farts after the manner now to be described.

Let A B, AC, and C B, in the annexed right-angled triangle,


represent the distance run, the difference of latitude and

jiercatob's sailing.
the departure made on any single course, A.

that the departure,

on the

line

B,

is

"We know

not the representation of ani-

surface of the sphere,

but the aggregate of

the minute

all

departures shown in

the diagram

one continuous
Let A bo he one of the elementary triangles in that diagram,
at p. 56, united in
line.

being one of the elementary de-

cb

Ac

and

partures,

the elementary

difference of latitude corresponding.

Then

since

a portion of a

e b is

parallel of latitude, it will be to a

similar portion of the equator or of

the meridian, as the cosine of the latitude of the parallel is


to radius (or 1), as was proved at p. 62 ; and this similar
portion of the equator measures the difference of longitude

between c and

b.

If, therefore,

to

b', till

the elementary distance,

the corresponding departure &

this difference of longitude,

portion,

shall

d V: Ml Ut.
cb-.JV;; Ae

c6

Bnt

we

A b,

he prolonged

V becomes

equal to

have the following pro-

namely

(Boo. 4, VI.)

of

cos lat. of c

.\AC=

it

ch
Ac'

:Ac

Ac-

= Acxseclat.of el

...

(1)

cos lat. of c b

It thus appears that if the proper difference of latitude

c,

be increased to

Ao

so that

A c'=Aex sec lat. of c, the

become increased to o' V, so that


c' J' = diff. long, of a and },
In other words, a ship having
made the. small diff. lat. A c, and the corresponding departure c b, must continue her course till her diff. lat, A c' lias
proper departure,

increased to

departure

from

A to

c'

b.

c'b,

will

A c x sec lat. of c, in
V may be equal to the
Now it

is

order that her increased


diff.

long,

made

in sailing

evident that if all the elementary

OigiiizM By

Google

mebcatob's SAILING.

64

arc prolonged in this manner,

differences of latitude

sum

of

all

to

long,

diff.

made

in sailing from

Consequently, to represent the din7 long, between


,

A and B,
A

the

the corresponding increased elementary depar-

tures will be the whole

the

diff. lat.

A 0 must be prolonged

C becomes equal to the sum of

all

till

the length

the increased elemen-

when the corresponding increased


represent the diff. long, made in sailing

tary differences of latitude,

C B', will
A to B. The business

departure,

from

finding,

theu, is to contrive

from A C, the proper enlargement of it,

means

AC

for

Wright

proceeded as follows

Taking the elementary differences of latitude each equal


to a nautical mile, or one minute of the meridian, commencing at the equator, and calling the enlargements Meridional Parts, he knew, from the relation (1) above,
that

Meridional Parts of l'=sec.

1'.

2'

= sec.

l'+sec.

3'

= sec.

l'+sec.

2'.

2'

+ sec.

4'=sec. l'+sec. 2'4-sec.

&&

3'.

3' -{-see. 4'.

&c.

And from

these equalities he calculated the proper enlargement of the portions of the meridian, increasing minute

by minute, from the equator, by help of the

table of natural

secants, thus:
Lat.

Mer. Parts of

1'

Sum

of nat. secants.

=1-0000000

S[er. Parts.

=1-0000000

= 1-0000000 4-1-0000002 = 2-0000002


= 2-0000002 + r0000004=3 000000(!
= 3-0000000 + 1-0000007=4-0000013
5' = 40000013 4-1-0000011 := 50000024

2'

3'

>

4'

&c.
It

&e.

was by summing up the natural secants in this way


first table of meridional parts was constructed.
If

that the

mehcatob's sailing.

65

we enter such a table with the latitude of A (preceding


we shall find against that latitude the enlarged or
diagram),

meridional latitude

tude of C, we also

in like manner, entering with the

find the

lati-

corresponding meridional latitude

A C the meridional difference


if A and C are on opposite

the difference of the two will be

of latitude, or the sum of the two,


'

sides of the equator.

It

plain that a table of meridional parts, constructed

is

after this method, will be the

more

accurate the

strictly

smaller the elementary portions of the meridian are taken


as, for instance,

by taking tlicm each half it minute in length,

was subsequently
But Dr. Hal ley contrived means of constructing the
in another way, which way involved no inaccuracy at

instead of a whole minute, as indeed

done.
table
all

and the

tables in existing use are all formed in this cor-

(See the Mathematical Tables.}

rect manner.*

Referring

now

to the diagram at p. G3,

we have the two

following proportions for the solution of problems in Mercator's sailing,

1.
2.

As

rail. (I)

Aa proper

And,

namely

diff. Iat.

(A C)

to

mer.

dep.

we

as in former cases,

more convenient
f

tan course

diff. iat.

mer.

think

work examples

diff.

diff. Iat.

it will

long.
diff.

long.

sometimes he

in this sailing without

logarithms than with them.

Examples
1.

A ship

N.~W. by

By

W.

in 'Mcrcator

from

lat.

229 miles

52"
:

Sailing.

:;

G'

thlf=

G' "W.,

sails

lat. and long, in ?


found to be 54 13' N. and
;

* For an account of Dr. Halley's method, and


liropTCSK of

Single Courses.

N., and long. 35"

required her

ex. 3, p. 60, the lat. in is

part of X.Tvipafu'i).

tiic

Navigation and Nautical Astronomy, in

for further details

inquiring student
'

is

referriJ

on the
t

<

(!.,

On-'s Circle of the Sciences."

Digiiizod by

Google

mer gator's
to find the

diff.

BAILI50.

we proceed by Mercator's

long,

sailing a

follows

latin

6*13'N.

52

Lat. left

Long,

Long.

6'W. Mer.

5 17'

Diff. long.

Her. parts 38871

6'N.

left 8fi

W.= 317

Diff. long.

3075]

diff.

= tan course* mer.

diff. 1st.

kCili ta

P0

1 *4988

miles"

40 23' W.

in.

1497
299
Diff. long.

The longitude

is

therefore the

found by mid-latitude

A ship from

2.
S.

33

8'

The

lat. is

sailing.

51 18' N., and long. 9 50' W.,

lat.

"W. 1024 miles

found in ex.

required the
2,

51" 18' N. Mer. parts

3;

37"

2:

N.

Mer.

diff. lat.

lat.

and long, in

page 60, to be 37

Lat. left

Latin

1'

317-28 miles.

Bame as that previously

Diff.

Iong.=
tan 33*

1201

1'

sails
?

N.

tan. course:

diff.

lat

8'= -6527
4021

Long,

0 GO' W.

left

Diff. long.

13"

Long, in

22 56'

6'W.=

W.

Sy

For the course

Inspection.

1305

_2Q
ig.

33,

785-8 miles.

and distance 256,

being one-fourth of the given distance, the traverse table


gives

diff.

diff. lat.

lat.

= 14

= 214-7, four

times which

is

858, therefore

and hence the lat. in is 37 N. The mebetween the two latitudes is 1205. For

18' S.,

ridional difference
one-fifth of this,

namely 241, as

diff. lat.,

and 33 as course,

the traverse table gives, under departure, 156-9, five times

which

is

784, the miles of

This makes the long,


long.
"When the number with
is beyond the limits of
more convenient to divide that

diff.

in 22 58', two minutes too great,

which we enter the traverse table


the table,

it

may he

little

67

meecatob's bailing.

number by 10

in this

nearest to this, under

way 1205

diff.

lat. is

On account

will

The

give 120-5.

120-8, the corresponding

departure being 78"4, ten times which

784 for the

is

diff.

long.

of small quantities being disregarded, the tra-

verse table does not always give results with the

same accu-

racy as computation.

Required, the course and distance between Usbant, in


48 28' N., long. 5 3' W., and St. Michael's, in lat.
37 44' N., long. 25 40* W. ?
3.

lat.

For
ten eourao
Tliff.

the cauric.

= diff.

long

~ mn

8t. Mlctiael

RTif

m.=10"44' Mor.

= diff. 1st.

difi. lat,

loue.=20 3T'=1237 miles.


48 28'
Mer. pts.

Uflliant, lat.

Diff.lBt. 644

1).

'

Otherwise by logaritlimB.
Diff. long.

12S7

tail

54 24'

The

diff.

long,

3-0924

2 -9474
1450

computations,

886

;er. diff. lat.

Inspection.

cos couraa.

244S

8,6)1 237(1 -3962=18* 54 23'

By

-v-

23'= -5,8,2,3)644 (1108 miles.

S334

lat=886

toe angle, in both

ia

about E4 23'4.

and the mer.

diff.

lat.

mer.
being found as above, seek in the traverse table for the
the diff. long, in
lat., in that diff. lat. column having
The page in which these

diff.

the corresponding dep. column.

aud the
are found will give the course: with this course
But
distance.
true diff. 1st. enter the table again for the
solution of
the traverse table is not well adapted for the
approximate
eiarnples of this kind ; it usually gives but
approximation
results, and, as in the present case, the

may

Digitized By

Google

68

irBBCATOB'fl SAILING.

not be very
is

In

close.

example the distance by the table

this

11 miles short of the truth as given by computation.

4 A ship from lat.


required the

lat. in,

51 9' N.

and the

Ans.

A Bhip

5,

sails

lat.

from

d
course N. 33 19' E.,

lat.

8.W. by W. 21G
made ?

sails

diff.

she arrives at

till

required the distance sailed and the long, arrived at

Ans.
her

and the

diff.

lat.

long,

is

134 miles

required the distance sailed,

in ?
at. 41 25' N.
Ans. dist. 1321 m il es
N.E. by E. from lat. 42 3 25' N., and long.
lat. 4G 20" N.: required
.

A Bhip sails

7.

15

6' AY., till

Ans.

A ship

33 19' W.,
long, in ?

dist.

423 miles

long, in 6 54'

W.

from lat. 51 18' N., long. 9 50' W., sails S.


till her departure is 564 miles : required her
Ans. long. 23 2' "W.

Compound Courses ly
In order to
the, end of

she finds herself in

the distance sailed and long, in

8.

1027 miles; long. 9 45' "W\


42 54' N. on the course S.E.E..

dist.

A ship sails from lat.

6,
till

miles

long,

9' N.
diff. long. 4 40' "W.
37 N. long., 22 56' W., on the
lat. 51 18' N.

49

31i,/-Latitude

find the diff. lat.

and Mercator's Sailing.

and the

diff.

long,

made at

a series of courses, or a traverse, we must register

the particulars of each course in a traverse table, as at page

and proceed in one or other of the two following ways :


1. Having found the diff. lat. and dep. made during the

49,

determine from these the direct


and find the diff. loug. duo to this
by either mid-latitude or Mercator's Bailing,

traverse as at page 49,

course and distance,


single course

as in the foregoing articles.

Or: the several entries having been made in the traverse table as before, find the balance of the diff. lafc. columns
2.

only
long,

we shall thus discover the


we proceed thus

latitude in

and

for the diff.

OigiiizM By

Google

::

69

SIEECATOn's SAILING.

From

the latitudes at the beginning and end of each

course find the corresponding mid-latitude, with which and

made during the


by mid-latitude sailing. The

course, deduce the

the departure

traverse becomes

diff.

But

known.

long,

long, being thus found

whole

for each distinct course, the

diff.

diff.

long,

due to the

Mercator's sailing be em-

if

ployed instead of the mid-latitude method, then there will

be no occasion for the insertion of any departures in the


table.

The following example, worked both by mid-latitude arid


will sufficiently show how the tabulated

Hercator's sailing,

quantities are to be arranged


1.

A ship

from

lat.

G0

9'

N., aud long. 1

7'

W-,

sailed

the following courses and distances, namely


1st.

S.B. by N. 69 miles.

2nd.

N.N.E. 4S

W. 7S

miles.

4tb. N.E. 108 miles'.

5th. S.E. by E.

Kequired the

N. by W.

3rd.

miles.

direct course

uO miles.

and distance, and the

lat.

and

long, in P

Tsaysbm
CouraoSL

Table.

DUE. lat,

iJiat.

N.
N. E. by N.
N. N. fi.

N. by W. J W.
N. E.
S. E. by E.

01)

57' 4

48

U-i

Direct course N. 3 4'E.


Distiiii'o

By

272

miles.

the traverse table the

Depmturo.

S.

E.
38-3
18-4

27 -8

41-6

74-6
76-4

108
50

W.
22-6

_"_

1747

27'8

22-6

225

152-1

diff.

lat.

225, and dep. 152-1,

gives for the course 31, and for the distance, 272 miles.

The computation

is as follows

OigilizM 0/

Google

hehcatoe's bailing.

70

Far
Lwi course

= dep-j~

diff.

kt.

ian34 5

22,5)U21('WfiO

dial.

4'

the distance.

= di

lat.

cob course.

5,2,8,4)225 (272 miles.

1350

1657

6u mid-latitude tailing.
Latitude left
Diff. tot.

3 45'

Sum

long.= dep. -rco3 mid-Iat.

Diff.

225 m.

Latitude in

cos 62

N.

6354'N.

4'= -46,8,4)1621 (325


14052

62

i sum, or inid-lat.

Correction

l'i

2'i

sailing is

For

the di.

long, by

diff.

c.

= 676

325

miles,

diff.

distance aa given above


of the departures

made by

sailing

to that arrived at.

case

is

Dif lung.

long,

325'2 miles.

made during

the tra-

on the supposition, however, that the

traverse is correctly resolved into

be

diff. tot. >

diff. lat.

It thus appears that the


is

by mid-Iat.

suilimj.

63 54' N.

Meridional

verse

long,

325 miles E.

Mercatw's

Latitude left 60"

Latitude in

miles.

1153

of lats.

the single course

and

in other words, that the balance

the aauie aa the departure that would

on a singlo course from the place left


is not the

But, as already shown, such

and consequently the

diff.

long, just determined,

must

M Eli CAT Oil' a


To

be affected with error.

71

BAILING,

avoid

error

tiiiB

it is

necesdary

proceed according to the second of the two methods

to

described above, that

is,

in

one or other of the following

ways

WoaK

oi'

ni

Pkkceding Example on mobe Cobbbct

i]

Pbinciples.
1st.

2nd.

Note.

The

By Mid-Latitude Sailing.
By Mercator's Sailing.

object of each of the following solutions

is

determine the difference of longitude correctly ; the difference of latitude is always accurately ascertained as above
to

but to give a complete form to the work, the longitude


table id annexed to the traverse table for finding the din",
lat.

and the

several departures.
1,

Courses,

Solution

B.

N. . by N.
N. N. E.

S b yw._ 4 w.

69

hij

mid-luiiladc

E.

S.E.byE.

60

W.

Mid.

Difl".

lun.

W.

60* B'

57 i

48

18'4

IDS

sailiii'j.

Departure.

Slat.

Tfl'4

410

'

liV'Sd
88* B'

"

61" S8-

v2-r

.'>.

liW

;,.v

BT ST'

L'.y

48*48- 174

B4'l'

127"

83' 68-

lE^li'

49

if'

Diff. lat.

Diff.

long.

B36

As the diff. lat. made on any single course does


much exceed a degree, there is no need for any correction

Note.
not

The two columns for dift'. long, are filled


traverse table by this rule
Take the inida course, and seek the corresponding departure in a
diff. left, column, against which, in the dist. column will be
found the number of miles in the diff. long. In the table
the course 60" and diff. lat. 38'3 gives dist. 77, and the course
61, with same diff. lat., gives dist. 79; we therefore take
78 for the course 60, as 60 37' nearly is. In like manner
of the mid-lats.

up from the

lat.

Oigiiized 0/

Google

mercator's sailing.

72
61

28', is

02

27', aa

tlie mean between 61 and 6!


mean between 62 and 63, while 63

regarded as
the

regarded aa 64, so also

is

2. Solution

64

and

43', id

7'.

by Ma-colof's tailing.

N. by W. J W.
X. E.
8. F..

by

T.,
I

Biff. long.

The two columns


from the traverse
given course,
lat.

for

diff.

number of miles
in, we have

By

table.

we seek

in the

60

left

diff,

loDg.

Latitude in

By
is

the

first

63 54' N.

mode

To

diff. lat.

in a diff.

we

find the lat.

find the

and long,

longitude loft 1 7' W.


Diff. long.33Gm.5 36' E.

1'

'

is

entering the table with the

in the dep. column,

it,

N.
Diff.lat.225m. 3 45 N.
Latitude

long, are, as before, supplied

for the given mer.

column, and against

Longitude

in 4 29' E.

of computation, in which the traverse

reduced to a single course and distance, the longitude in


4 18', which is IT in error.

In order now to
we have

find the

more

correct single course

and

distance,

I'ar Hit course.


t

mi course = diff. long-r-mer,

kit. loft 60" 0'


Lat. in 63 54'

Mer.

diff.

lat

Mor.

Per
diff. lat.

jiarta

4545

Dist.

Ilie

distance.

diff. Int. -L-

cos course,

ooa 3*' 6'=-8,VJ,8l225 (275

5026

1040

481

CIO

mk

73

mebcator's sailing.

Hence

the correct course

N.

is

3-1 56' E-,

and the

dis-

tance 275 miles.

Note.

It

may be instructive to the

learner to notice here

that, agreeably to the general practice, in

lumn headed
below

-5 in

we have

"lats.",

the

diff.

lat.

decimal above 5 by unit


of (S3 54', as

and in consequence of

it

aim

table,

at

little
it.

Now, although a

ought to do.

attention to minute accuracy

to

this,

the

the former column conies out 63 53' instead

is

in operations of this kind, yet

with very

forming the co-

disregarded every decimal

columns, and have replaced every

final latitude in

extra trouble,

By

fastidious

seldom absolutely necessary

when precision can be attained


always better and safer

it is

number in the
much more accu-

noticing the consecutive

the influence of the decimal

may

be

by the rough general principle of


by unit. Thus, it is
plain that each decimal in the column headed N. is very
rately estimated than

rejecting

nearly

5,

it

altogether, or replacing it

or

to unit: the

and that in the column S. is very nearly equal


correct "Longitude Table" above will

more

therefore be as follows

Lokgjtcde Table.

Me
60 8'
61 8'.}
61 51'
63" 5'i
64 22'
63 54'

Longitude
Diff.

Long

Lougitude

Her. V.

I..

50 i>o'

lefl

3-5

Diff.

E.

4546

4863
4756
4917
5090

1*
5

113
93
161
173
61

V w.
38'i B.

Long.

W.

78 '9
3S'6

48 '8
173
90-8

387-3
48-8

33S Ts"

We

thus see that from not taking a more correct estimate


of the decimals in the former work, the result was about
2 miles of longitude too little. If the several courses and

distances in the above traverse bo correct, the longitude

HEBCATOa's

now deduced

SAILIKC).

cannot err from the truth by more than a smalt

an accurate table of Mer.

fraction of a minute, provided

Parts has been used.


2.

A ship

from

lat.

66 14' N., and long. 3 12' E., has

sailed the following courses,

namely

N.N.E. E. 46 miles.
2nd. N.E. i E. .28

N. --"W.
52
N.E. by E. i E. 57
24 miles:

3rd.

1st.

required the latitude and longitude in


Ans.,
3.

ship from

lat.

lat.

N.E. by N.

1st.

E.

-J

N.N.W.

3rd.

N.W.by W.

68 24'

namely

56 miles.
38
46

Ans.,

5th. 8.
6th.
in,

by

30 miles.
20

W.

N.E. by N. 60

and the correct single

lat.

in,

course,

Having now

long. 7 53' E.

4th. S.S.E.

required the latitude and longitude


course and distance

N.

38 14' N., and long. 25 56' W., has

sailed the following courses,

2nd.

miles.

4th.

,",

5th. E.S.E.

40 2'4 N. ; long, in, 25 29" W.


miles.
N. 10 57' E. ; dist.

sufficiently discussed ITerc.itor's sailing,

we

are in a condition to explain the principles on which the

50

table for correcting the mid-latitude referred to at page


is

constructed.
Let

represent

I
/'

in

Then

"tin;

iK-i.per diiiureiiee

of latitude.

the meridional difference of latitude.


the

lat. in

which the

for tan course,

sailings,

to

meridians

dist. betvceii tho

the difference of lunyiuule uf

by

tli'ist

depart

live.

meridians.

and Hercator'n

plane, mid-lat,,

we have

departure

Lx

cor,

m=
'

cos

Hence, by dividing the proper diff. lat. I, by the meridional


din?, lat. V, we get cos m, and thence m, the latitude of the
parallel, the portion of which intercepted between the two

Digitized By

Google

3lECATOK'8 SAILING.
meridians

exactly equal to tho departure, the length of

ia

this intercepted portion

Lx
It

is

being

m = departure

cos

(see p. 57).

m and tho latitude of the middle

the difference between

parallel that is inserted in the table referred to at p. 59.

Note.

Before concluding the present chapter,

as well to notice that,


given,

when

and from knowing

in any example the

two of tho

also

departure, or distance,

lat.,

cliff,

lata,

from and

in,

it ia

it

difl'.

may be
long, is

quantities, course,

required to find the

such example cannot be worked by Mer-

cator's sailing.
The proper diff. lat. and the mer. diff. lat.
may be found, but not the lata, themselves the problem
must be solved by mid-latitude sailing, as in the following
:

instance.

A ship sails in the N.~W.

Mx.

departure

is

quired the

lafc.

By plane
3 28',

135

and her

miles,

from and in

sailing the

quarter 24S miles,


diff.

long.

hor

till

310 miles

re-

diff. lat.

and from the equation

found to be 208 mile3=

is

last

given above

cosm = ^B:=^ = -4355 = coa 64 11'.

Hence, the

310

mid-latifcudo corrected

is

i=64

11'

this is

greater than the mid-latitude unmodified, by the correction


in the table,

fore

namely by

the mid-lat.

ia

3'

G4

m id.

there-

8',

and

lat.

64

half diff. lat.

proceeding as in tbe margin, we

lat.

from

from

lat.

in

readily

and

determine the

From

in.

lata,

8'

>W
62 24'N.
65 52'N.

neglecting the tabular correction the

lati-

tudes in other books are made too great.

Again.

If,

with tbe

diff.

long.,

one

lat,

courae, dist., or dep., mid-latitude Bailing


to tbe finding of tbe other

Ex.
diff.

lat.

A ship from lat. 34 29'

long,

is

680 miles

be given, and the


not applicable

is

For example
TS., sails S.

required her

lat.

41 "W.,

till

her

in ?
B 2

Digitized By

Google

CUEEENT BAILING.
Mer.

diff. lat.

diff.

Jler.

Lat.

Hence

in,

the latitude

course = 680-~8693 =782

long.-f- tan

Lat. from, 34 29'

23

N.

CHAPTEK

1425

V.

TA KING

current act upon a ship, her rate of sailing

is

neces-

and in general both her rate and the


in which she would otherwise move through the

sarily affected

direction

_782

CUEEEKT SAILINGPLXING TO WIND W A ED


DEPAMOBE8.

]ia

2207

Mer. Paris

diff. lat.

N. Mer. Parts
arrived at is 23 G' N.
6'

by

it,

water.

If the ship

with or directly against the cur-

sail directly

rent, her rate oflly will

be

affected

but

she

if

athwart

sail

the current, both her rate of sailing and her coarse

subjected to

The

its

become

influence.

from the compass (the usual


marks the direction of the ship'3

course, as determined

corrections being made),

bead, and in this direction the ship

but the current carries her a certain other


aud distance in the same time, her actual motion

in a certaiu time
direction

moves a certain distance

being compounded of the two.

It

is

thua the

sameas

far

as position is concerned, disregarding the time of arriving


at

it

as

if

the ship had sailed the two distinct courses and

distances in succession, so that current sailing resolves itself


into a simple case of traverse sailing, as soon as the direc-

tion

The

and velocity of the current are ascertained.

di-

rection of the current, or the point of the compass towards

which

it

flows, is -called the set of the current

city, or rate, is called

and

its velo-

the drift.

The usual way of ascertaining tho


current unexpectedly met with at sea,
short distance from the ship

set
is

and

drift of

to take a boat a

and, in order to keep

it

from

OigiiizM By

Google

CURBE3TT SAILING.

77

being carried by the current, to let down, to the depth of


about one hundred fathoms, a heavy iron pot, or some other

a rope fastened to the stem of


the boat, which by this means is kept steady. The log is
then hove into the current, the direction in which it is
sufficient weight, attached to

carried, or the set of the current, is

boat compass

and the rate

hourly drift of the current,

is

determined by aid of a

at which it is carried, or the

given by the number of knots

of the log-line run out in half a minute.

Examples in Current
1.

in

A ship

sails

N.W.

Sailing.

a distance, by the log, of GO mile.",

a current that Bets S.S.W., drifting 25 miles in the

same time
This

is

required the course and distance

made good

the same as the following question, namely

A ship sails the following courses and distances


1st.

what

is

N.W. 60

2nd. S.8.W. 25 miles:

miles.

the direct course and distance

Couwen.

niat.

N.

N.W.

Diff.lat.

W.

S.

42-4

00
25

a aw.

Ci

23-1

19-3

Dep.

52

For this diff. lat. and dep. the course by plain sailing is
N. G9 38' W., and the distance is 56 miles.
1. A ship sailing at the rate of 7 knots an hour, is bound
to a port bearing S. 52 "W., but the passage is in a current

which sets

S.S.E.,

two miles an hour

it is

required to shape

the course ?

Here one only

of the two courses of the traverse

together with the resulting direct course

component course

by the traverse

we

shall give

two

is

given,

to find the other

solutions, the second

table.

OigilizM By

Google

CUEEBHT BAIUHG.

78

B A

Let

be in the direction of the port, and B the


place of the ship, B D in the direc.
tion of the current

B C=

A=B D =

Then

there

are

the side

7,

and

2,

B C

= 7.

ABC,

the triangle

given the Bido

and

2,

in the required direction


in

the angle

CAB=D B A=2230'+52 = 74
find the angle ABO.
In

3<y, to

we have

order to this,

a 7i 30'

9636

(p. 26),

ABO.
ABC.

in
i

.'.

ABC =
ABS =

15 6B'
52

ieCBS=67
triangle

ABC,

let

C,

59'

C,

and

the angle C.A B, measure the


same as in the above diagram and
;

let

Cm

then,

be perpendicular to

by right-angled

AB

triangles,

A C sinM. = C m, and B C sin B = C m

(1).

74 30' as a
Hence, entering the traverse table with A
2 as a distance, we get the dep. C i=l*9.
course, and A C

Again, with the dep.


7,

we get the

gives 68 for

The
given

course

CBS,

B=

m = 1"9,

is

C=
= 52,

and the distance B


added to A B S

10, whicli

the required course.

learner will observe

that the

solution previously

at once derived from the equations (1)

for

from

these

AC
BC
si

BC

N. E. by N, 18 miles in three hours, in


by S. two miles an hour required the
course and distance made good ?
Ans. course 1* points, or N. by E. J-E. dist. 14 miles.
3.

Bhip runs

a current setting ~W.

PLYING TO WIHDWABD.

79

24 hours amis the following couraea in a


S. lh miles an hour, namely
3rd. S. by E. 47 miles.
miles.
Current, S.E. by S. 36 miles,
made good ?
Ans. course, S. 11 50' AV., diat. 117 miles.
5. The port bears due E., the current aets S."W. by S.
three knots an hour, the rate of sailing is 4 knots an hour
4.

ship in

current setting S.E. by

40

lBt. S.AV.

2nd.

miles.

W.S.W. 27

required the direct course and distance

required the course to bo steered

Ans. course N. 51 E.
6.
S.

A ship sailing

by E. 42

made 55

miles,

miles, of

in a current has by her reckoning run


and by observations is found to have

diff. lat.

and 18 miles of dep.

required

the set'and drift of the current ?


Ans. set S. 62 12' W,, whole drift 30 miles.

Plying

it

to

Windward.

"When a ship bound to a port has a foul wind, she can reach
is, by crossing the wind on two or
courses, making a zigzag instead of a direct track.

only by tacking, that

more
This

plying to windward.

is called

Having

sailed

a certain distance as near to the wind na

she can, the ship tacks about, recrosaing the current of air
at the

same angle; and thus she crosses and recrosses

always at the same angle,

till

she arrives at her port.

Starboard signifies the righthand


lefthand side.

"When a ship

plies

aide,

and larboard the

with the wind on the

right the starboard tacks are aboard, and

when the wind

When

is

on the

loft

sails as

near as sho can to the point from which the wind

blows, she

the larboard, tacks are aboard.

is

said to be close hauled.

The

a ship

following exam-

ple will sufficiently illustrate the calculations usually neces-

sary in plying to windward, a subject in which the learner


some knowledge of oblique-angled trianwill perceive that

gles is requisite.

Digitized 0/

Google

PLY IK G 10 WINDWABD.

80

Examples
1.

Plying

in

my

Being within sight of

Windward.

to

~N. by E. 5 E.
up from the N.E.

port bearing

distant 18 miles, a fresh gale sprung

with

my

larboard tacks aboard, and close hauled within six

how

points of the wind,

about

and what

second board

be

will

must I run before tacking


distance from the port on the

far

my

In the annexed diagram

of

distance

the

on

CB

and

the place of the ship,

is

that

C the

first

board,

port,

the

that on the second.

The direction of the wind


marked by 10 A, to' C.

As

the ship

must be=6

w A C
also 6,
,\

is

is

and

v>

if 10

= 6 points, C B will he parallel to Aw'.

6 points,

B C A is

w AB

we have

points,

2\

4.

10'

Again,

0 A B is 8J

given the side

is 10,

BAN

is

m
m'

As

and since / C B
l, and icANis

is

4,

ABC,

Hence, in the triangle

AB

is

within 6

sails

points of the wind, the arc

bo made also

and the angles A and C


A C and C B.

18,

equal to 8 J-and 4 points respectively, to find


sin

points

sin 4 points

ain 8 points

sin

SJ points
-1505

Comp.

sin 4 pts. (C) Aritk.

::AB=1S

... 9-9979
... .1-2553

:BC=25'23

sin

81

pts. (A)

.1-4037

sin 7.} points

(B)

18

18

sin

3J

pts.

::AB =
:

AC

BO,

A C.

Oomp.

-1605

...

9'8024

sin i pts. (C) Arith.

(B)

1S

16-15

.1-2553
.

1-20S2

Hence, the ship must sail 10 miles on the first tack, and
then 251 miles on the seeoud, to reach her port. The course
on the second, or starboard tack, is G points
4 points

2 points, or
2.

N.N.W.

If a ship can

lie

within 6 points of the wind on the

larboard tack, and within 5| points on the starboard tack,

OigiNzM By

Google

FIXING TO WINDWABD.

81

required her course and distance on each

port lying S. by.E. 22 miles, the wind

being at

Let

S.W

A be the place of the ship, and B

that of the port, and let the

first

course

A C be on the starboard tack, the direction if


arc

10

A being that of the wind, and the


m = 51

If the arc

points.

be made equal to 6 points, C

Am',

to

tn'

parallel

be the other course, or

will

that on the larboard tack.

w A C = Ej
BAS =

.-.

Also

1,

i'CA

101,

wAS

and

Hence, in the triangle

= 22,

and the angles

o ii pta. (C) Arith.

ini

AB

22

I' 3424

'445G

The course C
by E.

I E.,

AS

2366

279

miles.

equal to

1.

side

AB

point and 41

Comp.

Bin 11 (or G) pts. (B)


:

AB =

22

-1118

9 -8198

1-3424

It is obvious, that

the hearing of the port

And,

AC=

i a

ain 44 pts. (C) Arith.

when

to reach her port on two tacks, she

other tack.

BO A =

we have given the

on the starboard tack is 1$ points, or


the course on the larboard tack,
N, is 6 points or W.NVW".

miles

being equal to the angle m'

till

Ali

8-8913

....

BC=2-79

S.

WOI

C,

A C and 0 B,

Comp. -1118

....

pt. (A)

AB

A and C

points respectively, to find

is

a ship close hauled

must

steer

is

on one tack

the same as the course on the

as the foregoing illustrations sufficiently

show, when the distance

A B and the

bearing of the port are

known, we may always work by the following rule


As the
sine of the angle between the two courses is to the sine of
:

the angle between the given distance and either course, so


is

that distance to the distance sailed on the other course.


3.

to

A ship is bound to a port 80 miles

windward,

which

is

distant,

and

directly

N.E. by N.1E., and proposes to

PLYING TO WIN DIVA ED.


reach her port at two boards, each within 6 points of the
wind, and to lead with the starboard tack: required her
course and distance on each tack

N.N.W.

Ans. starboard tack,

larboard tack, E.S.E. 1 E.,

W., 104-5 miles

1045

miles.

4. "Wishing to reach a point bearing N.N.W., 15 miles,


but the wind being at W. by N. I was obliged to ply to
windward the ship, close hauled, could make way within
(

6 pointB of the wind : required the course and distance on


each tack

5.

Ans. larboard tack, N. by W., 17'65 miles;


starboard tack, S.W. by S., 4'138 miles.
The port bears N. by E. i E., 18 miles
the wind
;

blows from N.E., the ship after runnmg 48 miles on the


larboard tack within 6 points of the wind, tacks about:
required her course and distance to the port on the second
tack P
Ans. course N. 57 35' W., dist. 49-58 miles.

Note.

"Whether

a ship,

when

cloBe hauled, reaches

point at two boards or courses, or, by more frequent tacking,


at

any number of boards, the actual distance sailed


the same.
ship

Thus, suppose,

A reaches the

boards

C,

distance sailed
pose,

point

is

just

first,

the

on two

C B; the
A C + C B.

is

whole
Sup-

secondly, that she tacks at

D, running

then tacking again, that she runs d

parallel to

e parallel to

C B,

A C, and so

she arrives at some point c in C B, and then sails on


her last course, c B. Then, because the opposite Bides of a
E C .*.
E, and e' c
parallelogram are equal d e

on

till

de

+ dc = A-C.

=D

J) d

In like manner Dd + e
e'+e B. Hence, the distance A C

D d + <*e+

A D+
=
^=0 c C B=
+ C B = A D+
,*.

ee'+e c+cB.
1

When the port is directly to windward there maybe some


advantage in working up to it by a succession of- Bhort
courses, as figured above for the wind may change, and
;

TAKING DZFABXUBES.

any change must be for the better, and it is plain that at


whatever intermediate point on the above zigzag path the
ship

may

be, she is nearer her port

by running the same

distance along

than she would be

C,

B.

Taking Departures.

At

commencement of a voyage before the

ship loses all

sight of land, the distance and bearing of some

known head-

the

land, lighthouse, or other object, the last familiar spot likely

to be seen,

is

taken,

and the ship

her departure from that

is

supposed to have taken

place, the direction opposite to the

bearing and the distance being regarded as the

first

course

and distance, and are entered as such on the log-board.


The bearing being taken by the compass, it is customary
for experienced navigators to estimate the distance by the
eye, but the more correct method of taking a departure is
to observe two bearings of the object, measuring by the
log the distance sailed in the interim between the observations, as in the following

Examples

examples

in

Taking Departures.

down the Channel the Eddyatone bore N.W.


and after running W.S.W. IS miles, it bore N. by

1. Sailing

by N.

E. required the course and distance from


the Eddystone to the place of the last
:

A c
-

observation ?

In the

annexed diagram,

the place of the ship at the

its

place at the second,

observed.
is

By

first

and C

represents
observation,

is

the object

the question the angle

NA

_gt-

B C is 1 point,
3 points, and the angle
the course of the ship S A B is (J

also

number of points
triangle ABC, we have

points. Consequently, for the

in the angles A, B, of the

TAKING DEPARTS EES.

84

A = 16 6 3 =

7,

=-=

sin C, i points, Arith.

16

10

Camp.

5,

AB =

18

the course from

1-2553

= 24-97

As

.-.

'1505

9-9916

sin A, 7 points

::

1*3971

to the Eddystone

is

N, by

E.,

B must be directly opposite,


Hence, the departure, or first course and

the course from Eddystone to

namely

S.

distance

by

is S.

W.

by "W\ 25 miles

the

lab.

and long,

left

being

that of the Eddystone.


2. Sailing

and

after

down the Channel

running "W. by

the Eddystone bore

S. 8 miles, it bore

N.W.

N.K.E.:

re-

quired the ship's course and distance from the Eddystone to

the place of the last observation ?

Ans. course S.S/W., distance 7'2

miles.

At .three o'clock in the afternoon the Lizard bore


N. by "W. $ W., and having sailed 7 knots an hour "W. by
N. I N. till 6 o'clock, the Lizard bore N. E. E. required
3.

the course and distance from the Lizard to the place of the
last observation ?

Ans. course S.W.fW., distance 19-35 miles.


4. In order to get a departure I observed a headland of
known latitude and longitude to bear JS". E. by N. ; and
after

running E. by N. 15 miles, the same headland bore

AV.N.W.

required

my

distance from the headland at each

place of observation ?

Ans.

The

first dist.

8 miles; second, 10-8 miles.

departure, and her voyage


commenced, she shapes her course according to
her destination, by aid of a Mercator's Chart, in which are
marked the obstacles and places of danger she must avoid.
Her hourly progress, as measured by the log, and the

being

ship having taken her

fairly

courses she steers from noon

noteworthy particulars,

which

is

till

noon, together with other

are registered

on the log-board,

a large black board properly divided into columns

OigiiizM by

Google

TAKING DEPASTURES.
for these several entries

leeway,

currents,

85

the result of the 24 hours traverse

&c, being allowed

for

is

determined

every noon, as in the foregoing pages, and the latitude and


in, by dead reckoning, ascertained.
"Whenever practicable, these are corrected by means of

longitude

astronomical observations, and the true latitude and longi-

tude found : the place of the ship may then be pricked off
on the chart, and from this place as a fresh starting point
the course is shaped for another stage in the journey.

specimen of a ship's journal

will

be given hereafter ; but as

the determination of the latitude and longitude of a ship,

independently of the dead reckoning, or the latitude and


longitude by account,

astronomy, -we must

requires

now proceed

a knowledge of nautical
to the second part of our

ESD OF THE NAVIGATION.

NAUTICAL ASTRONOMY.

NAUTICAL ASTRONOMY.
CHAPTER

I.

DEFINITIONS CORRECTIONS 0E OBSERVED ALTITUDES.

Nautical Astronomy

is

that branch of tha general

science of astronomy which enables us to determine the


situation of a ship at sea

It

is,

by means of

celestial observations.

therefore, entirely occupied with the solution of one

important problem, namely, the finding the latitude and.


longitude of any spot on the surface of the ocean

where the erection of a fixed observatory

of a place

is impossible,

and

at which even the astronomical telescope cannot be used.

because

Tfc is

we

are thus precluded from the advantages of

an observatory, and of such instrumental aid as can be


always supplied and employed on land, that observations
at sea

kind

must be limited in their extent, and peculiar in their


and it is on these accounts that a special system of
and
The
which follow, however, have no exclusive appli-

practical astronomy must be devised for sea purposes


hence the propriety of the name Nautical Astronomy.

definitions

cation.

Axis.

The

axis of the heavens is

tion of the axis of the earth

merely the prolonga-

the axis of the earth

is

the

diameter about which that body really turns from west to


the axis of the heavens is that about which the beavenly

cast

bodies appear to turn from

astronomy, as well as in

many

east

to

west.

In nautical

parts of general astronomy,

DEFINITIONS.

we may
to be,

87

regard these heavenly hodiea to he, as they seem


equidistant from the centre of the earth, and

all

situated in the apparent concavity surrounding us, called


'

the heavens: the points where the axis pierces this concavity are the poles of the Jieavens or the celestial poles.

The

learner need scarcely be informed that these are not

determinate physical points fixed in space, like the poles

of the earth we regard only the direction of these points,


not their linear distance: linear distances of points or
;

objects in the heavens do not enter into consideration in

nautical astronomy, which takes note of angular distances

The angular

only.

distance of two objects

the eye between the visual rays, or straight

one from each

object,

and meeting

is

the angle at

proceeding

lines,

at the eye

and

it

is

plain that at whatever point in the straight line from the


object that object be placed, the angular distance between

the two will remain unaltered.

supposed to be at the centre of the earth,


also the centre of our imaginary concavity ; and

the observer

which

is

In astronomy the eye of

is

the angular distance of any two celestial objects must be


the same however small or however great the radius of that
concavity is supposed to be. This angular distance is, in
reality, observed

by a

from the surface of the

earth,

but

it is,

certain correction hereafter explained, always reduced

to what

it

would be

of the earth

is

if

the eye were at the centre

the radius

the only linear measure introduced.

Equinoctial.

The

equinoctial, or the celestial equator,

is

that great circle of the celestial sphere of which the plane

is

perpendicular to the axis

it

is

therefore

marked out by

the plane of the terrestrial equator being extended to the

heavens, the poles of which are the poles of the equinoctial.

Meridians.

The

celestial

meridians too

are,

manner, traced by extending the planes of the

in like

terrestrial

meridians to the heavenB: they are semicircles perpendicular to the equinoctial, and terminating in the poles of that

great circle.

OigiiizM by

Google

DEFINITIONS.

Zenith and Nadib.

The

sphere -which

celestial

is

zenith

directly

spectator: a straight line from

is

that point of the

over the head of the

the centre of the earth,

through any place on its surface, if prolonged to the heavens,


would mark the zenith of that place. And the point in the
sphere diametrically opposite to this,

celestial

The

of that place.

is

the nadir

and nadir

line joining the zenith

evidently the axis of the rational horizon of the place

is

and

the points themselves the poles of the horizon.

Vebtical Cihcles.

The

vertical circles of

any place

are the great circles perpendicular to the horizon of that


place

they are also called circles of altitude, because the

altitude of a celestial object is the height of

it

above the

horizon measured in degrees of the vertical circle passing

through

It

it.

is plain'

that

meet in the

all vertical circles

zenith and nadir; and that the complement of the altitude


of

any

celestial

body

is

the zenith distance of that body.

Small circles parallel to the horizon are called parallels of


altitude.

The most important of the

vertical circles of

is

that -which coincides with the meridian

is

upon

this, its altitude is

altitude of the object

the greatest

when

the object

meridian, or below the elevated pole,

is

auy place

when an

object

the meridian

it is

its

on the opposite
altitude

is

the

least.

The

vertical circle at right angles to the celestial meri-

and which therefore passes through the east and west


points of the horizon, is also distinguished from the others
dian,

it

is

called the

meridian,

it is

on the prime

prime

vertical.

"When an object is on the


due north when it is

either due south, or

vertical, it is either

The azimuth

Azimuth.

due east or due west.

of a celestial body

is

the arc of

the horizon comprehended between the meridian of the

observer and the vertical on which the body


in this intercepted arc obviously

is.

The degrees

measure the angle at the

zenith between the meridian and the vertical through the

DigiiizKi by

Google

DEEINITIONB.
body.

Vertical circles are also frequently called azimuth

circles.

Amplitude.

This term

is also applied to an arc of the


the arc, namely, comprised between the east point
of the horizon, and the point of it where the body rises, or
between the west point, and where it sets.
Like the
azimuth, the amplitude is measured by an angle at the
zenith the angle, namely, between the prime vertical and
that which passes through the body at riBing or setting

horizon,

muBt be

but, unlike the azimuth, the object

when we speak

of

its

amplitude

in the horizon

whereas, whatever be

its

altitude, it always has azimuth.

Declination,
its distance

The declination of a celestial object is


from the equinoctial, measured on the celestial

meridian which passes through

it

as respects a point on the earth,


to a point in the heavens
trial meridians) all

meet

and

on the

terrestrial,

so that

what

ia latitude,

declination in reference

is

as circles of latitude (terres-

at the poles of the earth, or of the

equator, bo circles of declination

heavens, or of the equinoctial.

become

all

meet

at the poles of the

Also, parallels of latitude

parallels of declination

on the

celestial sphere.

Pot, ah

object

is

Distance. By the polar distance of a celestial


meant the arc of the declination circle, from the

object, to that pole of the heavens

the rational horizon.

When

which

the object

is

ia

elevated above

on the same

side

of the equinoctial as the elevated pole, the polar distance


is

evidently the complement of the declination, or, as

called, the co-declination

when the

object

it is

and the elevated

pole are on contrary sides of the equinoctial, the polar dis-

tance

ia

The
any

the declination increased by 90.

altitude of the pole, above the rational horizon of

place, is always equal to the latitude of that place.

the latitude
noctial,

is

For

the distance of the zenith from the equi-

and therefore the distance between the zenith and


is the complement of the latitude
and the

the elevated pole

90

DEFINITIONS,

same distance

is

equally the

complement of the

the pole above the rational horizon


fore,

altitude of

this altitude

The

equal to the latitude of the place.

the equinoctial below the horizon, or

there-

is,

depression of

above

its elevation

the horizon, in the opposite quarter,

is

the complement of

the latitude, or the co-latitude, which

is

therefore measured

by the angle the equinoctial mates with the horizon.


The circles and terms now defined comprehend all those
in most frequent use in Nautical Astronomy, and it is
always to be understood, whenever we have spoken of the
distance between two points, as measured od an arc of one
of these circles, that the angular distance, or the degrees

and minutes of that

arc is uniformly meant, and not the


Tho circles referred to having no
radii, the arcs referred to can have no definite
though they subtend determinate and calculable
We have now only to mention one or two other

linear extent of the arc.


definite

length,
angles.

circles of

the celestial sphere occasionally referred to in

nautical observations.

The
sun in

Ecliptic.
its

This

is

the great circle described by the

apparent annual motion about the earth

it is

in reality the path actually described by the earth about

the sun in the contrary direction. The ecliptic crosses the


equinoctial at an angle of about 23 27'i this is called the
:

obliquity of the ecliptic

it,

as well as the points of inter-

section, is subject to a small change.

The two points of

intersection are called the equinoctial points; the sun, in


its

apparent annual path in the

ecliptic, passes

through one

of these points on about the 21st of March, and through the


other on about the 23rd of September.

days and nights are equal at

all

places

At

these times the

where the sun

rises

because any point in the equinoctial, in the apparent daily rotation of the heavens, is as long below the
horizon as above, since tho horizon of every place divides

and

sets,

that and every other great circle into

The poles are the only

places

two equal

portions.

on the earth at which the

Digitized by

Google

91

DEFINITIONS.
sun,

when

nor seta

in either of the equinoctial points, neither rises

the equinoctial then coinciding with the horizon,

the sun revolves with


half of

its disc

its

above, and the other below

advance of the sun in


hours to sensibly

The two

centre describing that circle, one

its

annual path

is

The

it.

small

too minute in 24

affect this statement.

points of the ecliptic, 90 distant from the equi-

noctial points, are called the solstitial points, as the sun's

apparent motion at these points


almost stationary
of

so slow that he seems

is

he passes through them about the 21st

June and the 21st of December.

Celestial Longitude. The ecliptic is the circle on


which the longitude of every heavenly body is measured
the point from which longitude is measured is the vernal
equinoctial point, which is called the first point of the conAries;

stellation

and, unlike terrestrial longitude, it is

measured in one continued direction round the


sphere

celestial

bo that while terrestrial longitude can never exceed

180, celestial longitude

The 360

may be

of the ecliptic

is

of any extent short of 360.

conceived to be divided into

twelve equal parts, called signs


arc of the ecliptic of 30.

each sign

The names

is

therefore an

of the constellations

through which these signs pass, and the symbols by which


they are denoted, are as follows:
1.
2.

ryi

Aries (The Kaiu).

Taurus (The

Ball).

8.

8.

1)1

3.

Gemini (The Twice).

10.

4.

S3

Cancer (The Crab).

H, SI

5.

SI

I"30

fi.

rl(L

Virgo (The Virgin).

7.

Libra {The Balance).

Of

Scorpio (The Scorpion).


Sagittarius (The Arrow).

yf Capricornus (The
Aquarius

The

Goat).

Watcr-

T1 ' e Lior)12.

these, the first six signs are

Piswca (The Fishes).

on the north of the equi-

noctial, and the others on the south.


The belt of the
heavens about 16" wide, 8" on each aide of the ecliptic, and

in which these constellations aro situated, and within the

OigilizMDy

Google

DEPIHITIONS.

92
limits of

which the planets pursue their couraea being called

the zodiac, the 12 signs nre frequently called the signs of


the zodiac.

Celebtial Latitude.
is

measured from the

perpendicular to

it

The

body

latitude of a heavenly

north or south, on a

ecliptic,

the circles of latitude

all

circle

uniting in the

poles of the ecliptic.

Eight Ascension.

The

right ascension of a celestial

object is the arc of the equinoctial between the

first

point

of Aries and the point where the declination circle through

Thus, right ascension and

the object cuts the equinoctial.

declination in reference to an object in the heavens, corre-

spond to latitude and longitude of a place on the earth.

On

the earth, longitude

(that of

Greenwich

is

in this

measured from the


kingdom)

first

meridian

in the heavens, longi-

tude and right ascension are both measured from the origin
of the signs,

the

first

point of Aries, or where the ecliptic

crosses the equinoctial, hut always from "W. to B.

"We see from these

definitions that, as in the terrestrial

great circles, every great circle of the heavens

panied by another great

is

circle at right-angles to it

accom;

thus,

and longitude, declination and right ascension,


and azimuth, are all pairs of ares perpendicular to
each other. Those great circles all of which are perpendilatitude
altitude

cular to another great circle, in other words, those great


circles that all unite in

the poles of another, are frequently

called secondaries to the

latter

secondaries to the equinoctial

tude are secondaries to the

thus, the meridians

are

the circles of celestial

lati-

ecliptic

and

vertical circles,

or circles of altitude, are secondaries to the horizon.

No

measures in the heavens are the degrees, minutes, &c, of


a small circle

the distance between any two objects taken

by an instrument

a spherical surface,
points

is

and on
the shortest distance between any two

is

always the shortest distance

the arc of the great circle joining those points.

OigiiizM 0/

Google

TIME.

On Time

Apparent, Mean, and Sidereal,

The interval of time between two successive appearances


of the sun upon the same meridian, is the length of a day
not of a day according to

by the 24 hours
spoken

of, is

civil

the same time, yet


the sun
length,

ia

its

irregular.

and

reckoning, or as measured

of a clock, but of a Solar day.

The

interval

not uniformly of the same length ; for although

the earth performs each of

its

diurnal rotations in exactly

annual motion of revolution round

Solar days, therefore, vary slightly in

the mean of all these varying days that is


common day, and divided into the hours,

it ia

taken for the

minutes, &c., as shown by clocks and chronometers, and referred to in the


therefore, is the

common business of life the commou day,


Mean Solar day, being the mean of all the
:

Apparent Solar days.


The Day, whether mean or apparent, is divided into 24
equal intervals, called hours and each of these into minutes
and seconds; an hour, minute, Ac., of mean, or common
time, is not precisely the same as an hour, minute, &c, of
apparent time but the 24th part of the day is always called
an hour. "We thus see that the apparent day, though not
of invariable length, is a natural day it is the actual inter;

two consecutive passages of the sun over the


day, though of invariable length,
day it is not measured by the recurrence
is
of any natural phenomenon. There is, however, a natural
val between

meridian.

an

But the mean

artificial

mean day, is strictly invariable;


is called the Sidereal Day, and measures the interval
between two successive appearances of the same fixed star
on the meridian, and is the exact time occupied in one
The distance of the fixed
rotation of the earth on its axis.
day, which, like the artificial

it

day

stars is so immense, that the earth's change of place from

to day produces not the slightest effect

upon

tueir apparent

Lii

::

b,

TIME

94

HOUB-AHGLE.

whatever star be observed, and whatever part


the earth be in, it ia always found that the
between two consecutive passages of the star over
the meridian is uniformly the same in length the interval is
In the reckoning of astro23h. 56m. 4-09s. of mean time.
nomers, both the apparent and the mean day commences at
positions

of

crbifc

ita

interval

when

noon, the former at apparent noon, or

on the meridian, the

actually

instant

when

latter

the sun is

mean noon, the

at

the sun would le on the meridian if his

motion in right ascension were uniformly equal to his mean


motion. But the sidereal day commences when the first
point of Aries

is

on the meridian.

astronomical reckoning

is

carried

In each kind of day the


on from Oh. to 24h. But
is the same aa

the nautical day, in keeping a ship's account,


the

day, the reckoning beginning at midnight, counting

civil

12 hours

till

noon, and then 12 more

when a new day

begins.

till

the next midnight,

It will be observed, therefore, ihuc

the astronomical day does not

commence

.till

12h. of the

thus, August 15, at 9 o'clock in the


would be recorded in the ship's account,
August 15 at 9b. a.m., in astronomical reckoning would be
civil

day have expired

morning, or as

it

August 14, at 21h., that is, 8h. from the approaching noon,
when a new astronomical day, namely, August 15 commences. It may be noticed here, that "a.m." signifies
in the morning {Ante Meridiem)
the afternoon {Post

HouH-ANdiiE,

and " p.m." means in

Meridiem).

The angle

at the pole of the equinoctial

which a meridian passing through the centre of the sun


makes with the meridian of the place of observation ia
called the sun's hour-angle

from apparent noon; this angle

converted into time at the rate of 15 to an hour gives the


if the sun be westward of the meridian, and before noon if it be eastward.
In observations for the
It is the time shown by a sundial.

apparent time at the place after noon,

time at

sea, it is

object sought

the sun's hour-angle that

so that the time deduced

is

is

usually the

apparent time,

Digitized By

Google

CO BBE CTIONfl OP ALTITUDES.


which

is

03

mean time by

readily converted into

"Equation of Time," given

table for the

Nautical Almanac, in which publication

help of the

at p. 1 of the

all

the predicted

phenomena concerned in Nautical Astronomy are recorded,


like

the

common

occurrences of

The hour-angle

for

life,

any other

in

mean

time.

celeBtial object is, in like

manner, the angle at the pole between the two meridians,


one through the zenith, and the other through the object

which angle

is

evidently always the difference between the

right ascension of the meridian of the place,

and that of

the object expressed in degrees.

On

the Corrections to be applied to Observed Altitudes to

obtain the

True

Altitudes.

Altitudes of celestial objects are taken at sea by a quad-

rant or sextant, which measures the angular distance of the


object above the visible horizon of the observer.

observed altitude

but

This

is

the

the eye, instead of being above,

if

were level with the surface of the sea, the angular elevation
of the object would be measured from the sensible horizon.
Thi3 is called the apparent altitude, and is obviously less
than the observed altitude. The higher the eye, the greater
of course

is

altitude

a correction

the excess of the observed over the apparent

former to the

latter,

is

and

therefore necessary to reduce the


this correction is

Cobbection rou Dip. Let


andS the situation

server's eye,

EH

being the

horizontal

visible horizon being the

* The

line.*

is

E,

is

subtritctire.

the angle S

the

drawn from

II,

observer's

but the

AE

so distant, that the length of

be considered as nothing in comparison

is,

Then,

sensible horizontal line is in (strictness

subtended by

always

be the place of the ob-

tangent to the earth, from E,

nearest even of the heavenly bodies

may

of the object whose altitude

be found in angular measure, that

is to

that

is,

tho angle at

S,

immeasurably small.

L."

71

Z'J

'A

Lv

DIP OF TUE^nOBISIOS.
the altitude given by the instrument will be the angle

SEE';
'

the difference between

these two, namely,


H.

E H' is

the

angle

the Dip, or depres-

sion of the visible horizon,

and

istbatwhicb must be subtracted


from the observed in order to

obtain the apparent altitude of


S.

contact

HE

then

CE

complement of
is

H',

Draw C B from the centre


of the earth to the point of
and the angle C, are each the

B, and are therefore equal

that

ia,

equal to the angle of the dip.

Now
circle,

(Euc. 36, III.), if r be put for the radiua of the


E of the eye, we have E B ;
for the height

and A

2 ri+S s

But since A 2 is very insignificant in


comparison with 2 rli, it may without appreciable error be
rh. Now, from the
rejected, so that we shall have, E B=
(2

r+k)

Ji

right-angled triangle

EB

C,

we have

EB

(;-+A) sin dip; and, because the angle

never exceeding a few minutes, the arc


its sine

,.

dip

which

is

= E C sin C =
very sinaB,

is

may be taken

hence, equating the two expressions for

have

/
2

,
!i

;. ilip

j._pr or

for

E B, we

y~ very
-/Ml

nearly,

the length of the arc, to radius 1, that measures

the angle of the dip due to the height A of the eye.

This

of A likely to occur in practice, is converted into minutes, and the table of " Corrections for Dip"
arc, for all values

formed.

As
is

the number of minutes iu the arc which measures

the same, whatever be the radius of that arc, it follows

that the
is

the

number

of minutes or nautical miles in the arc

number of minutes in the

dip

and since

jj

is

A 1$
the

Digitized By

Google

COEBECTIO^S
measuring

length of the arc

v' 2 rh

is

EOll ALTITUDES.

the length of the arc

07

to radius 1, it follows that

AB

this, therefore,

calculated in nautical miles for successive values of


table referred to

Coebectiott
whose altitude

iB

may be constructed a little

being
It,

the

differently.

roa Semi-diameter. -"When the body


to be taken is either the sun- or the moon,

by the instrument is that of either


uppermost point of the disc, called the
lower or upper limb of the body a correction, therefore, for
semi-diameter must be applied, after that for dip, in order
the altitude furnished

the lowermost or

to get the apparent altitude of the centre.


is

This correction

the angle subtended at the eye by the semi-diameter of

the body observed

Almanac.

it is

given for every day in the Nautical

The moon, however, being

so

much

nearer to the

earth than the sun, her diminution of distance, in ascouding

from the horizon towards the zenith, has a sensible

effect

upon her apparent magnitude her semi-diameter measures


more when she is in the zenith than when she is in the
horizon, for she is nearer, by a semidiameter of the earth
in the former case than in the latter, and there is a gradual
;

augmentation of her diameter as she gradually ascends.

The moon

is

only about sixty Bemi-diameters of the earth

off when in the horizon, bo that her semi-diameter

the zenith,

is

when

in

about one-sixtieth part of the whole greater,

and the amount of augmentation for any altitude is found


by multiplying one-sixtieth of her horizontal semi-diameter
by the Bine of her altitude. In this way the table intitled,
" Augmentation of the Moon's Semi-diameter, is constructed. The number of seconds placed against the altitude in this table must be added to the horizontal semi1

'

diameter, given in the Nautical Almanac, to obtain the

Bemi- diameter proper to that altitude.

"With respect to the sun, his distance from the earth


is

so great that the augmentation of his semi-diameter,

as he
Hence,

increases

his

altitude,

is

practically

insensible.

CORRECTIONS FOR ALTITUDES.

OS

For the apparent


alt.

alt.

of the Sun's centre,To the observed

apply the corrections for dip and semi-diameter.

For
served

the apparent alt,


alt.

of the Moon's

centre.

To the ob-

apply the corrections for dip, semi-diameter, and

augmentation.

Coebection FOR Refbactiok.

As

the lower parts of

the atmosphere surrounding the earth are compressed by


the weight of the upper, the density of the air diminishes
the higher we ascend. A ray of light, therefore, from any
upon entering our atmosphere, meets with
an obstruction which becomes more and more sensible the
celestial object,

deeper into

it

the ray penetrates.

more and more out of

The ray

its rectilinear

course,

is

thus bent

and

path

its

through the atmosphere, instead of being a straight


is

The

deflected into a curve concave to the earth.

line,

direction

of the object from which the ray proceeds, being judged of

hy the

which the ray arrives at the eye,

direction in

and

so,

thus

we see the object raised above its real


except when it is in the eenith, regard its

erroneously inferred
place,

is

altitude as greater thau

it

actually

is.

The

correction,

therefore, for this refraction of the rays of light, is like that

for dip, always subtractivc.

The more

enter the atmosphere, the greater

is

obliquely the rays

their refraction

when

they eDter perpendicularly, they are not refracted at

hence when the object


greatest;

is

all

in the horizon, the refraction is

diminishes as the object ascends, and becomes

it

nothing at the zenith. In different states of the atmosphere


the refraction for the same altitude, is of courae different

the table gives the value of the correction for the meanstate of the atmosphere,

a second table

according to

and to

this is

sometimes annexed

modifying the corrections of the former

tiio

actual condition

of the atmosphere,

shown by the thermometer and barometer


place of observation

at the time

but this additional table

made use of at sea. It is however given at


mathematical tables accompanying this work.

is

as

and

but seldom

p. 340, of

the

DigiiizM by

Google

COBELCTIONS FUB ALTITUDEB.

Correction fob Pabaii, ax. Before the


celestial object

'jy

altitude of any

can be employed for any practical purpose,

must be reduced to what it would have been if taken not


from the surface, but from the centre of the earth, and
it

measured not from the sensible, but from the rational horizon of the place of observation. In the case of a fixed star,
the distance is so immense, that the radius of the earth

dwindles in comparison to a point, and there

is

no measur-

able difference between an altitude taken from the centre

and an altitude taken,


1

at

directly above the centre,

the same time, from a point

on the

surface.

But

as respects

the sun and moon, especially the latter, the anglo at the

body subtended by the radius of the

earth,

and which

is

called the Parallax in altitude, is of appreciable magnitude.

Let S be the object observed,

the place of observation,

A H the sensible, and C D the rational


horizon: the observed altitude

when

corrected for dip, semi-diameter, and


refraction, will

angle S

be measured by the

A H, which is the true altitude

of the centre above the sensible hori-

zon,aud8CD

will

be the true altitude

'

of the centre above the rational horizon.

The difference between


S B H, is
S

And

H-8 A H=A S

the true altitude

rational horizon

0 D,

two

these

angles, since

8CD =

C, the parallax in altitude.

S C D, of tho

centre above

the

ia

SC'Ds:SBH = SAH: + ASC,


Hence while the correction
the correction for parallax

the true altitude.

for refraction is subtraefcive,

is

additive.

The

horizontal

parallax is tho angle A


C this is given for every day
in the year in the Nautical Almanac; that for the sun
9", but that for the moon changes
:

never varies much from

Digitized 0/

Google

100

COBBECI10H8

considerably

EOlt ALTITUDES.

given both for noon

is

it

midnight,

atid

Greenwich time.

From
easily

the horizontal parallax, the parallax in altitude

computed

for referring to the triangle

ih

SAC, we

have the proportion

SC:AC::ainSAC:fdaASC.

orHC:AC::sinSAZ: an AS 0
AC

But -jj-^

And

sin

AH

we may

ing arc for the time


Par. in

And

C, tlic hor. parallax,

alt.

it is

in altitude
tion of the

cos S

A H.

A II -

cos all.

always a very

substitute the seconds in the measur:

we

in seconds

thus have,

= Hor.

Far.

in.

seconds

cos alt.

from this expression that the table for parallax


In the table headed " Correcis constructed.

Moon's Altitude," the joint correction

and parallax is given


parallax minus refraction.
refraction

The two

~~

and cob 8

A S C is

since the parallax in altitude

small angle,

it

for

both

exhibits the value of

corrections just explained (Kefraction and Paral-

lax) applied to the apparent altitude of

any point

in

the

heavens, reduces the apparent to the true altitude of that


if the observer's eye were at the centre of the
and the angular elevation taken from the rational

point, as
earth,

horizon.

Hence,

For the true altitude of the centre of Sun or Moon. To the


apparent altitude apply the corrections for refraction and

As already observed the stars have no parallax.


In taking from the Nautical Almanac the measures

parallax.

there given for semi-diameter and horizontal parallax, it

most not be forgotten that these measures are what they


would be if observed from the centre of the earth at the
Greenwich time recorded in the almanac. Now for the
moon, they vary slightly hut perceptibly from hour to hour.

DigiiizM by

Google

EXAMPLES OF COMIECTIO'S.

Ill]

so that for any intermediate time at Greenwich the corre-

The time

sponding values muBt be found by proportion.

at

Greenwich, and the instant of any observation or event


elsewhere, is the

event

it is

Gekenwich Date

of that observation or

found by converting the longitude of the place

of observation into time at the rate of 15 to an hour, as


already noticed at page 94.

Having now explained

all

the corrections neeeHsary to be

applied to on altitude observed at sea, in order to deduce

the true altitude, we shall proceed to a few examples: we

must

remark, however, that even the observed altitude

first

itself is affected

with error

it is

not that whieh an instru-

ment entirely free from all imperfection would give. Such


an instrument was never constructed by human hands. It
ia scarcely too much to say, that no chronometer, for instance, whatever the care and skill bestowed upon it, ever
showed exact time; nor did any quadrant or sextant ever
But this imperfection is
of far leas consequence than might at first be supposed
of but Httle moment whether a time-keeper lose or

accurately measure an altitude.

it is

gain, provided only

knowing

its

it

lose or gain uniformly, because,

error at any one instant,

we can

easily,

from
from

the uniform increase of that error, compute its error at any


other instant, and thence obtain the correct time. So with
respect to the sextant or quadrant, the index error, as

it is

being known, and there are several ways of determining it as will be hereafter noticed, the proper allowance
called,

for

it

can always be made, and the correct observed

alti-

tude obtained, as in the examples following:

Examples of Correcting Altitudes taken

A Stab. 1.
and the height

at sea.

If the observed altitude of a star be 42 3G',


of the eye 18 feet,

what

will its true alti-

be, supposing the index error of the instrument to be


18" svbtractive ?

tude
3'

Oigiiized 0/

Google

EXAMPLES 07 COSSECTIOKS.

102

eor.

Dip

Apparent

0"

43" 38'

OTjeerrea Alt

Index

3'
4'

18" 1
11" J

'

-0

7'2P"

28' 31"

ait

Refraction

True altitnde

4-

I'

42 27' 27"

The altitude of a star is 43 12', the height of the eye


18 feet, and the index error +2' 24": required the true
2.

Ans. true alt. 43 9' 11".


3. The altitude of a star is 16 33', the height of the eye
17 feet, and the index error +3' required the true altitude ?
Ans. true alt. 16 28' 42".
The Sin*. 4. On a certain day the observed altitude of

altitude ?

the sun's lower limb was 28 16', the height of the eye

20

feet,

the index error was

2'

was 16' 4"

Note.

Almanac,

required the true altitude of the centre ?

The

taken at

Observed

Index

alt. sun's L. L.

oar.

Dip

alt.

Befrac.

centre

alt,

was 16

the index error was

2'
4'

always

be

2S 10' 0"

-i

+9' 2"

I
J

.28

centre

38"
24"

+16' 4"
,

and par.

True

The observed

certain day

App.

.as

may

sun's horizontal parallax

9".

Semi-diam.

5.

was

38", and the semi- diameter

of the sun, as given for that day in the Nautical

25'

.28

1'

2"
39"

23' 23"

altitude of the sun's lower limb


33',

on a

the height of the eye was 17 feet,

+ 3',

given in the Nautical

and the semi-diameter of the sun,


for the day, was 16 17':

Almanac

required the true altitude of the centre ?

0.

The

altitude of the sun's

Ans. true alt. 10 45' 1".


upper limb was 47 2G', the

height of the eye 20 feet, the index error

1' 47", and the


Unite 'XI by

EXAMPLES Or COIIBECTIONS.

103

aim's semi-diameter 15' 49": required the true altitude of

the centre

Ana. 47

The Moon.

The observed

7.

3' 12".

altitude of the moon's

upper limb was 41* 23', the index error was +2', the height
of the eye 15 feet, the horizontal semi-diameter at the time
15' 10", and the horizontal parallax 55' 40": required the
true altitude of the moon's centre ?
Observed

Index

Dip

moon's U. L.

alt.

cor.
.

Semi-diameter

+ 10"forAug.
App.

alt.

and

"l

41 23'

0"

3'lr L-_17'

9"

-16' 20

centre

Refraction

par., or correction

41

moon's apparent altitude

True alt. moon's centre

6'

51"

^^

41" 46' 41"

S. The observed altitude of the moon's upper limb was


46 18' 49", the index error 6", the height of the eye 20

feet,

the moon's horizontal semi-diameter at the time 16'

and the

horizontal parallax 59' 7"

required the true

6",

alti-

tnde of the moon's centre ?


Ana. true
9.

The observed

altitude of the

alt.

46 38' 11".

moon's lower limb was

36 39' 46", the index correction +2' 17", the height of the

eye 22 feet, the moon's horizontal semi-diameter at the


time 15' 10", and the horizontal parallax 55' 33" required
:

the true altitude of the moon'a centre

Ans. true

alt.

37 35' 52".

Note. In the preceding examples the- horizontal semidiameter and the horizontal parallax of the moon, have
been considered as those due to the body at the instant of
observation.

In the Nautical Almanac

tlieBe quantities are

given only for every noon and midnight at Greenwich, and

they vary

render

it

sufficiently, at least

the

latter, in

the interval, to

necessary, if strict accuracy be required, to

make

LATITUDE FH0M A MERIDIAN' ALTITUDE.

101

allowance for that variation whenever the Greenwich time


intermediate between Green-

at the instant of observation

is

wich noon and midnight.

But

sea,

in finding the latitude at

the omission of a single correction amounting only to

a few seconds

is

not of much practical consequence, so that

the allowance alluded to

If the
be determined to the nearest minute, it is as
much as can be expected considering the difficulty of taking
an altitude at sea with precision and indeed it is as much
Still when the time
as the safety of navigation requires.
is

UBually disregarded.

latitude can

moon is some hours distant from


Greenwich noon or midnight, as we can easily allow for

of an observation of the

those hours, by a simple inspection of the noon and mid-

night horizontal parallax in the almanac,

do

so.

"When we come

fongitude,

we

Bhall take

we may

as well

of the problem of the

to treat

more exact account of the small

corrections of the moon's altitude.

CHAPTER II.
OS FINDING THE LATITUDE AT SEA FHOM A MERIDIAN
ALTITUDE.

The
at sea,

best method of determining the latitude of a ship


is

that which is deduced from an observed altitude

when on the meridian of the place. It


two accounts first, because the observation can in general be made with greater accuracy and
secondly, because the necessary calculations are easier and
fewer in number." The most desirable object to observe is
of a celestial object
is

to be preferred on

the sun, which

on the meridian at the

ship's apparent
noon, and accordingly the opportunity of taking his altitude
is

A star of

at that time should never be disregarded at sea.

known

declination

is also

a very suitable object

but

when

the stars begin to appear the horizon generally becomes too


obscure to be sufficiently well defined, a hindrance, however,

OigiiizM By

Google

LATITUDE PROM A MEHIDIAN ALTITUDE.

105

which may be sometimes removed by employing an

Artificial

The moon

Horizon to be hereafter noticed.

ia

not eo well

calculated to give tbe latitude with accuracy as the bud. or a


star,

because the moon's declination changes considerably

even in an hour, and as the declination of the body observed,


as well as its meridian altitude, must be known, if there be
much error in the ship's longitude by account, and consequently in the Greenwich date of the observation, there will

be a proportionate error in tbe declination, and hence in

The

tbe latitude inferred.

declination of a star

may be

regarded as constant, so that there will in this case be no


occasion for finding the Greenwich date of the observation :

and the declination of the sun varies so slowly, that even


a considerable error in the ship's longitude, and therefore
in the

Greenwich date of the observation,

will occasion

no

error of consequence in the declination at the time of that

The way

in which the latitude of the place of


deduced from the meridian altitude and decli-

observation.

observation

ia

nation of a celestial object

Let the

circle in the

dian of the observer at

and

also

EQ

If'

is easily

explained as follows

annexed diagram represent the meri-

his zenith,

his rational horizon.

Let

be the equinoctial, and


is elevated above the

the pole which

Then

horizon.

in reference- to an

on the meridian, S 'L will


always be the co-altitude, S Q or S E
the declination, and E Z or
the
object S,

HN

latitude.

Now

with

N, and the zenith Z, the object


must be situated in one or other of the four positions
marked S
S 3 S4 ; and taking these in order we liave for
respect to the elevated pole

the latitude

Z,

= E 81 + Sj Z,

EZ = ES! &,Z,

8,

B S

HN=HS

+ 8t B)

that is Int.

lat.

Jat

Iat.

=
=

dec.

zenith distance.

dec
senith
wnith dint
altitude

distance.

declination.

co-declination.

1 3

COHYEESIOS OF DEGEEE9 IKTO TIHE.

106

In

where the elevated pole

this last position,

the object and the zenith, the object


pole

"When the zenith


of it

between
the

is

eaid to be below

is

in the other positions it is situated above the pole.

is said

is

north of the object, the zenith distance

and when the zenith is south of it,


said to be south
hence, we have the

to he north

the zenith distance

is

following rule for finding the latitude from the true altitude

when the

When

object is above the pole.


the abject is above the

pole. It the zenith distance

and the declination have tho same name, that

is,

if

both

be

north or both south, their sum. will be the latitude.


If the zenith distance and the declinatiou have different

names, that

is,

difference will

if one be north and the other south, their


be the latitude, of the same name as the

greater.

Wlien the object


the

sum

is

below the pole, the latitude

of the true altitude

same name

and the

is

equal to

co-declination, of the

as the declination.

As it is necessary to know the declination of the object


observed at the time of observation, or at the Greenwich
date of
<fce.,

it,

we must know how

to convert degrees, minutes,

of longitude into time, from the relations" 15

15'= l m

15"

= 1*.

= lh

These relations suggest the following

rule:

Conversion of Longitude into Time. Eule. Multiply the


degrees, minutes, and seconds, each by 2.
Divide each

by 30 : the quotient, from the degrees, will be hours,


and twice the remainder will be the minutes we shall thus
have the hours and minutes in the degrees. The quotient
result

from the minutes will be minutes of time, and twice the


remainder will be the seconds we shall thus have the
time in the minutes of longitude. And lastly, the quotient
from the seconds will be seconds of time.
Example 1. Convert 34 44' 34" into time.
The double of this is 68 88' GS", and dividing each denomination separately by 3, cutting off the unit figure for
:

C0NVZB610N OP TIME INTO DEGKKES.

107

the 0 suppressed in the divisor 30, and remembering to


double each remainder, the operation will stand thus
3)8,8 8,8' 6,8"

The

division of the seconds is carried

on to decimals,

these being always used instead of thirds.


2. Convert 108 24' 22" into time.
3)21,6= 4,8' 4,4"

~7

12

i
'

36

13

7*

42''

30" in time

3..

84

4.

93 37' 4L"

5. 230 32' 10"

-47

S7'"47

is

B*>

SS*

50".

&

14m 30=-72.
15 h 22" 8-7.

The preceding method of converting degrees, &a, into


time, will he found much more convenient than that in

common

use.

In order to convert time into angular measure, multiply


the number of hours by

15',

the product

Divide the minutes and seconds by

is

4,

so

many

degrees.

and reckon every

unit of remainder as 15', if minutes be the dividend, and as

15" if seconds be the dividend.

be required to convert 8 h 14 23*


into angular measure, as also 5 h 19 37 s

For example:

let it

=
14"==
23-

45

5h

5'

19=

S"30'
45"

48 35' 45"

37'

75
4 45'
3' 15"
s
79 54' 15"

LATITUDE

10S

Sry ABOVE THE

FOT.E.

Latitude front Meridian Altitude of the Sim above the Pole.

Rule

1.

From the longitude by account, find the appa-

rent time at Greenwich

that

is,

the Greenwich date of the

observation in apparent time.

From

2.

I of the month in the Nautical Almanac,

p.

noon at Greenwich,
and from the hourly variation of the declination there given,
and the Greenwich date, find the proper correction for that
get the sun's declination at apparent

date

the declination at the time of observation will thus be

obtained.
3.

To the observed

altitude apply the proper corrections

which subtracted from

for reducing it to the true altitude,


f)0 will

4.

give the zenith distance.

Mark

zenith

is

the zenith distance N. or S. according as the

north or south of the Bun; then

tion and zenith distance have the


will

be the latitude

same

if

the declina-

mark's, their

sum

if they have different marks, their


be the latitude, of the same name as that of
two quantities.

difference will

the greater of the

Note,

After

the preliminary reduction of the declina-

tion to the time of observation, the

first

step in the work,

for obtaining the apparent altitude of the centre of the

or

moon from

sun

the observed altitude, comprehends the unit-

ing of the three corrections for index error, dip, and semidiameter, into one

when the

signs of these three items

little

them

n
But both the index error and the dip
known before the observation, their combined
known, and may therefore be written down as

are not all alike, the finding of the balance of

is

inconvenient.

being always
effect ia also

one correction.

DigiiizM by

Google

LATITUDE

BUS ABOVE THE POLE.

109

The Latitude from Meridian Altitude of the

Examples.

Sun above

the Pole.

1. March 4, 1858, in longitude 86" 34' W., the observed


meridian altitude of the auu'a lower limb was 46 48' 30"

(zenith N.), the correction for index and dip was

4' 6"

required the latitude?


1.

For

Hit app. time at Greemoicli.

Longitude by account

S6 34' W.

2'

3,0)17,2" 6,6'

Greenwich date (app. lime)

The

oh 46'"

must be subtracted, as the declination

variation for this time

decreasing (See Nautical Almanac).


2.

For

the dee!.,

Greenwich dale.

Dec. app. noon, Nauticnl Aim.

6 26' 4G" S.

Dlff. for

-5' 83"

Variation in
3.

Observed

App.

alt.

5MG"
For

s' 33"

the Latitude.

40 48'

alt. sun's L. L.

Index and dip.


Pemi-fliara.

.'
.

of centre

4' 6" I

16' S" J

True

alt. of centre

True zenith distance


Declination Q. Date

+ 12'

.46^

0'

33"

CO"

43

bV "

it

no

COBBECTION 01 DECLINATION.

JJoTE.^There is no absolute necessity to find the Greenwich date of the observation, in order to get the declination
at that date.

30,

If

we double the hourly

variation, divide

by

and then multiply by the number of degrees and fracof

tion of a degree in the longitude, the proper correction

the declination will be obtained

thus,

57""81

Hourly variation

23,0 )11,5-62

3-864
8G ro versed

68J

30832
2312
193
383" -87

Cor. of declin.

The

principle of this second

method of correcting the


The hourly

declination for longitude is easily explained.


variation

is

that due to 16 of longitude

hence, the double

is the difference of declination due to 1


and this difference multiplied by the degrees

of it divided by 30
of longitude

of longitude of the ship,

must give the proper correction


in the longitude amount-

Any odd minutes

of declination.

may be

ing to leas than a quarter of a degree,


as they will not

make

1" difference

disregarded,

in the result.

2. May 29, 1858, in longitude 31 17' W., the observed


meridian altitude of the sun's lower limb was 65 42' 30"

(zenith N.)i the index error

the eye 13 feet

1,

was 1'

9",

and the height of

required the latitude ?


For

QrtuwiA.

the app. time at

Longitude by account

.31"

17'

W,

2
3,0)6,2' 3,4

Greenwich dlte

5-

Digiiizcd By

Google

COBBECTIOiT Or DECLINATION.
2.

Noon

For the

Qrtemaxeh date.

declin.,

Declin.

Cor. for long.


in 2 h

21" 38' 12" N.

Declination

Increase of declination in 2 h 5 n

For

3.

Observed

alt. sun's

Indexanddip

the Latitude.

L. L.

Semi-diam

App.

olt.

Refraction

66" 42'

'

42"

+15' 49"

4'

+
06" 53'

of centre

par

~65 53'

True alt of centre

80

.24"

Trne zenith diat


Declin. Greanwioh date

Latitude

6'

45" N.

21 s 38' 12" N.

,45 44'

The declination at tbe time of observation


method in the Note as follows

is

found by the

Noon

declination

Cor. for longitude

Declination

21 87' 24"

Diff. for 1*

-t-22"-S3

48"

21 38' 12"

3,0)4,5-615

456C

+47"-EG

Cor. of declin.

3. September 23, 1858, in longitude 94 E., the meridian


altitude of the Bun'a upper limb waB 75 2<y (zenith S.), and
the correction for index and dip was 4' 42" required tbe
:

latitude?

DigiiizM by

Google

112

FHOM MKBIDIAN ALTITUDE.

TATITtJMS
1.

I/mgHude hy aoronnt

For Ok app, time

at

.94

E.

Greenwich dale, before noon


2.

Noondeclin.

/V

6" 10"'

Gr*Mn>kh

0" 2' 48" X.

3.
alt.

Semi-diam.

App.

For

16

in

1-

4'

0"

42'

alt. centre

74 50' 10"

14"

par

True alt. 'centre

True zenith

+3<

7fi' 20'
.

3.

in

the Latitude.

U.

Ind-anddip

Kef.

1''

in &

Cor. of declination

Obs.

dale.

Diff. in

Dkowsatios

the deelh.,

0" S' 24' P.


6"
6'

Cor. for B. longitude

Grermwh.

..

74' 59'

15

dist

Declination

Latitdi'K

.14"

5"

0' 05"
2'

S.

42" N.

58' 13" S.

example the Greenwich time of the observation


lG m before the noon of the 23rd. The hourly differ-

In
was

this

ence

is Hiibtraetive,

because the south declination, in pro-

ceeding from the noon of the 23rd towards the noon of


the 22nd, decreases.

As

the decrease of S. declination

the declination must


hove changed from K. to S. hi the interval.
In finding the correction of declination for longitude,
the learner will in general find the second method to be a
little more easy and convenient than the first, and the
work will be facilitated if he always prepare a blank form
exceeds the S. declination at noon,

Digitized By

Google

BLANK IOBM rOB I'lNBIXG LATITLBE.

113

of the operation previously to commencing it. Nor should


be neglect, when once the Nautical Almanac, or the book
of Tables is in hand, to make all the ubo of it he can in
anticipation of what he may want to extract thus, at the
:

time of taking out the declination, he should also take out


the semi-diameter, putting
blank form. The following

when

proper place in the

it

in its

is

a specimen of such a form,

the second method of finding the declination at the

Greenwich date of the observation


Blank form for

is

used.

the Sim.

NoonDeciin.

Diff.

in

1'

....

Cor, for long.

x 2

Decuhation-

3,0)

Cor. of decl. for long.

%* The
most

longitude

is to

"

be taken only to the nearest half degree, or at

to the nearest quarter.

Observed altitnde (L.

L. or

U.

L)

Index and dip


::'

Semi-diam,

App.

::'}

alt. centre

Re par.
True

alt.

centre

80
True zenith

dist,

Declination

LATTTtJBK

The same form


following example

will serve equally for

a planet, as in the

4. Jan. 29, 1858, in

longitude 58 37' E., the observed

Digitized By

Google

1U

LATITUDE TBOM

PLAKET.

A.

meridian altitude of Jupiter's lower limb was 49 18' 35",


(zenith K.), the index: error

of the eye 22 feet

was

+ 4'

10",

and the height

required the latitude ?

Declin. on Meriil. of G.

Correction for long.

Oxolinafios

13* 2' 27" K.

Diff. Id

1*

+5"'3

-21"

6" N,

IS* 2'

3,0)1,0-6

353
Long, reversed

Cor. for long.

The
is

correction of tlie dcoliuatitra

is

85|

20" -7

snbtractive because the longitude

E.

Oba.

alt.

L.

_ Ind.anddip.

49 18' 35

L
.

Semi-diam.

App.

alt.

"

-27'"
27"
+ 19" J
"I

Ref.-par.

True

alt. oet

True zenith dist


Declination

40" 42' 22" N.


13 2' 0' N.

Latitude

Latitude

from Meridian AUUnde of

a fixed

Star above

tie Pole.

As

already remarked, a star changes its declination so

slowly, that

any correction for longitude

as moreover a fixed star has

no

is

insensible.

And

parallax in altitude, nor

any diameter, the only corrections of the observed altitude


will

be those for index error, dip, and refraction

the rule,

BULB FOE LATITUDE.


therefore,

Rule
and

deducing

for

the

115
from a star

latitude

is

as

follows:

Correct the ohserved altitude for index, dip,

1.

the result will bo the true altitude, which


subtracted from 90 will give the zenith distance,
refraction

2.

Mark

the zenith distance N. or S. according as the

zenith is N. or S. of the star then, if the declination, token


from the Nautical Almanac, and the zenith distance have
the same marks, their sum will be the latitude if they have
;

different marks, their difference will be the latitude.

Meridian Altitude of a Star above the Pole.

Examples.

April 11, 1858, the meridian

1.

was observed

altitude of Arcturns
to

be 46 15' (eenith N-), the index

was +

correction

2'

10*, and the

height of the eye 20 feet


the latitude

Dip

16" 15'

10' 1
+2'
+ 2' 10"
-4' 24" J

Apparent altitude

1858,

1,

was

required the latitnde

Latitude

Note. The

'!

0"

280 S8'29"
1' 46"

56"

28 45'

-4' 11'

Apparent altitude
Refraction

Zenith distance

Zenith diet.

Star's decIin.Ap.il

observed

20", and the height of the

2'

eye 18 feet

Dip

-2' 14"
46 12' 46"

the

meridian altitude of Spies was


28 45' (aenith N.), the index error

Observed altitude
- 2' 20"

Observed altitude

Index

required

May

2.

43*48' 10' N.
19 55'

5"

Gl 23'

7" N.

St<uJ sdeclin.,Miiyl

10 25' 26" S.

LiTrTDi>E

60 57' 51" K.

IT.

63 43' IS" H.

time at Greenwich

passes the meridian of that place

the time at the ship

when

it

is

passes

when a

star or planet

very nearly the same as


tlie ship's

meridian; so

that, having the approximate time at the ship,

tain

we can ascerby a reference to the Nautical Almanac, what stars or

planets will be on the meridian of the ship about that time.

The time

of the meridian transit of each of the planets

actually given for every day of the year,

is

and the time of a

Digitized By

Google

TEAS SIT 01 STAES.

llfl

star's transit is

found by subtracting the R. A. (right ascenBtar, both of which

sun from the R. A. of the

sion) of the

are given in time in the Nautical

R. A. of tbe sun exceed that of the

But

the latter.

several stars

the ship at the same time


star actually selected

we may

latitude

of different names, and subtract


:

the result

the Btar.
star

the meridian of

for- observation,

previously find, approximately, what altitude the

star thus selected ought to have


star's declination to the

name

Almanac should the


24 " must be added to

star,

may be near

to prevent mistake as to the

from the almanac

may

By

is

in order to this, add the

by account if they are


they are of the same

if

the zenith distance or co-altitude of

the

time and the altitude the


be discovered some minutes before the time of
these aids

and the index gradually moved

transit, its altitude taken,

as tbe star ascends,

till

it

to descend, at which instant

Referring to the

we

illustration,

first

rind

appears stationary, and


it is

is

about

on the meridian.

of the preceding examples for an

from the Nautical Almanac, that on

April 11, the R. A. of Arcturua was 14 h 9 m 14 s, and that of

the sun, l1 IS 42 s ,
'

which

is

The

difference of these is 12 h

the time of meridian transit of the

50 32 s

star.

It

may

be looked for by aid of the approximate altitude, at about


18 or 20 minutes to 1 o'clock in the morning, making ample
allowance for error in the ship's time, and kept in contact

with the horizon till it ceases to rise. In the second example it will be found that the observation was made at
10* 48 19 s
.

But

in the case of a star (not a planet), instead of

making

a particular selection from the stars in the Nautical Almanac and then finding its time of transit, it is better to fix

upon the time, or rather upon the moat convenient interval,


and then seek in the almanac for the stars which pass the
meridian in that interval, making our selection from among
them. Thus, Buppose it were required to find what Btars
will pass the meridian of the ship on April 11, between 8

Digitized

B/Coogle

EXAMPLES OF FINDING LATITUDE.


Adding 8 h

and 10 o'clock in the evening.

117

to the B.

A.

we get the E. A. of the ship's meridian at 8 h


and adding 2 h more, we get the R. A. of the meridian at 10 h p.m. The stars whose E. A. lie between these
limits are those required.
If the sum exceed 24h the
of the sun,
p.m.,

excess is the R. A.

of the meridian.

On

the day pro-

1
m 42 s hence, the R. A. of
is V 18
h 18 42, and
lies between 9
42".
Within these limits the Nautical Almanac
a Hydra, 0 Ursaa Majoris, t Leom's, it Leonis, Regu-

posed, the sun's R. A.

each of the required stars

ll b 18
gives

Ac.
The learner need scarcely be reminded that the sun's

lus,

R. A. at Greenwich noon is not precisely the same as his


R. A. at any other Greenwich date but as the sun's mean
m a day, it would be needis only about 4
change of It. A. in the present inquiry.
;

motion in E. A.
less to allow for

When
altitude,

the horizon

an

is

too obscure fur the observation of an

artificial horizon is

sometimes employed.

This

consists of a shallow trough of quicksilver, protected from

wind and weather by a

glass covering or roof.

The observer

placing himself at a convenient distance from this, so that

the object and the reflected image of it may both be distinctly seen, the angular distance between the two is taken
and since the angular distance of the image below the horizontal plane

is

the same as that of the object itself above that

plane, the instrument, corrected for index error, will give

double the

altitude,

hence, dividing by
will

and there
2,

will

be no correction for dip

the apparent altitude of the object

be obtained.

We

Bhall now give a few examples for exercise in finding


the latitude from a meridian altitude of the sun or a star.

Examples for Exercise.


1. April 27,

1858, in north latitude, and in longitude

67* 42' W., the observed meridian

altitude of the

sun's

DigiiizM 0/

Google

lis

EXAMPLES FOB EXEECIBE.

s
lower limb was 48 42' 30* (zenith N.), the index error was

+ 1' 42",

feet : required the


Ans. latitude, 55 30' 50" N.

and the height of the eye 18

latitude ?
2.

August

14, 1858, in north latitude,

aud in longitude

51

the observed meridian altitude of the sun's upper


limb was 47 20' (zenith N.), the index correction was

~ V 47", and the

height of the eye 20 feet

Ans.

latitude ?
3.

Nov.

8,

required the

latitude, 57 15' 59"

1858, in south latitude, and in

N.

longitude

02 E., the meridian altitude of the sun's lower limb was


57 12' 30" (zenith S.), the iudex correction

and the height of the eye 30

feet

Nov. 21, 185S, in north

+ 1' 30",

required the latitude

Ans.
4.

was

latitude, 49" 7' 56" S.

latitude,

and in longitude

165 E-, the meridian altitude of the Bun's lower limb was
observed to be 47 38' (zenith N.), the index error was

1' 15",

latitude P

and the height of the eye 17 feet: required the


Ans. latitude, 22 21' 43" S.
'

5. March 2, 1853, the meridian altitude of Arcturus was


observed to be 47 24' 30" (zenith N.), the index error was

2' 10",

and the height of the eye 17

feet: required the


Ans. latitude, 62 37' 41" N.

latitude ?
6.

March

Hydra

12, 1858, the meridian altitude of a

was observed to be 39 24' 30" (zenith N,), the index error


2' 10", and the height of the eye 17 feet: required
was

the latitude

Ans.

latitude, 42 40' 4"

N.

July 10, 1858, the meridian altitude of Fomalhaut


was observed to be 63 38' 30" (zenith N.), the index cor7.

rection -was

2'

30",

required the latitude

8. April 17, 1858,

and the height of the eye 24

feet:

Ans. latitude, 3 52' 47"


in longitude

15

8.

W., the observed

meridian altitude of the lower limb of the planet Mars was


67 40' 30" {zenith N.), the index correction was
2', aud
the height of the eye 17 feet : required the latitude P

Ans.
9.

Juub

latitude, 12 24' 57"

13, 1858, in longitude 72 30' E., the

N.

observed

L'.'IJ

IC'J Cy

Cl

LATITUDE FROM THE MOON.

119

Vemia was. 30 4& 10"


was + 4' 20", and the

meridian altitude of the lower limb of


(zenith 8.), the index correction

height of the eye

24

feet

required the latitude ?


Ans. latitude, 35" 38' 0" S.

Latitude from Meridian Altitude of the

Moon

above the

Pole.

As

the declination of the

moon

varies

than that of any other heavenly body,

much
it

is

inoro rapidly

given in the

Nautical Almanac for every hour in the day, together with


the average amount by which

ceeding hour, that

ia,

it

varies in

10 of the suc-

one sixth of the whole variation during

that hour.

To
ia

what the moon's declination

find

taken,

we must

is,

when

her altitude

determine the Greenwich date, in

first

7nean time, of the observation

if

the

mean time

at the ship,

as well as the longitude, bo known, this of course

done

but

if

the ship's

is easily

mean time cannot be depended upon,

we must

then refer to the Nautical Almanac for the Greenwich time of the moon's transit over the Greenwich meridian,
and thence by means of the daily variation in the time of
transit, and the longitude, find tho ship's time of her transit
over the ship's meridian we shall thus get the time at the
when the observation was made, and thence, by means
;

ship

of the longitude, the Greenwich date of that observation.*

The Greenwich

date in hours and minutes being thus

* It may be as well
observation at sea

is

remark hero that tie Greenwich date of any


at once shown by the chronometer, provided conto

fidence can bo placed in its regularity.

Such

chronometers are now brought, that they may


for the determination of the
interval.

mean time

But an instrument of sudi

is

the perfection to which

in general

be depended upon

at Greenwich throughout a lon^:

delicate construction

is

vtvy easily

injured, and even variations of temperature will disturb in sonio degree

uniformity of action.

its

It is therefore considered as necessary to the safety

cf navigation to be provided

-ith

methods ef finding a

ship's position

on the

ocean, independently of the chronometer.

OigiiizM By

Google

LAIU'UBE

120
found,

we

I-liOM

THE

refer to the Nautical

iiUO>.

Almanac

for the

moon's

;it the hour, and correct it for the odd minutes


hy means of the " Diff. of Declin. for 10ro " before alluded

declination

to

the declination corresponding to the altitude will thus

he obtained.

We shall evidently get the


ship's meridian

ship's

time of transit over the

by applying to the Greenwich timo

of transit

over the Greenwich meridian the correction furnished by

multiplying the daily difference of time in the Greenwich


transit

by the longitude, and dividing the product by 360

the following Table, however, enables us to dispense with


this operation.
Table for finding the mi can time of the Meon't tratml over a given
meridian from, the titnt of the transit at Greenwich, and the
daily variation.

Daily Variation y the Tihb oi Greenwich Transit.

180

Although the above Table may he regarded as

sufficiently

accurate for the purpose intended, yet the learner

is

expect that the correction for longitude, which

gives

it

not to

by

Digitized By

Google

LATITUDE FEOM MOON'S ALT. ABOVE THE POLE. 121


inspection, has the
direct computation

same precision a3 if it were deduced from


that is, by multiplying the daily varia;

tion by the longitude, and dividing

by 360.

Certain tables

are absolutely indispensable in Nautical Astronomy, but

think
is

it

not

we

may be reasonably questioned whether the mariner


sometimes encumbered with a greater abundance

of this kind of aid than he really requires.

As

tables give,

in general, only approximations to the truth, the more


sparingly they are used, the greater will usually be the

The computation adverted

accuracy of the work.


is

too trifling and easy to render a table to Bupply

of much value ; and we insert

it,

as

it

to above,
its

place

occupies but

little

room, more in compliance with custom than from necessity.

Prom

the foregoing remarks, the learner will be prepared

for the following rule for finding the latitude of the ship

from a meridian altitude of the moon when above the pole.


Rule 1. From the Nautical Almanac, take out the
time of the moon's " Meridian Passage" at Greenwich on
the given day, as also the daily variation.
2.

From

table, or

the longitude by account, and the foregoing


by independent calculation, reduce the time of the
at Greenwich to the time at the ship when

meridian passage

the altitude was taken.


3. From the time thus deduced, and the longitude, find
the time at Greenwich when the altitude was taken.
[Note. These three precepts may be disregarded if the
chronometer at the instant of observation be consulted.]

4. The time at Greenwich being ascertained, take tho


moon's declination for the hour from the Nautical Almanac,
computing the correction for the odd minutes by aid of the

difference in declination for 10.


5.

From page III

diameter, increasing
tables.

meter

The

month take out the moon's semiby the " Augmentation" given in the

of the
it

correction for index-error, dip, and semi-dia-

will reduce the observed altitude of the limb to the

apparent altitude of the moon's centre.

122 LATITUDE FKOM MOON'b ALT. ABOVE THE POLS.


G. To the apparent altitudo of the centre, add tbo correcparallax in alt. minus refraction (See Table XVII.),
and the true altitude of the centre will be obtained. This
subtracted from 90 will give the zenith distance, which is
to be marked N. or S. according as the zenith is N. or S.
of the moon. Then as in the case of the sun, take the sum
tion,

and the declination

or the difference of the zenith distance

according as they have the same op different marks, and the


result will be the latitude.

Note.

The moon's

semi-diameter and horizontal paral-

lax are given in the Nautical

midnight

Almanac

for every

noon and

the corrections for any intermediate Greenwich

date may be easily estimated by taking the differences, and


then the proportional part of each difference for the number

noon or midnight.

of hours after

Examples : Meridian Altitude of the Moon above

the Pole.

1. May 17, 1858, in longitude 49 W., the meridian altitude of the moon's lower limb was observed to be 47 18'

30" (zenith

S.),

the index correction was

height of the eye 20 feet


1.

For

mean

the

Tinieatahip
Long. 49"

W.

in time

when

Itie

4* 20-5 Daily

+7'7

when alt. was taken

W.

Greenwich date of observation

1' 40",

4"

altitude

diff.

1"

xu

28

taken.

5S-4
84

Long, (reversed)

2256

+ 3 18
.

and the

required the latitude.

time at Greenwich

Moon's transit at G., liny 17


Cor. for long. 49

608

44 m

36,0)276,4(7"* frsor.

252
244
2.

For

the

Moon's Declination at 7

Declination May 17, at 7 h 23 EC 40"-7N.


-7' 8"-25
.
Decrease in 7 b 44

Dechkatiok

at 7* 41"'.

23 49' 32" K.

44 m al Greenwich.

Diff. in

10, -97" '33

i-i

38932
3893
428"

25

7'S *-25
,

Digitized By

Google

LATITUDE PKOSI MOOH'S ALT. ABOVE THE POLE.


3.

For

Moon's Hoi: Semi- diameter and Hor. Parallax at

the

Semi-din. at noon.

10'

19-7

DUE inl2*- 6"2 Hor.

~*
Son-DIA.

alT*~

in 8"

16-16-

Oorrtlonfor7

For

4.

Observed

"8

ln4

"1

ike Latitude

of Moon's L. L.

alt.

Index and dip


Semi-diam.

12"

~ 15

-"I

2'SI
'47

IS"'

of the Skip.

.47"

44"

'

+13

44

47" 32' 14"

18' 30"

-i

"J

Correction of moon's altitude

'-fi

1125

60' 35"

1'

+ i 6 ">S"^

App. altitude of moon's centre

+33' 56"

....

True altitude of centre

123

V 44'".

S9' 47" Diff.

dr. for? 44-

16' 16"

Augmentation

Hon.P.atTH4 m

4"

par.

S'l

inlj"

Zenith distance

48 11' 10"

Moon's declination

41 48' 50"

23" 49' 32"

17 59' IE"

S.

1S58, in longitude 60 42' "W., the observed


meridian altitude of the moon's lower limb was 30 30' 40"
2. Oct. 4,

(zenith N.), the index correction was

height of the eye 16 feet


1.

Moon's

For

tke

mean

transit at

<?.

Time at ship

that

in,

Oct.

21

52'8

+4

2-8

JJiff.

46-4

C0J

2784
23-2
36,0)2 80,

lie

to the degree

7" *8

eor.

and the

of nicety here

Greenwich date be taken to the

multipbed by
0''2 x

-2 (

horizontal semi-diameter

computed

It will be quite sufficient if the

nearest half-hour, the "Diff."

252

lfifi--6

* These small corrections for the

by 12; thus: 0 '-2 x 8+12

altitude teat taken.

Longitude.

25 6-6

horizontal parallax, need not

observed.

trftcn the

+ 7'8

long. 60 42' W. in lime


Greenwich date of obserr.

required the latitude ?

21 s 45 ,u

W.

Ora maick

time at

Oct. 3

Cor. for longitude 60 42'

and the

-f 5' 42",

it,

and the product divided

| = 4 "; und22"-5 x -| =

15".

Digitized 0/

Google

124 LATITUDE FBOM HOOK'S ALT. ABOVE THE POLE.


2.

For

the

Moon's Declination

Dcclin. Oct. 4, at 2 h

Decrease in 4 B, 4*

Decmn. atl h

55'" C

Oct. 4, at 1*

55 m, 6

7 43' 4S" -3 N. Diff. in lO

ffwemeieft.

<ti

- 15S"

9"'7

+ 1'

7 49' 58"

'5

'44

6340

N.

834

W"-U = l' 9 n -7
3.

/"or tlteMoon'i

Hor. Semi-diameter and

li'W

Somidiam. at noon

2h

Jlor.

3" o

DuT. in 12*

Observed

alt.

For

15' 56"

alt.

8'

j
(

48"

1'

56 m .

DifT.

2"'l

30 30' 10"

lS"-r

2"-l

30 48' 30"

+ ]&

+17' 50"

of moon's centre

Correction of moon's app. altitaile

Trne altitude of centre

6fri8"*5

Latitude of the Ship.

of Moon's L. L.

Index and Dip


Seinidiam.

Augmentation
Apparent

tlie

5S'B0"'G

1 5'

4.

Parallax at

Hor. P.

H. P. at

~S

ia**-

-n

Seuidiau. at

+47' 50"

25''

31 36'

90"
Zenith distance

Moon's declination

Note.

Prom

the foregoing examples tbe learner will

perceive that the principal object of step 1 in the operation,

that

is,

of finding the Greenwich date,

is

to enable us to

get the declination with the necessary accuracy at


instant the altitude was taken.

* The Greenwich date, lh E5 IOi 0,

is

As

Greenwich date

bo

the declination

4 m -4 short of

declination decreases as the time increases,

it is

<*

less at

i!

1'

tbe

may

and as the
than at the

that the correction of the declination at 2 b , for the

preceding 4""4, must be added.

OigiiizM Dy

Google

J1LANK
increase or diminish
it

POEM FOE THE MOON.

by

so

much

125

10 m of time,

as nearly 3' in

evident that the Greenwich date of the observation

is

should not err by

more than

two minutes of

a minute or

may

This date, as already remarked,

the truth.

in general

be got more readily, and with greater precision, from the


chronometer than from the longitude by account. Indeed,
the longitude by account, should not be employed in this

problem, unless

be known to

it

by

differ

than 30' of the

less

truth.

In step 3 of the operation there is no occasion for much


Greenwich date indeed, the correction for
always be roughly allowed for by a glance at the 12 h
by the Nautical Almanac, without

precision in the
ifc

may

differences furnished

formally computing for

as above

it

altogether as being of but


is

little

Blank Form far


1.

Transit at G.

Cor. for long,

the

The following

Moon.

Daily di

Degrees of long.

Longitude in time
G. date of obs.

.*

."

Dcelin. at above/four

Cor. for mimiies

36,0) ....(..'" Cor.

Ship's date of ob.

2.

often neglected

it is

moment.

the blank form of the necessary operations.

..

..

Dim

in 10 m

Minutes in G.d(rtc x

Deolim. at G. date

3. Moon's Semidiam. from Saut.

Aim.

To be

"(divided bylOf)

"Cor.for minute a.

corrected for the G. date

by inspection.
Moon's Hor. Parallax from Naiit. Aim.

To be corrected

for G. date

by

inspection.

* This

Tbat

may
is,

be got from chronometer.

remove the decimal point one place to the

left.

DigiiizM by

Google

EXAMPLES FOB EXERCISE.

126

.'

Observed altitude (L. L. or V. L.)

4.

Index and dip

Semidiara.

+ Augmen.

A]ip. alt. of

moon's centre

.'

....

." J

Correction of app. alt. (Table XVII.)

True altitude of moon's centre


90
True zenith distance

Moon's declination at G. date

Latitude

Examples for Exercise.


1. Aug. 30, 1858 * in longitude 129 Z( B., the observed
meridian altitude of the moon's lower limb, was 41 10'

(zenith N.), the index correction

height of the eye 18 feet

was 3'

40",

and the

required the latitude ?


Ana. latitude, 67" 7' 6"

N.

2. Not. 25, 185S, in longitude 22" 30' "W., the observed


meridian altitude of the moon's upper limb was 72" 12' 30"

(zenith N.), the index correction was

height of the eye 20 feet

2' 10",

and the

Ans. latitude, 40 47' 29" N.


Nov. 29, 1868, at 8 h 46m a.m. Greenwich mean time,
shown by the chronometer, the observed altitude of the

3.

as

required the latitude

moon's lower limb when on the meridian of the ship was


38 15' (zenith N.), the index correction was
2' 10", and

the height of the eye 20 feet


4.

required the latitude ? f


Ans. latitude, 50 9' 24" N.
Nov. 16, 185S, in longitude 82 30' E., the meridian
:

* The time of the moon's meridian -passage at Greenwich on August 29,


is

16h 107, which, according to

The learner

a.m.

time

is

See the

employed

work

of ex.

civil

reckoning, is Aug. 30, at 4" 10"-7

will not forget that

on shipboard the

civil

reckoning of

in the Nautical Almanac, the astronomical reckoning.

p. 123.

In this example the Greenwich date of the observation is given, namely,

Nov. 28, 20* 46.

Digitized by

Google

LAT.

TBOM MEE.

altitude of the

ALT.

BELOW THE POLE.

127

moon's lower limb was observed to be 64


+ 6' 40", and the

48' (zenith S.), the index correction was

height of the eye 22 feet

required the latitude ?

Ans. latitude, 24 42' 58"

S.

6. Dec. 13, 1858, in longitude 58 45' E., the observed


meridian altitude of the moon's upper limb was 43 25'

was

(zenith S.), the index correction

height of the eye 24 feet

6.

+ 5'

24",

required the latitude

and the

Ans. latitude, 48 24' 5" S.


Dec. 17, 1858, in longitude 18 42' "W., the meridian
was observed to be 52

altitude of the moon's lower limb

35' (zenith N.), the index correction was

height of the eye 25 feet

3' 40", and the

required the latitude ?


Ans. latitude, 59 5' 1" N.

Latitude from a Meridian Altitude below the Pole.

The sun

is

on the meridian of any place below the pole


is, 12
after apparent noon at
by the longi-

at apparent midnight, that

that place

'

so that 12 b increased or diminished

tude in time, according as the place

is

W.

or E. of Green-

wich, will be the'apparent time at Greenwich

that

is,

the

Greenwich date of the observation: the declination at this


time is found as in the examples already given, from the
noon-declination in the Nautical Almanac.
For a fixed star the change of declination in 12 k is insensible, bo that the declination will be the same as that given
in the Nautical Almanac.

For

a planet the declination varies sensibly in 12h, so

that, as in the case of the sun, the variation

must be allowed

for.

In the caBe of the moon the ship-time of transit over the


is to be found as in tho
foregoing examples this time increased by 12 h and by half

mid-day portion of the meridian


:

the daily difference of time

be the ship-time of her


passage over the opposite portion of the meridian that is,
of her meridian passage below the pole. The proper corwill

OigilizM Dy

Google

EXAMPLES.

128

reetion of this time for longitude being

then, made, the


Greenwich date of the observation, and thence, by aid of

the Nautical Almanac, the declination at that date,

The rule,

be found as before.

Bulb

1.

2.

Apply

and thence

is

to

therefore, is as follows

Find the declination of the object

of observation,

at the instant

its polar distance.

to the observed altitude the proper corrections

for obtaining the true altitude.


3.

To the true

altitude

add the polar distance

name as the
"When above the same the
object

will be the latitude, of the

Note.

pole,

the Bum

declination.

rises till it arrives

at the meridian, when, having attained its greatest altitude,


it

begins to descend

when below the

the object descends lower and lower


meridian,
to ascend.

when having sunk


It

to

its

pole,

on the contrary,

till it

arrives at the

lowest altitude

it

begins

only by Beizing the instant at which the

is

object appears stationary that its arrival at the meridian can

be detected at sea

but

may be as
may not be

it

rigorously speaking, this

meridian transit after

all

for it

well to notice that,

the instant of the

must be remembered

that,

besides the motion in altitude, there is also a motion in


declination, so that it

may happen, especially in the case


may cause the altitude

the moon, that this latter motion


be the greatest or least a

meridian passage.

little

before or a

little

of
to

after the

With regard to the sun and planets,


no moment but under particular

this circumstance is of

circumstances the meridian altitude of the moon, as fur-

nished by observation, on account of the rapid change in


declination of that body,

may

actually on the meridian

by

is

differ

1'

or

2'.

from the altitude when


The moon, therefore,

the least eligible object from which to deduce the

lati-

tude.

Examples

.-

Meridian altitude below the Pole.

July 2, 1858, in longitude 23 10' "W., the observed


meridian altitude of the sun's lower limb when below the
1.

Digitized By

Google

BELOW THE POLE.

ITER. ALT.

129

pole or at apparent midnight waa 7 40', the index correc-

+ 3' 20",

tion was

and the height of the eye 19

feet

required the latitude ?


For

1.

G. Noon

Declination at the

tlie

declin.

July 2

July 3

Or.

Midnight

declin.

.23
.22

3'

8" -5

23

34"

1'

16''

W.

Cor, for longitude

iii&taiit

52" -2 N.

3'

59' 16" -8

2)46"
July 2

18"

23

Declin. at instant of oba.

of oVse'vaiion.
1"

Diff. in

For

1533

H.

230

dip.

Semi -diameter
alt.

58'

6ti

44"

the Latitude

alt. Bun's L.

Index and

App.

2.

Observed

alt.

0'

Cor. of dec. for long. 17" -63

of the Ship.
0"

7 40'

57" 1

15' 46" J

7 54' 49"

of centre

EefrnctionParallax

True

"5

DO"
Polar distance

11"

N.

30"

6'

7" 4S' 13"

of centre

06 58' 44"

Polar distance

3" N.

74 47'

Latitude

April 10, 1858, the observed altitude of the pole-star


meridian below the pole was 41 36', the index

2.

when on the

4' 10", and the

correction was

height of the eye 17 feet

required the latitude?


Obs.

alt.

of pole star.

Indexcor. 4' 10"1


.-4' 4" J"
Dip.

41 36'

0"

16' 14"
'

App. altitude
Refraction

True

altitude

Polar distance

.... 41
....
.... 41
....

LiincDE

52' 14"
1'

5"

51'

9"

1 26' 44"

.43

18' 53" IT.

Declin. Ap. 10

S833'1C"N.

^
Polar distance

1" 26' 44"

130
3.

May

15, 1858, in longitude

altitude of the moon's lower limb

if 25'

1.

difference

My 15 2M2 ra -4

....

Moon's lower transit

Corrections for long. E.

Ship's date of observation

Longitude in time

Moon'sdeclin.atl2''
Correction for 7m

Polar distance

3.

Moon's nor. Semidiam.

i.

Observed

alt.

moon's

Index and Dip.

Semidiam.

L.

12,

31
b

7 m (Maybe got from Chronom.)

23 14' 19' N.

61 45' 41"

May 15

at

12 k

+ I8"'S5

+13-105

in

-7

16' 55". Hor. Par. 60' 42".

38"

.....

Polar distance

Lajhums

Note.

In

the horizon

QH

39'

9"

....

10

33' 30"

61

45' 41"

72

order that a celestial object

when

0"

+54' 21"

of moon's centre

it is

latitude of the place

25'

"

App. ah. of moon's centre


Correction of app. alt

Tme alt.

10-

Cor. for 7'.

Diff.

L
2' 29"

10

36,0 ) 264,0 {""cor.

+ Ang. +16'

+ 66 ra -l

Degrees of long.

14 45

2SM4' 6"-N.
+ 13"

Declin. at G. date

33

Dailydiff.

14> SB*

Greenwich, date of observation


2.

42' B., the observed

required the latitude ?

Moon's upper transit G.

Half daily

3r

when below the pole was


8% and the height of

+ 2'

the index correction was

the eye 22 feet

below the pole,

it

19' 11 "IT.

may be above

appears that the

must exceed the polar

excess being the true altitude of the object.

distance, the

On

account of

the varying state of the atmosphere near the horizon, the


refraction for altitudes below six or seven degrees, cannot

be estimated with accuracy. And as the polar distance


of the sun is never much less than 67, and that of the
moon never much less than 62, it follows that for a meri-

OigitizM by

Google

TAT.

ehom

alt. of

the pole stab.

131

dian altitude of 6 or 7 the latitude, in the case of the


pole, must not be leaa that 73 or 74, and in

aun below the

the case of the moon, not leaa than 68 or 69. Hence, such
meridian observations on either of theae two bodies are

and are therefore not generally


but in the case of the fixed stars, oppor-

restricted to high latitudes,


available at sea:

tunities occur in

all

below the pole

latitudes of getting a meridian altitude

sufficiently great to

allow of the tables

of refraction being used with safety. As the pole-star is


always above the horizon in latitudes north of the equator,

and on

cloudless nights

is

always sufficiently

easily recognised, it is the star

finding the latitude at sea,

Some

more frequently

when

visible,

and

selected for

north, than any other.

short useful tables are given in the Nautical

Alma-

nac, (pp. 527-9) for finding the latitude from an altitude


of the pole-star, whether it be on the meridian of the place
of observation or not the following is the rule there given,
:

(p.

568) with an example of its application.

To find

Rule

1.

the latitude

From

from an

Pole Star.

altitude of the

the observed altitude,

when

the error of the instrument, refraction, and


2' the result is the redueed altitude.

corrected for
dip,

subtract

Beduce the mean time of observation at the place to


the corresponding sidereal time, by the table at page 530,
^Nautical Almanac.
2.

3.

"With the sidereal time found, take out the first correc-

tion (Naut. Aim., p. 527), with its proper sign.

If the sign

be +, the correction must be added to the reduced attitude but if it be


it must be subtracted
in either case
the result will give an approximate latitude.
4. With the altitude and sidereal time of observation,
take out the second correction, (p. 528) and with the day
of the month, and the same sidereal time, take out the
third correction (p. 529). These two corrections added to
;

the approximate latitude, will give the latitude of the place.


Example: March 6, 1858, in longitude 37 "W., at 7

1'

132 LAT.

FItOSI

DECLINATION ALT. AND IIOUB- ANGLE.

43 m 35% mean time, suppose the altitude of the

when
and

pole-star,

corrected for the error of the instrument, refraction,

dip, to

be 46 17' 2S"

Ship mean time


Long. 37

W.

Greenwich mean time


Sita-cnl time at

Mean time

in time

required the latitude ?

7h

43 01 35'

23

10

11 36*

11

Greenwich mean noon

at ship

Acceleration {page 530, Naut. Aim.) for 10

"

I2 m

11

Co 42*

43

6*1

40*. 63*

35
41

46 15' 28"

Reduced altitude

With Argument

22

Sidereal time of observation

fl*

4G m 58',

First Correction

Approximate latitude
40" 17'

Arguments,

Arguments, March

0,

41

6h 41 ra

10'

7"

iC

5'

21*

+1'

(j' 1

{Second

Correction,

Third Cmreetion

+2' 31"

To find the Latitude when the Declination, Altitude, and


Hour-angle are given.

The hour-angle

of a celestial object at any place and at

any instant is the angle


at the pole included be-

tween the meridian of


the place, and the meri-

dian through the object


at that instant.

In the

let 7, be the zenith of the place, P the


and S the object observed, then Z S co-

annexed diagram,
elevated pole,
altitude,

PS

= polar distance, P ~ the hour-angle, and P

7,

the co-latitude of the place.

To

find this last quantity the throe former are

to be given, so that the solution

may be

effected

supposed

by case 2

of oblique-angled spherical triangles (see Spherical Trigo-

nometry,

p. 19.)

triangles

is

But the

the more easy.

following

method by right-angled

FOttMTJL-E FOlt

Draw S M, perpendicular
tbe oblique-angled triangle

wo have:
From the triangle P
P S, P M, for adjacent

Trig. p. 11.)

and

cos

P = cot PS

And by

P II, S

fern

PM

*,

CALCULATION.

133

to the meridian of Z, dividing

PZS,

PM8, ZMS:

triangles

two right-angled

into the

then by Napier's Eulea (Sph.


S,

by taking

for middle part,

parts,

tanPM=waPtanPS .... (I)


P S for middle part, and

taking the hypotenuse

M for opposite

parts,

cosPS= coaPMcosSM .... (2)


ZMS, by taking the hypotenuse

Again, from the triangle

ZS

for

middle part, we have


rosZS

= cosZMcosSM ....

Consequently, dividing f2) by (3),

SS - sUS

we

(3)

have,

- ZK - - PH z8 ~ ps

<*>

being determined from equation (1), and


Hence, P
then Z
from equation (4), the sum of these or their
falls between P and Z, or not,
difference, according as

will give

7i

Bringing equa-

the co-latitude of the place.

tions (1) and (&) together, the formula? for computation are
therefore,

tanPM =
cos.ZM

cos hour-angle x cotan declination


cob

F M x sin alt.

x coseo declination

!(>

If the object observed off the meridian he the sun, the

hour-angle
is,

is

the apparent time from the snip's noon, that

.from the sun's passage over the meridian, converted into

degrees.
The chronometer gives the mean time at Greenwich of the observation, and we thence find, by help of the

longitude by account, the ship's


affected only

mean time

by the error of longitude.

by applying the

of observation,

This

mean

time,

correction for the equation of time, given

in the Nautical Almanac, becomes converted into apparent


time, and thus the time from the ship's apparent noon, or

the hour-angle in time, becomes known.


But if the object be other than the sun, we must add the

DigiiizM by

Google

ALTITUDES NEAE THE MEBTDIAB".

134

sun's right ascension at the instant of observation, to the

apparent time after the ship's preceding noon, the Bum, or


excess above 24h , is the B. A. of the ship's meridian

its

the difference between this and the right ascension of -the


object
It

is

obviously the hour-angle in time.

proper

is

observe that in determining the latitude

^to

by computing the formula (A), there may be a doubt as


to whether the point
would lie between the zenith and

sum or

the pole or not, and consequently as to whether the

P M, Z M

difference of

is

the co-latitude; but in general, the

by account must bs near euough to the truth to


hesitation on this head.*
If, however, the object be near the meridian, this source
of ambiguity may be always avoided, and there is one additional reason for preferring an observati6n near the merithe higher the object
dian to one more distant from it
latitude

remove

all

observed, the less likely is the refraction to be disturbed

from

its

mean

of the sun,

it

In preparing for a meridian altitude

state.

sometimes happens that although an observa-

tion can be well taken a few minutes before or after noon,

yet that the sun becomes obscured by clouds

on the meridian.
the latitude

It

may be

is

When

when

actually

very useful, therefore, to know

how

obtained from an altitude mearthe meri-

dian: a rule for this purpose

maybe investigated

M is bo small that whether added to,

the result, in either case, differs so

little

as follows;

or subtracted from

P M,

from the estimated co-latitude, that

there seems no sufficient reason for preferring one totho other, the circumstance can occur only
is,

when the body is very near the prime vertical, that


W. And when P M is bo small as to make but

nearly due E. or due

little difference

whether it be added

when the body

stance can occur only

is

from Z M, the circum-

very near the six o'clock hour

Except in one or other of these positions the observation

circle.

made

to or subtracted

and the latitude deduced with accuracy.

however, when the


vai- lvi'I

iiiation is

the ship

J.il

itu-ic

and

dec-'

\r,r,

lien arc of

may be

It is necessary to notice,

mime?,

c-'iiiiriiry

then measured from the depressed pole P'

ZM 90 is the

distance of Z above the equinoctial, that

is

uo that

Ui.n,

the

P'M +

the latitude of

and in IhU case there can never be any ambiguity.

Digitized 0/

Google

ALTITUDES NEAR THE MEfilDIAS.


Keferring to the triangle

ZPS,

the fundamental theorem

of spherical trigonometry gives,

EinPZsinPScosP

eoaZS^eosPZcosPS

coaPZcosrg
- cobZS
sin p Zsjnps

.-.COB

Let z represent the zenith distance that S would have


meridian, and let 2/ be the difference

when actually upon^ the


between

tance

this meridian zenith distance,

and the zenith

meridian found by observation

8, off the

ZSrs+y, the above equation


=

Ma

~~

cos

<?

"

+ *)
sin

dis-

then since

is,

cosPZcobPS

PZ

sili

PS

Hence, subtracting each Bide from 1, and remembering


1 coa P = 2 sin"i P, we have,

(Plane Trig. p. 27J, that

m
.

Wow

HmPZsinPS + coaPZcos PS cob(3 + ^)


dn PZ in PS

**~
-OB(FZ

the difference

and the polar


distance

g,

co-lafcitude

distance,

fZ+ '0

rpZ akPS
P Z ~ P S,
is also

As

the object S

is

co-latitude

to the meridian zenith

the difference between the

and the polar distance

r
2ip-f?i 3 cas(z +z _

(^Tri e.p.2G).

between the

must be equal

because that

therefore,

cos 2

am

coa a cob

smz'

near the meridian, and as objects near

the meridian usually make comparatively but slow advance


in altitude, the difference z", between the meridian zenith
distance,

and that actually observed, may in general be

considered as

cos

sf

sufficiently

as equal to 1,

.*.

sin

small to justify our regarding

and thus writing the above equation,

= 2 sin PZ sin PS cosecz sin

Now the
in it

ia

arc z' being very small, the number of seconds


very nearly equal to the number of times sin 1" is

contained in sin

z"

consequently

we have

very nearly,

136

IjAT.

fkom

So. of seconds in

=
In

this

way the

neab the MERIDIAN.

alt.

z'=-~~i~BW PZ
.

sin

PS

cosecz sin-^

C0E deolin, cosec 2 sin 5 \ hour-angle.

TTT008

correction z', or the

number of seconds

to be applied to the observed zenith distance off the meridian, to reduce it to the zenith distance

on the meridian,

may be obtaiued. But this zenith distance z must be


known approximately before the formula can he used it is
;

supplied by the latitude by account

if this

do not

differ

from the true latitude by more than about 15', the true
latitude itself will be deduced with tolerable accuracy.

The method, however implied in the formula? (A), is the


more correct, though the work by those formulae requires
more references to tables. But if the corrected latitude,
furnished by the method just discussed be used in place of
the latitude by account, and the operation performed anew,
the second result will have all the accuracy desirable and
;

from glancing at the formula,

it will

be seen that, as

all

the elements of the computation except cosec

lat.

are the same in the

will

be very

trifling,

We shall

now

s, and cos
two operations, the additional work

as the following example further shows


give the result just obtained in the form of

a practical rule.

Latitude from an Altitude near the Meridian.

Since the logarithm of

is

5-615455, the formula

Binl

above expressed in words furnishes the following rule.

Rule

1.

To the

declination of the object, add the

tude by account when one


such

is

is

N and the other S

2.

lati-

but when

not the case, take the difference of the two

result is the meridian zenith distance

the

by account.

If the object he the sun, the apparent time from noon

the hour-angle. For any other object


A. at the instaufc to the apparent time since
the preceding noon, the sum, or its excess above 24 h is the

in degrees, &c.,

add the sun's

is

It.

Digitized by

Go

SUN KEAQ THE MEB.TDIAN.

137

B. A. of the ship's meridian. The difference between


and the It. A. of the object is the hour-angle.
3.

Add
1.

together the five following logarithms

this

The constant logarithm, 5 615456.


by account.
1

2.

log cosine of the latitude

3.

log cosine of the declination.

mer. zenith

4. log cosec of the

deduced from

dist.,

the latitude and declination.


5.

2 log

The sum of
is

the log of a

sin of half the hour-angle.

these logs, rejecting the tens from the index,


of seconds called the " Seduction,"

number

which subtracted from the true zeuith distance off the


meridian, gives the true zenith distance on the meridian.
"When this and the declination are of the same name, their
sum, when of

names, their difference

different

is

the latitude,

of the same name as the greater.

As

Note.

fractions of a second are disregarded in the

reduction, the logs used in finding

it

need be taken from

the tables only to the nearest minute.

Examples : Sun near

tlte

Meridian.

1. In latitude 56 40' N. by account, when the sun's


was 14 12' N., at 0h lGm p.m., apparent time,
the sun's true zenith distance was 12 4& N. required the

declination

latitude ?
Lfttitiirk

bj

])ec!i nation

Mer.

acct.
.

sen. dist. acct.

Half honr-angle

56 40' N.

OK

0-730975

I412'H.

CM

0-086523

Zenith dist. from ohs.

Cor. mer. zenith dist.

IM'liiintii'Ti

CO eo 10-170593
17-085638

42 28'
2

GO
Reduction

0'

396"

2-598184

log

0' 38'

42 40'

0'

5-615455

N.

50" 45' 24" H.

10-169903
'

Corrected latitude

9-738820
0-986523

42" 33' 24 N.
0" N.

14" 12'

17'085638

2-E96339

Digitized By

Google

SUIT ITEAE

138

The work on the

right

THE SEEEIDIAS.
a repetition of the operation

is

above, substituting the computed latitude for the latitude

by account

mer. zenith
increased

and as the former exceeds the

by account,

dist.

by

5'

that

is,

latter

by

5',

the

in the first operation, becomes

it is

42 33'.

395" differs from the former by


is 56 45' 25" M".

1",

As the

reduction

the more correct

lati-

tude

We shall

now

exhibit the

work of the same example by

the formulas (A) at page 133.


1.

2.

Tan P

CosZ

M = cob honr-ang.

x cot dec.
4"

0'

cob

9-998941

cot

10-596813

Hour-angle

Deolin.

14 12'

75 46'

75 46'

oa

14 12'

osec

47"20'

in

PM
M=

cos

PM

Declin.

Altitude

ZM
M
P

.'.

M ca

...
.

...

Colat.

lan 10-595754
alt.

42 SI' 35"

cob

9-890708

10-610289
9-866470
S -867467

75 46'

33 14' SB"
90"

Latitude

56 45' 25'

As the latitude here deduced is exactly the same as that


we may infer that both results are strictly correct.
The student ia strongly recommended to familiarise him-

above,

self

with both these methods of finding the latitude from an

observation of the sun off the meridian, remembering that


first method is applicable only when the observation is
made near the meridian the second method is generally

the

applicable, except

under the circumstances pointed out in

The reason that the first method is


somewhat preferable to the second, when the object is near
iB that the seconds in the angles may he dis-

the foot-note, p. 134.


the meridian,

regarded in taking out the logarithms, or rather that each


angle

may be taken

to the nearest minute only.

But the

OigiiizM By

Google

SUN REAB THE MBBIDIAN.

139

second method will always furnish a satisfactory test of the


accuracy of the result deduced by the first we shall now
work out another example.
2. In latitude 48 12' N. by account, when the sun's
declination was 16 10' S. at 0h 20 ra p.m., apparent time,
:

the sun's true zenith distance was 64 40' N.

required the

latitude ?
5-615455

Constant log
Latitude by acct.

.-.

....
....

Declination

Mex. Z.D.acct.

Jhour-aagle

48 12' N.
16 10' S.

cos

64 22'

coaec

230'

60

9-823821

eos

9-982477
10-044995

2sin 17-279360

557"

log.

0'

Z.D. ods

64" 40'

Her. Z.D

64" 30' 43" K.


16 10' 0" S.

Dedin

....

Latitude

S".

48 20' 48" N.

5'615455

Constant log
Corrected

lat.

....
....

Declination

M.Z.D

-.

4 hour-angle

Reduction

Correction of kt.

The work of
as follows
1.

tan

2 7 4610 8

9' 17"

Reduction

this

48 21'

cos

10* 10'

coa

64 31'

9-822546
9-982477

cosec

10-044452

2 30'

2 sin 17-279360

555"

log.

2-744290

Hence the corrected latitude


is 48 20' 45" N.

+2"

example by the formulas marked (A)

is

M=

cos hour-ang. x cot dec.

... 5"
... .16 10'
0'

Hour-angle

Dedin.

PM
Z

.73

...

0"

coa

0"

cot 10-537758

46' 29"'

tan

64" 34' 15"$

138 20' 45"

This

9-99S344

loHzm02
is

found on next page.

See foot-note, page 134.)

90

Latitude

48" 20' 45" N. as determined by the former


method.

Digitized 0/

Google

SUH HEAE THE MEBIDIAN.

140
2.

Cos

ZH
PM

cos

Declin.

PM

cosec dec. ain alt.

ZM
As

9-446249

16 10'

0"

cosec

25" 20'

0"

sin

10-555280
9-631326

cos

9-632855

6i34'15"2

cos

7346'2y"i

....

Altitude

two examples the corrections have been supposed to have been applied to the observed, to obtain the
true zenith distance, and as also the hour'angle in time is
considered to be known, we shall now work out a final
example in which are given the latitude by account, the
longitude, the observed altitude, and the Greenwich mean
time, as shown by the chronometer.
3. August 21, 1858, a.m., in lat. 51 40' N. by account,
and long. 2 9' W., the chronometer known to be 3G 9, 2 fast,
showing ll 48 m 32 s Greenwich mean time, the observed
altitude of the sun's lower limb was 50 36' (zenith N.), the
index correction was
V 20", and the height of the eye 20
in these

1'

feet

required the true latitude


For

i.

the Hour-angle.

G. mean time by Chrorj.

11* 48" 32'

36

Error of Chroo

12 h + II

G. date, Aug. 20

Equa. of time
G. app. time,

Long. 2" 9'

W.

App. time at
.-.

2.

Gf.

date

47 56

11

in time

ship, a.m. Aug. 21

Hour-angle

59

30

36 21"

= 23 m 33*,

44 57

or 5 55'

For Declination and Equation of Time.

Declin. noon Aug. 21


Diff. for

A.H

V, 49""71

Deolin. at G. date

12'8' 56"-9 N.

.\ for
.

12

+9

12" 9'

Eq. of time, noon Aug. 21

7"

-9

N.

2 m 58"'58

Note. The G. date being bo near the noon of Aug. 21,


the correction for Equa. of time is inappreciable. When the

OigiiizM by

Google

SDN NEAR THE SIEETDIAN.


time from noon

Observed
Index

alt.

made only

Sun's L. L.

and

cor.

dip.

3'

+ 15'
Apparent

alt.

Itefractiou

True

for the nearest hour.

50" 36'

+ 12'

51" i

47"

of centre

Parallax

of centre

alt.

True zenith distance


4.

Constant

the correction by means of

is considerable,

the " Diff. for l h " need be

For

meridian

off

the Latitude

39 11' 55"

5-615455
51 40' N.

9792567

Declination

12*

9*990161

.'.

M.Z.D.

9'N.

33 31' N.

acct.

\ Hour-angle

of the Slap,

log.

Lat. bj acct.

2"

57'i

10-196333

17'42645S

17' 27"
39" 11' 65"!
,'.

Mer. Z.D.

Deolin.

Constant

log.

Corrected

lat,

5-615455

9798247

0-990161
Corrected Mer. Z.D.
i Hour-angle

Reduction

10-202066

17-425458

1055"

1047"
'

.*.

Corrected Latitude

Correction of
.

We shall test the degree of accuracy


formula (A).

51"

lat.

3'

27" N.

of this result by the

SUK NEAB THE MEBIDIAH.

142

by Formula (A), page 133. "

Work of preceding Example


1.

tan

M = cos hour-ang.
Hour-angle

PIT
Cos Z

12

M = cos P M co
PM
.

PM

9'

7"

cos

9-997680

cot

30-666895

tan 10 '664575

77" 47'

5"4

77" 47'

6"!
7"

cos

12"

Declination

9'

9-325481

cosec 10-G76738

5"

sin

9 '889280

38 50' 14*

cos

9-89H99

77 47'

Altitude

ZM

x cot dec.
0"

5" 65'

Declination

50" 48'

S'i

38 56' 5I"i

90"

TauB
"VVe see

IjatitiT'H

51

3'

8"i

this result, that the

from

with the latitude by account, so

former method, even

much

as nearly

37 miles

in error, gives the latitude true to within less than half a

computing the corrected reduction.


Note. If the sun did not change his declination, equal
altitudes, taken one before and the other after noon, would

mile, without

correspond to equal hour-angles, so

that half the time

elapsed between taking these equal altitudes would be the

hour-angle at either observation.


*

But on account of the

The most troublesome part of a logarithmic operation ia proportioning


computer will, in general, find the following the most

for the tecondt; the

convenient

mode of

proceeding, namely

Disregard the seconds, and enter

the table with the degrees and minutes only, hut against the log tahen out

write the tabular difference.

When

all

the logs with their differences have

been thus extracted, then compute in each case

the seconds, remem-

for

bering that for every co-quantity the proportional part will be tubtractive.

The balance of those corrections for the


with the sum of the logs from the tabic.

seconds

may then

As repuds

be incorporated

a; i'hi,iclit'(d C'j/upk-

menls, the corrections for co-quantities are to be added, in other cases they
are to bo subtracted.
table,

it

may be

After the differences are

all

extracted from the

well to put the proper sign against each, to prevent mis-

take as to the additive and subtracts corrections.

Digitized By

Google

BkAHJK

FOKM

SUN MEAB THE MERIDIAN.

143

change in declination, this method of deducing the hourangle cannot he employed with safety, except under certain
circumstances.

When

the latitude and declination are such

that the sun passes the meridian near the zenith, half the
elapsed time between equal altitudes, a few minutes before

and

a few minutes after noon, will give the hour-angle with

sufficient

circumstances, the

in these

accuracy, because

sun's motion in altitude

so rapid, that the correction in

is

due to the motion in declination,


and the hour-angle,

altitude,

in a very short time

passed over

is

if the elapsed

time between the two observations do not exceed about 30 m

may

In high

be safely inferred.

the sun's motion in altitude

i3

latitudes, however,

where

very slow, if the change in

declination be rapid, the hour-angles

on contrary

sides of

may be very unequal for equal altitudes.


"Whatever minute3 of latitude the ship may have moved

the meridian

from or towards the sun, in the interval of the observations,


should be allowed for in taking the second altitude.
We shall now give the blank form for the foregoing operations.

Sun near

Blank Form.
1.

For

the

the Meridian.

How-angle.

G. mean time by chron.

Error of chron.

G. date, mean time


Eqaa. of time G. date

G. apparent time
* long, in time

for

W. +

for E.)

App. time at Ship

* It must
correct

rcnicval.'C'.'cd

lh;it

whatever error there

is in

hour-;ingle

lie

when converttd
may be

ever, in this angle

the loagitudc litre


it,

is regarded.

assumed

to lie

there will he the same error ia the

into degrees
(1

and minutes

the seconds, how-

BLANK FOJtM

144

2.

declin. (Naut.

and

of Time at 0. date.

E-/.

Aim.)

Diff. for

l*

Time

minutes from Q. noon

ia

THE MEEIDIAN.

8U1T TtEAB

the Dcclin.

-Poj-

Soon

(...-

eo)
Decliu, at Q. data

= ..'
.

.'

Equa. of time G. preceding noon


Diff.

in l h x No. of hours since that noon

Equa. of time at G. date


3.

.For the

Observed

Index

True Zenith Distance

alt. (L.

cor.

alt.

off Meridian.

L. or U. L.)

and dip

.'

."1

....

Semidiameter

Apparent

of centre

Befraction Paral las

True

alt.

of centre

True Zenith distance

A.

For

off

Meridian

the lalilvde

of the Ship.
5-615455

Constant log
Lat.

.'.

by

cos

aoct.

Mer. Z.D.

cosec

| Eour-angle

2 sin

CO

"

log

Z.D. offmerid.

Corrected M.Z.D.
Declination

lb will be sufficient to take the above logs to

minute

as alao tboae following.

tlie

nearest

LATITUDE

SUIT

NEAB THE MEBIDIAN.

1-15

Corrected Lat.
Declination

Corrected

M.

2.

D.

i Hour angle
Corrected Redaction

The

between this corrected reduction and the

difference

former, applied to the corrected latitude, will give the teiie

LATITUDE.

Note.

The

term " near the meridian" must not be

considered as always implying the same limit of distance.

If in ex. 3 above, the latitude by account had been nearly


equal to the declination, that is, about 12" N., the hourangle employed would have been

For

it is

much

too large for safety.

plain that in these circumstances, the sun's motion

in altitude, even
his zenith distance

when very near the meridian, is rapid:


when on the meridian is small, but when

a few minutes of time, the zenith distance is conNow in the investigation of the rule, it is
siderable.
off it only

between the two zenith

dis-

so trifling, that the cosine of that difference

may

we

see,

assumed that the


tances

is

difference

be regarded as 1 without any error of consequence


therefore, that in the

method would
It

may be

circumstances here supposed, the

be objectionable.

further noticed, too, that as

a small error in

the hour-angle would correspond to a comparatively large


arc of altitude, a comparatively large displacement of the

make
and the true altitude agree and thus the latitude inferred
would involve appreciable error. But, as already remarked,
the erroneous hour-angle

pole would be necessary to

when

the sun arrives at the meridian too near to the zenith

for the present

method to be trustworthy, on account of

the reasons stated above, the hour-angle

with

sufficient accuracy

by equal

may

be deterrnined

altitudes carefully taken

before and after the meridian passage

half the

interval

of time between the two observations being the hour-angle.

GENERAL FOES I FOE LATITUDE.

As the

sun's motion in altitude

when near the meridian

obviously greater and greater as his

is

meridian zenith

must in the present proThe following short table will show

distance decreases, the hour-angle,

blem, be less and

less.

within what time of the sun's passage over the meridian of

the ship, the altitude "near the meridian"

may

always be

taken.
Sun's Merid., Zenith Disc, or Differenco of Lot. and Docli

12" Ot SB" D 20"

O 25"

0'

30" 0 S5= 0" 40"

60 0 h

55'

We shall now gire a blank form, which may be followed


whether the sun be near to, or remote from the meridian.
Blank Pobm. Sun
(g* The

near

to or

remote from the Meridian.

hour-angle*, declination, equation of time, and true

alt,

to be

found as in the last Form.

Then the sum


tude, when the
the

sum mvnui

or difference of

lat.

90

and

is

P M, Z

declin. are of the

the latitude

is

the co-lati-

same name.

when they

And

are of different

names.

As

by account
whether the sum or

already noticed at page 134, the latitude

will in general

difference of

If either
latitude

be guide

P M,ZM,

PM

sufficient as to
is

orZM

to be taken.

be so small that the error in the


it, then, and then

by account may equal or exceed

only, can there .be


difference of

* The hour-angle
equinoctial.

any doubt as to whether the sum or


be taken and we shall be

PM,ZM should

litre is, of course, to be computed to seconds of the


The declination and equation of time are takeu from page II
Almanac ; and the "Diff, for l n " from page I.

of the Nautical

SUN IN ANY POSITION OFF MEEIDIAN.


apprised
is

117

our observation has been made when the sun

tbafc

too near the six o'clock hour-circle, or too near the prime

But without any reference

by
we know the position of the sun in referbls o'clock hour circle and prime vertical, all
may be removed by the following simple consinamely
^No two perpendiculars to a great circle of the sphere can

vertical.

to the latitude

account, whenever

ence to the

ambiguity
deration,

cross each other except at the distance of 90 *

When

1.

the Latitude

hence,

and Declination are of the same


name,

If the six o'clock hour

circle,

and the prime

vertical,

be

both on the same side of the sun, the difference between


F M, Z must be taken the result is the Co-latitude.

If the sun be between the

prime

vertical, the

result

2.

is

sum

of

six o'clock

hour

PM, ZMmust

and the

circle

be taken: the

the Co-latitude.

When

the Latitude

and Declination are of

different

names,

The sum
nished by

of

if,

90, is the

must be taken the result, dimiLatitude. [The sun, in this case,


;

arrives at the prime vertical before

it rises.]

Should the sun be actually upon the

then

should
0,

and

M will be
it

six o'clock

hour

circle,

0, and Z
will be the co-latitude.
And
be actually upon the prime vertical, Z
will be

M will be the co-latitude.

cosine of the hour-angle will be 0


cos hour-angle

In the former

case, the

in the latter,

= tan declination -r tan latitude.

"We think, with these precepts and directions, the mariner


can have no difficulty in determining his latitude from a
* Perpendiculars
of the horizon

to the meridian all intersect in the E. and

W.

point

these points are, therefore, the poles of the meridian.

2
Digitized Dy

Google

EXAMPLES: OBJECT OFF MEEIDIAIf.

148

when

Single Altitude off the meridian,

hia time is pretty

accurately known.

method juafc discussed,


no restriction as to whether the sun be near the
meridian or not; nor, being near the meridian, whether
it be near the zenith or not.
"When near the zenith, it
must, it is true, also be near the prime vertical but on
which side of the prime vertical, will be ascertained "by
Ib will be observed that in the

there

is

W.

noticing on which side the B. or

taken

and

also

point the altitude

by noticing that the motion

is

in altitude is

when the body is crossing the prime vertical.


Whether the sun be near to or remote from the zenith, the
observation should always be made when there can be no

quickest

doubt as to on which side of the


of the prime vertical, the body
in reference to the

gun

is

hour

six o'clock

And what

is.

or

circle,

is -here

equally applicable to

said

any other

celestial object.

In the following examples the learner is recommended,


for the sake of practice, to work out the solutions, by both
forms, of the cases in which the object observed

is

near the

meridian.

Examples far Exercise.


1.

At 18

Object

off the Meridian.

45' from apparent noon, in latitude 8" S.

by

account, the sun's true altitude was 74 16' (zenith N.),

and

his

declination at that time 23 27'

required the

latitude true to the nearest minute f

Ana. latitude, 8" 23'


2.

In

latitude 48 12' TT.

declination was 16

1C

S.,

8.

by account, when the sun's


0h 16 m p.m., apparent time,

at

was 25 20' ^(zenith N.) required the corAns. latitude, 48" 24' 5" N.
5 36 9 p.m., apparent time, in north latitude, the

his true altitude

rect latitude ?
3.

At

&

sun's true altitude was 35 4' 7", and his declination 10 54/
26"

N.

required the latitude ?

Ans.

kt 50 48' 2S"

2s

Digitized by

Google

EXAMPLES

OBJECT OFF JIEHIDIA3T.

110

4. At 10' 40 m A.M. apparent time, in north


when thesun'a declination was 163 12' 10" N., his
1

tude, S. of E., was 44 56'

5.

In

latitude 50 40'

latitude,

true

alti-

required the latitude ?


Ans. lat. 58 47' 8" K".

N. by account, when the sun's

declination was 11 44' 58" N., hia true altitude was 50 52'

29* at ll h 47

57', a.m.

required the correct latitude ?

Ans.
9h

lat.

50 47' 49" N.

30 m

6. At
a.m., apparent time, in north latitude, when
the sun's declination wa3 12 28' 40'' N., his true altitude
9. of E., was 41 30' : required the latitude ?
Ans. lat. 50 6' 1" N.
7.

At

7* 20m A.M., apparent time, in north

Mtude, when

the sun's declination was IS" 50* 10" N., his true altitude
N. of E,, was 24 20" required the latitude ?
:

8.

if

Ans. lat. 19 14* 53* N.


"What would have been the latitude in the last example

the sun had been S. of E. at the time of observation ?


Ans. 70 36' 9* N.

9. Oct. 29, 1858, P.M., in north latitude, and longitude


4 40* W., the Greenwich time, as shown by chronometer,
which was 5" slow, was 2 b 20m 43", the observed altitude of

the sun's lower limb was 46

4' 30*, and'

40',

the index correction was

the height of the eye 17 feet

required the

latitude ?

Ans.

lat.

12 50> 34" N.

10. At ll h 21" 32 p.m., apparent time, in longitude 0 45'


W., the observed altitude of the Pole Star was 51 22', the
s

index correction

3',

the height of the eye 26 feet

and for

apparent noon at Greenwich, on the day of observation, the


Nautical Altuanae gave the following particulars.
Rule, page 136.)
Sun's K. A. 6" 51"

I1V

star's

(See the

E. A. 1" V" 41"

star's

declin. 88 p 26" 56":. required the latitude to the nearest

Ans. lat. 51 47' N.


minute ?
11. Determine the latitude from the altitude of the Pole

150

PEOBLEJI OP DOUBLE ALTITUDES.

Star, as given,

with the other particulars, in the example at

page 132.

Latitude from

Two

Altitudes of the Sim, and the Time

between the observations.


It has been sufficiently

shown in the foregoing

article,

that when the time is known, the latitude of the ship may
always be found from a single altitude of the sun, provided

we know

But

his declination at that time.

if either

the

longitude by account, or the chronometer, be suspected of


error too great to justify confidence in the time at ship, as

deduced 'from them, then

will

it

be necessary to enter upon


Double Axtitudes.

the more complicated problem of

In

this problem, the exact time, either at the ship or at

Greenwich,

is

not necessary

it is

the interval of time only,

between the two observations, that it is requisite to know


with accuracy ; and this the chronometer, or even a good
common watch, if the interval be not unreasonably long,
will always measure with the desired precision.
In order to facilitate the solution of this problem of
double altitudes, various tables have been constructed, and

many

rules

and expedients devised; but

tve consider that

the direct method, by Spherical Trigonometry, while

more

accurate, is fully as short,

to the

Let

memory.*

S, S' the

and much

less

it is

burthensome

Its investigation is as follows

be the zenith of the Bhip,

the elevated pole, and

two places of the sun when the

altitudes are taken,

* The celebrated Delambre, after having carefully examined all the rules
with which he was acquainted

for

the solution of tins problem, came to the

conclusion that the rigorous process, by spherical trigonometry,


preferred, as well for brevity as for accuracy of result.

And

was

to

be

another high

practical authority. Captain Hater, entertains the

superiority of the direct over the indirect

same conviction as to the


methods.See " Encyclopedia

Metropolitaua," Art. Nautical Astronomy.

Digiiized by

Google

PROBLEM OP DOUBLE ALTITUDES.

lfil

then in the annexed diagrams the following quantities


be given, namely

The

polar distances

P S, P S'

"|

co-altitudes

hour-angle

Z S, Z S'
S P 8'

1 to find the co-latitude

will

P.

There are tliua three spherical triangles concerned, namely,


the triangles

P S"S', Z 3

S',

and Z 3 P, the

first

two, having

the great circla arc

S',

joining the two

positions of the sun

common

for a

base,

and the third having


for base the co-lati-

tude Z P.
1.

In the triangle

P 3, P 3'
from S

P S S', we may regard

as equal, since the

the two sides

change of declination in passing

to S' ib so small that

8'

may be

safely considered

as the base of an isosceles spherical triangle, of which each


side is

i (P S

polar distances.

which

+ P S'), that

is

half the

Hence drawing

will bisect the angle

sum

of the actual

the perpendicular

P and

the base 8 S f ,

have given in the right-angled triangle

PMS,

P M,

we

the side

shall

P 3,

M=

and the angle S P M, to find S


i S S'.
2. In the triangle P 3 S', there are now given the sides
P S, S S' or P S', S S', to find the-angle P S S' or P S' S.
3. In the triangle Z S 3' we shall have given the three
sides to find the angle Z S S', and since the angle P S S' is
known from the solution of the first triangle, we shall
;

thence have
the angle

PSZ =

P3

8'

- ZS S'

or

P S S' + Z 3 S'.

In the triangle PSZ we shall thus have given the


S Z, and the included angle, to find Z P.
Thus by the solution of three spherical triangles, we shall
4.

sides 8 P,

be

able to determine the co-latitude of the ship without the

LATITUDB BY DOUBLE ALTITFDES.

152
aid of

any but

tlio

common

logarithmic tables, and the

result will be rigorously correct, if the data are so, except

may be affected by the supposition that the


magnitude of the arc S S' would remain unaltered by our

in so far as

it

lengthening the shorter of the two polar distances by half

and shortening the other as much.

their difference,

It is

plain that the error of this supposition, always very small,

becomes less and less as the interval between the observabecomes diminished.
As the solution of the third
which two sides and the included angle are
given, may be effected in various ways, wo shall here give
the investigation of what appears to us to be-the preferable
method.
Prom the fundamental formula of Spherical Trigonometry
tions

triangle, in

we havecos Z

= coa S P cos SZ+einSP


p

some

arc

sin S

sin

SZ

cos S

P_

cosSP
cosZP=cosaP(oosSZ + BinSZtanSPco3S)

.-.

Now tan
:

S must' be equal to the cotangent of

cos

call this

arc a, so that

then the preceding equation becomes


COS z

in which

we

to sin (a

P=

COB

see that the numerator of the fraction

S Z).

Consequently we have

If S
;

should be greater than 90, tan S

so that

when

cos

of (1) will be negative.

is

In

equal

Bin a

tive

is

finally

C0BZp c,3P ain(BZ + q)

will

positive, the right-hand

be nega-

member

this case therefore (2) will

be

O^lzad

by

Google

LATITUDE BY DOUBLE ALTITUDES.


^

provided

we

still

(3Z

take coa S

coa

SP

Bin

153

(8)

positively, that ia, use the

supplement of 9 P. If S P and S each exceed 90, (1) will


be positive and (2) negative.
We shall now exhibit the work by the above method, in
an example, regarding the necessary preliminary corrections
for altitude, declination, and semidiameter, to have been

made.

Examples.

Latitude from two altitudes

qftlte Sun,

and

the elapsed time.


1.

The two corrected zenith

distances of the sun's centre

are

= 73 54' 13', and Z S' = 47

Z8

45' 61",

the corresponding polar distances are

Pa =

8142'K. andPS'=8145'N.

and the

.*,

4 {P3 + PS')

= 81' 43' 30",

interval of time between the observations is 3 h

required the latitude?

1.

In

ike triangle

PS8143'30"
SP

22

30

15 11
2
30 22

2.

In

the triangle

S8'44 30' 22"


PS' 81 45 0
8 PS' 45 0 0

PSS'

86

39

At.

S, to

.....

M 22

S8'=44

PM find the tide 8


...... in9-995455
sin 9-582840
Bin B-578295

P S S',

to

find ike angle

Comp.

Aritu.

sin
Bin

......
.

P S S'.

0"154291

9-995182

Bin 9
sin

849435

9-9992S8

h3

Ngitlzed

by

Google

LATITUDE BY DOUBLE ALTITUDES.

154

In

3.

Z S S',

the triangle

to

find the angle

Z S S'.

IS' 47 45' 51"


13

Arith.

Comp.

Bin

0-017369

SS'44 30 22

Arith.

Comp.

sin

0-154291

ZS 73 54

10 26

2)166
4

4
4

sum
sum
sum

of sides

83

Z8 = 9
SS' = 3S

13

11

Bin 0-203017

34 51

Bin

22 36 26
45 12 52

sin

9-794919

2)19-169596

iZSS'=

,\

4.

=
S' = 86
P S Z = 41

Z S
PS

.'.

In

S'

the triangle

= 11

0 42

ZS = 7S

13

ZS + :=B4

55

Note.

may be
given

39

26

PSZ,

to

find

first a,

thence OieaideZ P.

cot 10'Yl OSSO


j

The

and

9-C84798

Lat = 4B

49 55-

llonca the Latitude fs4B" 49" 56".

first

operation, namely, that for finding

angle of a triangle are given to find the third side


already remarked, a trifling

determination of S

S',

brevity of the work.

pute S

S',

in the

for the sake of

*
is

The sum

the

S',

replaced by a process similar to that marked (4) just


because, in both cases, two Bides and the included

jiite

but, as

amount of accuracy, in the

has been sacrificed to the superior

It

may not be amiss here to recomin which Z P is computed above,

manner

comparing the two

of the three logarithms

ifl

results,

the cosine of

P, consequently it

of the latitude.

OigilizM By

Google

LATITUDE BY DOUBLE ALTITUDES.

P8

0"

45

cob 9-840485

= 11

on 20
on
39

. ,rnrr;
cotlO'6S5JTT

PS'=S1

4S

PS' + = 93

24 D6=S'ip.*of3&"35'34"

COS 9 -150135

11' 39' 29" Ar. C. sin 0 604528

sj

^9090232

8 8'= 44 30 23

cot p-853195

ps'+ss

35

This example

155

PS

tail 10*335092

Bl" 42'

SPS'

which
only by

differs
1

from the former result

well suited to test the general trust-

is

worthiness, of the operation

marked

(1),

as the interval

between the observations, 3 hours, is tolerably large, and


the change of declination, V an hour, ia an extreme supposition.

In reference to the method of solution here exemplified


there are one or two remarks to be made, which deserve the
student's attention.

In the step marked

1.

from

Now to

its sine.

(2) the angle

PSS'

is

inferred

a aine belongs either of two angles

the supplements of each other, and it may be matter of


doubt whether the angle taken from the table, in connection

with this

the case before us, be acute, as

sine, should, in

we

have considered it to be in the above operation, or obtuse


we proceed to show how the ambiguity may be avoided.
The fundamental formula of Spherical Trigonometry give3
cosPS'

= toaPSeoaSS' + sinPSBinSS' eon PSS'


cos

Now,

is

it

we may

V SS'

"*J?*

a&

matter of indifference which of the two places of

the celestial object

we mark S

or S'

always consider that cos

P S'

so that in this formula


is

numerically greater

than cos PS; and consequently numerically greater than


cos P S cos S S'. And since the denominator is necessarily
positive, the fraction necessarily takes the sign of cos

P S'.

PSS' always has the same sign as cos P S'


and P S S' are always either both acute or both

Consequently, cos
bo that

P S'

* Inetead of taking the frappleraent, for the pnrposo of getting


ia tha nest column of the work,

above

30,

namely 3

24'- 2G",

we may take merely the

and takeout

the same thing as the aine of 93

24' 26".

excess of

its cosine, wliich,

its

sine

PS'+*

of oourae, is

LATITUDE BY DOUBLE ALTITUDES.

158
obtuse

distances,

other, the angle

the side

we

hence, if

two polar

P S',

that

always take for

whose sine
9' will

P S'

is less

that one of the


than the sine of the

always be of the same species as

they will be either both acute or both

is,

obtuse.

A3

respects the sun, however, these considerations need

not be attended to : whenever the two positions of that body


are on the same side of the equinoctial, both angles will be

either acute or obtuse,

the same

name

contrary name.

acute-

when the

as the latitude,

The

declination is

and obtuBe when

it is

of
of

from

sides of the polar triangle differ

equality in so trifling a degree, that the angles referred to

may

always be regarded as of the Bame species, except when

the sun actually crosses the equinoctial in the interval of


the observations ; and even then, each angle will be bo near
to 90 that, whether they be regarded aa acute or obtuse,

can make no difference of importance.


2. In low latitudes it may happen that the arc S 8,

if

pro-

P and Z, as in the
second of the diagrams at page 151. In this case the angle
PS Z will not be the difference of the angles PSS'.ZS S',
but their sum. It is plain that when the altitudes are both

longed, would cut the meridian between

on the Bame
only

side of the meridian,

when the

8 Z can be the sum

the declination being


that the sun would cross the meridian

latitude is so low

of the same name


between P and Z
;

also

for if it crossed the meridian

other side of Z, the great circle are S

S',

on the

when prolonged,

would necessarily cut the meridian still further from Z on


that side hence when the declination and latitude aTe of
:

the same name, and

the former, we

we know

may be

that the latter

is

greater than

sure that the difference, and not the

sum, of the two angles in question must be taken, when


both observations are on the same side of the meridian.
When, however, under other circumstances, in these low
latitudes, a

the

donbt occurs, we may remove

final Btep,

taking the

sum

it

by recomputing

instead of the difference, or

OigilizM By

Google

LATITUDE UY DOUBLE ALTITUDES.

157

and choosing that of the two results which


from the latitude by account, But a more

vice versd;

differs the least

convenient

way seems

to be thia

namely, to directly com-

P S Z, from the three sides of the triangle


Z: the polar distance P S, the co-altitude Z 9, and the
by account P Z, the operation being similar to

pute the angle

PS

co-latitude

that of step (3) above

the result will be an approximation

And we may

to what the Btep referred to ought to give.

remark, that as an approximation only

is

to

be expected,

seconds in the several ares need not be regarded

may

each

be taken to the nearest minute only.

When
may

the true co-latitude

combine

altitude

S,

is

thus ascertained,

with the polar distance

it

9,

we

and the co-

in imitation of the operation (3), to determine

the hoar-angle

when

PZ

Z P 9,

that

is,

the apparent time from noon,

the altitude nearest to the prime vertical was taken

correction for Equation of Time being applied, we


shall get the mean time from noon when S was observed.
"We here suppose S to be that one of the two positions of

and the

the sun which

motion in

is

the nearer to the prime

altitude is quicker

the

vertical, since

than when the sun

is

in the

other position, and consequently a small error in the altitude

has a less effect upon the hour-angle.


In .the example worked at page 153, we have proceeded
on" the supposition that the altitudes of the sun have both
but as, at sea, the ship
been taken at the same place
;

usually

on during the interval of the observations,

sails

it is

necessary to allow for the change of place, and to reduce the


first altitude

to what

it

would have been

if

taken at the place

of the second observation.


This

is called

the correction for the Bhip's run

it is

ob-

From the sun's bearing find the angle between


tained thus
the ship's direction, and the eun's direction at the first
:

observation; then considering this angle as a course, and

the distance sailed as the corresponding distance, find,


either by the traverse table, or by computation, as in plane

LATITCDE BY DOPELE ALTITUDES.

158
the

sailing,

diff. lat.

which the ship

this will be the

number of minutes by

has advanced

or

rather the ship's zenith

towards, or receded from, the sun in the interval, and will

number of minutes

therefore be the

to be added to, or sub-

tracted from, the first altitude, to reduce that altitude to

taken by another person at the


place of the second observation, and at the time of the first.
2. February 8, 1858, in latitude 35 N'. by account, when

what

would have been

it

if

mean time at Greenwich, as shown by the chronometer,


was ll 17 m 4* a.m., the observed altitude of the sun's lower
limb was 36 JO', and his bearing S.^E. after running 27
lower limb was 41 20',
the time shown by the chronometer being 2 h 38 18 9 p.m.

the

11

miles, the observed altitude of the

The

error of the instrument

was +2', and the height of the

eye 20 feet: required the latitude of the ship when the

second observation was made.


1.

For

the

polar distances PS, PS', and tit angle

Pint

Cor,

15" 18'

for28*i

Declisaiion

18

11

SPS'

28* 17- i*
4" S.

Mfc

47"'2e

1*

19

59

45

59

45

23J

U178

S.

00

PS=

Srfioecn them.

Observation.

...

G. Time, Feb.

NoonDeclin.

0452

104

1181
6,0)109,8-79
Cor.

18'

19"

Second Observation.
J 18'

G. Time, Feb. 8

Noon

Deoxibatios

5P

14

Declin.

Cor. for 2* |

14

10"

57 10

90
PS':

PS':

104
104

&

Diff.

1*

47"-89

24
S.

S578
2895

57 10

6,0) 11,9-73

59 45

IP

2)209 56 55

159

LATITUDE BY DOUBLB ALTITUDES.


From

Noon, Feb. T

Time

m
of 1st Observation 28" 17
2nd

...

3h

4'

5" 15'

U =

3'

21"

23 38

18

3 21

14

Interval of

Time

Hence the angle S P S', in

2.

For

the line allilvda

First alt sun's L. L.

2'

Index and Dip

App.

....
24"

True

Second

alt.

Parallax

alt.

36

alt.

of centre

sun's L. L.

....

36

Refraction

alt. of centre

..

12

22

39

0"

+13

51

33

51

59

Parallax ....

True

23 51

41" 20'

41

ofccntre

51

_______

....

Index, Dip, and Semi

App.

0"

IV

+13
"

86

alt. of centre

Effraction

the San's centre.

of

16 15

Semi- diameter

30"

degrees,

41

32

52

Since the angle between the sun's bearing at the first


observation, namely, S-JE., and the course of the ship afterpoints, the ship has sailed, in
is

wards, namely, N.E.,

the interval, within

Hi

points of the direction opposite to

the sun, a distance of 27 miles. With 27 miles dist. and


4 } points course, the Traverse Table gives 18' for the corresponding diff. lat., so that the ship has receded 18' from
the sun during the interval of the observation.
quently 18' must be subtracted from the

first

Conse-

true altitude

it to what it would have been if a second observer


had taken it at the place of the second observation at the
time the first was made. The true altitudes at this latter

to reduce

place are therefore


36 i' 39' and 41 32' 52"

.\ZS=

53" 55'

21"and ZS'=48

27' 8"

DigiiizM By

Google

LATITUDE Br DOUBLE ALTITUDES.

160

104 59' 45"

SPM

25

24

SI

18

30

10

Inthe

triangle

sin

9-613397

P S S'.

find the angle

to

15"

sin 9-886204

sin

to

find the angle

55

21

Aritb.

28

57

Arith.

51

26

25

28

56

T
SB

35

29

30

9-996904

Z 8 S'.

8"

53
48
2)150

of sides

9-985039

sin

...
.

ZSS',

48 27'

= 75
Z8 =21
SS'=-2B

sum
sum

9-628445

....

57 10

50

83

Zff

iBwm

9'

9-984952

sin

4S 28' 57' Arith.Comp. sin 0-125601

104

SB'

as isosceleB, ia

sin

P S S',

the triangle

SB'
PS'

14 28J
2

PSS'

S P 3", regarded

...

15

SPS'

5.

M = 35'.

finds

audSrM = iSPS' = 25''

53' 28",

PS

In

to

sides of the triangle

= 104"

4.

PMS,

In the triangU

3.

Each of the eqnal

i(PS + PS')

Comp.
Comp.

ain 0-092470

0-125661

sin

sin

9-564113

sin

9-9E6182

....

sin

2)19-438426

4ZSS'=31
*.

9-719213

ZSS' =:63 10 58
0
PSW
88 10

.-.

6.

In

PSZ

= 19

the triangle

59

P SZ,

2
to find fint

m and thence the tide

P8* ....
15"M'84" Ar.Comp.

ZS WVir
Lxr.=3S ,
Honoo

32'30"

Z P.
H

B -412ST8

sin 05C2O6S

a 9-7SM69

Bi

Bin 9-764410

the latitude ia 35

s5

3i>"

K.

* There are three references to the Tables with this are, namely, one in
etep 3 for the sine, and two in the present step, one for tangent,

and the

Digitized 0/

Google

BT

[LATITUDE

Note.
student

tudes

The
a,

BOX'S LI! ALTITUDES.

now

illustrations

161

given will convey to the

sufficient notion of the

and from the length

problem of Double Alti-

of the computation involved in its

solution, he will be prepared for the statement that it is a problem resorted to at sea only from necessity. This necessity,

however, can but seldom occur

bo long as the chronometer


can he safely depended upon, and the longitude by account
;

not grossly in error, the time at the ship can always be


obtained with sufficient precision to. enable us to get the
is

latitude from a single altitude of the sun, as fully explained

And

at pages 136 and 146.


single altitude,

the latitude thus inferred from a

and the sun's hour-angle,

is

in general

much

more trustworthy than the


altitudes,

latitude deduced from double


which should never be regarded as more thnn an
Trifling errors in the data of a problem

approximation.

may

accumulate to something considerable when they pervade a long coarse of operations. One of the two altitudes
in the present problem

we

are pretty sure

must be

affected

with error : the altitude, namely, which is corrected for


the run of the ship there is, of course, some difficulty in
getting the sun's bearing with precision, and there is a
;

further liability to error in the estimated distance sailed.*


other for cosine.

but

These

may

all

be taken from the table at one opening

be better to take out only twoaa sine and tangent; then cosine

it will

is at once got

by subtracting the tan from the

...

sine, conceiving the latter

10 + sinPS= 10-034952
10-572074
tan P S

sin

to be mcreased by 10, for cos

* If the sun's true bearing or azimuth at either place of observation


could be taken with precision, there would be no necessity for a second
altitude

for

we should then

given the polar distance

Iiave

spherical triangle Z

S, the co-altitude

S,

PS

in which are

and the angle

1, to find

the co-latitude Z P.
For the purposes of the problem in the text, however, the true bearings
of the sun

sary

and

of one place of observation

the compass bearings

two directionswhich

is

all

from the

other, are not neces-

are sufficient, because the angle between the

that is

wanted is

whatever be the variation of the compass.

unaltered iu magnitude

To obtain the (rue azimuth of

LATITUDE BY DOUBLE ALTITUDES.

162

The

sira'a

bearing can be taken with more accuracy

when

low than when high so that in this problem the bearing


when
is always taken with the less of the two altitudes
;

therefore

it is

the second that is the less altitude, and the


taken, the point opposite to that of the

sun's bearing

is

ship's course,

from the former position, must be used in

reducing the second altitude to what

taken at the same place as the


operations

For

is

the blank form of the

Blakk Fokm.
1.

would have been if


and the latitude will

it

first

The following

apply to that place.

ike

Two

Latitude from

P S',

polar diilances PS,

Altitudes of the Sun.

and

the polar angle

S P S'

letmcea them.
First Observation.

6. Time

.>....

NoonDeclin.

Cor. for hours

fr,

noon

.'

Diff.

in 1"

Honrs from noon

Correction

.'.

Second Obiervation.
G. Time

NoonDeclin.
Cor. for hours

fr.

noon

."

.*

Diff. in l*

Hours from noon

Deolihatiok

pg

Correction

3)

i(PS + Pff)=

used for

P 8,

.'. .*

in step 3.

Interval of Time, converted into Degrees,


.

.*

.'

.."

BPS'

.-.4

S PS'

=.

..'..*

= SPM.

the sun, and thence to deduce the latitude as indicated in this note, the
correction for variation must of course be applied.

bt double altitudes.

LATITU.DE

2.

First

alt. (L.

L.

Index aud Dip


Serol-diamater

>r
.

For

the true altitudes

of the

Second alt (L.L. or O.L.)

. .*

U- L.)

Index and Dip


Bemi-diamutar

.'

App.

alt.

Tnia

The

alt.

.'
.

."

....

of centre

Refraction

Pmallns

of centra

sun's bearing when the leBs altitude

was taken having

been observed, and the course of the Bhip, or the bearing of


the place of the greater altitude from that of the less being
known, find by addition or subtraction, the angle between
these two bearings.

"With this angle as a course, and the

distance between the two places as a distance, find the cor-

responding

diff. lat.

from the Traverse Table

taken as so many minutes, add


altitude, according

to,

and

this

diff.,

or subtract from, the less

as the ship has advanced towards, or

receded from the sun.

The

less altitude

being thus cor-

we

rected for run, subtract each altitude from 90, and

shall

have

ZS=
3.

In

the triangle

and ZS'

P M S,

to

find

*.

S Iff and thenee2SM=SS'.

=
In

.' .

."

the triangle

the angle

PSS'

PSS

to

find

1
.

Note. As pointed out in the preceding page, the polar


distance, used in this third step, is taken equal to the half

sum

of the actual polar distances.

Digitized 0/

Google

LATITUDE BY DOUBLE ALMTTTDES.

164
In

6.

sum

i sum

the triangle

ZSS',

to find the

angle

ZSS'and

ZS'
ZS

Arith.

Comp.

sin

SS'

Arith. Comp.

Bin

thence

PSZ.

'

of sides

ZS

.........

sin

ZSS' =

.-.

PSS' =

In the

6.

triangle

P 8 Z,

to

find first

a.

and

thence the latitude.

If instead of the sun the object observed be a star, step 1


is

of course dispensed with, as the declination

is

got at once

from the Nautical Almanac, and in step 2 there is no correction for semidiaineter and parallax
the remainder of
the operation is the same. But instead of taking two altitudes of the same star, a far more practicable and trustworthy method of finding the latitude is to take simultaneous
;

altitudes of two distinct stars.

advantages
1.

No

This method has. several

allowance

is

made

for run

error involved in the course sailed

sun

is

of the ship, and thus all


and the bearing of the

avoided.

There is no risk incurred of losing a second observafrom unfavourable weather.


3. The hour-angle, or the angle at the pole between the
2.

tion

Dill

:-.'J

Cy

LATITUDE FROM 8IMTJETJOTE0U6 ALTITUDES.

two polar

distances, is given at once

16S

by -taking the

differ-

ence of right ascensions of the two stars so that neither


the Greenwich date nor the time at ship requires to be
;

As, however, the polar distances of the two stars


differ considerably,

the aideS

S'

in the triangle S

was found

P S', in

may

cannot here be computed as

it must be found,
a way similar to that in which Z P

in the case of the sun or of a Bingle star


in the triangle

P S Z. But

done, an example will Buffice to

make

after

what has been

the operation intel-

ligible.

It is proper to notice, however, that if there be

but one

observer, so that the altitudes, instead of being both taken

at the same instant, must be taken in succession, the prac-

must be managed as follows


The altitude
must be taken, and the time noted by a watch
the altitude of the other star must then be taken, and the
tical operation

of one star

time noted.

After a short interval, the altitude of the


second star must again be taken, and the time noted: we
shall thus learn the second star's

motion in altitude in a

and may thence, by proportion, find what its


was when the first star was observed so that we

given time
altitude

shall have the altitudes of both at that instant.

Latitude from the Altitudes of two Stars taken at the

same

Ex. In latitude

38

!N.

time.

by

account, the altitudes of

a Pegasi and a AquilEe, taken at the same instant, on the

Bame

side of the meridian,

were respectively

29 49' 27* and 57 29' 50*

the index correction was

41

feet

also,

15",

and the height of the eye

the Nautical Almanac gave the following

particulars

OigiiizM By

Google

LATITUDE FBOH SIMULTANEOUS ALTITUDES.

166

a Aquils.

Pegasi.
Declination

14 22' 50" N.

Right Ascension

22fc 67 m

Require J the latitude

For

N.

19 h 43- 15"

polar distances PS, PS', and

the

8" 28' 2"

Declination

6*

polar angle 8 PS'

tiie

ielween them,

14 22' 50"

PS'=75

37

2"

8 28'

= 81

PS*

10
22" 57"

_. .
Bight Ascenmons

3:

6'

SPS'intime =

19

i2

15

13

51

13-

...
,-.SPS'

For

2.

Observed

alt.

Apparent

12 45"

48 27' 45"

Z 8', ZS.

ri true zenith distance*

the

ofS'

Index and Dip

15'

29 49' 27"

ofS

57 29' 50"

29 42 54

alt.

42

True altitude

2 3'= AO 18 48

3.

In

PS'
SPS'

*=

27

45

21

23

PS =

81

81

58

102

40

21

SS'=

ZS=

75

37'

32

37 20

PSS', tojindSS'.

75 37' 10"

48

PS + =
PS'

* This is

the triangle

10"

21

102

40

21

47

47

tan

cos

8 -821586

cot

10 '412677

23 Ar. Corap,

marked P S, instead of P $,

...

10-591091

eoa
sin

9-3950S4
0-442922

gin

9-989290

cos

9-827290

for the reason assigned at p. 1 55.

OigiiizM By

Google

4.

In the triangle

PS S',

SS' 47 47' 14"

5.

In

PS' 75

37

10

SPS'48

27

45

PSS' 78

13

32

the triangle

to find the

to

0-130384

sin

sin 9-9S6175

find the angle

167

angle PSS'.

Comp.

Arith.

...
...

Z S S',

ALTITUDE 8,

S IMULT ABE OU 8

LATITUDE FROM

sin

9-874205

sin

9-990764

2 SS' and

thence

PSZ.

ZS'60 18' 48"

ZS

32
88' 47

Arith.

Arith. Comp. siu 0-130384

70

21

41

44

21

34

27

41

11

22

13

32

57

50

,-.

= 100
= 78
= 21

Z8S'
PSS'

.'.PSZ

6.

50

In

the triangle

21

P SZ,

to

find a

50

57

32

37

20

+ *=41

44

29

ZS
ZS

...
...
...

81" 31' 53"

PS
PSZ

The blank form

sin 0-268333

20
14

Isnm ZS = 37
isnm SS'=22
1ZSS'=

Comp.

37
47

44

29

12

29

and

sin 9-786799

am 9-584194
2)19-769710

nB-884855

Bi

thence the latitude.

tan 10-827204

cos

cot 10-794480

9-967276

At. Comp. sin 0-800021


.

sia9-823324

sin~9

for the foregoing operation

791353

ia

aa follows

BLAHS EOEM: AIT. OP TWO STABS.

163

Blank Fobst.

Latitude from Altitudes of two Stars taken


at the

Ejgg"

That Btar

is

polar distance is the

I.

For

to be

same

time.

marked

S'

of which the sine of the

less.

P S,

polar distances

the

PS', and

Declin-ofS

the polar angle

of

Polar dint.
II.

90

PS=

A. of S

PS'

'

SP S'.

S'

90

ofS'
SPS'intime

alt.

.*.

For

2.

Observed

of

Index and Dip

Apparent

alt.

Refraction

3.

SP

S'

the true zenith distances


.

.'

."

..'

ZS, ZS'.
of.S'

. .

Inthe triangle P S 8',

to

find

S S'.

FS'
SPS'

Ar.Comp.

P8

si

..

FS =

s=
The remaining

steps, namely,

those in the form at page 163.

4r,

5,

and

6,

are the

same as

LATITUDE BY D01TB1B ALTITUDES.

Examplesfor Exercise in Double


1.

In

latitude 1 34/

169

Altitudes.

N. by account, two corrected zenith


and 19 59'

distances of the sun's centre were 54 39',

the corresponding declinations were 5 31' 6" S., and


5 28' 54" S. ; the interval of time between the observations was 2 !l 20m

required the latitude ?

Ans. lat. 1 29' 28" N.


2. In latitude 35 27' N. by account, when the mean
time at Greenwich, as shown by the chronometer, was ll h
1710 4* A.M., the'observed altitude of the sun's lower limb
was 36 14', and his bearing S. E. After running N. E.
27 miles, the observed altitude of the lower limb was

41

24'.

the time at Greenwich being 3 U 38" 18 s p.m.

error of the instrument was

20

feet.

Declin.

.it

The

and the height of the eye

The Nautical Almanac gave


G. noon procoding lat observation, 15 3S' 0".
following
G
IS 10 32
'
,,

Uiff.

in It,

-i6"-5

ift

Required tho latitude of the place whore the second observation was made ?
Ans., lat. 35 20' 2" N.
3. In latitude 53 30* N. by account, the corrected zenith
distances of Capella and Sirius, both observed at the same
time, were
Sirius.

Capella.

Zenith distance ZS'nSD*


Polar dbtanca PS'

14'

24"

11

SO

Zonith diBtnnco
Polnr rtintaneo

ZS

72*

F8=10S

S'

28

48"
40.

Also the difference of their right ascensions was lh 33 m 45':


Ans., lat. 53 19' 23" N.
required the latitude ?
4. In latitude 28 10' S. by account, the sun being ob-

was taken shortly afterwards,


the chronometer at the instant showing 9' 49 m 20" and
when the same chronometer showed 10 h 44 45', the altiscured at noon,

its altitude

tude was again taken.

In the

first

observation the altitude

LATITUDE BI DOUBLE ALTITUDES.

170

of the upper limb was 45 S3'

the lower limb was 42"

The

being N. E.

8' 30",

in the second the altitude of

the sun's bearing at the time

N.W.

run in the interval was

ship's

J "W. 6 miles; the allowance for index error and dip was
4' 30", and the Nautical Almanac gave for the Green-

wich noon of the day


Sun's declination 16 34' 4" N.

Sun's semidiameter

Diff. in

42"'8

1\

15' 52".

Required the latitude to the nearest minute at the place

where the

first

observation was taken ?


Ans.,

In

Note.

28

lat.

0' S.

the foregoing examples a single altitude of

the celestial object observed, has uniformly been regarded


as the altitude at the time

but as

take an altitude with precision,

accuracy
three or

it is

in

not always easy to

customary, where

required, to take several

is

five

it is

much

usually

altitudes

pretty rapid succession, that

is,

within a

minute or two of each other, and to note the corresponding


times the intervals should be as nearly equal as practicable.
The mean of the altitudes is then taken as the altitude
:

corresponding to the mean of the times.

The

learner

is

to understand, however, that in taking a

set of altitudes, it

is

not the chronometer which

is

directly

consulted for the corresponding times: the chronometer

is

never removed and carried about, but a good seconds watch


is

always employed.

Tho mean of the times by watch,

cor-

responding to tho mean of tho altitudes, being found, tho

watch is then carried to the chronometer, and its error on


the chronometer ascertained this error being allowed for,
wo have the time by chronometer corresponding to the
;

mean of the

altitudes; or the error is found immediately

before the observations are taken.

Digiiized 0/

Google

VARIATION OF TDK COMPASS.

CHAPTEE
OH"

The

171

III.

THE VABIATIOK OF THE COMPASS.

aDgle by which the compass-needle when unin-

fluenced by local circumstances

north and south

line, ia called

deviates

from the true

the variation of the compass

at the place through which that north and south line passes.

The

variation

is

different at different places,

variation was formerly easterly

in

and

ia

seldom

At London

long constant even at the Bame place.

1659

needle then pointing due north and south

it
:

the

wa3 zero, the


then slowly

it

deviated from the plane of the geographical meridian to-

wards the west, the deviation increasing

when

till

the year 1819,

the westerly limit, 24 42', appears to have been

attained.

Since then

it

has been slowly but irregularly

returning, the variation at present being about 23 West.

On

shipboard the angular departure of the' compass-needle

from the plane of the geographical meridian, is the combined


effect of the variation properly so called and the local attraction of the ship itself, which in iron vessels must of course

be

Contrivances have been introduced to

considerable.

neutralise

this

an account of the most


be found in the article on " The Com-

local attraction

efficient of these will

paBS," in Mr. Grantham's " Iron Shipbuilding," in Weale's


Series of Rudimentary Treatises.

To ascertain at any place the amount by which the compass direction deviates from the direction of the true north
and south

obviously a matter of much practical imthe following article will be devoted to the

line, is

portance at sea

consideration of

it.

YABIATION FBOM AN AMPLITUDE.

172

Variation determined

from

the observed

Amplitude of a

celestial object.

In order to discover to what extent the compass is in


error, it is plain that we must possess some means independent of that instrument of finding the true bearing of an
object the difference between this and the compass bearing
will be the variation, or the error of the instrument.
At sea, the object must be one of the heavenly bodies : if
it be in the horizon, that is, just rising or setting, the
;

bearing

is its

bearing

is ita

amplitude;
azimuth.

if it

be above the horizon, the


object is rising, the true

"When the

is always measured from the E., and when it is


from the ~W., and towards the north or south

amplitudo
setting

according as the declination

N.

is

To compute the amplitude

or S.

only necessary to

it is

know

the declination of the object, and the latitude of the place.

For
and

P be the

let

zon
,

elevated pole,

Q' the equinoctial;

H H' the hori-

the zenith, and

then

S be the body

if

at

rising or setting, the perpendicular


its

to

declination, the

will

he

opposite

angle 0, at the east or west

hypotenuse

will

point of the horizon, will he


the co-latitude of Z, and the
be the true amplitude ; so that in the

right-angled spherical triangle

S x sin 0, that
.-.

is sin declin.

Bin

0 D 8 we have sin S D =sin


= sin amp. x coa lat.

amplitude= declination
cob latitude

Since refraction causes objects to appear in the horizon


when tiiey are on the average about 33' below it, the compass bearing should be taken
star selected, is

about

when

the sun's centre, or the

33'-f. dip above the

sea horizon; so

Digitized By

Google

VABXATION FB01T

AMPLITUDE.

AIT

173

that allowing 16' for the sun's scmidiameter, the observed

altitude of the sun's lower limb should be about 17'

It

is

to be observed that

if

dip.

the true amplitude, found

by

calculation, and that taken by the compass, bo both N. or


both S., their difierence will be the variation but if one be
and the other S. their sura will be the variation. The
;

JH.

variation

is

E.

when

the true amplitude

is

to the right, and

W. when it is to
to say,
is

the. left of the compass amplitude, that is


E. or "W., according as the suu'b true direction

it is

to the right or

of the compass direction.

left

Buffieient if the variation

Examples.

It will be

found to the nearest minute.

is

Variation of

the

Camp ass from an Amplitude.

February 20, 1858, the rising amplitude of Aldebaran,


taken at sea with the azimuth compass in true latitude 27
1.

36' IT., was E. 23 30'

compass

N.

required the variation of the

Declination of Aldebaran, Fen.


log sin amplitude

(Kant Aim.

20, 1868,

= log sin doclin.log

Doclin.

IS* 13' 21"

Latitudo

27

116

....
.

p, 379.) 10 13'

cos latitude

sin 9

21" H.

+ 10

olS

cos 0-D17K3

TruoamnlitudoE.- IB 22 33 N.
23 SO
0 N.

Compass amplitude E.
Variation

S"

or

V
7'!

The magnetic or compass E. has


P 7' 7" from tho true E.
towards thoK., banco tho magnetic
N. muat hare deviated this amount
E. from the true N.

27" K.

receded

V..

tho line amplitude being to the right.

July 10, 185S, the star Eigel was observed to set 9


50' to the N. of the "W". point of the compass, in true lati2.

tude 48 10' N.

required the variation of the compass ?


8 21' E4" S.

Declination of RigeL July 10, 1858,


Deolin,

Latitude

8 21' 54"

48

Trrw amplitude W. 12

Compass amplitude W.
Variation

10

35

56

50

....

sin

0 '16280

cos

9-82410

ain

9-33870

S.

If.

22" 25' 58" W., or 22" 26' W., the true amplitude being to the

left.

Digitized By

Google

FBOM

VABIATIQTt

174

AIT

AMPLITUDE.

3.

February 15, 1858, in latitude 43 36' N. true, and


W. by account, the setting amplitude of the

longitude 20
sun's centre

was observed

apparent time

1.

be

to

W.

6 45'

For

the declination at time

App. Time at

SMp

loDg. in time

W.

of obaervation.

App. Time at Greenwich

12 89'

Sun's Noon Declin.


Cor. for 8 b 10

Declination

N. at 6 h 50 m p.m.

required the variation of the compass ?

5T'S

12

6 b 50*

,+1 20
.

10

DifE.

B.

1S-51--82

32 54

41456
864

S.

423-20

2.

For

True Amplitude.

12 33'

Declin.

Latitude

True amplitude W.

Compass amplitude W.
Vabiatioh

the

43

36

17

28 S

45 S.

...

....

sin

9-33704

cos

9-85984

ain

9-47720

24 13' W., the true amp. being to the left


of the compass.

The blank form

Blank

Foitu,

for these operations is the following

Variation of Compass

from Sun's

Amplitude.

[The true amplitude is always measured from the E. when


is rising, and from the "W. when it is setting
and towards the N. or S. according as the declination is

the object

N.

or S.]

Digitized By

Google

;:;

YABIATION

1.

For Sun's
Time
Long.

AW AMPLITUDE.

ritOJI

175

declination at time of Observation.

at Ship
in.

.*

Time

Greenwich date
Sun's NoonDeclin. at G.

.'

Diff. l k

....

Cor. for Time from Noon

No. of hours

Declination

2.

For

>

Cor.

the true

A mplitudt, and

tJience the Variation.

Declination

sin

I
j

....

Latitude

True amplitude
Compass amplitude

Vabuhox

~"

cos
sin

When the

N. or both

amplitudes are both


the

S. this is

diff.

of

sum

the two, otherwise

it is

E. if the true amp.

is

to the right,

left,

of the com-

and W.

if to

the

their

pass amp.

Examples for Exercise.


1. Jan. 1, 1858, the rising amplitude of Spica, in latitude
16 21' S. true, was observed by compass to be E. 16 3' N.

required the variation of the compass ?

Ans. variation 26 55' E.


2.

In latitude 21

14' If. true,

when

sun, reduced to the time at the ship,


rising

the declination of the

was 19

18'

amplitude was observed to be E. 35 20' 9.

(i"

S., its

required

the variation of the compass ?

Am variation 14 34' "W.


3.

March

11, 1858, at about 51

56 a.m. apparent time,


was observed to be E. 6 36' N.
of the ship was 10 2' S., and her longitude

the sun's rising amplitude

the true latitude

by account 168" E.

required the variation of the compass ?


Ans. variation 10 38' E.

Digitized 0/

Google

VAIUATION FBOJI AN AZIMUTH.

176

Nov. 15, 1858, at about

4.

45 p.m. mean time, the


15 40* S. ; the true latitude

sun's setting amplitude was

W.

of the ship was 31 56' N.,

and her longitude by account

75 30"

5..

W.

required the variation of the compass ?


Ans. variation 6 27' "W.

Sept. 18, 1858, at about 5 h 50 m a.m.

mean time, the


was E. 12 10' N. the true latitude
and the longitude by aceount 72 15' W.

sun's rising amplitude

was 47

25'

required the variation ?

Ans. variation 9

&%

E.

Variation determined from the observed Azimuth of a celestial


object.

Azimuth like amplitude is an arc of the horizon it is the


measure of the angle at the zenith included between the
:

meridian of the observer and the vertical through the object

In N. latitude the horizontal arc ia here regarded


as measured from the S. point of the horizon, and S. latitude
from the N. point towards the E. if the altitude be inobserved.

creasing,

and towards the

W.

if it

In the diagram at page 172, let

be decreasing.

S'

be the object : the arc of


Z S' is the true
given the three sides of

the horizon which measures the angle

To determine

azimuth.

it,

we have

the oblique-angled spherical triangle


altitude

S'

Z, the

the angle

polar distance

PZ

3'

may

ment

of

is

2(90

the final result to

PZ

Z S' P

namely, the co-

and the co-altitude


by an operapage 160: the supplement
S',

therefore be found

tion similar to that marked (5) at


of this angle is the true azimuth.
to gives half the angle

S',

As the

or \ Z,

we have

operation referred

and

that the supple-

only to change sin in

cos.

Ngitized

by

Google

PHOM AN AZIMUTH.

VAETATIOIT

Examples.

177

Variation of the Compass from an Azimuth.

1. April 20, 1858, at about 9 L a.m. apparent time,'[tae


50', and
same time,

altitude of the Bun's lower limb was 36

his bear-

ing or azimuth, by compass,

S.

at the

31 E.

The

true latitude of the ship was 50 12' N., and the


longitude by account 13 "W. required the variation of the
:

compass, the correction of the altitude for index and dip


being 4/ 31"?

For

I.

polar distance at time of observation.

the Sun's

Time at Ship, Ap. 19


lions-inTimeW.

21*

...

App. Time at

Decliii.

Noon, Ap. 19,

Cor. for Sa 11

10'

+ 18

15"

0-

+ 52
21

ft

52

56

10328
1033

rS'=7i

Polar distance

0,0)113,6 '1

+ 18' 56"

2.

For

Ooscrred altitude L. I.

Semidiameter

- 4' 31'
+ 15 57

Apparent

of centre

Index and dip

alt.

Effraction

the five co-altitude.

36 50'

True altitude

0"

"1

11 26
J

37

1 26

-1

- Parallax
37

10

0 16.-.Co-altitui.eZS'=52 0 59'44"

OigilizM By

Google

PKOM AH AZIMUTH.

V A II I ATI OS

178

For the true Azimuth, and thence Ike Variation.

3.

78 30' 50'

Polar distance

40

Arith.

39

48

Arith.

2)171

18

30

~~85

62

39

15

32
45

39

35

51

16

29

30

Co-altitade
Co-latitude

Bum

sum- co-alt,

sum -

Azimuth

59

co-lat.

sin 0-09768

Comp.
Comp.

sin 0-19376

sin 9-73211

9-S5ES6

sin

2)19-87940

.....

cos 9-93970

The variation is

K. or W., accord-

True Azimuth

S.

59

Compass Azimuth

S.

31

0 E.

20

C W.

Variation

B.

ing as the true azimuth

is

to the

right or left of the observed azi-

muth, just aa in an amplitude.

June 9, 1858, at about 5 h 50 m a.m. apparent time, in latitude 50 47' N. true, and longitude 99 45' W. by account,
the bearing of the sun by compass was S. 92 36' E., when
the altitude of his lower limb was 18 35' 20"; the index
correction was + 3' 10", apd the height of the eye 19 feet
2.

required the variation of the compass

For the polar distance

1.

Time

at Ship,

Loug. in Time

June

Cor. for

29

June
.

9,

Declination

22

of ohiervation.

Vp- 50 n
6

56'

0,

38" N.

39
29

Diff.

+ 12"'08

+6

App. Time at G., June

Declin. Noon,

at time

8,

22

53

44

16

90

Pour

Distance

67

Digitized by

Google

TABIATION PBOM AH AZIMUTH.

2.

For

tlit

Observed alt L. L.
-

Index and dip


Seraidiameter

App.

-t-

18

35'

20"

1'

+ 11

15

of centre

alt.

179

trot eo-altUude.

Refraction -Parallax

40

47
.

True altitude

3.

For

the true

azimuth and thtncethe variation.

16'

Polar distance

67

Co-altitude

71

12

Co-latitude

...

39

13

...

S3

44

->S

17

31

48

49

31

28

103

34 E.

92

36 E.

10

58 W., the true azimuth being to the

sum

ram- co-alt.

sum

Tim

-co-lat.

Azimuth

S.

Compaas Azimuth S.
Variation

8'

40
0

Comp.
Comp.

Arith.
Arith.

sin 0 '02378
sin

0-19911

left of

the observed.

The computation marked (3) in each of these two examples


But
haa been conducted in imitation of that at page 160.
there is another form for finding an angle A, from the three
sides a, b, c, of a spherical triangle, which is somewhat
shorter than that above, namely, the form

being half the

sum

of the Bides.

If

we

call

the altitude a,

Digitized By

Google

VABIATIOH EEOM AS AZIMUTH.

180
the latitude

and put

I,

and the

s for the \

sum

co- declination or polar distance

of these three, the formula, after

an

obvious transformation, will give


sin \

Azimuth

V~ ^
i

cos

The work

cos

of step (3) above, by this formula,

Polar distance
Altitude

Latitude

...
...
...

4 sum
sum- Polar

o,

disk

67"

3'

is

0-02378

47

20

Arith.

Comp.

cob

50

47

.Arith.

Comp.

cos 0-19911

63

18

48

15

32

method of working the step which we


in the following blaul; form
It

as follows

16"

IS

is this

shall indicate

Blank Fohm.

Variation of the Compass from an Azimuth.

[The azimuth ia to be estimated from the S. in N. Iat.,


and from the N. in S. Iat. towards the E. when the altitude
increasing, and towards the W. when it is decreasing.]
:

is

1.

For flic declination.

2.

For the
(Tj.

t.

G.

Soon

Diff. in 1

No. of hours x

Tj.)

Correction

im. Wlian the object


in is

got

and

Polar Distance
*

true altitude.

L. or \l.

Docliu.

lit

iiic-

fa a star, the dcclionco from tho Nautical Alms-

r,lti:jdo requires

no correction

amidiamelor and parallax.


.

The Greenwich

date,

mean

time,

may he

neter, properly corrected for gain or 1 0*8.

obtained from the chro-

TAHIATIOK FEOM AN AZIMUTH.

3.

For

181

Asimuth, and thence the Variation-.

the true

Polar distance

."

Altitude

Latitude

Aritli.

Comp.

cos

Aritli.

Comp.

cos

2)~.T7.
J

sum

Polar dist.

1 Azimuth

2
True Azimuth

Compass Azimuth

55"

V,"])C!i

i'm

Ltuc:

azimuth

is k,

the left of the compass azimuth,

subtract; n-lien to the right, add.

Yakiatios

Note. "When the object observed is on the meridian, its


bearing by compass will be tlio variation, which will bo ~W.
if the meridian be to the left of the compass bearing, and
E.

if it

be to the right,

Examples for Exercise.


July 20, 1858, in latitude 21 42' N. true, and longitude 62 E. by account, the sun's observed azimuth was
S. 100 16'
at 1 h 4 n> a.m. apparent time ; the altitude of his
1.

lower limb was 23 36', allowing for index error, and the
height of the eye 24 feet : required the variation of the com-

Ans. variation 3 42' ~W.

pass ?

2. October 28, 1858, in latitude 36 18' S. true, and longitude 15 30' E. by account, the sun's observed azimuth was

at abont 6 b

SO"

p.m.

of his lower limb was 12

35',

allowing for index error, and

If . 86 34"

W.,

the height of the eye was 30 feet

mean time the


j

altitude

required the variation of

Ans. variation 10 361 "W.


1858, in latitude 25 32' N. true, and
longitude 85 W. by account, the sun's observed azimuth was
the compass ?
3.

November

8,

TABIATION FEOM AN AZIMUTH.

1S2

W., at about 4h 15 m

S. 58 32'

of his lower limb was 15

+1'

20",

p.m.

37',

mean time;

feet: required the

Ans. variation 5 26' E.

variation of the compass ?

May

the" altitude

the index correction was

and the height of the eye 15


21, 1858, in latitude 52 12'

W. true, the sun's


azimuth by compass was S. 82 58' "W., and the altitude of
his lower limb was 23 46'.
The chronometer showed the
Greenwich time of the observation to be l7 h 56 m 34 s May 20.
The index correction was +2' 30", and the height of the eye
12 feet required the variation of the compass ?
4.

Ana. variation 9 24' E.

Note.

The objeet of the preceding

tion of the compass

the

N. point

is

articles

on the

varia-

to determine the angular departure of

of the instrument from the true

N. point of

the horizon, at the time and under the circumstances in


which the amplitude or azimuth is taken. If no provision
have been made for neutralizing the influence of the ship
itself on the needle, the variation thus determined will be
compounded of variation proper and of the deviation from
the position in which the needle would otherwise settle,
caused by the local attraction. In iron ships this local attraction is of course considerable, and it is a great deal
influenced by the position in reference to the meridian in
which the ship is built. To determine the extent to which
the deviation affects the variation proper, experiments must

he made before the ship proceeds to

sea,

by turning her head

in different directions, and comparing her compass with

another compass on shore.


local disturbances

To

free the variation

from the

thus ascertained,

artificial magnets, and


recommended by the Astronomer Eoyal to be employed in the manner directed by him
in a pamphlet to be had of the publisher of the present

small boxes of iron chain, are

treatise.

An

account of the necessary operations for Com-

pass Correction will also be found in Mr. Grantham's work


on " Iron Ship-Building," before alluded to.

183

FINDING THE TIME AT 8EA.

CHAPTEE

IV.

ON FINDING THE TIME AT

The
first

determination of the time at sea

consequence.

SEA.

is

a problem of the

It is indispensably necessary to the dis-

covery of the correct longitude, which indeed

is

nothing

interval between the ship's time and Greenwich time at the same instant, converted into degrees and
minutes. As in most of the other problems of Nautical

more than the

of the quantity sought we have


Astronomy, so here
generally some approximate value, more or less incorrect,
and this is turned to account in tho operation for finding
the true value. At first Bight the statement would appear
:

contradictory, that erroneous data could aid in conducting

to correct conclusions; but Nautical

Astronomy abounds in

instances where very material errors in the values with

which we work have no practical influence upon the results


arrived

The reason

at.

is,

that these erroneous values never

enter directly into the mathematical portion of the inquiry

they merely serve the purpose of suggesting to us certain


other quantities with which they are connected

The

which are
employed in the computation and which are such
be incapable of error beyond a very limited extent.
longitude ly account, and her estimated time,

actually
as to

ship'3

never enter into the trigonometrical calculation of any


nautical problem, claiming accuracy of result: but for the

preparatory reductions for the sun's


equation of time, they

and

it is

declination, or the

may be used with

every confidence

only for such like purposes that they are used.

These quantities vary so

little

in a considerable interval of

equivalent

time, that an error in time of one hour


error of 15

of longitude

clination to the extent of 1'

the error will not average

will

not

and as

affect

to an

the sun'B de-

to the equation of time,

1*.

Digiiized by

Google

TIME 7B0M SUN'S ALTITUDE.

184

Time deducedfront an Altitude of the Sun.


Keferring to the diagram at page 172,

if

3 be the place of

the sun at the time of observation, there will be given in the


spherical triangle P Z S, the co-latitude P Z, the co-altitude
Z 8, and the polar distance P S, to find the hour angle
Z P S, which measures the apparent time from noon.
As at page 179, putting a for the altitude of S, p for its

polar distance,

of all three,

we

shall

sin

which

for the half

sum

have for the hour angle P.


co s a

p.

and *

for the latitude of Z,

am (s~a)

C09 I

derived from the formula following (Spherical Trig,

is

18):-

by obvious substitutions.
The hour angle P being thus found, and converted into
time, we shall have the apparent time at the ship ; and by
applying the equation of time, shall thence get the

mean

time at the ship, as in the examples following.

It

Note.

will

liability to

mistake,

prevent confusion, and consequently


if

all

the time at a place at any instant

be always measured from the noon at the place preceding


that instant

that is, if it be always converted into time


Thus, 10' 20 m a.m. in civil reckoning, means
10h 20 m past the preceding midnight it is better to regard
it as 22h 20m pflBfc the preceding noon, pushing the date one
day back, so that 10' 20 m a.m. Jan. 4, is the same as 22"> 2001
;

p.m.

Jan. 3.

Examples.
1.

In

Time from an Altitude of the Sun.

latitude 50 3<y

N.

true,

and longitude 110

"W".

by

mean of a set of altitudes of the sun's lower


limb was 11 0 50", the mean of the corresponding times
account, the

Digitized by

Google

TIME PROM BUS'S ALTITUDE.


"by

the

was 4

watolt

S' 20", and


mean time

From
7 m 42 s .

6'

56" S. Diff. in

For thi true

altitude

and polar

a,

44" 1

- 7'

Index and dip

+15

58 J

....

alt. centre

-4
11

centre

7 m 42"

Equa. of time*
Cor. for 12s .

Long.

HOW.

Mean time at

&

12

Of.

0"

Declin. at G.

6'

+11

00

-85S"

12
10-296

0" p. m.

5 nearly.

56"

S.

Diff. in

47

18

43 S

18

43

90

Polar distance

7" 52"

4 h 45 m
+7 20

Time per watch.

39

4 25

Diff. in l b,

+ 10

Eqda. of Trail

Deolikatiok

04

11

Eef. -Parallax
alt.

15' 58".

distance p.

11 0' 50"

Scmidiam.

App.

noon.

Q-.

+58". Equa. of Time,

1",

+0"-85S. Semi-diameter,

Diff.

True

Noon

was

error of the watch ?

Obs. alt

Cor.forl2>>

185

p.m., the index correction

and the

the Nautical Almanac at

Sun's Declin. 0

1.

45 m

1'

the height of the eye 20 feet: required, the

at the Bhip,

+5S"i
124

1
',

702
5

6,0)70,7"
11' 47"

* This

is

taken from page I of the month

in the Nautical

Almanac

there will be no sensible error in regarding the time as apparent instead of

The Almanac itself directs whether the reduced equation


added to or subtracted from the apparent time at the ship.
mean.

is

to be

Digitized Dy

Google

186-

TIME FE01I

2.

For

mean

the

Altitude

Latitude

18

43*

63

75
64

50
52

34

ieom-alt.

50

error of the watch.

Arith. Comp. cos 0-196489

30

90

ALTITUDE.

and

25"

4'

2)151

Polur distance

isum

STJH'S

time at the tkip,

11

Arith.

Comp.

sin 0-000006

cos 9-385411

Bin

9-956812

2)19-588718
i

Hour

angle

36

= 72
In Time = 4 h

.'.Hour angle

0'

48'

36

48 m

-7

Equation of time

Apparent time at ship

6*

52

Mean time at ship.


Mean time per watch.

4" 40 14-

45

0" 4 m

The student

Watch fast

46*

for

mean time

at ship.

will readily perceive the object of finding the

error of the watch.


ciently

sinO-769359

good one to

The watch being assumed to be a suffibe depended upon for regularity during

the short time occupied in performing the foregoing calculation,

when the operation

is

will still

making the
show what the time

finished the watch

proper allowance for the error

where the observation was made comparing


the chronometer which is never
its situation
we shall at once see by how
much it differed from the time at the place of observation, at
the instant that observation was made that is, we Bhall get
what is called the error of the chronometer on mean time at
at the place

is

it,

therefore,

now with

disturbed from

A memorandum being made of this error, so that


we mny always be able to allow for it when consulting the

the place.

The

sine of this is cos 18' 43".

exceeds 90, instead of the sine of


excess,

it

And whenever

the polar distance

we may always take

the cosine of the

and Urns avoid the trouble of subtracting from 180.

Digitized 0/

Google

TIME FBOM SON'S ALTITUDE.


chronometer,

we may

at

187

any future instant learn the time

at that instant, at the place

left,

provided,

at least, the

chronometer can he depended upon for regularity during


Hence, hy again finding the mean time at
and as before the error of the chronometer on that

the interval.
ship,

time, the difference of the errors will be the difference of

longitude in time between the two situations of the ship.

But we must

defer further remarks

on

this subject

till

next

article.

Since the determination of the time at sea requires that


the altitude of the object observed should be taken with

more thau ordinary accuracy, a


purpose

is

single observation for this

seldom considered as sufficient;

it

is,

therefore,

usual to take a set of altitudes, and to employ the mean of

the whole, taking the mean of the corresponding times by


watch as the estimated time, as in the following example.
2. August 16, 1858, the following observations were

taken in latitude 36 30' N. true, and longitude 153 E. by


3' 5", and the height of
account the iudex correction was

the eye 27 feet

required the

mean time

at the ship, and

the error of the watch ?


Timet per Watch.
4"

40-

0' r.M.

44

17

6)210

32

Attt. Sun's L. L.

24" 18'

"We are to proceed as if the observed altitude of the sun's


lower limb were 23 44' 48", and the corresponding time
per watch 4h 42 m & p.m.

TIMB PBOM

188

Obs.

For

alt.

L.

12"

8'

+ 1__

....
.

alt. centre

Eqna. of Time

4"'

23

23

17'

Diff.

-9

Cor. for 18 hours

52

20

-2

50

24

-49

81

4" 42m 6* r.*.

Time per Watch, Ang. 1C

-10

Long. 153 E.

M. Time

"1

50

15

alt. centre

Parallax
True

distance.

23 44' 48"

Semi-diameter

App.

and polar

Index and Dip.

Eef.

ALTITUDE.

BITIT'S

the true altitude

at 6.. Aug. 15'

12

18

Noondecliu. at G.

30 nearly

14"

5'

13" S.

Diff.

-47"'13

4713

"H

Cor. for 181 hours

Declination-

13

Polar distance

7G

32

50

41

19

3770

236

90

Agreeably to what

is

recommended

6,0)87,1-9

in the

Greenwich, at the instant of observation,

noon preceding that instant, 24


to bring this about,

same as

if

we bad

1'

is

-14' 32"

Notb

at p. 184, the time at

measured from the Greenwich

being tacitly added to the time per watch,

and the date

therefore put one day back.

This

is

the

actually subtracted the longitude in time from the time

par watch, getting for remainder (neglecting the


5 b 30 preceding the noon of Aug. 16, which
the noon of Aug. 15.

is

6')

- 5 h 30

that

is,

the same as 18 h 30 m after

TIME

IT

BOH

For mean time

2.

SUM'S ALTITUDE.

at ship

awl

error of the watch.

24"

Altitnde

23" 50'

Latitude

36

30

Arith.

7G

20

Arith.

2)136

29

14

14

52

21

28

Polar distance

""'68

isum
.}

Bum

44

-alt.

Comp.
Comp.

cos

0-094837

sin

0-012804

cos 0-563811

sin 9-844919

3)19-521501

IHourangle

35"

12' 1"

sin

9760750

2
.'.

Horn angle

InTime

= 70
= 4"

may be remarked

36*

45

44

42

Oh

It

24

41

+4

Equation of time

Mean time
Mean time

3>u

38"

here that

which,

the polar distance

Bince

Apparent time at

Watch
an.

bhip.

at ship.

per watch.

iW for mean time at ship.

error of a few seconds in

of course,

is

to

be

expected

the estimated time and estimated longitude are


both to some extent incorrect, will make no appreciable
difference in the value of the hour-auglo deduced.

distance is always a

polar

large

never

arc

much

The
less

than C7", and for large area the tabular differences of the
sines are always small. Whatever error there may be in the
polar distance, there will be half that error in the i sum,

and

in the h

sum

alt.

hut as the logs connected with

are log cos and log sin, their errors oppose ,one
[See, however, the remarks at page 190.]
It appears, from the Hourly Diff. in the declination, that if the
combined errors of the time and longitude in the foregoing
example amounted to so mueli as l' of time, or 15 of
longitude, the error in the polar distance would be 47*.

these

another.

Suppose, for greater convenience of calculation, we assume


the error to he 44', and let us see what efFect such an
error would have upon the resulting time at the ship.

Digitized By

Google

TIME FHOM ara'B ALTITUDE.

190
Altitude

23

EC

latitude

36

Polar distance

J sum
I sum

,.
alt

21"
Aritb. oomp. cos 0-094837

30

76

36

68

14

30

44

2*

.sin

35

12'

14"

28

Aritb. corop. sin 0-012826

cos 9 "569014

0-844902

2)19-521579
1

Hour

angle

.-.Hour angle

=70

24

In time

41 38*

4h

Consequently, there

Apparent time at

ship.

only 2 a difference in the ship's time,

is

although the estimated time from which, with the longitude,

has been derived,

may have been

three quarters of an hour


wrong, and the longitude nearly 3 in error, both errors
it

supposed to conspire.
The watch is, therefore,
s
Should it happen that the estimated time
slow.
so far wrong as to differ from the time deduced as above
by so many minutes as to cause the change of declination,
due to those minutes, to amount to several seconds which
may be ascertained by glancing at the "Diff. fori ','' it
will be prudent to re-calculate the declination and hour
being

C 3 m 40
ia

aDgle with the corrected time: the second result


course, be

more

correct,

will,

of

and may be depended upon to within

if the longitude by account be so


as 4 or 5 in error, provided, that is, that the latitude

a second or two, even

much

be correct, and that the altitudes have been taken with care.
Tbe student is not to suppose that, when his computed
time
is

differs considerably

from that by watch, that his result

therefore the less to be

depended upon, or that the

necessity for repeating the calculation

the

more

ecliptic,

imperative.

In certain

the error of the watch

is,

on that account,

positions of the sun on the

may amount

to so

much

as

an hour or two, and yet a repetition of the process, for


may be unnecessary while in
certain other positions of the Bun it may be prudent to
further correcting the time,

TIME THOM BDN'S ALTITUDE.

101

repeat the work, though the error of the watch amount to

By comparing the Hourly Differences

but 10 or 12 minutes.

of Declination for the months of

those for

June and December, with

March and September, the reason of this will at


it must be remembered that a repetition of
first result only when the error in

once be seeu:

the work can correct the

due to the error in the time,

declination,

amount

is

of sufficient

to affect sensibly the logarithmic calculation.

The

circumstances the most unfavourable for strict accuracy


are those

but

where the

" Diff. for

sum, in the above operation,

3 or 4, from 90,

little,

"

is

at

all

and

at the

differs

same time the

considerable, as in the instance next

following.

Note.
that

all

In repeating the operation,

we have

distance,

the i
cosines

to do

is

it

will

be noticed

to apply the correction to the polar

and half that correction to the ^ sum, and also to


alt. ;
and then to take out the sines and
or, which is better, to proceed in the manner to be

sum

presently noticed.
3. The mean time used in computing the declination
was l 40 m the declination found was 1Q 36' 30* S., the
altitude was 50' 48" N., the true altitude 29 38' 26", and
11

the equation of time 8 m 45-7 subtractive from apparent

time

required the error of the watch ?

20* 38' 26"


48
0

Altitude
.
Latitude
.
Polar distance

91

86

SO

Hour angle

17

35

45}

12

Arith. comp. cos (H0B263


Aritb. comp. sin 0-000171

SO

Hour angle
In time
Eqna. nf tin

/.Meantime atahip=

.-.

Error of Watch

= 32".

DigiiizM by

Google

TIME FROM SUM'S ALTITUDE.

192

Suppose, now, that this error gives a correction of


the polar

mended

then, applying this correction

dist.,

as

+22*

91" 36' 52"

Polar distance

Aiith.

0-19926S
Comp. sin 0-000172

Bum
sum

88

39

.cos

alt.

56

23

13

sin 9

Hmir angle

17

85

214

Bin9-4S0283

35

10

43

in

recom-

we have

above,

S'840o93

920538

2)18-960566

.-.

Ab
it

Hour

this is

angle

3s

less

=&

20" 43' in time.

than the apparent time before deduced,


31 m 57 s.

follows that the error of the watch is

We

take this opportunity of showing the practical advan-

tage of using the logarithmic tables as recommended in the


foot-note at p. 142.
arcs,

we

Disregarding the seconds in the several

Bhall take out the several logarithmic values to

the

degrees and minutes only, writing against each the tabular


difference to 100" : we Bhall then multiply each difference

by the number of seconds which has been reserved, cutting


two figures from the right of the product for the division
by 100, and Bhall then incorporate the aggregate of these
quotients, previously marked + or
as directed in the
foot note, with the sum of the logs extracted.
In repeating
the operation, we merely have to increase three arcs by 22",
11", and 11" respectively
we shall therefore have only to
multiply the differences against the arcs by these numbers
in order, cutting off two figures as before. If the cutting
off be postponed till the Bum of the products is found, strict
accuracy will be secured to the final figure of the result and
this is the plan we shall adopt in what follows.
off

OigilizM By

Google

BLANK rOHM TOE TIME

194

tation about the proper sign to be written against the tabular difference

every sine

+, and every

is

cosine is

but

complements always require a change of the sign that otherwise would be written.
that

It will be

remembered

in the

above

was not the sine of 91 36' that was taken from the
but the cosine of 1 36' so that the tabular difference

it

table,

only

G would have been marked

it is

the complement of

is written down.
It need scarcely be
remarked that when seconds are to be subtracted from the

that cosine which

arcs, the signs are opposite to those


if

which would be annexed

We

the seconds were to be added.

now

shall

give

the

blank form for computing the hour-angle, and for correcting


the

first

result in the

Blank Foem.

manner here

explained.

Time at Ship from the Latitude and

Sitri's

Altitude.
1.

For

and

the true altitude, jwlar distance,

Oba.

App.
Ref.

equation of time.

alt.

Index and Dip

....

8emi-diam.
alt.

centre

Parallax
True

alt.

centre

Equa. of Time

.'"

op Time

Diff.

Cor. for the boors


Eqiia.

'_!_V

hours

77' Cor.*

Time per watch

.*

Long, in time

Mean Time

at G.

Noon

Deolia. at G.
Cor. for hours past noon

nearly.
.

.'

."

DilT.

....

..."
x

hours

Dbolikation

90

"

60)

Polab Distance
The Nautical aim. directs whether the equa. of time

Cor.
is

additive or

Digitized 0/

Google

FBOM SUH'S

2.

For the mean time


-

and

error of the match.

[Secomh reserved.}
Comp.

195

AIiTIXDDE.

at ship,

Pt*. for tea.

Biff.

sin

....

Bqoa. of Time

Mean time at ship.


Mean time per watch.

Car. of this time

fast

slow

{See extra

[Extra work for correcting the first

Sea. of cor.

for

M. time at
ship.

worh bdov.)

result.}

Pit. for the sea.

..(.."

= Cor. of
= in

hour angle

.'

* If the polar distance exceed 90", the comp. cos of the excess

is

to

he

taken, in which case this sign will be plus.

This difference is to be taken oat of the Table at the same time as the

TIME IBOM ALTITUDE OF A STAB.

196
Note.

In

taking an altitude, for the purposes of the

present problem,

it is

desirable that the object should

as nearly due E. or due

W.

be

as possible, because in that

a small error in the altitude will have the least


on the time : the nearer the object is to the
meridian, when between it and the prime vertical, the less
situation,

influence

favourable

is

tropics,

the observation to accuracy in the deduced

When, however, the

hour-angle.

place is between the

and the declination of the same name as the latisun to the meridian will not be

tude, the proximity of the

an objection
in altitude

since

under tbeee circumstances, his motion


rapid for a good observation at

is sufficiently

any point of his course.

Time deduced from

ail

Altitude of a Star.

"When instead of the sun, the

object observed

is

star,

though the trigonometrical computation for finding the


hour-angle remains the same, some of the preparatory work,

The

in step (1) of the foregoing form, is different.

mination of the sun's hour-angle gave


apparent time : but a

no information

deter-

us at once the

star's hour-angle alone

as to the time of observation

can give ua
;

yet

if

the

star's Might Ascension be also known, then, combining this


with the hour-angle, either by addition or subtraction, we
shall know the B, A. of the meridian ; and then again sub-

tracting from this the E. A. of the Bun, we shall finally obtain

the sun's hour-angle, and thence the time at the ship

when

[See Note, p. 197.]


E. A. at the time of observation, and the mean

the star was observed.

The

star's

sun's E. A. at Greenwich noon, are both given in the

Nautical Almanac

and therefore the

latter being

reduced

we shall have 'the


B. A. of each object at the same instant; and as just
explained, the Btar's hour-angle at that instant being found,

to the Greenwich time of observation,

DigiiizM by

Google

TIME TBOM ALTITUDE Or A ST Alt

we shall have

the E. A. of the

197

mean sun and of the

meri-

dian of the ship at the same instant, and therefore the mean

A single

time.

example will

sufficiently illustrate

what

is

here said.

Note. Eight

W.

from

ascension, bo

to E., or from the

it

remembered,

first

is

measured

point of Aries easterly

from CP up to 360, that is, in a direction contrary to the


apparent diurnal rotation of the heavens: when therefore
a star

to the

is

added to

its

when
With

is

it

W.

must be
and
must be subtracted.

of the meridian, its hour-angle

E. A. to get the E. A. of the meridian


to the E., its hour-angle

regard to the sun, whether

it

dian, its B. A., subtracted from the


will give the sun's hour-aogle

The student must

especially

be W. or E, of the meri-

E. A. of the meridian,

from preceding noon.

remember, that whenever we

Bpeak of one E. A. as being subtracted from another, with

a view to obtaining a third E. A., it is always tacitly supposed that 24 h is added to the second when the first is
greater than

it.

And whenever

another to get a third, 24 h

sum

if it

is

exceed that quantity.

one E. A,

is

to be added to

always suppressed from the


It is plaiu that there

is

no

displacement of a celestial object by increasing its E. A. by


24*, or by 360 if the E. A. be expressed in angular measure.

The

hour-angle of a star, or planet, or of the moon,

least angular distance

be to the W. or E.

is its

from the meridian, whether the object

but the hour-angle of the sun is usuallymeasured westward, that is, from the preceding noon.

Example.

Time from an Altitude of a

Star.

April 22, 1858, in true latitude 42 12' N. and longitude


by account 44 30* E. when the mean time per watch was
8 h 2 m p. m. the observed altitude of the star Arcturus, eastward of the meridian was 73 48' in artificial horizon the
error of the instrument was + 7' 34": required the error
:

of the watch ?

DigiiizM By

Google

198
1.

TIME PEOM ALTITUDE OP A STAB.

Forthe

e
polar distance, and the R. A. of mean sun.

trite attitude, the

73" 48'

Observed Alt
Index cor.
.

-.

+7

2)

Apparent

When the

36 56' 30 "

artificial

horizon

Long. 44J E. in time

Mean Time

used, there

8>"

58

at G.

A. mean sun at G., noon


1\ + 9" 86 .-. for 6"
A. MEAN BUN AT TIS1E OP

no correction for Dip.

0*

0 nearly.
2" l" 0"51
4-59-1 6

OfiS.

is

twice the app. alt.

is

Biff, for

Is

correction being applied, the result

Time per watch

R.

'

Tkdb Alt

Not*

The indei

0"

84

73 55 34"
86 57' 47"
17'

alt

Befraction

19 55'

Stab's E. A. 14" S 14', Declih.

6" N.

90
.".

Polas niBiisoE

= 70

54

329,ft8Cor. for

14

The R. A.

of the

mean

26 Watch i!oa on

ann, at

mean noon

main timo

at place.

at Greenwich, is given at

page II of the Nautical Almanac, under the head of "Sidereal Time."'

DigiiizM by

Google

BLANK rOKM POE TIME EBOH A

199

STAE.

In this additional time the E. A. of the mean sun would


be increased by about 2 s as appears by referring to the
" Diff. for l h ."
m 24". *
Hence the error of the watch is
In the following blank form we shall provide for the insertion in step 2 of the tabular differences, as in the above
,

But

example.

since a star's declination

constant for a

is

considerable interval of time, there will be no correction

required for change of polar distance; a correction, however, for

change

in

the

mean

sun's B. A.

may be

necessary,

as in the preceding example.

Blank Foem.

Time

at Ship from the Latitude,

and a

Star's Altitude.

FarUte true akiiude,

1.

tlx polar distance,


io

and

por watch

the S,

A. of memiaiwt.

.."..*

Thub Alt.

Note.When

the allituda

i*

taken in the nrtiHcbd horiion,


is no
The index

there

correction lor Dip.

correction

made, the result

is

beiug

twice the

Staji's

The mean

sun's daily advance in

It.

quently his hourly increase in R. A.

In the next

comp. cos of the excess

is

to the "Diff." will be

+.

The

A.
is

is

romt

to be taken,

additional correction of the time

omitted iu most

di

uniformly 3" 56"- 55 ...

it is

exceed

when the

conse-

90, the

sign annexed

here noticed is improperly

books on Nautical Astronomy.

determining the longitude of the ship,

9**86.

step, if the polar distance

For the purpose of

of the first importance that the

ship's time should be obtained with all possible accuracy.

example, the corrected mean time at the ship

is

8 b 16- 24*.

Io the above

EITBCT OP AH EBBOB IN LATITUDE.

200

For

2.

t!ie

mean

..*..'.

time at tie ship,

and

."

error of the vateli.


Tab. DiJ.

Ptt.fo?

Sres.

Cot for second*

Note.

The error in

in the time per watch,

watch

In

still

this

the bud's R. A., due to this error

may now be

allowed

and the

for,

further slightly corrected.

and the preceding problem, the latitude

to be correct

assumed

is

but, in general, the error of a mile or so in

datum will have but very little influence on the time.


We shall, by way of illustration, suppose the latitude in the
example solved above to be 1' below the truth, and examine
The tabular
into tbe effect of this error on the time.
thiB

differences, already before us, will enable us to

but

little

We are
latitude,

and

Tah.

and therefore 30" to tbe J

191

765

alt.,

to find the " parts" for these addi-

tional seconds, just as the parts

found

do this with

trouble, as in the margin.

to conceive 60" added to the

sum, and also 30" to the \ sum

for tbe seconds before.

pearo from the result, that 17

It apis

Parts.

273 +

_
2

were
Cor.

11*60
22950
8190

)j

to be

DigiiizM by

Google

EFFECT OF AN EBBOB IN LATITUDE.

201

subtracted from 9-688521, thus reducing it to 9 683504


sin 28 50' 58"V . hour-angle
57 41' 56"
31 50m 48* in

'

The tabular

time, within about a quarter of a Becond.

difference 382, against Bin 28 51' 2", enables us to get the

correction for seconds due to the difference

thus

1700 -=-382 =

subtracted from 28 51'

17, at

once

the number of seconds to be


If the latitude had been 60"

4,

2".

too great, then the signs of the "parts" in the margin

would

be changed

all

382

1700

.*,

= 4 = the

seconds to be added to 28 61' 2"; that

is,

number of

the i hour-angle

would have been 28 51' 6", and the hour-angle, 57 42' 12",
is 3 h 50m 49 a
If the latitude be assumed to be 2' below the truth, then
which in time,

each of the parts should be doubled, and half the sum


this half sum is obviously tbe same as the

taken: but
whole sum

33,00 above,

number

= =

the
3300 -j- 382
9
hour-angle, which
and consequently the hour

hence

of seconds of correction of the

angle

is

therefore 28 50' 53",

angle

is

57" 41' 46"

= 31

J-

50"1 47 s in time.

"Without cor-

recting the time in this way, through the correction of the


hour-angle,

we may

at once apply to the former the correc-

tion for the seconds of arc in time

but

thus 18"

= 1* of time

as fractions of a second of time are disregarded, as well

in the original result, aB in the correction of it, the corrected

time might on this account


It

is

still

err

by nearly a second.

easy to see, from what has

readily the navigator

may form an

now been

said,

how

accurate estimate of his

error in the time, arising from a small error in the latitude

from which it has been deduced. But if the error of latitude amount to several minutes, the correction for the time
found in

this

manner

will

be only approximative since the

tabular differences are not constant from minute to minute.

If instead of the latitude,

it

be the altitude that

is

supposed

to involve a small error, the correction of the time due to


that error

may be found

in a similar manner, but with even

less trouble, because the only arcs affected

by the error
K

will

HETECT OP AN BKEOB IS ALTITUDE.

202
be the
Tab.

sum, and the i earn

273

Thus,

alt.

increased by 30", and the 1

'

correction

margin

= 3*

will

and

be found

15600

-j-

Bum

alt.

as in the

382

= 41"

in time nearly, so that a small

error in the altitude will have a

much

greater effect

the time than an equal error in the latitude


cases it appears that

the altitude

be diminished by 30, bo that the

will

2)311 40

will

if

the i euro will be

be iucreose(i bv

Pan*.
22850
8190

Diff.

765

npon

but in both
an error in excess {not exceeding 2')
:

produce about the same error ou the time as an equal

error in defect, the errors in the time having opposite signs.

Beyond

2'

of error, whether in the latitude or in the

must be regarded

tude, the correction of the time

alti-

as only

approximative.

^Examples for 'Exercise.


1.

March

15, 1858, the

mean of a

set of altitudes of

the

1
sun's L. L., in lat. 16 28' 30" N-, and long. 99 30

account, was 10 36' 20", the

W., by
mean of the corresponding

times per watch was 6 h 45 a.m., the index correction of


the sextant was
required the

watch

2'

30",

mean time

and the height of the eye 22 feet:


at the Bhip, and the error of the

Ana. mean time at nhip 6 h 56 m 14"

error of watch 11* 14* bIow.


April 26, 1858, in lat.29 47' 45" S.,and long, by account
the mean of a set of altitudes of the star Altair
was 25 14' 20" to the E. of N., the mean of the times per
10*,
watch waB 2 h 12 m 30* a.m., the index correction was
2.

31

1' E-,

and the height of the eye 20 feet required the mean time at
the ship and the error of the watch ?
Ans. mean time at ship lh 51"; error of watch 21 m 30s fast.
Note. In a similar manner may the time be deduced
from an altitude of a planet; the only difference being that,
:

as in the case of the sun, the observed altitude is to be corrected for parallax and semi-diameter as well as for refraction.

Digitized Dy

Google

CHAPTER
ON FINDING THE EEBOB AND

V.

OF THE CH HO NO METER.

EA.TE

In the preceding chapter we have

discussed at

some

length the interesting problem of determining the time at


aea,

and thence the error of the watch.

If the time thus

determined be compared with the time shown by the chronometer, we shall in like manner, by taking the difference of
the two times, find the error of the chronometer on mean
time at the place of observation. It is still more important,

however, to know the error of the chronometer on mean

time at Greenwich; and this may be easily ascertained provided the longitude of the place of observation be pretty
accurately

known

for, as

already seen,

if

the

mean time

at

the place and the longitude of that place be both known, the
esacfc

time at Greenwich

is

very readily obtained, and the

difference between this time and that

meter

is

the error on Greenwich

mean

All chronometers have an error

kept of

its

time.

this is always accurately

determined, usually at an observatory


is

shown by the chrono-

where a memorandum

performance as a time-keeper.

The purchaser

always receives a certificate stating how much the chrono-

meter was too

fast or too slow, for

at the Greenwich

mean noon

mean time

at Greenwich,

at a specified date

how much it gains or loses, on the average,


mean time that is to say, its daily rate.

and

also

in 24 hours of

The chronometer, accompanied with the proper

of its

error

and

daily rate, is takeD to sea,

certificate

and after any

its daily rate being multiplied by the


of days elapsed and the product called the accumulated rate being combined with the original error, we
are enabled to apply the proper correction to the time

interval of time,

number

actually indicated by the chronometer,

the mean time at Greenwich.

and thus to ascertain

EEBOB AND BATE OF CHRONOMETER.

204

lor example
account 38 45'

Suppose on August 21, in longitude by


when the mean time at the ship, as
last chapter, was 7 h 24 E1

found by the method explained in


p.u., that the

chronometer showed 10 h 2 m 34 s

and that

the following certificate from the Greenwich Observatorv


stated
Aug.

1,

Mean Noon at G.

Daily Sate.

Error of Chron. 2" 4-76 Fast

Bequired the mean time at Greenwich corresponding to the

mean time

at ship ?

Time at Ship Aug. 21


Long. 3S 45'

W.

in tim

Time at G. Aug. 21
n Aug. 1 to Aug. 21, at

9>>

l" ss 20* 9 k

59 m, or 20* 10 h

= 20V.

Correction for Daily rate

In 20"

For original error

Whole

correction

Chronometer showed

2
10*

67-83

2= 34'

Meah Time at G.

Hence, assuming the mean time at ship to have been correctly determined, and the chronometer to have maintained
its rate, the longitude by account is 36' of time in error
that

is, it is

38 54'

W.

too

little

The whole

so that the corrected longitude is

correction of the chronometer for

the 20 days elapsed, that


at Greenwich, being 2 m

is, up to mean noon of Aug. 21


4>75 + 52" subtractive, we may

henceforth employ the following

memorandum

Digitized By

Google

EBBOB ASH BATE OF OHBOHOMETEB.


Aug.

21,

Mean iVoon. at

205

Daily Kale.

G.

Error of Chron. 2= 5G"-75 Fast

^-B Gaining.

But, although implicit confidence

may be

placed in the

original correction, yet we have no security that the daily


rate may not have changed.
It is of importance, therefore,
from time to time to examine into this matter, and instead

of taking the invariability of the original rate for granted, to

In order to
the navigator must wait till his ship
some port or harbour, where it can remain for
If the place have the advantage of an

ascertain the rate at subsequent periods anew.

do

this

efficiently,

arrives at

several days.*

mean time there can always be obtained


mean time must be found by the methods

Observatory, the
if not,

the

explained in the last chapter, using the

artificial

horizon for

taking the altitudes ashore, or else in the way hereafter

The mean time at the place, upon comparison


with the mean time at the same instant as shown by the
chronometer, will give the error of the chronometer on mean
directed.

time at that place.


for the

mean time

A few
let

days after this set of observations

another set be taken, and the

mean

time again determined, and compared with that shown by


the chronometer: the error of the chronometer on
at the place will be again ascertained

the two errors (or their sum,

how much

if

mean time

the difference between

of contrary names) will

Bhow

the time-keeper has gained or lost in the interval

between the two times of observation from which we can


readily find, by proportion, what has been its average gain
or loss in 24 hours of that interval that is, its daily rate.
;

Similar observations should be

made

at intervals as long

* The next best method to this, is to compare the Greenwich time, as


shown by the chronometer, with the Greenwich time as determined by
Lnnar observations to be discussed in next chapter. The difference of the
; and
;

times will ehowtho error of the chronometer on Greenwich mean time

subsequent observations being taken, and the difference of the times found
in like manner, the daily rate of the chronometer, in the interval of time
elapsed,

may

be inferred.

206

EBBOB,

AND BATE OE C1IBONOMETER.

as the ship remains at the place


different daily rates will thus

average of

all

and

it

be deduced

is

probable that

it is

the mean or

these which must be regarded as the daily rate

of the chronometer ; and on the day of the Bhip's departure


a fresh memorandum is to be made of the error of the
chronometer on Greenwich mean time, at the corresponding

Greenwich

date,

and of the

daily rate thus determined.

"Whenever an astronomical clock can be referred to, the


mean time at the
a daily comparison of

necessity for taking observations for the

place will of course be superseded

the chronometer with the mean-time clock will show the


daily rate of the former, which, if not uniform, will enable

ub to determine the mean

daily rate

or the comparison

may

be made at equal intervals of two or three days.


The chronometer itself is not to be carried ashore for the
purpose of comparison
this office for

The

a good seconds watch

following, from "Woodhouse's

an

serve as

Days.

is

to

perform

it.

illustration

the place

Times of mean Noon.

Sept. 8

is

5>

18
21

55

11-97

5
5
5
4

24

55

23-B2

11

54

IS

54

46-93

54

59-46

Astronomy,
Cadiz

Citron, too

11* 64- 18"18

30'8S

p. 804, will

skw.

Differeuca.

41-82

12"64

29-18

16-11

13-07

12-53

0-54

48-03

12-51

38-18

11-85

65-64
Here the sum

of the differences in 16 days

is

65"-64,

and

mean daily rate, estimated by dividing the


s
] 025.
sum by the number of days, is
But both the error of the chronometer on mean time at

accordingly the

the place and

its daily

reference to that

rate

mean time

two following problems

will

may be found without any


at particular instants, as the

show

Digitized 0/

Google

EBEOB OE CHBOHOM. BY EQUAL ALTITUDES.

1.

2b

The

ike

207

Error of the Chronometer ly equal Altitudes


of a Star.

declination of a fixed star is constant,* so is the time

during which the earth performs a rotation on

its

axis

hence, if equal altitudes of a fixed star be taken, one before

and the other

after its meridian passage, the meridian itself

will bisect the angle at the pole

distances,

and therefore

between the two equal polar


between the

half the time elapsed

taken at the same place will make


known tho exact time when the star was on the meridian.
Now, the chronometer may surely be considered as sufficiently

two observations

regular to measure the interval between the observations

with the necessary accuracy, bo that

the chronometer- times

if

of the two observations be added together, and half the

sum

taken, the result will be the chronometer-time of the star's

meridian passage.

But the
which

it is

li.
;

A. of the Btar

and

if

is

the E. A. of the meridian on

from this E. A., increased by 2ib

than the mean sun's E. A., we subtract the latter


preceding Greenwich noon,

we

shall

if less

the

fur

have the mean time at

the place at the instant of transit nearly, as at page 198.

And

applying to this the correction for longitude in time,

we shall have the mean time at Greenwich nearly.


As in deducing this time the buu's E. A. for the preceding
noon was employed, we can now, by means of the "Diff.
for lh ," find what correction of this R. A. is due to the time
past that noon just determined, and apply it to the mean time
of transit nearly, to get the more correct time, just as the like
correction was applied at page 199. The difference between
the time just found and the chronometer-time of transit will
be the error of the chronometer on mean time at the place.
The following is an example
:

* That

is, it

varies insunsilily diiuins the iuterval of time between the

two observations here taken.

Lli j

-.'"J

b,

IBBOB OF CHB01TOM. BY EQUAL ALTITUDES.

208

Obstyva&mt on
Altitudes

T..

Arclwia, Nov. 20, 1858, in longitude 08 30'

the Star

43"

1
30

43

50

Sum

by Chron.
f

10'

43

'.

Times shown

an J IV.

of Meridian.

11" 68- 47-

18

of Times,

so, 7a 42,

65

11
s

Hence the Chronometer-time of the

15u

star's transit is

3 m 51*.
Arctuma R. A. Not.

SO

B- 13'

14*

(to

be increased by

aim
R.

A of mean sun at noon

Wean Time

of tnuuit at place
LorB.eS-SO'El.intimn

Mean time

at Greenwich

IB

20

4S

21

43

2fi

84

IS

14

24*,

as B. A. o!

is greater.)

+10-70

Diff.furl*

nearly

S3S0

25 nearly

107tl

Cor. for lfii*...I84-00

= 2-41-

Subtracting therefore this increase in the sun's E. A. for


the 15 h past the noon,

when the B. A. was

aa above,

we

hare
Mean time
Mean time

of transit at place

shown by

Sl h

46" 41*

chron.

IS

SI

En-orofett.onmeanT.atplaco

42

10

aa

Mean time at G.

In taking the equal altitudes, the best


having selected the

15>>

ll"

15

Error on mean T. at G.

mode of

'

51

1ft

proceed-

which should be at a
considerable distance from the meridian, that is, about three

ing

is this:

or four hours, take

its altitude

advance the index so that

star,

roughly with the sextant, then

it

may

point to degrees and

minutes without any fractions of a minute


the illustration just given, the index

then waiting

till

is

suppose, as in

advanced to 43

10',

the star has attained this altitude, let the

EH HO It OF CHEOSOM. BY EQUAL ALTITUDES.

209

time ll b 55 47* be noted. Now advance tbe index to say


43 30', watting till this altitude ia reached and again note
the time ll h 57 57'.

an

In

additional 20', and wait

like
till

manner advance the index

the altitude 43 50'

noting the time 12* 0m 7 B, and bo on

as

till

is

attained,

many

altitudes

and times before tho meridian passage have been taken as

may

be considered necessary.

Then without disturbing the index from

its last position,

till this last altitude is furnished by the star on the


other side of the meridian, the time 18 7 m 35' being noted
and linked with the time when the equal altitude was
before taken and proceeding in this manner, moving the

wait

11

index 20* the contrary way, after each observation, till


arrive at the altitude 43 10' at first taken, the series of

we

observations will be completed, and the times corresponding


to each pair of equal altitudes will have been noted.

If the

chronometer have gone uniformly during the interval be-

tween the

first

and

last observation,

tho

mean of the times

corresponding to any pair of equal altitudes will be the

same as the mean of the whole, that is, it will be the same
as we should get by dividing the sum of all the times by the
number of pairs, and taking half the quotient. But should
there be a Blight difference, the latter result is to be regarded
as the chronometer-time of the star's transit.

The student

will

not

fail

to notice that this

method of

equal altitudes has tho advantage of not requiring any corrections for the index error of the instrument, yet after the
first
is

of the altitudes,

when

the star has passed the meridian,

taken, the shifting the index of the sextant to

place

may not

its

be accomplished with strict precision,

it

former

would

therefore be better to take each of the altitudes, before the

meridian transit, with a different sextant

to take the first

altitude after the transit with the sextant last used,

and the

remaining altitudes with the other sextants used in reverse


order.
The indexes all remaining untouched, we have
sufficient

security

that the altitudes on one side of the

210

HATE OF CHEONOM. BY EQUAL ALTITUDES.

meridian are really equal to the corresponding altitudes on


the other side, presuming the accuracy of the observations.
i

By

the same method of equal attitudes

may

the time,

by

chronometer, of the sun's meridian passage be deduced,


but on account of the sun's change of declination, in the
interval of the observations, a separate computation for the

influence of this change on the time

we

becomes necessary

think the determination of the time from a single

tude of the sun, as explained in last chapter,

is

alti-

to be pre-

ferred.

2.

To find the Hate of the Chronometer hj equal Altitudes


of the same Star, on the same side of the meridian, on
different nights.

It has already been stated (page 94) that the interval


between two consecutive transits of the same fixed star over
is uuiformly 23 h SB"1 4'09 of mean time

the same meridian

consequently the return of any fixed star to the same meridian

ia

exactly 3 m 55'

on account of the
not only

is

il earlier

strict

at every reappearance.

And

uniformity in the diurnal motion,

the star thus accelerated in its return to the

meridian, but equally in


diurnal path.

its

return to any point in

its

It follows, therefore, that if an altitude of a

and the time by the chronometer be noted,


after the lapse of any number of days the same
on the same side of the meridian, be again taken,
and the time noted it follows that if we divide the difference of these chronometer times by the number of days,
the amount by which the quotient differs from 3 m 55s, 91,
will be the daily error of the chronometer.
For example,
June 6, 185U, at 10b 30 m 12 s by chronometer, and on June
12, at 10 G m 40*, a star on the same side of the meridian
star be taken,

and then

altitude,

ta

had equal

altitudes

required the rate of the chronometer ?

Digitized 0/

Google

211

KATE OP CHBOUOM. BY EQ.OAL ALTITUDES.

12

10

6 Days elapsed
Daily

diff.

is

40

32

54-33

diff.

55-91

is less

As

greater.

1*68 Gaining

plain that the chronometer

it is

23

of chronom.

the daily difference

when

S)

bychro-i.

Trno daily

EaW

It

10 h 30 12'

6 Time by Chronom.

Jane

than

it

must be gaining wlien


ought to be, and losing

in all cases of taking altitudes for

the time, the nearer the object observed

that a single altitude, if carefully taken,

me;m of
must

to the

is

and in the present case

vertical the better,

several altitudes.

to the prime
it is

probable

preferable even

is

If several be taken, the

be read of, and to do this without a


second or two of error, is no easy matter; but in the case

altitudes

all

of a siugle altitude only, the reading off

is

unnecessary

the index should be clamped for that altitude, and the


sextant left untouched

which,

if practicable,

till

the second observation

is

taken,

should be ou a night when the state

of the atmosphere, as indicated by the barometer and ther-

mometer,
first

is

nearly the same as it was on the night of the

observation.

there

is

Of course

here, as in the former problem,

to be no correction for index error.

If different stars
instrument, the

are

mean of

each with a different

observed,

the rates, furnished by the several

pairs of observations, is likely to be the

more correct

rate.

In the foregoing remarks and directions we have said


nothing as to the choice of any particular star or stars,
merely observing

that,

whatever star be selected,

its posi-

tion in the heavens should be as near to the prime vertical

as possible

10 or 12

degrees,

its altitude,

however, should never be less than

because of the changes to which the

refraction at low altitudes is subject

of entire indifference which star

is

but

selected

it is
;

not a matter

for as the

L'l

more

IZ'IdC,

OS BINDING THE LONGITUDE AT

212

SEA.

rapid the motion of an object, the less does any small error
in marking its exact position affect too time corresponding
to that position, the nearer the star

the better: 80 that when

its

is

position

to the equinoctial

in other respects

is

favourable, that star which has the least declination should

always be chosen in observations for time.

CHAPTER

VI.

OK FEUDING THE LONGITUDE AT

The
place

SEA.

longitude of any place on the surface of the globe

ascertained as soon as

we can

discover

is

the time at that

and the time at Greenwich at the same

instant,

since we have only to convert the difference of the two


times into degrees and minutes, reckoning 15 to the hour

to effect the object.

How

to find the time at the place

is

a problem that has been sufficiently discussed in Chapter


IV., and

it

is the office of the

chronometer,

when

properly

corrected for error and accumulated rate, to furnish the

time at Greenwich.

But the time

Greenwich as well as
the time at the place may also be found by direct observations of the sun and moon, or of the moon and a star independently of the chronometer: that is, it can be found by
what is called a Lunar Observation. This method of finding
at

the Greenwich date of an observation and thence the longi-

tude of the place where the observation was taken will be


discussed in the next article

in the present

we

shall infer

that date from the chronometer.

Longitude by Chronometer.
After what has been taught in the two preceding chapters,

but

little

ciples of

need be said here by way of explaining the printhis method ; an example will best convey the

LOMGITTJDE BY CHHOKOMETER.

mode

213

of proceeding, the learner bearing in mind that when

the time at Greenwich


longitude

August

is

E.

16,

when

ia less

than that at the place, the

greater, the longitude ia

W.

1858, in E, longitude, observations were

taken of the sun, as recorded at p. 187. (Ex. 2), when the


ehronometer showed 6h 36 m 40 a a.m. On July 14, the

on Greenwich mean time had been


and its daily rate, to be 3'*5 gaining

error of the chronometer

found to be 2 m 20 a

fast,

required the longitude of the ship ?

r-

a.

- t

DigiiizM Dy

Google

LOHSITTJDE BY CHBOWOMETEB.

214

Note. The mean time at the ship is found by the calcupage 189, the sun's declination being corrected for
the Greenwich time, here inferred from the chronometer to
be 18 h 32 25 8 3. In the operation at page 188, the Greenwich time is estimated from the longitude and time by
account : neither of which is necessary here.
In correcting the chronometer-time for error and rate,
it will be observed that we have first applied the correction
for the time up to the noon of Aug. 15, and have then
lation at

beyond

corrected for the hours


is

the

way

this date.

In strictness this

in which the corrections should

If we had computed the gain upon 32 a


should have treated the time as

compound

at a sort of

interest.

if it

It

18'

be applied.

36 m 40s, we

had been accumulating


is

true that in general

would not lead to any practical error, but if the original correction, the number of days elapsed, and the daily
this

rate,

be

all

considerable, there

might be an error of a second

or so in the Greenwich time.

Blank Fobm.

Longitude by Chronometer.

[DaUJ'TimobyChron.
Original error

Actum,

in

days elapsed

Time corrected

to

.'

..

Days elapsed x

.' }

..

...=.."..

Aconm. rate
..

Daily
Noon

Mean time at GatErrwicn

mean time

...

Boon of Date

Correction for time past noon

"With the

Dailyrale

.,*

at

rate
.

.*

x time past
.

Greenwich thus determined, and

the altitude, observed at the above chronometer-time, find


now,- by the proper form (pages 194 or

199), the corre-

sponding mean time at ship : wo shall then have


* The day is considered to commence at the preceding Greenwich noon,
nnd the time shown bj the chronometer is the approximate time after that
noon.

Digiiized by

Google

LONGITUDE BY CIIEOWOHETEE.
Mean time

at snip

Mean time

at

Greenwich

Longitude

In

time

..'i
i

Inks tha

difference:

tho long,

less,

is E.,

Lonlutude = . . "

..'

if G. time is tlie
otherwise it isW.

-."

E. or "W. according as Greenwich time


greater than ship-time.

which

is

is less

or

was 30

2',

Examples for Exercise.


1.

June 2 the true

altitude of the sun's centre

when the chronometer showed 5 h l m 0* the latitude was


40 5' N. The chronometer on May 20 was 45" slow for
Greenwich time, and its rate 2 1 losing. The sun's decli;

S,

nation at the time of observation was 22

0' 17"

N., and the

corresponding equation of time was 2 m 31 s , to be subtracted


from apparent time required the longitude of the ship ?
:

Ans. longitude, 7 29' 49" W".


2.

May

19, in the afternoon, in latitude 42 16' N., tlie

mean of a set of altitudes of the sun's lower limb was 43 55',


mean of the corresponding times by chronometer was
7 n 0m 56". On March 17, at noon, the chronometer was
the

l m 18' too

fast for

Greenwich mean time, and its rate was


error, and the height

7 S '8 gaining : the sextant had no iudex


of the eye was 25 feet:
Sym's Btcl. C.

lnnrs.

5,;i;r iinr.ii.

Diff. for l*

+ Sl"-38

Kri ;vli,:n w' Ti.f (miIi.

3- 4D-5

from app. time).

Ditt fori', 0"1B

required, the longitude of the ship f

Ans. longitude, 55 44' 45"

TV".

August 20, 1858, in latitude 50 20' N., when the


chronometer showed 2 b 41 m 12 B the observed altitude of the
star Altair was 36 59' 50" W. of the meridian; the index
correction was +6' 28", and the height of the eye 20 feet.
On Aug. 1, at noon, the chronometer was l7 m 45 a slow on
Greenwich mean time, and its daily rate was 4*'3 losing;
3.

required the longitude of the ship ?

Ans. longitude, 141 35' 30" E.

Digitized Dy

Google

LONGITUDE BY LUNAR OBSERVATIOBB.

216

Longitude hj Lunar Observations.

In the foregoing article we have explained how the longitude at Bea may be determined by aid of the chronometer, an instrument of human contrivance, and consequently
liable to those accidents

constructions of

man

and derangements to which


It

are exposed.

is true,

as

all

the

we have

previously shown, the errors and irregularities of the chro-

nometer may from time to time, as suitable opportunities


be discovered and corrected but such opportunities
frequently offer themselves, only at wide intervals, and
during these intervals the mariner has to assume that his
occur,

time-keeper has uniformly maintained

its rate,

as last deter-

mined, and that through whatever changes of climate or

he may have passed, and whatever

fluctuations of weather

hidden influences may have been in operation, nothing has


disturbed this assumed regularity.
ordinary circumstances he
safety, as far us skill

may make

And

in truth, under

this assumption with

and mechanical ingenuity are conmay be regarded as a masterpiece

cerned, the chronometer

of artistic construction, but of so delicate a character that

the greatest care


in

which

it

is

necessary to preserve

leaves the

of which

it is

it

workman's hands.

kept in an apartment by

in the condition
It

is

accordingly

the chronometer-room out

itself

never taken during a voyage

soft cushions, and, like the compass,

it is

imbedded in

suspended upon gimbals,

may not affect it by jerks and


and the atmosphere around it is, as far as possible,
maintained, by means of lamps, at the same temperature, so
so that the motion of the ship

vibrations,

that

it

cold.

may not suffer in its


But notwithstanding

action from varying heat


all

theae precautions,

and

it

is

evidently most desirable to be provided against accidental


injury,

tion

and even against possible imperfections of construc-

to have, in fact, some means to resort to beyond the

reach of accident, and where

all defect

of workmanship is

iONGITTTDE BT L17NAB OBSERVATIONS.

an absolute

impossibility.

Such,

217

means can be furnished

only by the unerring mechanism of the skiea.

The

moon, and stars supply to the mariner a celestial chronometer


and when all other resources fail him, he
may read off his time from the dial-plate of heaven; but to
decipher its indications requires some degree of scientific
knowledge, and involves no inconsiderable amount of mathesun,

matical calculation

in the present article

we

shall investi-

gate the theory, and exhibit the practical application, in as


simple a manner as

we

problem of finding the

can, of the

time at Greenwich, and thence the longitude by the Lottab

Observations.
It

may be

well, however, in

to convey to the learner

a few preliminary remarks,

some general notion of the leading


upon the mathe-

features of this inquiry before entering

matical details.

And first we may observe that of all the heavenly bodies the
moon

is

that whose apparent motion

is

the most rapid, and

consequently that whose change of place in a small portion of

time

is

most

easily detected.

The

beat

way of estimating

the change of place of a moving body in a given interval of


to

measure

end of the

interval

time,

is

is

describing

referred

is

and at the
from some object directly in the path it
its

distance at the beginning

the further the object to which the motion is

moving
it or recede from it in a given interand consequently the more difficult is it to

situated out of this path, the leas does the

body advance towards


val of time,

estimate accurately the difference of distance


interval

is

when

that

small.

Now, the immediate object of a Lunar Observation is to


measure the angular distance at any instant between the
moon and some known object, either directly in or very
nearly in the path she is describing. The theory of the
moon's motion is now so well understood, that what her
distance will be from such known object at any future
instant can always be predicted, and although her motion

Oigiiizod By

Google

lON&ITUDE BY ITTSAB OB 8EBTATTOITS

218
is

not strictly uniform, yet

it

is

sufficiently so, that if the

two instants three hours apart


be previously computed, her distance at any intermediate
instant can be found by proportion, and conversely an intermediate distance being found by observation to have
place, we can in like manner, by proportion, discover the
intermediate time corresponding to that distance. Now, the
distances from the object at

distance of the

moon from each

of the several stars lying

in or very near her path, as also her distance from the sun,
are carefully

computed

for every three hours

of every day

in the year, and for several years in advance, and the results
are all inserted in the Nautical Almanac ; these " Lunar

Distances" occupy from page XIII. to page XVIII, of


every month.

An

observer at Bes, wishing to

know the time

at Green-

wich, measures with his sextant the distance of the


either from the

and

selected stars,

moon

sun, or from one or the other of these


after reducing the observed to the true

distance, in a way hereafter to be explained, he refers to


the Nautical Almanac for that distance, recorded there on

the given day, which

is

the nearest distance preceding, in

order of time, to that be has obtained, against which will

be found the howr, Greenwich mean time, when that recorded


distance had place, and he further knows that his distance
occurred at a more advanced period of Greenwich time.
To find how much more advanced, be takes the difference
between the recorded distance at the hour just found, and
the recorded distance at the third hour afterwards, as also
the difference between his distance and that in the Almanac
at first found

then as the former difference

is

to this, so is

3 h to the additional time required. But a shorter way of


computing the proportional part of the time will be explained hereafter.

From

this brief sketch of tbe course to

learner will perceive that there


finding the time at Greenwich

is

be pursued, the

nothing laborious in

by a Lunar Observation

ON CLEARING THE IUHAE MSTANCE.


may be

except the work that

observed to the true distance


fessed, involves

to clear the observed distance

is

and

and

In the annexed diagram

must be con-

this, it

As the

calculation.

from the

effects

refraction, the operation is called the

Clearing ike

219

necessary for reducing the


;

aome amount of

object

of parallax

problem of

Lunar Distance.

Z be

let

the zenith of the place

Z M, Z 8 be the two verticals on


which the objects are situated at the time m, s, the apof observation, and let

moon and sun, or of the moon and a


M, S, be their true places.
moon is depressed by parallax, more than it is

parent placeB of the


star,

and

let

gf'As the
elevated

by

true place,

M,

will

>^

be above

apparent place,

its

refraction, its

m; but

the sun or a star being, on

X
\

__.

the contrary, more, elevated

by

refraction

than

pressed by parallax, its true


place, S, will

be below

its

The apparent zenith


the usual way, that
the corrections for

A*"*"*^

de-

5,1

apparent place,

distances,

m,

t.

Z s,

are got at once, in

by applying to the observed altitudes


dip, index error, and semidiameter, and

is,

subtracting each apparent altitude thus obtained from 90

the apparent distance between the two objects, m,


the great-circle arc

is,

the problem
that

is

given

is

s, is

s,

that

the observed distance itself : and

to compute from these the

the great-circle arc

In the

tme

distance,

S.

spherical triangle Zj*all the three sides will

be
hence the angle Z, or rather cos Z, may be found in

known quantities. In the spherical triangle


sides, Z M, Z S, being the true co-altitudes,
by applying the corrections for parallax and

terms*of these

M S two of the

obtained

refraction to the apparent altitudes, and subtracting each


L 2

Oigiiized Dy

Google

IHYESTieAMOlT OF
from 90, are known; bo that the expression for

result

MS

Z MS, will involve the true distance


unknown quantity. Consequently, by equating
we shall have an equation in

cos Z, in the triangle


as the only

the two expressions for cos Z,

which
S is the only unknown, and this may therefore be
determined by the ordinary operations of algebra.' Let
the apparent altitudes and the apparent distance be repre-

sented by the small letters

and d; and the true

a, a',

alti-

tudes and the true distance by the capital letters A, A',

and

triangle

then (Spherical Trig. p. 5) we have


s, and then from the triangle

Zm

(cos

a ma

...

a sin a

cos

But (plane
cos

from the

first

ZMS,

d+coB

)
'

aa

COB

CUH

cos

cos a'

cob

Jl,

cos

+ syi A sin A
i

A cos A'

Trig. p. 30)
(o

+ o') =

2 cos

a+a'+d

+ a')rvd

j-

4-sm Aain

j-cosi-| (o

= 2C03C06
by putting*
coa

for

(a + a'+d).

D=

:
L

CO

Consequently

cos

cos 3 coa

A cos A

cos a'
(a

fd

COH<A +

co

A')

Subtract each side of this equation from 1, then since

l-coBD^SsinHD.andl + cos (A + A ) = 2
1

we

shall have, after dividing

by

2,

cos'

J (A + A")

rSYIBTlGATION OP POBMUL^.
ain s i

(A + AO

coa5 i

"

1808

cos

al
'

cos

a cos

era a' cos-

221

A ooa A'

a'

4(A + A')

Now, let the fraction within the brackets be represented


by Bin C, then the expression becomes
11

sm*iD=rf*(A+

A') cos'C

..sMD = cosUA+A')cobC
Hence the

formula! for finding the true distance

are

the following, namely,

cos
sin i

D = cos

a cos

a' cos- i

(A + A')

cos

(A + A')

....

(I)

It may be satisfactory to the learner to mention that in


assuming the fraction above to be equal to the square of
sine (sin2 1C),

some

some

fraction is

we do no more than assume

that the

And we

positive quantity less than unit.

are justified in this assumption from the following considerations

The

1.

minator

fraction is positive.

is

factor in the deno-

obviously positive, since neither of the altitudes

can exceed 90.


tive

For every

Every factor in the numerator

is also

posi-

the only one of these about which there could be any

doubt

is

the factor cos s

but to prove that 2 s can never be

so great as 180, conceive the arc measuring the lunar dis-

tance to be extended both ways to the horizon

the arc

thus completed would measure 180, and the ends of it are


cut off by the perpendiculars to the horizonthe altitudes which are respectively less than those hypotenusal

ends, because in a right-angled triangle, whether spherical

or not, the perpendicular


2-

unit.

is less

than the hypotenuse.

The fraction is not only positive, but it is less than


For if it were equal to unit, sin5 D would be nothing
j

Digitized by

Google

INVESTIGATION OP FOBJCim:.

and

if it

were greater than

which no square

unit, sin

would be negative,

can. be.*

The preceding

formula? were

first

given by a French ma-

The computation of the expression


requires, we see, nothing but cosines the

thematician, Jf. Sorda.

under the

radical

result of tbia computation

In the

however, a sine, namely, sin C.

is,

band member of the other expression there

right

occurs another cosine, cos C, and a cosine already employed

the final result being a sine.

It

would be

as well, perhaps,

to postpone the change from cosine to sine

the very-

till

that in the arrangement of the work there should he


no interruption to the vertical row of cosines, in which case
the row of figures would terminate with two sines that is,
might be as well to use the formula) under the following
last, bo

it

alight

change

cos s cos

Qt~d) cos A cos A' i

a cos a'

cos
sin 1

(A + A')

cos3 i

D = cob 4 (A + A") sin C

[...(II)
J

* These remarks should not be regarded as SBperfluous.

Id following

the steps of a mathematical investigation, the learner should exercise that


caution and circumspection which
fied

an interpretation of

he might

is

often necessary to prevent too unquali-

symbols

iiis

instance, in the inquiry above,

for

hastily conclude, in the absence of such caution, that the

arrived at conveyed

a general truth in

spherical triangles Z

M S,

imposed, as in the diagram

spherical trigonometry

"being

s,

formula
the

two

any whatever, partially super-

the above rem arts show that this would

be

an inference. The author of tbia work himself committed


a mistake of the like kind, manyyearaago, when writing on the present subStarting from the second expression for cos D above, namely from
too unqualified

ject.

f 2 cos s cos

oosD=i

cos

(s

~d

a cos a

cos

cos

A cos A'

(A + A

,1

and thus got the following formulie

he replaced the quantity within the brackets by 2cos-C


cos 2 C,

(A+ A '

1 f-coe
J

1, that is

for the true distance,

by

namely,

= ^^(^c^AoosAcos a
o' cob (A + AO

cos

cos

D = cos 2 C oca (A + A')

cos

which are true however only under the limitation that


90.
("Yorag-aTrigonometoy," 1841, p, lfli.)

A + A'is

less

than

Digitized 0/

Google

APPLICATION OS THE T-OB.HULB.

each,

cos L cos a

1 cosZ -

from

we

1,

"

cob [a

cos

D = [ cos d

'cosP

cos (a

= A + A',

and

.-,

cos

cos D

A cos A'

cosAcosA'

a')

+ cob (A

~ A')

(Plane Trig. p. 30), that,

+ wsQ=2cosi(P-t-Q)cosi(P~Q)

(P~Q) = A',

Let i (P + Q) = A, and
P

cob

coi

a')

(AwA

cos

_ cos (A ~ A') cos D

<*

cos cos a'


.'.

page 2

A cos A' + sin A sin


cob A cob A'

a (o *W)

Now we know

at

have,

a' <x$d
=
A' coaD

+ sin a sin

cos a'

Referring to the two expressions for cos

and subtracting

= A rJ A',

or else

(A + A') + coa (A

In like manner, cos (a + a

then we shall have,

= A + A', and P = A A'

mA')=2 cos A cos A'

+ cos (a

a')

= 2 cos a cos

a.'

Hence, substituting in the above value of cos D, we have,

If,

instead of subtracting,

expressions for cos

"We

shall

now

to 1,

we

we add

each side of the two

shall get, in like

illustrate the

manner,

use of these formulae by an

example.

Application of the

Formula for Clearing

the Distance.

Suppose the apparent distance of the moon's centre


fronl a star to be 63" 36' 14", the apparent altitude of the
moon's centre, 24 29' 44", and the true altitude 25 17' 46",
1.

Digitized By

Google

APPLICATION OF THE FOEMUL2E.

224

also the apparent altitude of the star, 45 ff 12", and


true altitude 45 B' 15" : required the true distance p

Here d = 63

14",

A = 25
The work will be

= 24 29" 44",

17' 45",

as follows

A'

44
12

2)133

11

10

66

37

$~d

61

its

9' 12".

Part*.

Tab. Diff.

29

45

A 25

= 45

By ikt Formula (U).

1.

35' 14"

a 24
a'

= 45 8' 15"

oos 0-040919

Comp.
Comp.

cos 0-151655

coa 9 '598860

cob 9-699398

96 +

212+

4224

2544

+ 6768

.487
11
99
212

17

45

cos 9-956268'.

A' 45

15

cob 9-848472

A-f A' 70

26

2485
561

4455
3180

10631

39-595372

39pattsforsecs.

38,63

2)39-595333

19-797667

,(A + A')35

13

C 39

48

37

..

cob9-9122I0-i-

cos

9-886457

175)6500(37"

525

+ 94 pte.

253 1250
37"

'or

9-806348 +
. 1

D=

81

32

.'.

B=

63

171- ...
.

sin

9-718558

35

343)6100(17 "4

843

2670
2401

The minus
it is

put before cos i (A+A*) to imply that


to be subtracted from the quantity over it ; and the

plus sign

is

sign

is

annexed to

it

to indicate its addition, to

the

quantity similarly marked below.

Digitized By

Google

APPLICATION OF THE

2.

d6S" 35' 14"


a 21 29 . 44
15

a + o' 69

38

56

a/va'20

a'

12

By the Formula

Tint,

coa

114835 +

nab

con

317772 +

nat. cos

986701+

log 5-899058

792907
39

28

25

17

45

A' 45

A
A + A'
A fx* A'

Comp.

1988478

70

26

nat. cos

331903

19

50

30

nat. cos

910634

log

787701+
63

35

log 3-891812

15/

sum 1275537

225

POP.MTJL.X.

(IV).

nat. cob

....

6-105693

log 5-896363

152801

In the preceding operation we have not actually exhibited


As never more than two cosines
added together the parts for seconds Bhould be
* but in comparing this

the parts for the seconds.


are to be

incorporated into each at once

method with the former, an estimate Bhould be made of


what is here suppressed, in reference to the extracts from
the table of logarithms.

(See p. 230.)

* In the Navigation Tables which are intended ta


will be

found a very convenient table

the trouble of correcting for seconds


table

may

also

be fonnd useful

irf

accompany

this work,

natural cosines, by aid of which

is scarcely

worth mentioning.

for other purposes.

This

Tho author has

before expressed his disapproval of the exuberant supply of tables with

which most of the books on

He is

this subject abound.

persuaded that a

reference to a variety of tables, in one and the same operation, begets con-

tusion and perplexity

more

especially

when any

of these require to be

modified, in every case of practice, by supplemental tables in the margin.

He

inclines to think that the navigator

who

has to work ont an important

where even a small inaccuracy

problem, such as that in the text


sequence

would

is of con-

rather have a model to go by which should confine his

attention to a single table, the use of which he

is

well acquainted with,

APPLICATION OP THE rOEMUL.

226
It

some advantage

is

in this second

distance that the cosines of a +.


in the

same column, or

a',

in adjacent

way

of finding the

+ A' always occur


columns of the table

A ~ A'.

Also the first and last


same column or in adjacent columns, as do the two middle logs. The table of
natural cosines, as given in the accompanying volume of
tables, is moreover more easily employed* than the table of
log cosines.
On these accounts some may possibly prefer
the method now given. Both the methods might be abridged
so do the cosines of a

logs occur in like

by the

a',

manner

at the

aid of special tables: but these are sometimes so

perplexing, involving two or three small marginal tables of

and requiring bo much tact and judgment in


we think the rigorous methods by the common
Indeed as a general principle

corrections,

the use, that

tables are to be preferred.

the fewer the tables employed in the

computations of

Even the logarithmic porwork might we think be advisedly-

nautical astronomy the better.

tion of the foregoing


replaced by

stand thus

common

arithmetic

the operation would then

even should he have

to

perform a few independent arithmetical operations,

than hare his mind perplexed by turning from table to table for the several
items he

is to

put down

more

especially

after all, without certain changes

Under
besides
to

when these

and very

careful

are not to bo obtained,

and vigilant scrutiny.

this conviction, the author has here proposed a

little

method which,

arithmetic, requires reference only to one table, very easy

consulta table of natural

cosines.

It will, however, be understood that the preference here given to the

arithmetical operation in next page, instead of to the logarithmic work in

the

last,

is

merely a matter of individual taste and opinion.

The com-

puter who uses the method in the test, will employ logarithms or not,

.is

he thinks best

Digitized By

Google

APPLICATION OF THE EOBMCXJE.

dW

35'

II"

a ii

29

441

45

12/

a'

nat. cos

a+a'GQ

38

56

nat. cos

a~a'20

39

28

nat. cos

17
8

451
15J

70

26

nat. cos

A' 19

50

30

nat, cob

347772 +
792607 Multiplier

25

A' 45

A + A'
A~

227

444835 +

(to

be nrrersed)

935704 +

1283476

Divisor.

334003 (To be subtracted from


940G34
belaa)

quotient

1275537 Multiplicand

706297

8928759
1147983
25511
7663
89
1

8984332

787704
334903 (Subtract)

1125663

452801

2,8,3^7,6 ) 10100895

1026781

nat. cos S3" 4' 35"

=D

Should this nat cos be negative,

the tvpplemmt of the

angle answering to

9039

it

u the

tables will be D.

8*8*
55

An arithmetical operation like the preceding most not be


judged of by the eye, in a comparison of it with a logarithmic process in the latter the Sogers are a good deal leas
exercised, but the mind a good deal more.
It may be proper to add here that the sign + or
;

annexed to any quantity, implies that that quantity is to be


added to or subtracted from the next marked

algebraically

quantity below
tities

may

be.

it, whatever the prefixed signs of the quan"Whenever any of the cosines are negative,

bcxe poa cleabihg the distance.

228
that

is,

sign

is,

when any

of the angles exceed 90, the negative

of course, to be prefixed.

logarithms are taken, are

the

final result

is to

The numbers whose

regarded as positive

all

whether

belongs to a positive or negative number,

be determined as in the common " rule of signs" in

multiplication: only an odd

number of negative

can give a negative result.

may be further noticed that the

It

quantities

cosines are all treated aa whole numbers, and not as decimals.

The

operation

by

in a rule as follows

this

second method

easily expressed

is

"SjyLLfor Clearing the Apparent Distance.


1.

"Write down, in order, the apparent distance, and the

apparent altitudes

and take the sum and difference of the

latter two.
\

2.

Underneath, write the true altitudes

m anner their sum and


3.

Referring

now to

taking in like

difference.

the table of natural cosines, take out

the cosine of the apparent distance, as also the cosine of

each
4.

sum
and

Bum and

difference.

Take the sum of the

first

and second cosines, then the

sum

of the second and third, and lastly the

of the fourth

fifth.

These sums will give three numbers. Multiply the


and third of them together, and divide the product
by the middle one, performing the operation either by logs
or by common arithmetic
the result the cosine of the
sum of the true altitudes being subtracted from it will be
5.

first

the cosine of the true distance.

In taking out

Note.

If this cosine be negative

the supplement of the angle in the tables

is

to be taken.

the cosines, the best

way

of pro-

ceeding will be this : Having found the column headed with


the degrees, take first the seconds; and having written the

proper correction for these on a

slip

of paper, place this

correction under the cosine answering to the minutes,

write

down the

and

result of the subtraction.

OigilizM 0/

Google

EXAMPLE BY THE
"We
shall

shall give

It "CLE.

another example worked by thia rule, and

then sketch the blank form for each of the two

methods of finding the true distance.

29 27' 5", a'


25
Given the apparent altitudes a
51" the true altitudes
29 25' 30", A'
26 41'
99 58' 58" required
and the apparent distance d

2.

60"

35",

A=

the true distance ?

d99

56' 58"

a 29

27

nat.coa

51

60

fill

a + a'55

17

56

36

14

25
41

301

i'25

173352 +

395943 Multiplier

A 29
A

26

A + A'56

A ~ A'

35

iiaL cos

(to be reversed)

998023 +

iS^D*

nat. cos

43

55

nat. cob

557484
998863
1556347 Multiplicand

349593
4669011
1400712
77817

14007
47
15,6,7,3 1 1 1 8)6162246(

4701954

1120
1097

393171

557481

{Sultracl)

PIBST BLANK FORM

230

NEW

If instead of actually multiplying

1IETHOD.

and

we take

dividing,

the logarithms of the three numbers, the extracts from the


table will be aa follows

S'5975S6

83
*

3-804843

G'192010
83 '7

fl-195069

44

11-16

2-77

1-95

?' 21

'

5-507633

= Comp. of 6-1S51E7

6-192106

5-594582= log

557484 (Subtract)

503
lll)~79 (71

6-192106

393171

164313 = nat.

oos

1
99 27' 26"

777
13
1st

App.
App.

Blahk Fona for

dist.

alts.

clearing Ute

Lunar

Distance.

." uat. coa

'{'*"**']

Sum

app. alta

Diff.

app. alta

nat. cob

Multiplier

True

nat. cos

alta.

Divisor

(To be subtracted from

Sum

true aits

nat. cos

Diff.

true alts.

nat cos

quotient

Sum

Multiplicand

beW).

Divisor) Product (Quotient

nat. cos of True

Distahce.

Note. The " quotient" may be found aa here indicated,


by common multiplication and division, using the contracted

Digged

0/

Google

BEOOHD BIuLSX TOBM

BORDA's MKTHOD.

231

methods, or by taking the logarithms of the three numbers,


thus

Log Multiplier
Camp, log Divisor
Log Maltiplican il

,10

Log Quotient

2bd Blaitk Fowi /or clearing


App.
App.

the

'

Comp. coh

App.ttlt.

Oorap. ooj

being rejected front the indes.

Lunar Distance

dist.
Bit.

sum

npp. diaL

(Borda's Method).
Tab.I>\ff.

Parti JOrtKj

+
+

TroeaJt.

Bum

true

alts.

AutfleC

+.

Parts for ee

4 true distance

Thee Dwtasce

By comparing
that

if

in the

first

the two forms the student will perceive


the multiplication and division be per-

will be the same number of


first method the
made with much greater facility, and consequently the work is completed in less time, and -with less

formed by logarithms, there

references to tables in each: but in the

references are

trouble

and, aa both methods are equally accurate giving

Digitized 0/

Google

OH TAKING THE OBBE STATIONS.

232

the true distance to the nearest second

we

the first method,


on the ground of superior

think, claims the preference,

simplicity.

But

it

may be

remarked, that whichever method be em-

ployed, an error of a few seconds

one or two minutesin taking


very

little

or

much

of even so

tlie altitudes, will

limb

as

hove but

influence on the resulting true distance, provided

the observed distance be taken with accuracy.


valuable peculiarity

This

is

because, in preparing to take the dis-

tance, the sextant can be previously set to a division


easily read off, the observer waiting till

on the

the anticipated

distance has place, at (he instant of which the altitudes

be taken by two other observers

may

and any small inaccuracy

either in the readings off or in the observations themselves,


will

be of comparatively

'But,

little

consequence.

instead of a single observation,

take the

mean of

anticipated distance

is

it is

always best to

3?or this purpose, after the first

several.

taken, with the corresponding

tudes, the index of the sextant can be

alti-

moved a minute or

two, according as the objects are approaching to or receding

from each other, and another observation of the distance,


with the corresponding altitudes, taken, and so on: the

mean

and the means of the corresponding


from which the true distance is to be

of the distances,

altitudes, are those

computed.

It

is

distance from the

of much more importance to deduce the


mean of a set, than to so deduce the alti-

tudes, since strict precision in the latter

indeed, as

we have

is

not indispensable

already remarked, the altitudes


.

may be

each in error to the extent of even one or two minutes,


without materially affecting the result of the computation.*
*

The reason of

formula

ak

(III),

has for

may ha

The fraction in the


numerator and denominator numbers consisting of

this

explained as follows

If the last two or three figures of each

or seven places of figures each.

bo equally increased or diminished,


cannot be materially altered
of a, a' produce's

and

it

it is

an equal alteration

is

plain the value of the fraction

equally plain that a small alteration

in A, A'.

Again, the fraction always

Oigiiized by

Google

ON TAKIN& THE OBSERVATION 9.

may

It

233

happen, however, from the want of qualified

assistants, that

by the same

both distance and altitudes must he taken

In

observer.

this case, having set his seitant

to the anticipated distance, shortly before this distance has

him take the

place, let

altitude of each object, with another

instrument, noting by the watch the corresponding times.

Let him again observe the altitudes and times Bhortly

after

taken, having already noted the time

the distance

is

the distance

itself.

of

Then, by proportion, as the interval

of time between the two altitudes of the same object

is

to

the interval of time between one of thoBe altitudes and the


distance, so is the difference of the altitudes to the correction

to be applied to the one altitude spoken

what

it

would have been

Of

distance.

course, a

of,

to reduce it to

taken at the instant of the

if

mean

distance,

and a mean altitude

But

of such object, ean be inferred from several, as before.

may

the obscurity of the horizon


altitudes altogether

in this case they will have to be deter-

The method

mined by computation.
will

preclude the taking of the


of computing altitudes

be explained hereafter.

Examples for Exercise.


1.

The apparent

distance

and the true altitudes'A,


d

2.

the apparent altitudes

Ans. D =83 20' 54".


The apparent distance, the apparent altitudes, and the

true altitudes are as follows, namely


differs

a, a',

namely:

D?

Eequired the true distance


*

d,

A', are as follows,

= 83 57' 33", a = 27 34' S", a' = 48 27' 82"


A = 28" 20' 48", A' = 48" 28' 49"

from unity by a very small

being very small.


cos

&

(1

+ p)

The formula

cos

fraction

is

that

is, it

cos

by the very small

equal to I

(a~<) + cos (A~A'J

equal errora

these errors, therefore,

here destroy each other, bo that the only error remaining


multiplied

is

therefore

M-jp

Now oos (fja') and cos (A~A') have


fraction

j>.

And

is

that in cos fama')

similarly of form (IV).

Digitized by

Google

EXAMPLES OP

234

C LEAKING

3.

THE DISTANCE.

= 72 42' 20", a = 20= 13' 20", a' = 81" 17' 20"


A = 20" 10' 13", A' = 32 2' 14"

D?

Required the true distance

The apparent

D = 72 33' 8".

Ana.

'

distance, the apparent altitudes,

and the

true altitudes, ore as follows, namely:

d=
1

5G 50' 31",

a= 58 4' 85", at = 23" 3' 1"

A =58* 3' 59",

The apparent

=23" 51' 41"

D?

Required the true distance

A'

Ans.

=56

16' 27".

distance, the apparent altitudes,

and the

true altitudes, are as follows, namely

d = 108 14' 84",

= 24 50', a' = 36" 2P

A =25 41' 39", A'= 36* 23' 50"


Required the true distance
5.

The apparent

D?

Ans.'D =107 32'

distance, the apparent altitudes,

true altitudes, are as follows, namely

d = 33* 3V 21", a

A=

= 28" 24' 58",

28* 23' 14", A'

D?

Required the true distance

Determination of

tJie

tude,

As

1".

and the

a'=

61' 36' 50'

= 62 2' 0"
1

Ans.

D 33 5& 48".

Greenioich Time, and thence the Longi-

from

Lunar

Distance.

already stated. (page 218), a variety of

Lunar

^Dis-

tances axe given in the Nautical Almanac for every day in

the year, and for intervals of every three hours.

such an interval the motion of the

moon

in its path

During
may be

considered as sufficiently uniform to justify our inferring,


without material error, what the distance would be on any
intermediate instant, by proportion, or on the other hand,
to any intermediate
evidently troublesome to wort a proportwo of the terms are degrees, minutes, and

what the time would be corresponding


distance.

Bui

tion in which

it is

Digiiized by

Google

ON PBOPOBTIONAL LOGARITHMS.

235

To save

this trouble

and the third term hours.

seconds,

such proportions, Dr. Maskelyne, a former ABtro-

in all

noroer Boyal, calculated a table, called a table of Propor-

Logarithms:

tional

which

Tables

it

will

accompany

explain the principles of

be made of

be found in the Navigation


volume : we shall here

in

this

its

construction, and the use to

it.

Peo fort ion At Logarithms.

The

number

of seconds

10800, and if a be the number of seconds in any


3\ then log 10S0O - log a ia what

ia

portion of time less than


is

to be understood

by the proportional logarithm of a.

Hence, contrary to common logarithms, the greater the

number
fact,

a the less will be its proportional logarithm.

these logarithms are analagous to what in

logarithms are called arithmetical complements

the log the less

Prop, log

its

arithmetical complement.

= com.

log

IOSOO com.

log

a = com.

10
log

proportional logarithms are complements of the

logarithm a

not to 10but

to com. log 10800.

actually equal to 10800, then prop. log. a

just aa in

common

logs, if a log

In

common

the greater

As
,

common
If o be

= com. log 1 =0

be actually equal to 10,

its

complement is 0.
"We thus see that a table of proportional logarithms of the
numbers required is constructed by simply subtracting the

common log
that

ia,

of each number from the

Ammon log of 10800,

from 4-033424.

let the

between two consecutive lunar disbe D, and suppose the


an intermediate lunar distance determined

difference

tances in the Nautical Almanac


difference between

at sea, and that of the two distances in the Almanac, which

preceding, in order of time, to be d : then


(#'') of time must be added to the time
of this nearer distance to obtain the Greenwich time of the
is

the nearer to

it,

to find what portion

observed distance,

we have

the proportion,

Oigiiized 0/

Google


ON PROPORTION AT, LOQAEITHMS.

23G

D
tog

,*.

a;''

log 3 h

,*.

8>

log 3 b

log

a*

+ log d

log

& = log D log d


= (log3 ' logd)1

that

(tag

8* log D)

is,

P. log a>

where by

= P.

log

(I

T.

log D,

and D, are meant the mkmSot- of seconds in

these several quantities.

The

D iB inserted in the Nautical Almanac, between

P. log

the distances there given at the beginning and end of every


three hours,

from P. log

so that

d,

by subtracting

this proportional log

taken out of the table of proportional loga-

rithms, the remainder will be a P. log, answering to which


in the table will be found the portion of time to be added to
the hour of the earliest distance, in order to get the Greenwich
:

Suppose

were required to find the Greenwich mean time

at which,

mean time
it

of the observed distance. J?or example

the true distance between the moon and a Pegasi would be


41 14' 58" on January 22, 1858. It appears, by inspecting
the distances in the Nautical Almanac, that the time

be between noon, that

is

0 h and 3 h

Distance at noon

40 29'

Given distance

11

Difference

Tims

after noon

8"

14

58

45

50

SI- 10"

must

the nearest distance,

preceding in order of time the given distance,

iB

therefore the

P. bgofdiff.

2987

P. log

6941

P. log

2954

But, although the moon's motion during the whole of the

3h

uniform to render the interval of time, thus


determined by proportion, a close approximation to the true
is sufficiently

interval, yet to obtain the interval exactly, a correction for

the moon's variable motion during that interval must be


applied.

The

Take the

is found as follows
between the P. logs against the two

correction

difference

distances in the Almanac, between which lie given distance

Digitized By

Google

OH PROPORTIONAL LOGARITHMS.

237

lies. Then with this difference, and the approximate interval,


found as above, enter the short table given at p. 526 of the
Almanac, and the proper correction will be found. Thus, in

the example above, the

P. log at noon

is

298V, and the

P. log at 3 h is 2986: the difference between these is 51.


Turning to the table at p. 526 of the Almanac, we find
opposite to l h 31 (the nearest to l h 31 m 10'), and under 51,
the correction 16s which, added to the approximate interval,
;

l h 31 m 10^, because the P. logs here are decreasing, gives


hence the
l b 31 m 26' for the true interval from noon
Greenwich mean time iB l h 31 26 a
Proportional logarithms may be advantageously used in
:

many

common

other inquiries in which

proportion would

And as in ordinary logarithms, we may


else be necessary.
always avoid subtraction by taking the complement of the
P. log to 10-0000, and then rejecting this amount in the
sum. Por example,
The observed altitude of a celestial object at 3 h 28 44.'
was 20 3', and at 3 h 38 m 20s the altitude was 20 45' what
was its altitude at 3 h 33 m 47 s P
:

Ffrttalt

Second

...

Time

SO* S'

20 45

Difr

**

Difference

A 9"
5

36'

3h

3
**

SB" 14"
SS 20
0

3>-

Time

nt icq. alt. 3

Difference

30

26"

33

47

Arith. pomp. P. lag S72T0

I*.

42'

log

15520

P. log

-6320

p. I05

-lino

First
20"

W 0"

AititudontS* 33" 47'

Having thus shown the use of proportional logarithms,

we may now

proceed to detail the operations necessary for

obtaining the longitude by a

Lunar Observation.

Longitude from a Sun-Lunar.


1.

The

first

thing to be done

ship's account, or

is

to get, either from the

from the chronometer, the approximate

LOXGITUOT: FROM A SUN-LUX All.

238

Greenwich date of the observations

by means of which the

semi-diameters, horizontal parallax, declination, and equation

of time, at the instant of observation,

may

be ascertained

near the truth for the purpose in view

sufficiently

for these

quantities vary bo little in even a long interval of time, that

a considerable error in the Greenwich date can affect their


value only in a very slight degree.
2.

The next

step in the

work is, by applying the necessary

corrections to the observed,


altitudes
3.

we

to-

obtain the appareutj and true

and the apparent distance of the

centres.

These preparatory operations having been performed,

shall

then have data

both the

sufficient for finding

mean

and the mean time at Greenwich, at the


instant the observations were made, as in the following
time at the
examples

ship,

On

February 12, 1848, at 4* 16T.M. mean time, by


estimation, in latitude 53 30' S., and longitude by account
39 3(y E-, the following lunar observation was taken
1.

Moon's L. L.

Sun's L. L.
Olia. nit.

29"

Indezcor.

29

The height
Mean

17'

26"

10'

15

16

2C 40-

25

of the eye was

10

39

10

20

time at Skip, Feb. 12

'

Ntarut Lbnba.
Obs. dirt. 09' 27' 30-

20"

feet

50

Longitude E. in time
Estimated mean time at

Eeferring

now

16-

38

40

38

we

take out the

declination, the

moon's hori-

to the Nautical Almanac,

two semi-diameters, the sun's

26

99

required the longitude ?

zontal parallax, and the equation of time

DigiiizM by

Google

LONGITUDE THO.M A
18" S.

13" 52'

0.

daclin. at

fori*!

15

61

57

13

Diff. for I",

Eq. ay

+ 14"'

DO

Folab Distance 76

Moon's Hor. parallax at G. noon 58' 3

Correction for 1"

| of

Diff.

12*

239

8TJB-LUJTAB.

49"

-87

l(b
S.

Tun.

4087

2499

33"

for^*,

13"

Moon's semi-diam. 15 G8

For

1.

Oba.

Dl f

Semi

the

2S16'16"

alt. L. L.

"
+

lfl 111

A pp. alt.

Apparent and True Altitudes.

+
t 1,49

29 S7

contra

nit.

-133

Bet -Parallax
True

29 25 3D

centre

Time

2, .For (A*

at Sftip,
Tofi. Z)r/. Pll.forsccs.

3 0-225612

76

79

33

16

-alt. 50

45

i Honranglo 29

25

Hour

60

14

Polar

dist.

a 0 '012814

Comji.

e 9-259268

1141 -

7920

urn

9-691246

2
angle 68

55 m 21"

Equation of Time

Meam Time

at Sun-

17115

+ 14

33

54

Apparent time at Ship

LONGITUDE FBOH A SUN-LUNAE.

240

3.

Oba.

For

(list.

the

True Distance,

Bun's seml-dlim,
Moon's + Aug.

+ 16

13

+16

Greenwich Time, and the Longitude.

the

BB"26'40", App.

99"

diet.

\
J APP-

a"8

5?

Si" nut

29 27

coa 173352

(25 50 51

Sum app. alls.

55 17 50 nat. cos

app. alts.

3 38 14 nut. coa

563295

385043 Multiplier
Diff.

998023

1567318 Divisor

Troe dibtihce 9B'27'26"


atnoon<Naut. Aim.)

98

Diffarence

an timcat Greenwich

au time at flHp

LosoitddiE.

If the estimated

3S

19

26

l6 32 35"
4

30"

mean time

19'

nat. con

164318

Prop, log (N. A.) 2725

Prop, log

5612

Prop, log

2887

54

45"

at Greenwich,

namely l h 38m

had been taken from the chronometer, we should

now be

correct* Greenwich time,

namely

able to infer from the

ln 32 m 35 s, that the error of the chronometer .on Greenwich


mean time is 5 m 25 s fast.

Note.

When

the estimated time at Greenwich,

upon

which the preparatory operations are founded, differs considerably from the true mean time at Greenwich, it will be
prudent to glance at the results of those operations with a
view to discovering whether this difference of time can

DigiiizM By

Google

LONGITUDE PHOM SUN-LTTNAB.


cause any appreciable modification of tliem

241
that

to say,

ia

whether, 1st, the sun's declination requires any additional


correction of consequence in reference to

time at the ship

as

its

influence on the

explained at page.; 190; and,

fully

2nd, whether the additional correction of the moon's semi-

diameter can have any sensible effect on the distance.

In the example above, the 2 m 35", by which the estimated


Greenwich time, namely, 2J- hours, exceeds the true Green2" in the
wich time, authorises an additional correction of

declination,

and therefore of

correction, however, is

+ 2" in the polar distance

this

readily seen to have no sensible

mean time at the ship. The change in the


moon's semi-diameter, which diminishes only 4" in 12 k is

influence on the

equally insensible.

may be

It

further remarked,

that in determining the

interval of time from the proportional logarithms,

we have

not here taken account of the correction of that interval for


the moon's variable motion, which correction as noticed at

page 237

is

"We think it
Blank Form to be

given in the Xautical Almanac.

right, however, to introduce it in the

hereafter given, as in certain caBes, especially in low


tudes,
will,

it

may

considerably affect the longitude.

of course,

remember that an error

The

say

of,

lati-

learner

2',

longitude, does not place the ship 2 miles out of

in the

its

true

on the equator
the error in
distance would be 2 miles X cos lat., aa shown at page 53
such an error in the present case would but little exceed a
position, except it be actually

mile.

In the foregoing example the time at the ship has been


if this body be too near the
is taken for its altitude to be safely
for this, purpose, the time must be inferred from

deduced from the sun, but


meridian when the lunar

employed

that of the moon.

Now

in this case

it is

desirable to find

the time at Greenwich before finding that at the ship, that


to perform the operation marked (3) above before that

is,

marked

(2), instead of after it, for

the right ascension and

242

LONGITUDE

SUM-LUHAB.

i'UO-U

declination of the moon change so rapidly that an error of


but 9 or 10 minutes in the time may cause an error of so

much

aa

4s

in the resulting longitude.

therefore, that

when time

is

It

is

desirable,

computed from the moon,

to be

that the Greenwich date of the observation should bo as


accurate as possible

May

2.

we

an example.

shall give

22, 1844, at llb IS, estimated time, in latitude

50 48' N. and longitude 1

W. by

account, the following

moon being

lunar observation was taken, the

meridian

Mom's

57 63'

alt.

Index

57

The height

Longitude

53

30

May

21

22

52

now

Ols.

24i feet

(list.

6"

56 2G'

35

42

59

31

25

required the longitude ?

23* 15

....

in time

Estimated mean time at Greenwich

Referring

2"

20

of the eye was

at ship,

W.

Nearest Limbs.

L. L.

22 53'

0"

+30

cor,

Mean time

E. of the

Sun'i L. L.

Ola.

4
19, or 41 m before noon,

23

to the Nautical Almanac,

we

May, 22.

take out the

following particulars, the sun's declination not being required,


Bince from the proximity of that body to the meridian
proposed to deduce the ship time from the moon.
Moon's seml-diam.,

May

21,

midnight

Moon's semi-diam. at 23* 19",


Moon's Hor. par.

May

May 21

21, midnight

15'l"-3

+4

-15

15 5

"4

55'

T"-S

+ 15-2
Moon's Hor. par. at 23* IS",

May

21

55

22-8

DUE. for 12*


for

Cor.

DUE. fit 12*


for

it is

+4"

41-

-4

-25

4 '15

+16"-1

41Cor.

-9

15 -2

Digitized 0/

CoogI

LONGITUDE ISOK SUN-LUNAR.


noon,

Suri'a semi-diameter at

For

1.

Oba.

+15

30"

67' 68'

h. L.
4' 49"

A pp.
Rof.

ilOOH.

alt.

Dip
Semi.

alt.

*'

Dip.

Semi.

fi'4
f-

alt.

centre

Parallax

-Bef.

App.

centra

True

2.

Obs. dlit.

Sun> semi.
Mooiia

10" \

15

Augment.

contra

alt.

Parallax

Truo

248

May 22, IS' 49*.

Apparent and True Altitude).

the

SDK.

For

the

SS* 25'

centre

True Distance and Time at Greenwich.


SI", App.

49

15

+ Aiig. +15

alt.

11 J

App

58" 56'

dist.

'

Sam

31"

4
3

30

23
81

31

,,(63

nat

coa 5S5489

eol 154260

nat

699T49 Multiplier.

,58*

B'

59'-

Difference 33

(a

51

45

Bum

81

55

Difference

34

12

44 nat. cos 140402


14 nat cos 327048

Sfl

nat. coa 818912

973173 Diiicor,

987445 Multiplicand

5304670
870T01

87070
772
S8T
87
9,7,3 l,7,270Sfl6T<

695681

0839032

140402

930655

655229

875355

6141

= nat coa 66"

16'

25"

LONGITUDE FROM BUN-LUNAH.

244
True distance

DM.

Aim.)

nt 21' (Naut.

5S 16'

25"

55

15

30

~1

49

2h

7 ra 42-

23"

7 m 42"

Interval after SI*

.".Mean time at Greenwich

P.L.ofDiff.

3321

F. I.

4712

F. L.

UflT

May 21

Having thus got the correct mean time at Greenwich when


the lunar distance was taken, we can now deduce the right
ascension and declination of the moon with greater precision,
as follows;
A.

jlffiw ftin'i X.

S h 58- S3>-|

Noon, 21at
Cor. for 23* T

+3

4*

23>>

4B*

Moon'flRA.

21'

7k

SS-

65 40

+ 16

Cor. for74:!*

17=

13*

Cor. forT-12*

5'12"K.

Mooq'bDcc. 17

4 10

ft.

Polar distance

Fur

3.

Altitude
Latitude
Polar dist.

Hour angle

Mean time

51'

48

55

50

*
.

at Ship.

36

28

44

Tab.

Comp. cos 0 '199263


Comp. sin 0-0195S7

May 22
or May 21

at ship,

Dig,
65

23

at Greenwich 23

Longitude

Mean Time

45"

23"

50
72

Mean time

.",

the

W.

in time

Lonoitcbk W.,

The constant hourly

difference in the

moan

sun's

li.

199), from -which the difference corresponding to any

A,

is

S'^SSflfcee

number of hours,

Digitized By

Google

LONGITUDE JBOM BDN-LUNAH.

mean time

If in this example the

before the

mean time

We

at ship

245

had been found

Greenwich, the resulting longitude

at'

would have been about

4' in error.

conclude these illustrations with one more

shall

example,
3. September 2, 1858, at 4)* 50 m 11 s , as shown by the
chronometer, in latitude 21. 30* N., the following lunar

observation was taken, the height of the eye being 24 feet ;-


Obt. all. Sun's I. L.

B8"

40'

+2

Inderoor.

Obi. diit. N. L.

Ohe. alt. Jfiwrfi L. L.

30"

32"

52'

+3

10

Required the longitude

Sun's Noon Declin. at G.

20"

65"

32'

40

10"

10

56'

46"'5 N.

Diff. for 1"

54"'96

forfi*
for 10'"

25-35

Equa. oftime

for4 b 50 m

Cor.

EquA. or time

Diff. for

+ 0"796

l*

3.85

corrected

59' 35"

Moon'aHor. Parallax

-1

Diff. for

be easily obtained.

But there

2"

Cor. forfl 11

Hon. Par. corrected

minutes and seconds

may

59

12 b
5h

for

+y-7
+2"

37

is

a table

for fur-

nishing this difference in the Nautical Almanac, page 530.

The
noon,

difference
is

7" 42'

(by

is

8' 1",

between the Moon's R. A. at23h and at the following


,

Kant. Alm.)
16".

Also,

+ 2m

5',

the proportional part of which, for

the difference between the two declinationa

the proportional part of which for7m 42'

is

1' 2".

is

LOS G1TU11E FBOM BW-IrTHFAB.


For

1.

Qba.

Dip

Semi.

*'

15

Apparent and True Altitudes.

49'-)

51

2.

Sun's

(he

L. L.

alt.

For

the

Mean Time

at Ship.

58 3' 15"

alt.

21

Parts

30

Comp.

coa

0-081822

82

39

Comp.

sin

0-004124

2)162

30

54

81

15

27

coa

9-182196

sum -alt

22

22

12

ain

9-580392

iHonrangle

14

30

Latitude

Polar

diflt.

4
i

for

tecs.

1131

1369511

36963
6132

18-7BS034

311
2

Hoot

29

angle

Meak

tijie

J6 "

3 or

at Ship

4* Apparent time at ship

55

[The hour-angle deduced above


for the ship time derived

from

is

it

rather

emailtoo small

to be depended upon, as

accurate, except in particular circumstances.

But, as noticed

at p. 196, when, as in the present example, the place

observation

is

between the

name

the same

tropics,

and the declination

as the latitude, the hour-angle

is

of

of

may be much

smaller than under other circumstances, without affecting

the accuracy of the result.


exceeds

Table

2\

When

the sun's hour-angle

as in general it should, it

XVIII

may be found by

of the Mathematical Tables, from twice the

sine of the hour-angle.

(See p. 255.)]

DigilizM By

Google

247

LONGITUDE FHOSI BUH-LUSAB.


3.

Oba.

For

dial.

the

True Distance,
0"\ App.

05 81'

H mi.

Hun's

Moon's

+ IS
+ Aug. +10

54
[
J

29

Apt>-

the O. Time,
66'

dist.

and

the Longitude.

53

4S

3T

02

32 nat. coa

25

10

tiuni

405850

20" nat. cob

3'

5S
"^ ha

03S297 +
370553 Multiplier.

000556

S nut. cos

865250 Divisor.

03

49

18 nat. cos

24

B7

12 nat. cos

040223
006652
S57424 Multiplicand.

416426 nat. cos Si" 28' 27"

5S14S4

Tree

65 23' 27"

distance

Dirt, at 3> (H. A.)

66_ 2*

Infcmdoftiwe
Correctionp. 526 (N.A.)

Mean time at
Mean time at

Green.
ship

Longitude W.'in time

And

P. L. of Diff.

2537-

66

... P.I.

4701

.P.I. 2167

+1

3"+l

49

19

55

35

S3

the error of the chronometer

mean

23

'~V4S^ 18-

ia

Longitude 43 2G' W.

52 s fast on Greenwich

time.

L'l

-.'"J

L-.

BLANK EOEM SON-LUNAB,

248
It

now

merely remains for us to give the blank form for a

sun-lunar.

Blank Foem.

Longitude ly Sun-Lunar.
>

Estimated mean time at ship


Estimated longitude in time

Greenwich date

. .

Sim's noon declin. at G.


Cor, for time past G.

..

.."

. .

.."

..'

for E. and + for W.

(May bs had from Chron.J


Diff.

.."

forl>

....

noon

Hoars past noon

. .

Declin at G. date
90

C,0>

Folae EiaiMSCE

Cor of dctlin.

Sun's semi-diameter

..'

Equa. oftima(p. I..N.A.)..Cor. for time past G.

Moon's iemi-diametor

.."

noon
..

Diff. for 1"

..'

>(

Equa. oftUieatG. date

..'

.."

.."
.

Hours past noon

..

.." Cor. of Eq. of time

Moon's Hor. Parallax


Cor. for time past G. noon

..'

.."
.

Hob. pas. at G. date

Diff.

for 12

.."

11

. .

Hours past uoou

.." Cor.

1.

Far

the

Moon.

Sou.
See Blank Form, p. 113.

2.

For

the

ofHor, Par.

Apparent and True Altitudes.*

Mean Time

See Blank Form, p. 126.

at Skip, from the Sun's Alt.

See Blank Form, p. 195.

The blank forms

for there it is scarcely necessary, at this stage of

the learner's progress, eren to refer to

the operations for deriving the

apparent and true altitudes, whether of the sun or of the moon, from the
observed altitude, are of such frequent recurrence, and, moreover, are so
simple and obvious, that there can be no necessity to consult a form for

them in working out the present problem.

Oigiiizod By

Google

3.

BLANK TO&M
For

SBTT-LTFNAH.

249

True IHatance.

the

Correct the observed distance for tbe two semi-diameters,


taking account of the augmentation of the moon's
diameter, the same as in step 1;

semi-

the result will he the

apparent distance, with which and the apparent altitudes


proceed as in the

Form

For

4.
..

True distauco

Next

earlier dint. (Naut.

230

at p.

to find the true distance.

the Longitude.

.."

..'

Aim.)

F. L. of diff.

..

P. L.

ic

True interval of timo

Timo

of oarlior diat.

.."

..' n/ltr

time of earlier

Tabid at

diat. iu

p. St

Naut. Aim.

Mean time at atrip


Longitude in time

Lokohode

..* ..'

.."

Note. When from the sun being too near the meridian,
or from any other cause, the time at tbe ship must be
deduced from the altitude of tbe moon instead of from that
of the sun, then the true distance, and thence the mean
time at Greenwich, should be obtained be/ore the mean time
at ship

is

computed, as in Example

determining the time


following

The Blank form

2.

from the moon's altitude

for

the

is

Time at Ship from, the Moon's Altitude, and Time at Oretnmcfi.


II.

A. at 0. noon

A at
.

Cor. for time paat noon


It.

A, at 0, date

r.

It.

Moon's II. A.
hour

tlio

forroiu.

and

sec.

A. at G. date

Deo. G. date

PoUR

dietakct;

m3
Oigilized 0/

Google

BLAKK H0E1I
3.

Truealt.

For

.'

the

sum

at Ship.

Tab.

Comp.
Comp.

Polar dist

BTJH-IUHAB.

Mean Time

."

Diff. Pit. for sees.

coa
sin

sin..

alt

Cor. for sees.

Honr angle
.'.

Hour

Bin

angle

or in time

.'

Moon's E. A.

{Sum if W.,

E. A. of meridian

Mean

sun's E. A.

Mean time

In the

Note.
from the
although

effects
all

Diff. if

E.

of Merid.)

at ship

work for clearing the observed distance

of parallax and refraction, the cosines,

decimals,

may

always be treated as whole

numbers, as in the examples already exhibited.

It

may

sometimes happen that when tho cosines of the apparent


distance,

and the sum of the apparent

trary signs, they

sum

may be

altitudes,

have con-

so nearly equal that their algebraic

(in this case their numerical difference)

in the place of the leading figure.

may hare a 0

It is beat always to

0 in the resulting multiplier; and in


employing the multiplier, as such, to put the 0 in the unit's
place, just as we should do if it were a significant figure,

actually insert this

commencing the work of

multiplication, however, with the

next figure, rejecting, as in


of the multiplicaod.

all

other cases, the unit's figure

It will generally be found that in the

Oigilized 0/

Google


EXAMPLES

251

BtTH-LTOAE.

subsequent division the first place of the quotient will also


be a 0, and that a significant figure can be given only after
Attention to

cutting off the unit's figure of the divisor.


these particulars

is

necessary in order to avoid writing the

a place to the

figures of the quotient each

before

left

its

true place.

These remarks are not to be regarded


distinction of cases, because there

ia

that the computer has to bear in

as pointing to

any

no such distinction: all


mind is, that each of

divisor, and multiplicand,


which number of
by suppressing leading zeros,
numbers be per-

the three numbers, multiplier,


is

to

consist of at

least

six places

places is not to be diminished

unless, indeed, the operations with these

formed by logarithms, when the leadiug zeros

are, of course,

to be rejected.

Of the two

factors

marked

and multiplicand,

multiplier

either may, of course, be placed under the other.

It some-

times happens that the right-hand places of the latter are


occupied by zeros

when such

he better
make it the multiplier, and the other factor the multiplicand; for in reversing this multiplier the zeros have*no
:

is

the case,

it will

to

"We

influence.

shall

now

give an example or two for

exercise.

Examples for Exercise: Longitude from Su?i-Zunar.


January 21, 1858, at about ll a.m. estimated mean
time, in latitude 40 16' S., and longitude by account
11

1.

106 3ty E., the following lunar was taken


Obi.

aft.

Sun's L. L.

Obi. alt.

63 17'

The height

May

+V

Indexoor.

Ols. ditt, N. L.

70" 27'
Indeicor.

%'

20"
15"

of the eye was 17 feet : required the longitude to

the nearest minute


2.

16 9' 30"

+ 2'

Indexcor.

Moon's V. L,

Ans. longitude 105 44' E.

18, 1858,at 4* 30" p.m.

mean time by estimation, in

Oigiiized 0/

Google


LONGITUDE FEOM STAE-LUHAU.

252

and longitude by account 58


was taken

latitude 14 2ff N.,

following lunar

83 28'

20

10*

The height

3(y E.,

the

Obs. alt. Sun'i L. L.

Indexcor.

4'

Obs. alt. Moon's L. L.

Obi, dist.

69 IS' 10"

Index

cor.

+2' 20"

N. L.

71 36'

Indexcor.

1'

40'
30"

of the eye was 24 feet: required the longitude to


Ana. longitude 56 50' E.
?

the nearest minute

September 11, 1858, when the chronometer showed


4h 30m Greenwich mean time, in latitude 48 38' 7* N., the
3.

following lunar was taken


Obi.

alt.

Sun's L. L.

Obs. alt. Moon's L. L.

cor.

2'

Ob*, dist.

Index

cor.

2'

N. L.

60 33' 28"

28" 11' 10*

62 8'

Index

10'

5-

1'

The height of the eye was 24 feet: required the longitude,


and the error of the chronometer on Greenwich mean time ?
Ans. longitude 39 15' W.: error of chron. 6 m 5&>fast.

Longitude hy a Star-Lunar.

When
and a

is that between the moon


between the moon and the sun,

the observed distance

fixed star, instead of

the computations for the ship's time become a


fied.

In the case of a fixed

star,

little

we have nothing

modi-

to do

with

either parallax or semi-diameter, nor does the declination, as

given in the Nautical Almanac for the day of observation,


require any correction to adapt

observation

from any
sions

is

made.

it

when that

But whenever time

celestial object other

must always enter

is to be deduced
than the sun, Bight Ascen-

into the work.

The

star's

hour-

is obtained exactly as the


is obtained, but the former, in itself, can
no information as to the time, which is necessarily

angle at the instant of observation


Bun's hour-angle
give us

to the instant


EXAMPLES
the hour-angle of the

in astronomical reckoning

253

BTAB-LUNAB.

mean sun

at that instant, and which

always the time past the pre-

is

ceding noon.

Now, without any direct observations on the sun, this


hour-angle at once becomes known, provided we know the
E. A. of the sun and tho E. A. of the meridian at the instant
referred to. The B. A. of the meridian is obtained from that
of the star and the star's hoiir-augle at the instant If tho
star be to the E. of tho meridian, its E. A. (or this + 24
:

')

diminished by

and
its

its

hour-angle

is

the E. A. of the meridian

bo to tho AV. of tho meridian, its E. A. increased by


or the excess of the sum above 24b
is the

if it

hour-angle

E. A. of the meridian.
The E. A. of the meridian being thus obtained, we have
only to subtract from it (increased by 24f if necessary for
this purpose) the E. A. of the mean sun in order to get the
l

mean
mean

sun's hour-angle from preceding noon, that is, the


time after that noon. These matters, however, have
been sufficiently dwelt upon in Chapter IV, and after what
has been done in the preceding article the student can
require no additional instructions to render the following

work of a

star-lunar intelligible.

Examples

Star-Lunar.

August

7, 185S, at about half-past 3 o'clock in the


morning, in latitude 49" 40' 2f and longitude by account
61 Zff W,, the following star-lunar was taken
1.

Obt. alt. Aldtbaran E.

of Meridian.
32"

Indgxcor.

The

17'

Obt.

10'

IS

alt.

MoorCi L. L.

32

24'

Into cor. + 2

height of the eye was 20 feet

Obt. did.

10

N. h.

41 27'

40"

Iudex

cor.

SO"

20

required the longitude ?

DigiiizM 0/

Google

LONGITUDE rHOM STAE-LUNAIt.

254
Time

at ahip, Aug. 6

Estimated mean time at G.

W.

Longitude

15" SO-

.10

38

in time
.

33"
Horiaoutal puraUflx 60' Si"
Moon's somi-dkmeWr
Hourly diE
S CS" SWISS
+ 9'-8S6
Wean suu's R. A., noon, Aug. C
IS
+ 3 IS
Correction for 19* 36- .
11

Mean

sun's K. A. at ost. timo

Star's II.

A.

88T04

Star's declination

193'= 3 13"

For the Apparent and True

1.

Ota.

Altitudes.

alt.

50"

26'

Dip

App.

32

alt.

Refraction

10

32

32

alt.

56

may be

+ 12

19

39
49

29

28

38

32

alt.

Cor. of alt.

Troe

* This correction

+ 10J

Aug.
App.

Tree

33

16

28

33

alt.

obtained from the Table given at page 530

of

the Nautical Almanac.

The Table here


ascension

is

referred to shows

increased in

a given

by how much the mean sun's

mean

interval of

time.

Increase of E. A. In 18a
in

36"

in IS 11

36

may be more

3= 7"273
5 -914

lu general the correction for the increase of the


the time past Greenwich noon,

right

In the above

example the quantities taken out of the Table would be the following

S-IS*

mean

sun's E, A., due

expeditiously found

by

t<j

help

of this Tablo than by working for it as above.

The hourly

difference of the E. A., to

in the Nautical Almanac,

is

s
L>

-S56u

if

four places of decimals, aa

giveu

the additional decimal be annexed

to those in the text, the decimals in the correction for IB* will agree with

tbo=e here extracted frore the Almanac.

Digitized By

Google

25S

LONGITUDE PttOM BIAE-LUHAE.

32*

Alt. (Star)

2.

For the Mean Time at S

8'

66"
cos 0-16*

Comp.

Latitude

49

40

Polar dUt.

73

46

34

i earn alt.

45

38

49

973

ir3 h

R. A. of meridian
H. A. of mean snn

Meantime

[As noticed

may be

at skip

43735

10094

206+

56 H 24 s B. of meridian

24 to be increased bj 34

31

15

29

15

1'

at page 246, the hour-angle determined above

otherwise expeditiously found, so soon as the result

19-386017

is

obtained,

by entering Table XVIII (Mathe-

matical Tables) with 9-3860X7:

3h 66

9-384678

Tab. numb, ne
Tab.

thus

9-386017

Given number

133900* (24-

8)

11136

Hence, the hour-angle in time

is

3U

56T24B

as above.

And

manner may the hour-angle in time be alwayB

in this

determined.]
*

Two

lores

r.ie

nlwnya to be annexed to the rcmninder.

I0S6IT0DE FEOM

256

For

3.

True Distance,

the

Oba. dist

41

Jloon'saomi.

-f

Aug.

94

30"

16

43

B T A E- L U K A E

Time and

the 0.

App.

dist.

Sum

the Longitude,

(33

10

13" naL cos 7467S1


28

64

49

37

nat. Cob 426354

41

list,

41' 41'

1172145 Mult.
'J

True

cos 999082

lit.'

tSS

2S

tia

37

34

10

12

;)8

1125315 Div.

Kum

lint,

cos 9DO730

1412420 Multiplicand.

230244

0
Interval

S3

P. L. 5248

30"

30- 24' after 18 h

2D-

748649 nat. co. 41"

SW 33"

2103

..P.L."^

1"

lh

if!

+4

Correction (N. A.)*

Truo interval

40

712

18

Mean timo at G.
Mean time at snip
Long. W. in timo

As

10

30

24

ID

29

15

48

18

9 .:

lasomns

in former cases, this correction

of the Nautical

that for 21

'

is

Almanaw

The

CO' 17' 15"

is

difference

W.

got from the Table at

page 526
18'' and

hetween the P. L. for

13, the P. logs heing decreasing;

and under

this difference

in the Table, ami against the interval l b 30 m , -we find the correction

4,

Digilized by

Coogl


LONGITUDE IROH BTAB-LTJKAB.

257

September 18, in latitude 28 45' 11" S., when the


chronometer showed 5 h 12 m 30 s Greenwich, mean time, the
2.

following star-lunar was taken


Ois.

Anlaret W. of

oil.

Meridian.

Obi.

53 18'

Index

eor.

alt.

40'

Moon's . L.

Index

+i

cor.

Obs.

N, L.

dixt.

10"

41 SO'

10

56 7' 40"

20

Index

+4

cor.

50

and the error of the chronometer,


the height of the eye being 20 feet ?
".Required the longitude,

Mean time

at Q.

by chronometer 5 12 30*

Moon's semi-diameter 14' 58" Horizontal parallax 64' 44"

Mean

18

sun'a R. A., noon, Sept.

Correction for 5"

Mean

12-5

II* 48" 27*'47

(N. A., p. 580)*

Bim'a B, A. at Q. time

Star'a E.

+ 51-34

by chron.

49

11

19

l&> 20" 45'

Star'e Declination

28

2" 8.

90
Polar distance

1.

For

the

A3

58

52

Apparent and True AUiluda.

Moon.
Oba,

alt.

Obs.

Dip

App.

41 40'

alt.

--V
4' 24"

Dip.

14

alt.

App.

4-10

alt.

Cor. of alt-

True

"

At the page

of tho Nautical

12"

Cor. for 5"

alt.

Almanac here

referred to,

.....1

we

49-2824

Correction for 6"

12" 30'

30"

58

+ 10

-9713

hi

44

258

LONGITUDE FItOU flTAB-lUNAB.

2.

Polar

Por

62 10'

Star'e alt.

Latitude

15

Ike

alt

72

24

15

20

13

54

Star's hour-angle

42

Mean Time

at Ship.

21"
11

(list.

44

>r

Comp.cos 0-057136

1-265

Comp. ain 0-046834

5074

2 b 50- 59'
16

20

19

11

44

E. A.

11

49

19

of

mean sun

45

B. A. of meridian

* If, stopping here, we enter Table XVIII of the Mathematical Tables


with the number 9-123209, we shall get the hour-angle as at page 255,

Given number

9-123209

Tab. numb, next less

9-118468
5)

...

174100(59-

40475

Hence tho hour-augle


disregarded.

is 2 h 50"' 58',

as above, fractions of a second being

lOMGITUTE PBOM STAH-LUNAB.


3.

Oba.

For

the

259

True Distance, the 0. Time, and the Longitude.


S0 h 12" 30"

dint.

Moon's somi.

+ Aufi. +15

8
|

App.

diat.

App.

alts.

27'

60*
J 62
1

Sum

38" nat cos

S52510

11

u M u
Bl

SO

not. COS

07MM +
483071} Mult.

10

19

02

lilt,

888786

cos

013352 Div.

M 08I72B
09

S85830

9M1M

Multiplicand

87028*

3310418
723283

18082

Interval of time
Corr. (S. A. p. 520)

1^1

27" 28' alter 3 6

True interval

l!"

Mean time at G.
Menu tinrn at ship

fit

27-28-

22

25

Long. B. In time

51

57

.'.

Lunuitude

23' 41' 15" E.

Mettnt.atG.byoh.

12

30

Error or chron.

14

53 Blow on Greenwich

Ab

this error is considerable, it will

mean

time.

be proper to ascertain,

and allow for, its influence on the Sidereal Time, or the


B. A. of the mean sun. By turning to page 530 of the
Nautical

or

Almanac, we find that the correction

acceleration of B.

A .for 14m

68'

is

+ 2"

the

hence the true

Digitized By

Google


BLANK FOEM STAB-LUSAB.

260

mean time
longitude

at ship is 7 b

ia

22 23 s, and consequently the true

28 43' 45" E.

If the work of the multiplication and division be performed by logarithms, instead of by common arithmetic, as
above, the operation will be as at p. 230, or as foDows

482076
004104

log

5-683047

log

5-956216

-J

1-88

013352

Arifli,

camp, log 4-089387

23"5

Blahx Fobm. Longitude


Estimated mean time at ship

Estimated longitude in time

Estimated Greenwich date

Gf.

date

Mean

=~ 2a

moon's semi-diameter

J"
.

m
{

.'

+ 46

by Star-Lunar.

for B. and

for

W.)

(Ma; be had from Chron.)


Hot. parallax

sun's R. A., or sidereal time at G. noon

Cor. forG. time past noon

Mean

-941

46
log 5-678696

477195

At

68

(Naut Aim. p. 530)

.*

.'

.'

sun's E. A. ot Greenwich date


Star's B.

A.

......'

Star's declination

."

90

Polar mstanoz

1.

Par

the

Apparent and True Altitudes.

Sua.

Moon.
Obs. alt.

Dip

Semi

App.

.'

."I
>

alt.

Cor. of alt.

Trae

alt.

oigiiizea 0/

Google


BLAHE F 0 EM
2.

For

the

Mean Time

B T AH-Lf
at

BAB

261

Ship from Star's Altitude.

Tab.

..,.< ...

Star-salt.

Latitude

Polar

Comp.
Comp.

dirt.

2)7.7

sum

cos

Parts

/m MC3
.

sin

sin

r~~

alt

_.j(-ifE.

or + if

W.

of

ridiau)

R. A. Of meridian

. A. of mean sun

-f

Mean time

Mann's soml.+Aug.

.."

if

than R. A. of sun)

bB roI,traeted 11001

A.ofmer.)

at ship

8.

Oba.dlit.

bj 2i\

(t

less
<l

For

the

True Distance.

..'

,."\App,

..

..

)App.

dist.

..*

..'

..

slta.{

Multiplioand
Multipliur reversed
Dlvtenr) ProducKQuotient.

= nnt. cos sum of true idts,(3ub.)

EXAMPLES

2G2
i.

For

the

STAE-LITKAB.

Qreemekh Mean Time and Longitude.

True distance

'

."

Next preceding diat. (Naut. Aim.)

2>ijf."

P. L. of diff.

TTTTT
Interval of time

."1

Correction (Naut.'Alm. p. 526)

.
.

?.t. ...

.'

P. L.

. .

Trne interval

Time

of preceding dist.

Mean time at Greenmoh


Mean time at ship
Longitude in time

The

,',

difference

Lohaitude

.'

,"

between the above mean time at Green-

shown by the chronometer, will be the


error of the chronometer on Greenwich mean time at the
wich, and tbe time

If the error have been found at any-

instant of observation.

previous instant, the difference of the errors will be the

accumulated rate during the interval

and

this divided

by

the number of days in that interval will be the daily rate.

Examples for Exercise : Longitude h/ Star-Lunar.

August

5,

1858, in latitude 24 18' N., and longitude

by account 11 16' E., the mean time at ship per watch


being llh 15 m p.m., the following star-lunar was taken
:

a Fegasi E. of
Meridian.
Obs.alt.

Index

Moon's L. L.

4635' 0"

+1 30

cor.

Obs.alt.

Index

cor.

Diet.

5626'10"

Obs. diet

Index

cor.

N. L.

9482'I0"
+4 10

The height of the eye was 24 feet required the error of the
watch on ship mean time, and the longitude P
Ans. error of watch 18 m 22'/<wi ;
:

longitude 11" 9' 15" B.


This

is

the difference between tbe P. L, taken from the Nautical

Almanac, and the P.

L. next following

it is

required, in conjunction with


EXAMPLES
September

2.

chronometer,

5,

263

when the
mean

8 24' S.,

1858, in latitude

known

S T A H - L I" >" A 1 1

to be 7 m 2' slow on Greenwich

time, Bhowed IS 11 40 m

September

8",

4, the following star-

lunar was taken early in the morning

Aldebm-an E. of
Moon's L. L.
Obs.

3030'0"

alt.

Index

Obg.

+40

cor,

Dili, remote

57"88'20"

alt.

Index

cor.

The height of the eye was 20

+2
feet

Limb.

Obs. diit. 05 4' 42"

20
:

Index

cor.

~2

10

required the additional

error of the chronometer, and the longitude of the Bhip ?

Ana, additional error of the chronometer

7* slots;

longitude 66 57' 30" "W.

In

all

the foregoing examples the

mean time

at the ship

has been deduced from the altitudes employed in clearing the


but, as already remarked (pages 104, 242),
;

lunar distances
neither the

moon nor a

star is so eligible for the determina-

and even the sun,

tion of time as the sun;

proximity to the meridian, or to the horizon,


favourable

position for

the purpose,

either from

may not be in a

when

the distance

it and the moon is taken.


Now, as in determining
the longitude, it is just as important to know accurately
the time at the place of observation, as the time at
Greenwich, it is often necessary to observe for ship-time

between

either before or after the lunar distance

taken, and thence

is

to deduce the time at the place where, and at the instant

when, that distance was observed.


chronometer performs an important
with

all

needful accuracy

And
office

here, again, the


it

furnishes us

with the interval of time between

observations for ship-time and those for the distance, which


interval is of course not affected

by the error of the chrono-

meter, and only in a very minute degree by

which, however,

if

known, may be allowed

its daily

rate

for.

If the time at ship be determined at a place A, and the


the approximate interval of time, for finding the oorreetion of that interval
given at p. .12? of thn Almnnse.

Oigilized 0/

Google

OBSEEYATIOKS TOE TIME.


lunar distance be taken at another place B, the interval of

time between the two sets of observations corrected for


the difference of longitude between A and E being added
to the time at A, if the ship was at
before it was at B, or

subtracted in the contrary case, will give the time at

when the

ship was there

that

when the

is,

distance

was

taken.
The' following skeleton form will sufficiently indicate

steps are necessary to find the time at

was
at

at

B when the

what

distance

from knowing the time when the ship


A, the interval between the chronometer times when

was observed

there,

and at B, and the

between

difference of longitude

and B.
Mean time by chronometer when

at

k
,

.*

......

B
Interval of time by chronometer
Correction for gain or loss in that interval

Interval of time corrected for rate


Diff. long,

of

Mean time at

and B in time

Interval of time corrected for

ship

when at

ut Greenwich

diff. long.

when

at

Longitude of ship in time when at

It has already been remarked, that although an altitude


from which the time at the place where it is taken is to be
deduced, should be measured with all practicable accuracy,

yet for the purpose of clearing the lunar distance merely, a


like precision in the altitudes is

not indispensably necessary.

But circumstances may arise, from an obscure horizon or other


causes, which

may

preclude the observations for altitudes

altogether, though the distance


* If

B is

to the cast of A, this

may be

must be added

In

readily taken.

if to

the west, it

most

be subtracted.

Digitized 0/

Google

COMPUTATION OF ALTITUDES.

283

such circumstances, the altitudes for clearing the distance

must be determined by computation.


In order to compute the altitude of a celestial object at
any instant, we muBt know the object's hour-angle with the
meridian at that instant, and this requires that we know the
time.

corrected

If the object bo the sun, the time itself

the equation of time

is

the hour-angle

but

if

for

the object

be the moon or a star, the hour-angle will be the difference


between the B. A. of the object and the E. A. of the
meridian at the proposed instant and to get these right
ascensions, the time at the place for which the altitude
is required must be known.
To find the time at a place B, where u lunar distance is
taken, by means of the time at a place A, where altitudes
;

are taken, the foregoing blank form suffices.

And

for deter-

mining the time at A, ample directions have already been


given in Chapter IV.

The time

at

when the lunar

distance

was observed, and

thence the hour-angle of each object with the meridian being


found, the declinations at the time, and the latitude of

being also known,

it will

responding true altitudes;

be easy to compute the corand thence, by applying the

usual corrections for altitude the contrary way, to get the


apparent altitudes when the distance was observed

we

shall

have

all

that

is

so that

necessary for the determination of

the true distance, and thence the longitude of the ship when
at the place where the distance was observed.

How the true

and apparent latitudes of an object are to

be computed when the object's hour-angle, its declination,


and the latitude of the place are given, may be explained as
followB

Compulation of Altitudes.
Eeferring to the diagram at page 151, or to that at page

172, we have, in the spherical triangle

PZS,

the following

Digiiized by

Google

COMPUTATION OE ALTITUDE 8.

266

namely

quantities given,

distance

P 8,

co-altitude

The

co-latitude

P Z,

the polar

and the hour-angle P, given, to determine the


that is, there are given two Bides and the

ZS

included angle of a spherical triangle to determine the third


side.

Formulas for the solution of this caBe have already been


If in imitation of what

investigated at page 152.

we put

done,

tanZPcoaP

we

= cot =

....

is

there

(1)

shall have

Bin o

It will be observed here that of the trigonometrical quantan

tities

case, that
tive,

is,

is

when

and therefore

formula (2)

P, cos

become negative

will

P, cos P, the only one that can ever

if in this case

we take

cos

positive,

,.it

= " ZP 7n ';- 3P '....(3)

"We do not Bay that

it

(2), as well here as at

cases

but then, in using

the hour-angle exceeds

must be employed, because the

form

it,

page 162, is applicable to all


where it may be replaced by

(3), the influence of the signs of the trigonometrical


tities

the

become

may always be employed when

which
90.

"When BUch happens to be the

cob P.

the honr-angle exceeds 90, (1) is nega-

muBt not be overlooked.

"When cos

is

quan-

negative,

cot a will be negative, so that the angle a will be the supple-

ment

of that furnished

by the Tables:

this supplement,

added to S P, will always give an angle o'+8P, such that


Bin (<'+ S P) will be the same as sin (a S P) ; but by
using the latter the trouble of taking supplements is avoided.

P exceeds 90, a will necessarily exceed S P, otherwise


the sine of the altitude would be negative, which is impos-

"When

OigiiizM By

Google

COMPUTATION 0* ALTITUDES.
sible. #

And

it

267

may, therefore, be further observed that

S P can never be subtractive when the latitude and declinanames in fact, for the hoar-angle in
this case to exceed 90, the object must be below the horizon.
Note. Since the altitudes employed in clearing the
lunar distance are not required to the same degree of
tion are of contrary

precision as those used in finding the time,


cient if they are

computed

be

it will

suffi-

to within 20* or 30" of the truth.

Examples of Computing Altitudes.


Given the co-latitude

1.

SP = &t

13',

7i

P=51

and the bour-augle

the polar distance

56',

P= 33

30', to find

the

altitude of the star.

ZP

51" 48'

tnn 10 103548

Cos 9-791596

33

so

cos 9-121107

a Ar. comp. sin 0'163iaB

43

22

.10

cot 10-024M5

61

13

10T

35

30

BP
b

+ SP

True

alt.

12'

59

12

Refraction

Arp. alt.

S P

S"

sin 9 9339B4

35
44

If the object had been the sun instead of a

have had to have subtracted

J5"

from

sin 9 979200

star,

we should

this result for parallax,

bo that the apparent altitude would have been 59 12' 39".

Although, as stated above, the true altitude need not be


computed to extreme nicety as regards the seconds, yet small
corrections such as this, to reduce the time to the apparent

must not be neglected : the relative measures of


the true and apparent altitudes must be scrupulously prealtitude,

served, as the formula for clearing the observed


sufficiently

whose
* It

On

implies.

altitude is to be

may be remarked

this

account,

computed

is

distance

when the

object

the moon, the correction

since, aa there noticed, (2)

in reference to the formula (2) page 152, that

would

lie

exceed 90, such a case cannot exist

negative if S
for cob Z

is

P and S were each

to

always positive.

n 2

Digitized by

Google

COMPUTATION OF ALTITUDES.

268

of altitude, applied to the true altitude as above, gives a

which should be regarded aa only the approximate

result

apparent altitude

because, in the Tables, this correction

is

adapted to the apparent and not to the true altitude; so

when the approximate apparent

that,

altitude is obtained

from the true, as above, we Bhould again refer to the table,


entering it now with this close approach to the apparent
altitude,

and take out the true correction

o it

the correction

previously applied belonging to an altitude somewhat too


great.

For example: Suppose that

in the instance above, the

object had been the moon, and that its horizontal parallax at

the time had been 54' 50" ; then, referring to the table of
" Correction of the Moon's Altitude," entering it with this
horizontal parallax, and with the true altitude, 59 12' 9", as
if it

were the apparent

correction to be

altitude,

we

find the corresponding

which must be regarded as an

27' 30",

approximate correction only, thus

Moon's true altitude

Appro iiiunte correction


Approximate app.

alt.

Cor. due to this app. alt.

Affabknt Altitude

.-.

And

even this

is

a second too great, as the Table shows

so that the correct apparent altitude is 58 44' 17".

In the

case of the sun or a star, the approximate correc-

by so much as a second from the true


and therefore need not in general be modified.
September 2nd, 1858, in latitude 21 30' N., and longi-

tion will seldom differ


correction
2.

tude, 43 IS'

W., by account, the distance between the sun


and moon was taken, but the moon being near the horizon it
was resolved to find its altitude by computation. The mean
time at the ship, as determined from altitudes of the sun, was

found to be lh 55 m 35'

required the altitude of the

moon ?

L.'i 1

?j

L'v

COMPUTATION OF ALTITUDES.
Mean lime

at ship

Longitude

W.

lime

in

lireeuwich Hate of obs,

.Venn Sun'i

and

A.,

J[,

eh

B. A. of meridian

46

10

55

35

12

11

or

12

Cor. for 48" 47*

Declination

20

40

41

45
5

1"

10

51

52

28
30

Polar

61

Diet,

95 1G' 15"
35' r, l

59'

Diff. for

+2

Correction

50

Hob. Pah. G. date

Latitodo

SI* 30'

Ilournnglo

S5

18

0"
15

Polar- disc.

01

S*2

3P

13

11

for

+5"-7

5b

+2

of the Moon's Altitude.


cot 1O-1W0O2
coa

3-063134

cot

0-3G7730

Tbue

alt.

1st cnrrcetioii

9 564075

2nd correction

30"
22

44

49
U

IS

ai.t.

8P sin O1I3083

50

3rd correction
Arp.

At. comp. sin 0-011501

6' 35'

Approi.npp.alt. 4

is

12"

37

ii

The following

28 e 10'

at4 h

Declio.

45

and Sour-angle.

21

Moori'a Hor. Parallax

10

47 -4*

IS" 28-

Moon's Houa-isaLB

45 22"fil

10>>

Jfor. Parallax,

12

Moon's R. A. G, dato

47

Cor. for 48 47'

12

48

R. A. of meridian

R. A. at 4*

63

Mean Son's R. A.
Mean time at ship

Moon's R. A., Declination,

55 35*

A. of Meridian.

H. A, at Greenwich, noon
Correction for 4 s 48 47'

269

1*

40

sin 8-388659
1

4
22

the blank form for these operations

COMPUTATION OF ALTITUDES.

270

Computation of the Moon's Altitude.

Blank Eobm:

1.

Mean

For

the

Greenwich Date.

time at ship

.'

Longitude in time
GltEKHWIQI! 1TF. OF ODS.

For

2.

Mean Sun's

the

A.,

and E. A. of Meridian.
h

E. A. at Greenwich, noon
Cor. for G. date (Naut.

Mean ann's B, A. at
Mean time at ship

Aim.

p.

530)
1

G. date

^Add)

R. A. OF MERIWAJf

3.

For Moon's K. A., Declination, Hot: Par., and Hour-angle.

R. A. at hour of G. data
Cor. forminutos

,,

Cor. for mina. and aces.

and sees

Moon's B. A. at G. data
A. of meridian

Moou-a hour angle


in degrees

..'

Moon's hor. parallax


Cor. for time post noon

'

..'

.'

*T

90

/torn

snsttr

polar dim.

."
.

.."
. .

Hob. Pah. at G. pate

4.

Declbi. G. dato

\ (Bob. las

Hoob-ahole

Deelin. at honr

.." ..*

..*

DifT. for IS'

for tiioa past

noon

For the Moon's True and App. Altitude.


a At. oomp. sin

a + SP

jt3T

Tho

corrections on tho

right arc taken from tho


table of " Corrections of tho

'

AIHt

"

h'

'

ZJ . L t
alt.

and then with tho

roctod app.

cor-

Appro x. app. alt


2nd collection
App. altitude

*"

..

..

to bo applied to

'

true alt

at

Arr. altitude

alt.

* If thia remainder exceed 12 k, subtract

f The

sin

it

from 24\

lower sign to be used only when the hour-angle exceeds 80,

which

can never happen when the latitude and declination have contrary names.

Digitized by

Google

COMPUTATION OF ALTITUDES.

Blank Fobm:

For

the Greenwich Bait.

Mean time at ship


Longitude

For

the

Mean Sm'i

.*

time

in

Greekwich

2.

271

Computation of a Star's Altitude.


1.

due of

obs.

R. A., R. A. of Meridian, Star 1 Hoar-angle,

and Polar
E. A. at Greenwich, noon

Distance.
.

.'

Cor. forG. date (Naut. Aim., p.530)

Mean
Mean

sun's R. A. at G. date

j-(4oW)

timo at ship

K. A. of meridian'

(Svh. less

B. A. of the star (Nant. Aim.)

greater)

Stab's Hour- angle in Time

* or.

Star's declin. (Naut-Alm.)

.'

from

00

Polar distance

3.

For

the Star's

True ami Apparent A

Itilude.

i + SFain

cot

Folardliit.

BSPf

Trck altitude
Refraction

,.'

..

.."

..

..

sin

Apr. altitude

If the object be the Son, the

mean time

at ship,

when

the observation for the lunar distance was taken, corrected


for the equation of time at that instant, will

be the apparent

* If this remainder exceed

12\ subtract it from 24*.


The lower sign has place only when the hour-angle exceeds

90.

Oigilized 0/

Google

COMPUTATION OP ALTITUDES.

272
time at ship

that

the sun's hour-angle

is,

this

beiug found

the computation for the true altitude will be the same as (3)
above, from which the apparent altitude is obtained by

adding the refraction diminished by the sun's parallax in

The preparation

altitude.

the sun,

is

therefore as follows

For

Mean time

for the step (3), in the case

of

the Surtt Hour-angle.

at ship

Longitude in time

GilBEBWICn DATE

OTP

OBS.

Sun's noon declin.

DHf. for l*
Cor. for G. date

Cor. for Grcenwieh date

Declination at G. date

PoiAR DBTANOS
Equatiou of time at G.,

Diff. for

lb

Cor. for G. date

Eqao, of time at G. date

Mean time

at ship

Then proceed to

calculate tho true altitude as in step

for a star, adding refraction

minus the parallax to the true,

to obtain the apparent altitude.

Examples for Exercise


1.

In example

3,

page 245,

Computation of Altitudes.
it is

required to compute the

true and apparent altitudes of the sun

when the lunar

dis-

tance was taken.

Ana. True altitude 58" 53' 10" ; apparent altitude 58 53' 40".
2. August 16, 1858, iu latitude 36 30' N., and longitude
* If thifl exceed

12\ subtract

it

from 24".

Digitized 0/

Google

COMPUTATION OF ALTITUDES.

273

153 E.. by account, when tlio mean time at ship was 4'
45 m 44s required the true and apparent altitudes of the sun ?
1

Ans. True altitude 23 50' 24" ; apparent altitude 23 52' 26".


3. April 26, 1858, in latitude 29 47' 45" S., and longitude

by account 31
star Altair

7' E.,

the distance between the

the

was taken, when the mean time at ship was l h 51 m

A.M., it ia required to
star to

moon and

compute the true altitude of the


and thence to deduce the

the nearest minute,

apparent altitude ?
8' ; apparent altitude 25 MY 3".
1858, in latitude 46 15' N., and longitude

Ans. True altitude 25


October

4.

2,

by account 56 24' E, a star-lunar was taken, when the mean


time at ship was 5 h 32 m 12 s a.m., it is required to compute
the moon's truo altitude, and thence to deduce the apparent
f

altitude ?

Ans. Truo altitude 49 22' 17" apparent altitude 48 44' 14".


Note, In computing altitudes as above for the purpose
;

of clearing the lunar distance,

it will suffice if

the true

alti-

tude
for

is obtained to the nearest minute ; but the corrections


deducing from this the apparent altitude should be

applied with care, the seconds being always retained.

In-

deed, if the true and apparent altitudes are obtained with


precision, and we equally increase or diminish these
by even so much as a minute or two, the resulting true
lunar distance will be affected in but a very trifling degree
by the change, inasmuch as the relative values of the

strict

altitudes will be disturbed but in a very trifling degree.


Also a few seconds, any number, for instance, not exceeding

10",may bo added

from the apparent lunar disend of the work the resulting true

to or taken

tance, provided at the

distance be corrected for the overplus or deficient seconds in

the apparent distance.

By

so modifying the apparent quantities as to cause the

trouble in taking the parts for seconds,

we may save a little


when the logarithmic

method of clearing the lunar distance

is

seconds in each to be a multiple of 10*,

employed

but in

OigiiizM By

Google

274
the

mode

dat's wohks.

the

of operation

more

ship's joubnai.
specially dwelt

upon in

this

work, such changes would produce no advantage.

Having now discussed all the more important problems


of Nautical Astronomy, with as much fulness of detail as the
limits of the present

rudimentary treatise permit,

it

merely

remains for us, in conclusion, to give a short account of

what at Beais called a "Day's "Work ;" and to exhibit a brief


specimen of a Ship's Journal, as*promised at page 85.

CHAPTER
day's

As already

woek at

VII.

bea: the ship's journal.

noticed at page 84, as soon as a ship has taken

her departure

and her voyage

courses on which she

sails,

fairly

as indicated

begun, the several

by the compass, her

hourly rate of sailing as determined by the log, together

with the other particulars, leeway, currents, &c., affecting her

on a large black board,

progress, are all recorded in chalk


called the log-board.

book,

These are afterwards copied into the logall corrected for leeway and varia-

and the courses being

tion of the compass, each corrected course, with the entire

distance

sailed

on

it,

being known, a reference to the

Traverse Table gives the corresponding diflbrence of lati-

tude and

departure.

The

difference

departure due to the whole traverse


the direct course and distance

made

is

and

of

latitude

and

then found, and thence

sailed, as

Lastly, with this direct course

explained at page 48.

distance, the difference

found either by parallel, mid-latitude, or


and thus the position of the ship at the
end of the traverse is ascertained. These operations are
regularly brought up to noon of each day
they comprise
what is called a Bay's Work, the result of which is the
position of the ship at noon by dead reckoning.
of longitude

Mercator's

is

sailing,

DigilizM By

Google

day's wobks.

the

275

ship's joubnai.

"Whenever astronomical observations

for latitude or longi-

tude are made, a distinct record of the result of these


inserted in the log-book
is

always closed at noon, and a fresh account opened, a

latitude or longitude,

interim,
it

is

but since the ship's daily account

determined by observations in the

brought up to the following noon, by applying to

is

tbe latitude or longitude, by dead reckoning,

made

in the

between the observations and that noon so that in


what is recorded as the result of observation at

interval

strictness,

noon,

The

is

made

often

up, in small part, of the dead reckoning.

noon being determined in this manner,


the chart is referred to, and the place where she is being
pricked off, Bbe takes as it were a fresh departure from a
known spot, and her course from it is lien shaped, as at
first, in accordance with her ultimate destination.
"When
ship's position at

the log-book, thus completed, furnishes a

this is reached

Journal of the voyage.

As

a specimen,

we

page of such a

Bball here exhibit a

journal, subjoining the necessary day's work.

Note. The initial letters, H, K, and F, stand for Hours,


Knots (or miles), and Fathoms respectively. The fathom is
not a fixed length of tbe
it is

feet,

log-line, like

the

Jcnot

sometimes

the eighth part of the knot, or something beyond six

but

this is

more convenient

it is

knot, which

is

little less

supposed to bo

so that, as

its

to take the tenth part of the

than six

feet, for

the fathom

and

length in the following specimen*

the knot represents a mile, the

fathom will

represent one-tenth of a mile.

The

result of the day's

work preceding the day

the following page of the journal applies,

is

to which

supposed to stand

thus:
DC
Course.
ET.

68 E.

Loot

Diat. [int.

57m.

.18"

Long. Oba.

In Btrktnesa a fathom is 6 feet.

276

THE

DAY'S WOHKS.

SHIP'S JOUBNAL.

EXTRACT STROM A JOURNAL OP A VorAQB ZHOU

Conrson.

Win da.

S. N. B.

E.

Mundav,
18, P.M.

Si-jh.. 12,

weather.

tvinark*,

Out

by E.

E. E. 1 N.

first

reef tofwills.

riiynls, and flymajib.


Light broozes and clear

N.

MICHAEL'S TOWARDS

Si.

Mi><l,<ratc ,iud clear


1

j'DHtowoathar. Swell from


allow

':

it

drift

of 24

a royals and
E.

N. E.

E. 8. E.

K, E.

E. N. E.

K.

E.

1
D
10

.H.

flyinejlb.

Tackad.
Ditto weather.

Ditto weather.

Midnt.

TueedV,
1)

Sept. Id, A.M.

Modern to and clear weather.


Fresh brcotcs.

In top-

gnllmit sail.

t
ff

'1

In

s
a

10

reef topaaibl.

21

W.

73" 2"

Flyfnff clouds,

Frc-ib

Lonf. bvchron.atD, .u.,

Noon

(ii'iit

^tmriu breezes and clnudy.


In second reef topialla.

-i
(.

iTtiloa

with light

and squally.

jib ami in sjvuikiT.


uuon by mor. alt.
MS" ' X.
Variation by
ailmulh, SO' W.

Di.ivli

tat, nt

In order to complete tliis page of the journal, the day's


work must now be computed the compass courses recorded
:

abovo being corrected for leeway or the angle of deviation


which the action of the wind sideways causes the ship to
make with the fore-and-aft line the distance, diff. lat. and

departure due to each course,

are to

be taken from tho

traverse table, aud thence the whole distance,

mencement of the traverse

to the end, as at

and
com-

diff. !at.,

departure found, as also the compass course from the

page 49.

This compass course being corrected for variation, gives


the true course, with which and the distance

from the traverse

table, or

we

are to find

by computation, the true

diff.

day'b -wohks.

THE

and thence by mid-latitude or Mercator's

diff,

long.

These

277

bhip'b jouiwal.

Int.,

differences, applied

sailing,

the

to the latitude

and

longitude determined by yesterday's work,

make known

and the latitude and longitude o

the place of the ship;

the place next to be worked for, being also known, it will


merely remain, from these data, to rind the bearing and
distance of the spot to be reached,

The

accordingly.

day's

work

is,

Leeway.
N,

&

Diat.

u.

i:.

J2-3

U-5

61

10-3

4'8

n-e

N*
E.*a

141

W.

W.

B-6
B-8
S-l
11 !

4'0
8. E. J E.
B. bj E.

to shape the course

K.

N. by E. i E,
S. i, by 11
S. ]]. l,y

and

therefore, as follows

Departure.

Difl

6D
tOL'KBE

N. 6a E.

14-0

7-1
6-8

(Swell)

TRDE

13- 0

1S'7

<B

24 "0

31-1

Compui
Di.it-'ince

cmirso N. Sfl'E.
03 miles.

"With the course C9, and distance 63 miles, the traverse


table gives 22'G for the difference of latitude: hence
Lat. left
Diff.

3S

S0'N.

43'

23

kt.

Lat. nv acct.

Meridional ports 2134

Uc 09=

2-6051

N.

2523

Diff. long.

The departure made from 9h

75-548

noon is nearly 14 miles


with this and the mid-latitude about 38, the difference of
longitude is found to be about 17' E. hence
a.m.

till

I OBSERVATION

AT BOON

day's wobks.

278

the

ship's jqubfal.

Having thus got the latitude and longitude of the ship at


may from these determine the course and distance
iu the present
the port or place to he worked for
case the lizard, in lat. 49 58' N., and long. 6 11' "W"., as

noon, we

to

in ex. 3, p. 67.
Lat. ship (bjalis.) 38 4G' Mer. parts

49 58 Mer. parts 3471

Lat. Lizard
/.Diff.Iat.

2527 Long.

22*45' W.

ahip.

Lizard

W.

= G72imIea:Mer.diff.lat. 944
long. 17 34=1054m.
= tan 48' S"^! con = -6872)672(1007
Diff.

944)1054(1-1165

N. 48 9'
Consequently, the work

and the distance


1007 miles.
for the day heing thus
completed, we write the following results at the bottom of

Therefore, the course

the page

tt.

69'

E.

Hist.

Lnt. acct.

flSm.

2S" 43'

Note.In

32" 55'

W.

it is

22' 46'

W.

points, is applied to the direct course resulting

resolving the traverse

but when the variation

both before casting up the log.

The

may

When the
way

is

ship

to the

As

is

to the right.

wind

on the starboard tack, the allowance for leeand when on the larboard tack it is to the
when it is westerly it must be
of the compass course, and when easterly

left,

left

"VVe shall only further add, that

when

the day's

very considerable, and no observation for longitude

has been obtained, the difference of longitude

more

from

expressed

to be estimated.

regards variation,

allowed to the

is

is

is

be corrected for

direction of the

suggests the direction in which the leeway

right.

Dist. 1007 miles.

given in degrees and not

in points, like the leeway, each course

run

Bearing nod dist. of


J
Lizard N. 1S V E.

Long. oba.
B8* 40'

the foregoing day's work the correction for

the variation of the compass, as


in

E.,

is

correctly determined

by working

made

will

be

for this difference

agreeably to the principles explained at pages 71, 72.

DigiiizM by

Google

TABLE OF CONTENTS.
NAVIGATION.

J.

INTRODUCTION.
On

plane trigonometry ami tlm principles of logarithmic calculat

C HAPTER
Definitions:

T.

The compass The log The dead- reckoning


CFTAPTflR

Theory of piano

Parallel sailing

eiiilin"

Mid-1

Single

ntit'1*a

couraoa Compomul

s.i-iimt;-

TI.

Application

course

of the

latter

to ninRle courses

Mercator's

Bftilin;;

Ti':i--

!:-

iMi'ikUmde.

anil

irercator':

sailings

Current sailing

Tljiug to windward Tiding ilejurtiu-cs

II.

NAUTICAL ASTKONOJ1Y.

Definitions of astronomical terms. Corrections to be applied to

observed altitudes. Dip, semi-diameter, parallax, refraction

7'J

TABLE OP CONTESTS.

280

CHAPTER
Ou finding the latitude

at

Bee.-

H.

From a meridian altitude From an


From two altitudes of the sun

altitude off the meridian

From simultaneous

altitudes of

two

fixed stars

.104

CHAPTER m.
On

finding the variation of the

compass:From an amplitude

From an azimuth

171

CHAPTER
On

finding the time at sea

IV.

From an altitude

of the sun

From
1S3

an altitude of a star

CHAPTER
On

finding tho error

sides of the

V.

and rate of the chronometer

transit over the meridian

From a star's

From equal altitudes on

meridian Froni equal

altitudes on the

opposite

same side
203

of the meridian on different nights

CHAPTER
Ou

finding the longitude at son:


observations.

On

clearing

VI.

By the chronometer

by lunar

the lunar distance by

Borda's

........

method; by a new method. On proportional logarithms.


computing altitudes

CHAPTER
Specimen of a

Bhip's

Ou

212

VII.

journal.A day's work at sea Corrections

for the variation of the compass,

and

for leeway

Oigiiizsfl

274

By

Google

WEALE'S

ME.

PUBLICATIONS FOR
RUDIMENTARY

for

Hi.'

V;-

1801.

SERIES.

nf l-arini'iV.

In

12m".

fli-niy

RUDIMKKTA

li

Gen.

Wooilr

.villi

V.

it?, clotti, i>rirc

Is.

Gel.

IKOLOOY, by Major-

PORTLOCK,

In <!omv 12mn.

\v..

F.lt.S..

*c.

t. cl .Hi,

[!:-..

price

its.

with
Ri dim j;N'rAJiv.i,.-..MLM-:i:AJ.u(;v,
Mr. DANA'S AiMitinna. 9vo1a.ini.

JJTTD I M EN TAliY. ft
Mi n

"|>f:i:

M ECH AN ICS,

:.

k\'

u'.'i

i'.iujv,

by
hy

rudimentary. t*V on' GALVANISM


-It
ANIMAL AND VOLTAIC ELECTRICITY

SNOW

hi Sir

W.

1JAR1U3.

2^UDIMe|TtaRY.'

9,

||' ,;..||1T

l.'ni .

wl

IV.

D I M E K T A R Y.
RTJ TELEGRAPH,
History of

10 ^IAGN>.i'! S '
W. SNOW IIAKI'.IS,

;\

;)

,;,:,[;,

the,

1-Mi-i'

11*.

by E.

In ilvmy limn, with' W.iuilcoi,:

IN.

ELECTRIC

H IOHTOS, C.E.

.'

-|..tli,"i.i

ica is!

RJTDIMENTARY.-

I'l^i-,..].:

I,v

c.

i;.

!!Ll;'si'.i.!..

Ill il.-rllv

limn,

V. ['!l

RUDIMENTARY.
^}__

Orders

(if,

l.y

C.E.
W.li.ili 11:1. rl.i'li. pi'irc Is.

10.

ARCHITECTURE,

W. 1I.LEED3.

RXJDIMENTAIiy!' ''i^ ARCHITECTURE


xl>

Bfctlu of, by T. BURY, Areliilect


Jolin WeIe, 59, High Holborn, London,

W.C.

rudimentary

WEAJjE'S

JyjE.

series.

RUDIMENTARY. 18, 19. ARCHITECTURE,


IHncipIos of Design in by E. L.

demy I2mn.

In

wLtli

GARBETT,

Woodruff

clntli,

Architect, 2 vols.

price Is.

RUDIMENTARY. 22. BUILDING,

Art

of,

by E. DQBSON.C.E.
.1.

!i:

,,

Urn

iritli

>.

W.ir.-im;

"PUDIMEK TA'liY. 23.


J-^

In

tl.Tiiy

li-w.

2-1.

An of,

TILE- MAKING, *C,

rii.h

RUDIMENTARY.

vols.

26 MASONRY AND
by E. DOBSON,

of,

demy 12mo, with Woodcut a,

In

LI:;.

Wi:i.'li;.it". clu'li. |irlr-e

2fi,

B TO N E- C U TTI NG, Art

-i-i

r]-:tlj,

;.

BRICK-MAKING,

by E. DOB30N, C.E., 2

C.E., 2 vols.

>

RUDIMENTARY. 27. 28. PAINTING, Art


or

GRAM M

\U

COI.0

Ill-

DBA IKING DISTRICTS AND LANDS,


h

.
l

Juifi. -,HrU

\V <;:!

t-

1
:

Act

by G. D.

of,

priiv

fid.

M E N 'LA RY.ljii. PRACTICE


RU1U
DRAINING AND SHW.IGK
TOWNS AND

OF

BL'II.D-

111-'

ING9, Art of, by

of,

GEOKGE FIELD,

NG, by

D.DKMPSEY, C.E.

3e'~ wellsinkinci
Rudimevi Try.'
Artof.byG.K.
AND BORING,

BURNE1.L. OE.

TJUIVBlENTAAiV.
-t^

MESTfl, Ait

of Ihe,

JJUDI m"eNT ARyJ

OF INSTRU-

by J. F.

d
33. -

ftUDIMENTAllY. 35.

BLASTING

JA A Nil vUA];i:VIN<l. AND ON STONE,


J BURGOYNK. Ban., G.C.B., K.E.
.in. vV~
RUDIMENTARY. 30, 37,
]..

-.

f.

ARY OF TEKM3

Mtcl.miicnl

Eii^sitets,

|.,,,

38,

tiv

ROCKS

Lifii.

, rt f.

(ir'i.

fir

i.

39. DICTION-

used bv Architect*, Builders, Civil sod


surseyori., Ariista, Ship- build era, Ae..

>,

Iligb

Hclbom, lOldou, (

WE ALE'S RUDIMENTARY

JJK.

In draj linn,

ilh "\V.i

'l

,l,.:-t~.

RUDIMENTARY. -12.rim

hi

j-!:

\y ,:,;,. jt,

"DUDIMENTA II Y. -43.

rvlw

e.l'Ah,

SERIES.

Is.

COTTAl.i:

ISO, Treatina

J1UILD-

on,
.

-I

,11,.

j.

AND

TL'lil/I.Ail

11

pnnitu.

OOliSON,

-.

C.F..

EUDIMENt ART. 45. LIMES, CEMENTS,


y.Ov.

NELL,

J-*-

Al:i-, (.u_\v;::i;ti:,

.MASTIC-,

by G. K.

k,:,

DUE-

O.K.

AND

KKl'AFlilVi CnllMON' IHIAiJS.l.y ll.I.AW,

RtTDIMEfflf^AKR^ S^S,^ CONSTRUCTION AND ILU'MiNATlON OF LIGHTHOUSES, bv


% ...x
in
4 A N -U
'

-.

]:t

.-

.l.T.y

. .1-

.mill W. odculH,

Y.
EUDIMENTAB
ARCHITECTURE,

clurh,

]i:ici! 1".

53.NAVAL

51, 62,

Prinoiples of the Science,

by J. I'EAKE,

RUDIMENTARY AND EI.EMENTAE Y.-~o3*.


-PliArrKAf, iMN.-TlilTTlON

c-nci-wly Klii'wl nf Siiipa

pUDIMENTARY. 53"* ATLAS


lo ditto,

drawn

unit

isn

of 15 Plafes

urnvpil to n Scale for Practice. Ferine


II lii'U;:Allan Till' v lis h:ul

Ol'i'.ni'iv,!

in tkrr.o

rwiianw

Pmrt

l'lLi'tfl.

I.,

2s

II., 2s.flit.

Part

III., 2a. 6A.

"

"

n-TtFrw,..-. lS eiiis. cl-tfT i.Viri-

InioJiiv lini".

R.

KIPPING,

-RUDIMENTARY. .04. MASTING, MASTIV MAKING, AND EIGGING OF SniPS,


In il.'mv

T)

1N1I.

In-

l-Jiy..-,.

UDIMENTA
.1

[i
'

i.l,

^V.T-.ii.-.LM. rlrili,

I_l

iiy

[.

price SS. Bd.

EON SHIP HUILDiliANTIJAM.

V'. .54*.

N.A.

anri

C.E.

jIKV.I rr.i:st:-ni!i,in !.["];:. i. s,

John Wealo,

159,

Higb Holborr, London, SV.C.

Digitized by

Google

RUDIMENTARY

WEALE'S

JJE,

In

domj

l'imr..

58. WARMING AND


TOM LlNSON^2vola. iol.

hy CIIAIil.HS

Tn demy ISnm.

wisii

RUDIMENTARY.
K.

clntli.

il

5!).

STEAM

price Is.

TOILERS, by

ARMSTRONG. C.E.
irllli Wo klouts, elotfc, price it.

In ilemi- lirao,

pUUIMUXTAUY.
J-fc

ninth, price 2b.

n<;<.<l:-:.i;.

jili

RUDIMENTARY. 57,
VENTILATION,

SERIES,

01.

(in,

LAN 1) AND

ENGINEERING SURVEYING, by T. BAKER, C.E., 2 vole.


^IrToemy

l-Jmn. ivilli

Wnodau".

'pTicalsT

cl<>il7,

RUDIMENTARY AND ELEMENTARY 62.


J-^

PRINCIPI,i:S

OF IIAII.WAVS,

lite

f...

Una of IhO

Construction.

loi

By

ISe-

Sir

11.

19

RUDIMENTARY-^62*.BAIL WAY WORKING IN (JIIKA'J 1SU1TAIN.


[.- ..ii...
'H-UI
1

Sisicislisul

-'A

and

1 ;

A L r.ni.niNTJ?.

tt!

Kn C iiies,

Mtinliim.rv

.f

nt

lb;. C.ii^i-.ir-ri.'m

Tible of

Dctiils,

V l||

-i.

Mm;

Powers,

Hien

Ilia

nod Imphr

M.icliinea,

MENTAEY. 66. CLAY LANDS AND


demy

\V'i

07,
RUDIMENTARY.
11
III

1-Jni

iv itl;

'.

.but:;. !>]nt!l, prjcoS--.

E.

11.

HMNIiON, M

In il-mv

l-l-.n-t.

CLOCK AND
CHICKS AM)

08.

WATCH-MAKING. AND ON

BELL9. by

i'lUIR.'ll
>

'
.

>

lirlr-i 1m.

['1

HY. 71.-l.'IANOL-Oi:TE,
RUDIMENTA
L
-

stntcli'

the,

I'l.iyii'.H

r.ji

i:

by

(.'.

C.

Iu-

SI'llNCER, Mils. Dr.

In demy I2mo, wilb Sleel Ensrivings ami Woodcuts, cloth, prico

RUDTMENTAEY. 72, 73,74,75,75*. RECENT


TV

fossil sin-.!,!,?
WOlJilWAKU, ..I'tl.,:

.m nt-.it^i

lirir.

.i,',i:i,M^),by

r.r

Mus. 4

v,:

in 1.,villi

samugi,

Sitppl.im.m

TiTd,7:i7v :U,,~7witVYi7777n7.,:^,777tl777~[7,^

RUDIMENTARY. 76,
GEOMETItY. by

J. F.

77.

DESCRIPTIVE

HEATHER,

SI. A.

2 vols, in 1.

RUDIMENTARY. 'tV'." ECONOMY

FDEL, by

T.

3.

1-Jk.o.

-'

vol?. ;n

PLIEIITO

it

OF

PRIDEAUX.

. itli Wt.ciltiiM, dull-, prico


"9.
C.F.NEFiAf, PURPOSES.
John Weals, 9, lllgli Holborn, Loudon, W.C.

In *miiv

RUDIMENTARY. 78,
Al

STEAM

_=.

AS AP-

WE ALE'S RUDIMENTARY

jyjE.

^UDIMEJs In

<.KA\

I'LATLS

..!'

SERIES.

LOCOMOTIVE ENLOCOMOTIVE

UEill'iillV'S

to

In diimy 12m,>. with IVun

uls, ntotli, price 1b.

79**. ON PHOTOGRARUDIMENTARY
Jl
1'JiY.

.:.!!

!i

L'

I:j tt

.,n,l

in-..

"RUDIMENTARY.
-iL
LANilj

3 vols, in

!!!;

Kl;-J M.

U:c;il

>t!r, ]ivii:

.-:

,4*.

tin",
L ;

l:

.c:il

i.,b-

u;.

JkJIl'i

hi'

EMBANKING
1'. !_;.;-.

L
In

demy 12mn, with Woodcuts,

UDIMENTAKY.

"R

J-l
I17

cloth, prico 2a.

Si*.

Si,

POWER

OF

wateii, as applilo to drive n.OUt mills,

JOSEPH GLYK

.11

ili'inv 1-Jnin, chilli,

|ii it-.!

Is.

KUDIMENTA11Y. 8;i. 1;00 K- KEEPING, by


JAME3 II ADDON" M.A.
1

In

demy

mo, with VWnkiits. price 3s.


83", 83 (bis)
and Distribution of, by,SAM[JfcL

82",
"RUDIMENTARY.
J-V
GAS, on

'J

COAL

the 5Isn,iLiCtiira

iii;t;iii:s,c.E.

-CONSTRUCTION
SHIPS ASH
Ill

I'O ii'.;

ifniHv 12hii>,

\Y. i;i-ANt),ofIIrttl]i.

t,y

cl.illi, pri.'fi

tJ.R.i.

RUDIMENTARY M. ARITHMETIC,
numerous Esntnples, by Prof. J. R.
In

demv

r.inl'i.
ISH) o,

clnth, pi
H:
!

"RUDIMENTARY. -84*.
J-*;

by

I'rof.

(n ,1,'MV

ldft

l*.

xviiU

YOUNG6d.

KEY

to the above,

J. R.

rjiiio,

RUDIMENTARY.
ARITHMETIC,

U.
EQUATIONAL

c-'f.-.li,

lui'V

85.
Questions of Interest, Annuities, &c, by

vr.ini'M.r.Y.

John Weale,

68,

High Holborn, London, !W.C.

Digitized 0/

Google

|^E.

WHALE'S RUDIMENTARY SERIES.


Iu demy 12mo,

cloth, price IB.,

SUPPLEMENTARY
RUDIMENTARY. 83*.cv(i*ti..!iai
iurnnE.
...

Lf-i.n

<

! Siimili' li.:?;-:*,

Annuillus,

-!)i.f,

Lnuarilhnia

v.

&c &c, by W.

Ill I'.-I.BY.

1 86, 87.

ALGEBRA,

GEOMETRY,

by

ANA-

AND
RUDIMENTARY7 '91^92!' ^LANE2i-ola.ini.
SPHERICAL TRIGONOMETRY, fry the s.imo.
In

demy

Wcndr.uK

with

cloth, pries

RUDIMENTAIiY. 93. MENSURATION, by


T.BAKER, C.B.

XV

Iii

demy ISmn, with WoodcuW,

cloth, price la.

RUDIMENTARY. 96. POPULAR ASTRONO MY. By the


In

1I011EKT MAIN, M.R.A.S.

linv.

dcmvl2m", with Wood chip,

clotli,

prico

U.

RUDIMENTARY. 97. STATICS AND DYNAMICS,

fry

T.

HAKElt, C.E.

In rtony \<>mn. with SJO W.raricuti, 0]


98, !W\
AM! I'KAtjnUAl. i:: N ~ li i/l -Tl
bv T. liAKKl!, {:.];. .wid UN TOOLS

El! DIM HNTARY.

In dooiy

Of

2s.6d.

11 At.'

AND M Al'l!

.1

M-Ii,

N KS, by

12tno, ilntb. price Is. Gd.

RUDIMENTARY.
TABLES,

till, 1'i'iro

.MECHANISM

i'

JAMES NA31IYTH, C.E.

100*.

compiled for practical

iisb

NAVIGATION

with tba BfrnvB.

RUJ.)IMENTARY^ 'wL 'dVpFEK ENTIA I.


CALCULUS,

bj

J^i'lvjill^S,

"lli^i!:

!!m|1.'.-,-i',

U-.\L

Digitized 0/

Google

WE ALE'S RUDIMENTARY

J|B.

~)

SERIES.

UDIMENTAliY. 102. INTEGRAL CAL-

5UDIMENTARY. 103. 'INTKGf! AL CALJUDIMENTAEY.

ALGEBRA, GEO-

1*1
M r.NTARY. 106. SHIPS' ANCHORS
DUD1MJ
1|

J-V

FOR A

TM^NTAW^ ^lort^'^TEOPOLITAN
RrmBU1LDINCS
Net, mil
ACT
in v ,;:,cnl j.pcratt.in, with
2Sth, 1880, for better su^iilying of

Act duted Augiut

tho

Gas

to the

Mctrnpoti".

In

rlflmy lsimo, cloth, price Is. CJ.

NTAHY.

litK

MKTRO

LOCAL MAKAGIJML.NT ACTS.

All tlm.Acta.

im.iTV AMI PALl' MJ l.-ini' .ins.


In

drm? 12mo,

cloth, price la.

RUDIMENTARY. 110. SIX RECENT Legislative ENACTMENTS, for Conductors, Merchants,


Tradesmen.
_______^
pries
In demy
RUDIMENTARY. 111. NUISANCES RE-

(iml

Is.

12nio, cloth,

MOVAI. AM) HISEA^K


In

demy

T> UDIMENTAliY.
CINE,

I'U

AX WE

AiiTiLi.r.Kv
LL, B.A..
1-l .li-.nv

T)

V KNTI

N ACT.

112. DOMESTIC

MEDI-

KSMlVISfi 111-ALTI 1, by M. ItASPAIL.

In

demy lamo,

Tl UDTMENT ARY.

l'lli-I

12nio, cloth, price Ia.6d.

(is-

cloth, price la. Gd.

113. USE
i

^ino. ,,-tli

ti: n,

bv

OF FIELD

i.-.-.u.-vaa.

W,;.-.,.<.c;A*, el ,:1>,

Hamilton

jiri Is

r,:.

UDIMENTARYS 114. ON MACHINERY:

Itudini!Mit!i>T
.ind on Hi,! \V:.rkiii.:

l-;ifti;-.eut.irj..I'

M.H'l.in.':-/,

l'riiici^lr:;

by C.

i).

<

the Gonstmclion

Ai'.E L, C.T..

In royal Ito, cloth, price 7a.

RUDIMENTARY. -115.ATLAS OF PLATES


fid.

OP SEVERAL KINDS OP MACHINES,

llimtri.'.ic

17 very valuabla

iil.ili^.

John Weala,

69,

Iligh Ilolborn, London, W.C.

Digitized 0/

Google

WE ALE'S RUDIMENTARY

R.

RUDIMENTARY.

11G.'
n

smith.

SERIES.

TREATISE ON

of Sound,

ROGEK

by T.

MAGNET.

ISy 1

RUDIMENTARY. IIS.
i:m;:m:::i;im;

>i- Nf.uii

111}.

ON THE CIVIL

n amfhic'A.Iij d.

stevln-

RUDIMENTARY. 120. ON HYDRAULIC*

RUDIMENTARY. 12 ON RIVERS THAT


CAHKY SAND AND Mi l), nnd an ESSAY ON KAV1CABLE CANALS. 121 ami 122 Num. ti-.Hlier, 2s. 6d.
I

In

demv

lim.i,

Wn.i.li-itls, cl.itli,

TDUDIMENTARY.
AND

J-"

.JIUXEKY,

123.

f. iii.rl.'il

In dtlmv

ON

on Dr.

pried la.

fid.

CARPENTRY

Ki.li'snn'i

Work.

4(0, clo'h, urtci! 4". Gd.

-RUDIMENTARY. 12.1*. ATLAS


IV

in .lefail

tn

'*

In

lb,.

" er

AM)

CAI;Pi:NTIlY

in cliilh In

demv 12ml,

of PLATES
.JOINERY. 123 and

vol.

willi iv.wrtriits, clnrlt, price Is. ai.

RUDIMENTARY.
ON ROOFS
H
PUBLIC AND PRIVATE BUILDINGS,

JTOR

124.

founded on Dr.

Robisou'M

Work.

TJUDIMENTAKY 124''. RECENTLY CON-*'

STIiL'UTKD 1I10N IIOOI'S, Alius


til

.1

"11V

1-'!1 .

.-.-];!,

:V<-...d--.'.

-.

f]-h.

of plates.
firii-s

a^.

RUDIMENTARY. 125. ON THE COMBUS-

TION OF COAL AND THE PREVENTION OF SMOKE,

nV'iiin-.llv

I'wirjliv OsihiiMvd, by

:li1

ciiaki.es

WILLIAMS.

wye

"

2<" I LLL VTFAITONS


M. iS'l AM
RVDlWILLIAM?CiMlir.-iTIUN OF COAL,
'l

fo

2 vols,

125

bound In
In

and

126,

1,

demy

12nio, with

Woodcuts, oloth, price

RUDIMENTARY.-

Is, Cil.

ofPRACTICAL
INaiiciiitectuha

127.
stiiuci [iins in tiii: aim

modelling.
Jolin Weale, SO,

II q!i
i

Ilolbom, London,

W.C.

DigiiizM By

Google

WEALE'S RUDIMENTARY SERIES.

E.

demy

In

Wpndcuts.

12 mc, -ilh llii-rsvlnijs null

RUDIMENTARY.

THE

128.

TEN BOOKS

M. VITULVIH UN CIVIL, MILITARY, AND


NAVAL ARCHITECTURE, translated by J03EPII GWILT,
111-'

dnmy 12mo,

In

Tsa Enquiry
\n;xyA
Emu
THE PRINCIPLES OF
;

'v.

into

lil'.AUTV IN lililiCIAN AHC11I-

ABERDEEN. Ac Ac

T}

THE

UDIMENTARy! 131.

J-^

AXD

MEHCHA.v:

foTT.-^iTr-.rii:,;;

thcr will)

;it

lie iip|ir-'\iiniiti'

demy

In

.if ;ii'.v

.jn/iniltv

..f

Cum

lug is

mid Mtllwork.

11-1.ti.-h

with Wnidcuta,

12mi>,

MILLER'S,

ready clukuneii,

rAi:.M;:u^

Vaji.'
v:U m ni'

Si-rli: llin

clnth. jirico 2s.

6i

JIENTAHY.-l 32.TR E A TISK ON THK


RUDI
.Eiikction in- invm.i.iNt; jiiursK*. WITH si-nci.

ncArioNV ijcant ri'ir.- nr


Ac.,bv

Si.

11.

In

HliOOKS,

demy

litmo,

27 I'lnriM.

with Woodcuts, cloth, prlco

-RUDIMENTARY. Vol.
-U.

vAiiious material.-,

Tin'.

AtcV.itisd.

In

demy IZmo,

by R.

have Woodcuts,

to

5a.

TREATISE
ON
LAMBORN.

1.

THE METALLURGY OP COPPER,

II.

cloth.

"RUDIMENTARY. Vol. 2. TREATISE ON


-l**

TIIE

METALLURGY OF SILVER AND LEAD.

In iIctiiv

!uvi.

RUDIMENTARY AND ELEMENTARY


t.i

y.,].:t. TKF.Ansi.;

ON

ir.ns

..

^,

i-K-.-li,

METALLURGY

AND

3
INC.

PLATINUM, IRIDIUM,
In demy l'2m-\

EUDIMENTA RY
Vol.

to

(las-.

ASSAY'-

to li.lvn Wco.lc-itB, cloth.

ELEMENTARY.

A ND

5.TREATISE

NIC KEL, COBA LT,

up

Ac.

ON

TIIE

MINING OF ZINC,

TIN,

Ac.
_

In

il.-i--.-

Jini.i.

(..

Iiiia.i

V,-,.

vlt-nH.

rl.

.[-.,.

"RUDIMENTARY AND ELEMENTARY.


JA

V..!.

n-.-Tlil'.ATI^i:

In

demy

ISm'n.

ON COAL M1NI.NO

RUDIMENTARY. Vol.
TALLURGY,
WATT, F.R.S.A.

John Wealo,

Kiculpcv

with Woodcuts, cloth, prlco


Practically

59,

7.

ELE

treated

by

High Holbora, London, W.C.

Oigiiized 0/

Google

OF EDUCATIONAL 'WORKS.

\V,jr]rMt;:.

15IK.1, v.i:!:

!:i

dlh. prire

2s. (id.

OUTLINES OF THE HISTORY OF GREECE.

-U 3 V,\
-

,5.

Iq demy

Map

Hisin, Willi

("\UTLINE
V

7,

11.

IIAJil^/I'^,

Iilv anil

.if

Woodcuts, cloth, pr:co

2s. .Cd

OF THE HISTORY OF ROME.


8. By W.
In

demy

HAMILTON,

L>.

2 vols.

larao, cloth, price 2a. el.

pHRONOLOGY OF

CIVIL

AND

ECCLESI-

\J asvh >-. iMSTOiiv. :.ni-:ii.'.Ti:i;i:, art, and CIVILISATION, from the earliest period to the present. 9, 10. 2 volfl.

demy 12mo. cloth,

In

price Is.

GRAMMAR OF THE ENGLISH LANGUAGE.

VJ

By HYDE

lli

CLARKE,

D.C.I..

1\ Ry HYDE CLARKE, D.CX

12, 13.-A New Diotionnrr of tins L^-IL;!, T. v.-.w


Htm. iL:)..vi, UYXI'OO in,:,!:, k ri.!,l';'l ni .re ll::m in
IIVIIH CLARK-'., ll.C.T...
vul-.inl.

k.l.y

;!

In

demy 12me,

cLith, price Is

Q.RAMMAR OF THE GREEK LANGUAGE.


ly

H. C.

HAMILTON.

HAMILTON,

LI81I LANGUAGES^-lti, lb\ By H. K.


In

demy 12mo,

17,

pEAMMAR
VJ

cloth, price 2s.

"DICTIONARY OF THE
U
GREEK LANGUAGES.

ENGLISH AND

13. By U.K.

HAMILTON,

"

Oe THE LATIN LANGUAGE.

-19.-By

the Rov. T.

GOODWIN,

LI8H L AKGQ AQ E3 .20, 2I.-By


In

demy

LANGUAGES 22
LATIN LANGUAGES.
WIN, A.ll. Vol. II.
.

In

the Key. T.

GOODWIN,

I2ruo, cloth, price Is. 63.

OF THE

TilCTIONARY
.BY

A.D.

demy I2mo,

'
'

23.-I

cloth, price la.

QRAMMAR OF THE FRENCH LANGUAGE.


John Weale,

S3,

High

ilolborn, London,

W.C.

Digitized By

Google

JR WE ALE'S EDUCATIONAL

SERIES-

/GRAMMAR
OF THE ITALIAN LANGUAGE.
VI
a.

jii

demy

ELWEB.

lima, doth, price

TICTIONARY OF THE
U
AND FHENCII
ELWES.

2s.

ITALIAN, ENG-

L A.SGUAGES.-28, 29. By A,

LISH,

Vol.1.

demy

In

ISmo, cloth, price 2a.

niCTIONAU-V OF THE ENGLISH, ITALIAN,


-L'

AXD

LANGUAGES.BO,

FitENOll

31. By A.

ELWES.

MlAlLMAil OF THE SPANISH LANG L* AGE.


In

demy 12mo,

cloth, price 4b.

niCTIONARV OF THE SPANISH AND


ENGLISH LANGUAGEa.-85,3fl,37,38.-lly

A.

ELWES.

4 Tola, in 1.
In deiny 12mo, doth, price

la.

Q.RAMMAR OF THE GERMAN LANGUAGE.


In

dmiy 12mo,

clolh, price Is.

GERMAN READER. 40. From

/CLASSICAL
\J

the boat Authors.

n,

DICTI0NAEIE^ OF THE ENGLISH, GERE.

MAN, AMI
HAMILTON,

:;

]m:NC!T L,\N(U:Af!!:S. 41,

vols.,

In

separately,

demy 12mo,

Is.

42,

43.- By N.

each.

cloth, price 7a.

OF THE HEBREW AND

-niCTIONARY

J-* ENGLISH LANGUAGES.H. 45.C ante iiunji the WW&WvA


and Rabbinical vuvlK
u-i=. !ii>;;p(]ier with :h-> iIrium,-. nbWi
may bo bad separately for Is.), by Dr. BEE SSL A U, Hebrew Fro:>

DICTIONARY OF THE ENGLISH AND


IIEUREW LANGUAGES. 48. Tel. III. to complete.
In

demy 12mo,

cloth, price la.

FRENCH AND ENGLISH PHRASE BOOK.


John Weak,

59,

High Holborn, London, W.C.

Digitized By

Google

12

WEALE'S CLASSICAL SERIES.


neatly printed on good paper.

Tliose

pi

rjEEEK AND LATIN CLASSICS. A Series

Volumes

cmi:;ii;iii:i;

1'iijilt,

'.In'

princ'iiul lircfk

of

and Latin Author*.

y n,.:,^ [u Kn^i.-ti, i.Linoipilly

1-^

r.c^.^iiji.ini'.Li

pri"

S'.!'.i

t.

.1

and

apclic.itii.-

fur tliu

L'nivei'-iii,^

i>xfi>rd, Caiulj.-i.i;:...

t.f

}'.<: n;l:i!r:;l]
(.([it-.; -.i-. Alj. : iv,-.:i. w.
SJ;i:..|in iIll C:.!!,'^ ut li,::";i-.i,
Cork, Ualwuy, Winchester, and Eton, and tbe great Scii. [-> ;>.;
U.iriw, l[uj.-!iv, ftc iUn f.irl'rirate Tuition and Instruction, and
l

fur [lie Library, as

f.'lli-.'H

LATIN SERIES.
In

il.-Tiij- 1-Jiiio,

imards. prirc is.

LATIN DELECTUS. 1. Extracts

ANEW

from Classical Aulbors, with Vocabulaiics and Explanatory

Explanatory Notts

in English, and a Geographical Index.

demy ISmo,

In

boards, price In.

QOENEL1TJS NEPOS. 3. With


In demy limo, boards, prlee

English Notes,

IB.

IRGIL. 4.The Georgics, Bucolics,


In

VIRGIL'S

with English

demy 12mo, boards, prloe 2s.


U. 5. ( 0 n tlio same

j-E N' EI

pkn

a-

the preeedipg).

6. Odes
HORACE.
Notts,
Analysis

and E^podaB; with

and Explanation

and

in iteniy limo, boards, prior

Englisli

of the Metres,
ls.fi.1.

HORACE. 7.Satires and Epiatlea, with English


Notes,

In

demy

I'Jino,

fcc.

hoards, prico la. 6d.

Conspiracy

SALLUST.

of

8.

Catiline,

Jugur-

thltw War, with English Notes.

ijERENCE
lu

demy I2mn,

boards, price 2a.

TERENCE. 10. Phoniiio,

Addplii, and Hccyra,

with Engllah Notes.

plCERO. 11. OratTous 7'

S^illa>r A^chlis, and_ror_ll


In

demy 12mo.

plCERO. 12.First and Second Philippics


V
tlon< for Mil",

l..r

Marcoll.*.

tie.

Ora-

JjB.

WE ALE'S CLASSICAL
In

QICEEO. 13.De

Jemy

SERIES.

12oio.

Officiis.

TTJVENAL AND PERSITJS. 15. (The

"

indeli-

ca'a par ts eTpun B ad.)

I IVY.

In ilamf ISinO, boards, price an.

16.

Books

i. to v. in
English Nolfa.

_J
In

i!

l'Jmii,

board u,

J^IVY. 17. Books

xxi.

!n

,].:,)

[irici

and

two

vols.,

_
with English

]'J."

JACTTUS. 18. Agricola;

Germania; and An-

CELECTIONS FROM SUETONIUS


CREEK

SERIES,

with

Is.

xxii.,

ami

(he

ON A SIMILAR PLAN TO THE LATIN

jj-ENornc-N.

a.

VEXOPHON. 3. Anabasis, iv. v. vi.

vii.,

with

English Motes.
In .ferny 1211)0, bf'irdi, price Is.

rUCIAN. 4. Select

Dialofraea, with
SotM.

J OMER.

English

In doinj liiim, boutta, pri In. en.


Iliad, i. to vi., with English

5.

Notes.

JJOMEE. 6.^iad,viLto^

Jn

my

l'Jiiu'.

b un!::, rriri:

JJOMEE. 8. j^Iliad^
John Wealc,

59,

High

Isi

six. to ixiv.,

with English

irollnrn, Tendon,

W.C.

Oigiiizsd Dy

Google

WEALE'S CLASSICAL SERIES.

JJH.

dvmv limn,

In

Iwards, prfea 1. 61.

OMER. 9. Odyssey, i.tovi., with English Notes.


in

demy limo,

nri

bniirds,

1*. CJ.

gOMIiR. 10. Odyssey, vii. to

xii.,

with English

JJOJIER ll! Odys'sey'sUrto sviitwith English


JJOMER,

"li-'^dysaey*' xix/ to^'sxiv.

and

PL A.TO. 13. Apology^ Crito, and Phauio, with


i,

In demy 12 mo, boards, price

TIEROBOTUS. 1-1.
-11

I).-

I'.-l

in

11k

!n i!,T:iy

i.

C.-ai,'.

ii,
the

Mil:.'

|-Jnif>. Imili'.I:.

HERODOTUS. 15.

of tli'vm.Kiiin'.

iir

-.

fi.l.

with

iv..

iii.

ln.lWt.'d to lira Grace the

In. Od.

with Engli.h Notts.

Dnta

Ensjlisli

Notes.

0 r Devon-lii.

demy 12ma.

In

HERODOTUS. 16.v. vi. ami part of vii.


1J

t.j

lliiCraci tho

Dill;-.!

nf IVvniisilire,

demy lamQ.

In

HERODOTUS. 17.Remainder of vii., viii., and


is.

li-ol.'Jited

In

1-Hn ftrac* 11m

demy

Duke

of Dciv.nsliire.

12mo, hoard*, price

gOPHOCLES. 18. CEdipua

1b.

Rex, with English

g OPHOCLES.1 9. ffidipuT'CoIonaaus.
g OPHOCLES. 20. Antigone.
g OPHOCLES. 21.Ajas.
In dcrayiamo.-

g OPHOCLES. 22.Philoctetes.
In dt-mv laoin. boards, price

URIPII>ES.23.Hecuba,
In

demy

Is.

6d.

with English Notes,

IBrao.

JUEIPIDES. S4.Medes.
In

dpmf

ISroa.

gUJUPIDES.25.Hippolytus.
John Wehle,

ED,

High Halbom, Lan fan, W.C.

J|K.

WEALE'S CLASSICAL SERIES.

J^URIPIDEs"-^2&Akcstisi with

English Notca.

In demy 12wo.

]URIPIDES.27. Orestes.

EURIPIDES. 28. Excels

from the remaining

r\yx.

COPHOCLES. 29.Extracta'from the

remaining

Plgya.

la

demy limn.

^SCHYLUS. 30.Prometheus Vinctua.


Ia dam y ISmo.

^SOHYLUS. 31. Persic.


In

ffi

demy l&no.

SCHYLUS.32. Septem contra Tliebus.


In

demy

IStno.

JgSCHYLUS. 33. Choephone.


SCHYLUS.34. Etimenides.

SCHYLUS.30.- Supplice

DLUTARCH. 37. Select Lives.


J^RISTOPHANES. 38. Clouds.

j^EISTOPH A NEs! 39. Frogs.


In demy lSmo.

"ARISTOPHANES. 40. Selections

from the

iviuiiliilne ('rpjDBdlea.

in

i-'ir.i:,

(Mill Js,

'JHUCYDIDES. 41. I.,


ili'i::

v-

^HUCYDIDES. 42. II.


John Weale,

60,

12

jii

i.M Is.

with English Notes.

'

High Holboro, Iiondon, W.C.

Digitized By

Google

16

WEALE'S CLASSICAL SERIES.

j^R.

ipHBOCEITUa 43^^rt

Idyls.

Induy lima.

JJINDAR. 44.
In

gO CRATES.

demy 12mo.

15.

lQ dBair 12ln'

JJESIOIX46.

MR. WEALE'S PUBLICATIONS OF WORKS

ON ARCHITECTURE, ENGINEERING, AND THE


FINE ARTS.
Atlas, folio Volume, wilh fine Plates,

acmuMEK,

nut yi S n

ttes.

A tlm

la.

'

HEN'S CHAPEL,
Fir-o 1'l.itss

prim 1

WORK." -THE

'B RITISH GOVERNMENT

fr.ih,

price 3 10a.

Architect.

Atliis folio, -1 livriii.*!!^, nubii;,!.^ in MuClh). nl 10G rsala


Il'mtmtnl by beautifully executed
iu Kii-larnl.
Engr.iviijgs, sonie of wbleh ure coloured.

I.ni-Kis

well, or 1

(i

GOVERNMENT

CPANISH
^

.iu!m;mi:sts

[agni:,

i'i:iiLii':i

ai:

aux

ilrnl

ToM...

manm

Pa kt

IK' nil
I

WORK."

i[iiKrTUMc)UK ijk
he i.a kation.-Pabt

i-utAis

l'L-,ivji:r:n (In T-'Id.i., (;in:i,i(l:i, Aic.il.'i

3.

dc

lldii'inis. 1'aBT 2.
T. U'dn,

Ciinmdi, Segovia.,

CateSi1:l-

Part 4. Santa Mail* de AlMli do Henares, Casa Lonia da


&a. This work surpasses in beauty all

Valencia, Toledo, Segovia,

imhler folio plates, 'ith tent also


j

..._..

u
1

.
1.

ii

Iwd

irn

T.-i

and elegantly hnlf-bound in ufm, ni!>, .CIO 10*.; Plates in


ami it-::; in i-n;i:'n;il -im. uiU'-i.oiiiid Inmorucco,
C.il iiiitiicr f.ilin,
jrill,

7 1-.; I'la'cs in

in (loth extra, bo-rds

C"!'inil'i.H-

and

VICTORIA
T'HE
IN CANADA.

leltared,

I'nlii., :irni

BRIDGE,

i:i

iusN.-riiil 1;:..

14j. ed.

AT MONTREAL,

Elabcraifly illiistratoil by views, plans,


;:iid (lot ii'si.J !ke
t-^lil'r itii tin: i llu.it r,t (, im
lohlnexy snd Contrivance used in thoco

clm-i.-ioxn.
ol

On U

MOKTiiN

M.P.,

l'l--,TO.

WARD LADD

BETT3,

John Wealo,

B>,

THOMAS

liii.YSiE Y, and

F;:, l: -=.

High Holborn, London, W.C.

ED-

COCKERELL'S WORK.
PROFESSOR
THE TEMl'I.L/S
PA M MILL
AT
HI''

AND

-KGIVA,

.II'I'ITF.!:

IINII.'S

LFlUUlUUrt A

ill-' _\ I"., [.!.,)

1'

iiAeS.-K,

SEAlt

ARCADIA.

l'lllGALEIA, IN

It is propo;ed to publish

the.

and Worki of the

Life

JSAMBARD KINGDON BRUNEL,

lato

F.R.S.,

PRACTICAL HOUSE CARPENTER.

I'HE

theirdBtiils: 7. DtsLs;:!-: fur Modi. Vv'iii'.i-i-.v!;, witii their ilo'iiils,


for Villa Architecture. Tha whole amply described, for the
use of the Operallve Carpenter and Builder. Firstly -written and
published by
PAIN. Sccondlv, willi Modem Designs,
and Improvements, by 3. H.
3, Architect.

and

WILLIAM

BROOK

In

A
a-

18131 will

be published a volume

DIGEST OF PRICES

of

in 12tno, entitled

Works

Civil

in

Engi-

neorlng and Hallway Engineering, Mechanical Engineers e,


Wr, n-ht and l.:;:;."L lnm W- rks, L?
i
r ;u:d Wi.c

Tuiils,

AAIRY,

ASTRONOMER ROYAL.

1 1

F.R.S.. &<s.

Result, of Uiperimeuta on Iho Dlsturbanca of

ths'L'niii

pji3

in Iron- built Ships.

In a sheet,

3a..

in case, 3s. 8J.

ANCIENT DOORWAYS AND "WINDOWS


i

'-

(1-

In 1

xnmidiH

Arr>in;;;d

Aiv!i'ir.^;KV.

l.i.ithic

vol.

Ii-m

111-- <

In

ilhn[;;il'!

Ill-;

iliff.

rent ntvTn-i

l.-nij.i-^r ii.iln - !:-f,..-.Mii1i.- i; .

I'

'

imperial 410, with 20 fine Plates, nemly half-bound in

ANCIEIs
NCIENT

DOMESTIC ARCHITECTURE.

Prlnein
of the late Sir
application of a

modern

<diriri;s.

The stained glass

NGL

CA

facsimile.

N C

49.

(3d.,

In

an ir case, or In a

N AM NT.

It
V.
I
1 1
l' I
<
I
A
-fl Wherein aro figured the Bainta of tho KuirlLsli cali'iidur, wit li
their upproprint* emblems; tho different stylus of stained glass;
and vinous mi-r.-il i.v::i!>(il and in n u:ii ul.-i Hs! il in fliLirdifl.
J.ihn Wi nk', 5ii, in ;h llolborn, London, W.C.
I

Digitized by

Google

WHALE'S WORKS ON ARCHITEC-

abt

rjil.niicil, 4

FINE

KNCINI.i.i.IM;,

TT;i:!.,

tn 4to, with

Ali'l'S,

,S-c.

j 0 Engraviofia, somo of which s


"li^irml copies, lialf-houudin morocc

I-.,

ARCHITECTURAL PAPERS.

410, In.

^RNOLLET, M.

Report

"

l:i-iti>.

'1

cm his Atmospheric

Hi Plates, 7k. na.

TMOSPHERIC RAILWAYS. THREE


l'01:T3

hiijimv.i!

ri,

Atmi>si]ll.l-iBRail-.i^ys.

BARLOW,
In rue.

rmislTiirliii;: find

MALLET,

li.

ay S inncnslon

on

Niagara

tlie

Itrid ga

4 to, vcrr Hunt balf-iuorocco. 13s., witti Kngiiviiigs,

CHARLES, R.A.,
BARRY, M SIRIW^U
Ard,i:.u'u-^.
Hy
......
rho TnnlUrs'
Su.iilen "f
ITlllv.Ho:

I'lii;

IV.

lj]

ill

"'

Cluli-ilous
lull
,t,...

i:kv,>tiju-, an.l
i.;iiiv,itiju-,

Sii-ctic us,
tk-ctie

RE-

IV u iking

C.E.

W. Observations

P.

I tai I

In

methods of

l!v

If.

Lnjji-f.vint;;;

&c.
LKKI1S;

flan-:,

ot

dfUila.
li.

In 1 Vol., large Bvo. wllli cikiisiml Hales, lialf-ni.iiorrri, price 1 Is.


(J. G.)
OS Til!-: !>lL-T!tlOT OF C]j:Vh.l.AM) is mjutii

GEOLOGICAL TREATISE

BEWICK'S

InBto.wIib Plata. Price 4a.


S.
on Geometrical Drawing,
embracing Practical Geometry, including iliu une of Drawing
inioiit
Or; cou^ruciinn and n:;r uf rici\s, Oi'Uiognplik Preami Elementary Dencilnttvc Geometry.

Work

BINNS, W.
lush-

..

lection,

In 4tn, wild

;, IliiiS

L-.Lt.v- I'Uti-N, el,

1 lis. 8d.

i....:-.i-l1s,

F I E L D, J. M., M. R.
BL A S HT ION:OF VASES, STATn:S, fHISTS,

&c.

List.,

SI.

I.I

I.

I'KlillA

In 8vo, Woodcuts,

J. M
BLASHFIELD,
Tin: 1!istiii;y

.uvular or
OF AXUENT AND

gODMER,
BnHn.M.

R.,

li

Is.

M. R., Inst,, &c


ami m a m:fai'ti-i;1',

Ml IDF.RN TEKIT A

C.E. On

li.ii.-sbi- 3 f.-t,

amilili;
r.Heii

d-j-.,:n
r

L'OTTAS.

l;irEc

j'w:,lf

ttie

COTTA,

Prop ilaion

munificent Plate, 3
.:

;;j f,..-t

loan Inch,

t .irimiing Flan nml Elenui.n.


,1.
Tl i.i UVrk ,1T!,--, P.K:,Mi:S.

J*ii WoaJc

of Vessels

59, Iligli Il jlb ni,

of

feet

LOMJDX

Engraved mid

elnb-J-

London, W.C.

Digitized by

Google

19

KNCISliLHINC, FINL ARTS,

Tl'IU-],

BRIDGE.

JJ

Han

and Elevation, on a
STAINES liHIDGL;

10 fee. to en inch, nf

scale

of

E^-r.ivins.

fin^

Tlw*riti.flUenENSIE3.
In rami Bra, with vrry elaborate Pit tea (fold td), L lOi.

wi:h

UUisi!-;i!i-ii!f,

In 1

ill

An

Account,

Bridge across the River

ilui S!is[i-usi(in

bound

vol-., royal Hi-o,

AM)

nf

Wm. T. CLARK, F.K.S.

BRIDGES.
TIMlSElt,

SUSPENSION.

BRIDGES,

-O

Dsnubp, by

3 vols., lulf-iuoroec",

rii-ic'

lOi.

THE THEORY, PRACTICE,

in-iinT.cTn:t: fnncn-.s or sniNi-:, ikon,


AMI WIltK; iviih Kxn;jjli-a un I!..' Principle oi SUS.

pensiou; Ilhislni'cJ by

iS l-JriKniTLn^s nr:S !!

Woodcuts.

In one Urge 8vo volume, wiLh explanatory Till, and

BRIDGES.

I 'lutes

Bound in

rujjrihini; detail! and mea-ured diiuenatons.


ntiTurni, uniform wUh ilin preceding work, price

ti

hatf-

2 10s.

SUPPLEMENT

TO "THE

THEOKY. PRACTICE, AM) ARO H1TECTDSE OP


ri:i!i-.;i:s or sroM'., ikon, timukk,
wiee, ami SUSPENSION."
largo folia Engraving, price 7*.6d.

gRIDGE mmss

the Thames.

SOUTH WA RK

1 [urge folio Engraving, price 5a.

BRIDGE

across

Thames.

the

WATERLOO

STONE BR IDG B.
1 very large Engraving, price

BRIDGE
-D

acrosa the

Thames.

fin.

VAUXHALL

IRON BRIDGE.
1

very large Engraving, price

4s. ed.

gRIDGE^crosath^Thames. HAMMERSMITH
rge

(Ve UPFER'

BRIDGE

PHILADELPHIA,
1

BRIDGE

1J

SCHUYLKILL)
known spin

the greatest

large Engrnrins, price 3s,

Urge Engraving,

BIUDGE.

ON

PENSION, by

no uk nux

THE
Sir

I.

price 3a.

at

of one arch,

fid.

SCHUYLKILL)

fHILA-

at

(the
DELl'iUA,&.vcred.

fii.

PIUXCITI.E OF SUS-

BRUNEL,

In

lUo

ISLAND OP

large Engraving, price 4n.

BRIDGE. PLAN
1

and

ELEVATION

of

the

PATENT IRON BAR BRIDGE over the River Tweed, neat

Berwic k.
31 Plates,

BRIGDEN,
Of

ami Views qf
Henry VIII.

R.

Interior

8eft on

John Weale,

folio,

la.,

boards.

Decorations,

Church, Lancashire, erects

59,

Details,

in the tetgD

High Holborn, London, W.C.

WORKS ON ARCBITECEE&INQ, FINE

AltTS,

(te.

j^RITTON'S

VIEWS

(Jolin)"

(Join ii

JJRITTON'S

In

j^EODlE,

Rules

.Tl.Ct,Ll,.HUS

views

v'e'

ot

for Rangiug Railand witln.m T:,bl-.

CHAIN PIER at

i.)

1>y

WEST

the

of

14CATHEDKALS.

410, 2s. Gd.

R., C.E.

'

!':[i!si'T-'cri

.IK of

Brighton,

TIIOM

IS..

R. SUPPLEMENT.
BUCHANAN,1XAMP1.F.H
1>V MullKK.V 'HJUl^ AND
T.-sl Mi

PIlAl

M AC III
euiri,

bv

-v... ;mi,1

:.!

i-.,.

i'liiti'S .11

;ni!..-,

,.,i

'IS
HiLfpi.-meutim- V-ltuno ta .Mr. HLXK[E"3
Hi;i:ilASAM "On Mill-Woik am! (Jihtr Machinery,"
;.

.,(

iL

T Ei

t",

Sic

'

f.

Tins

>

rk

n't'

IS

risisl:;

l'l:il(

'-

gURY,
(CALCULATOR (THE)
*J
I

and

Chan'T's

lICII .U;, S'>N

In

pALVER,

Or,

nuiu>i-:ifs

ml UliAILI.KH

8w,

TIMBER MER-

nnra:.

I.iANI',.

[.(

n r wii.liasi

Wi.-li.-A.

Plates, cloth hoards, Ts.

E. K.,

Gil.

R.N. TJIi; CONSERVATION

imi iMi'iiovF.ur.NT or TinAL civihs.

In t'o, hiU-bound, pilea 1 5.

Of
A SERIES
WINCIM^-

OWEN
p ARTE
^
OF 'I1IU AS- MINT

B, Architect.
PAINTi;i) UL.AKi

11,

Ti;n i;vnn:m:Ai.,
In

i:,:,-ii;.;i.,

4to. 17

L'i

[.:,!

John WeaJe,

59,

ot.

wc.,i-wa

Flat, half-bound.

p ARTEC, OWEN'
^
of tub i.nuucii
;,!,!.

II!

!(.,

of

LMli . .K
I

joiin
A

,c

I,

[iliisimH.in-

7s. Gd.

Architect
st.
f.f L!-i

'

ACCOUNT

the baptist,
,T< mi,.

High Holburn, London, W.C.

21

In 4tn, ivilh 13 ErigraylngH, 1 18,

pHATEAUNEUE,
V*

A.

Architect. Architec-

de,

a Series of very neat examples of Interiors


;
rosi.loutos in the rial nn style

turn Dotuosiica

and Exterior*

->f

ALE,"l NIG S" J ONES^VoHNSON,


p HIPPEND
LOCK, and PETHER.Old English and French immyj

inent!;: c:[i.ri.=

iiii:

lUlilc.^uson

l'laiosof slafcurate

it)5

(i

r in:i:i:l(j

d Edition with wldliiMH, price 1

^JLEGG, SAM., C.E. A PRACTICAL TEEATACTURE AND

nLEGa*' SAjT,' *C.E.AECHITEC-TUEE


V
MACHINERY. An
Form and
For
mm
Eaaay on Propilcly of
of Students and Scliuol maulers.

the

portion.

fn

QOLBURNS, Z.On
One very

pONEY'S
V

Steam Boiler Explosions.

View

Interior

(J.)

Church of
In

fld.

of the Cathedral

Paul.

St.

on card board,

4to,

OP
Pro-

8t.i, Is.

larga Engraving, price 1b.

la.

Q 0 WPEE, CDiagram of tha Expansion of Steam.


In

<ine vol. 41".

wIPh 20 Folding Plates, price 1

la.

pEOTON AQUEDUCT. Description

of

the

New York

Croton Aqueduct, in 30 Urge detailed and engineering explanatory Platia, with tet in the English, German,
and Frr-r.ch J.n:,:l rii;i":. by T. N(;IHiAMKli;, C.Ii.
In

demy 12mo,

cloth, extra

DENISON. A

bound and

lettered, price 4.

Rudimentary Treatise ou Clocks

account of (be Weitmtn; with a full


v-iu, l.'j iiiuiUMi hkckkt hi:m#on, m a,
Fouitl. Edition re-written and enlarged, irtm En-..vhus.

and Watches, and Bella


st-rci.K-it -"!

Q.C.

D0WNl's!'V:^0{]^:V' AixiiHtct'-^^t
bition Building.
the Great Fvhihir.i.:n,

The Building
1851

'

-S

erected in

Hyde

InhiPa. k
'

!or
=

Finns, Elcr.nif.iis, Scctlnnid. and


and the Wifk int: and Meuaured Di-an-ines.
I

DRAWING

BOOKS. Showing

FU|icrior mctlir-ii of

Diawing

mi

to Studeuta the

sliidDiviii^.

BOOK. COUES ELEMENDRAWING


TAIttES DE LAVIS APPLIQUE A r/AKGIHTEcTU HE;
fulii)

v,,l'l!:i,'.

.inil tinie,

e llt;lil:itii( 1'J

|.|:...br,:.,(.-lv

.ilt

very finely Biecuted, by tha bet

i..

,l

fillips,

artists in

-:
i

Franco.

Parla.

John

\V,

r,il,

Jl..llu:n, Luv.^oli,

W.V.

3.

WOltKS ON ARCHITEC-

"WEALE'S

/IE,

_
.

TION;

folio

shadows and
France.

-COURS

APPLIQlifi

fil.fiM.EN-

A OKNGML.NTA-

volume, containing 20 elaborately engrave 1 l'in


in
tints, very finely executed, by the butt artists In

Pa

1.

1b.

RAWING BOOKTEtUDES

PROGRES-

SIVES KT COMPLETES D'AliCUI'i'ECTUKfci DE


LAVIS, pnr J. B. TRITON j Iarg folio, H fine Plaied, comprising
tLe

LJuliT.-;

aii'l

i'-M-ms

purterraii.

!:n,uli!ingi, i:h p:-=.:ll.:^. m-mmi-jiiLs,


f Aicli[titti:i-i!,
of thnlr proportion, art of shadowing doors, balusters,
ftr., &c.
1 4a. Paris.
i

Ac,

ill

ECKSTEIN,
Chimneys;

Smoke and

willi

oluih Ij.JIldH. lullnrril,

G. F.

."i:..

Practical Treatise on

remarks on Stoves, ins coimumiiiion of

Coal, Ventilation, 4c.


1'tatfs,

imperial Svo,

ELLET, CHARLES,

C.

price. 7s.

E, of the U. S.Report

on iho Improvement of Kanawha, and incidentally of ihe


Ohio River, by mama of Arllflcial Lake?.
In Svo, with Pistes, price

EXAMPLES
a

12a.

of Cheap Railway Making;


murium and Balkan.

In one

vol. 410, 43 I'iateB,

with dimension*, extra cloth boards,

"PROME,

Lieutenant-Colonel, r'.E.
Outline of
th Method oi conducting a Trigonometrical Surrey for tba
I'lH-iimi-.i!) uf Tiipfigmpliical Plans
and Iris mictions fir filling in
Ihe Interior Detail, both by Measurement
J..-c: ]ng
)lili;iiry
Keconnniaaanees, Levelling, &c,
together with Colonial Sur;

In royal 8v, with Platen nnd Woodcuts,


i

roTPli, prti

FAIRBAIRN,
APPLICATION OP

'?.

Scc<>nii Bullion,

in Cltl'iL riMli

W,

U.E.,

lifi,

"thITthi;
'LUID3, by JC

a Weal*,

69,

mncli

Ifls.

F.E.S.-ON

THE

CAST AND WROUGHT IRON TO

High Holborn, I^ondon, W.C.

ss

pWILT, JOSEPH,

Architect. TREATISE
Tin: i:Qi-[T,iTiiiiuM op Ar.c'in;^.

In Sn,

clo:li

TTAKEWELL,
-*

S.

art., will, R l'l,

t.

4k.

J. Elizabethan

ON

<!.!.

Architecture

by pnr.iUnH r,( Dirton H-.11S0, llaili.-ld, Lnn*million, in England, and Hid 1'alni/.. LVIlj L'miccliai is

tlliHTur.d

loat, mil
at Rome.

HAMILTON,

P. S., Barrister-at-Law, Halifax


Nov* Sctia Nova Seitii considered as n Field fur E.nl-

JJEALD, GEOHGE, C.E. System

Two

Vols.,

demy ISmo, is

TJOMER. The
Hymns
by

tlie Heir.

of

T.

of Setting

Out

cloth extra boards and lettered, prion


lis. ad.

Iliad and Odyssey, with the


Hnmor, Edition wilh an accession of English notes
L. LEARY, M.A.

Ft.

~
"SON, JOSEPH, C.E.
C.E.- The Working
JJOPKINSON,
h EngTBTlngs, clolh boir.ru.

Kn

11

Third Edition,

Explained by the

;i;u.

HUNTINGTON,

J.'

lfia.

of

It

B~ c7e7 TABLES

and

RULES for Facilitating tlie Calculation of Eirilmork, Land.


Curves, Distances, and Gradients, required In tbe Formation of

HUNTINGTON,

J. Ii

C.E.

THE

TABLES

OF GRADIENTS.
10 Plains, 8vo, bound, E*.

TNIGO JONES, Designs


*-

and Cliiimiry

i-.T.c

of

tin:

for

Time

wiv,

(c ni|
Thlr. nfl, an tliat Eugliah
ferred Into Irtili, and fiwwwl.
l

o,

t
*

GATES,

EON,

Glasses

irlci tin: lit.

Statute Measure
Mra.uve

be trans-

with B Bnnartnf!

DESIGNS

OF

H)ln;i:s. FALIh.UH.NU,

niE ROYAL

Chimney
t.'lJ

TBI8H. PI antation\Vd British


-*

ORNAMENTAL

AND lltON-WOEK OF

L'AIUCS, ivird mm,., ml.iir Designs.

WEALE'S WORKS ON ARCHITEC-

[R.

ENGLVEE1UNO,

TEJKE,

JEBB'S,
In 3

fl i.!

,],,;

Pla'.cs.cWh ho:irdi, 2 2".

John, Bart.

Journal

Arnij- under ihn Iluke of Welimi ,,,,-1 lil4. n iih an Accnimt


Mr;:,!i--l!ci).
Mr .):>!IN T.
,:,] -:d and editid liv
!>

era

lintrtonin Spa.ii), In: ;.,.,


nf tin, Lin- ..f Tr.jTfis

.tONi;*.

ARTS, &o.

Modern Prisons.Their Cou-

Colonel,

vols. Svo, willi 28 elan-irate

TONES, Major-Gen. Sir


ot Ibe Sieges carriea by the
v 0 j r!

liy

Wrt]-;,!,.

ICCB Thiul F..1H..,,.


H AUK Y n. ,H)Si:-,

Lii-ut-G^n-re.1 Sir

l'.-.rt.

J^ENNEDT InD^HACKWOOD'S

Tables for

JDTG, THcSlA^^he^^tolBte^'fl Guide;

KNOWLES, JOHN.

F.li.S.The Elements and

A "
Tni,itis<j

Practice of
Niiv.il' Arc'iitra-Mre; or,
15 f
liUin;:,
il(!hi- theoretic*
ili.-nreticaland practical nn tln> 1w-t
'

ih-Iti c

'

Tubles of

niiii'.'n

.i.

rn,

Sliip

<n

-i!.^ .'stsl

il

S=!iiitli[i2-i,

The Third Edi

411-1. ten of s lino anil an c ],i,i ,.

>

l,.IM,..n,Ki,

('_;.

fvl.

.-ill,

i
s

th

T OCOMOTIVE ENGINES.

jtr^e a[la fujlo


in -.,.

ni

1,:.U

ballM,

The

Principles

and Practice and Explanation of the Machinery of

E ngines

I'

Losouialivit

operiitlon.

An

Account of the

MANUFACTURES AND MACHINERY.


Prnjirfss of, in Great Uritiin, as exhibited chirfiy in ChronoloRicil noitcoH of soma Loiters Tuti'iit granted for Inventions and
ImproTempntB, from theeari i-^t I'm- -s (.. the n-i'.-n nfQ iiecn Au ng.
lliTiV.i.Ms.'ilil.

JJAY, R. C, C.E.Method
Imperial

4lo,

with

of setting out Railway

est

fine Illustrations,

half-bound in mor-ccii, 1

cloth bourns, 1 Es or
61.

Hi

TtTETHVEN, CAPTAIN ROBERT.


-L-l

MERCHANT

OF A

to the Education

chant H-'rvii-:.
PwiinmilHT an

:!

nf Y"ii'i-

(IFFIi;l'i;.

onicr;

THE LOG

i:t, Hcfcri-niw
Ymitti of the Mer-

Vicr.'<!

ants tl>c

liy i:ni!i;i; r y, ]:T!iYK\


0;i,iif ;1 c, ii!|i;miv".. Si't-vitv.

(.

nniiiJit in

t!i.-

Itl

rovl SisTiirt\

TV1ETHVEN, CAPTAIN ROBERT.NARR ATIVEK WRITTF:-. 11Y SKA COMMANDERS, 11,1. VSTBATIVE OP Til E LAW OF STUUM-i, TJi,- " Il^niieini "
-I'l

Hnrrlc

mo

nflfigl, with

Diorama.

1'ii-t 1, bir.i' ^v...

MURRAY, JOHN,
*

DlT

a'

bm

'

Ke ",i " i,,fI

John Woalo,

69,

l'..r'

in

A
'J,

j:i

i:i.i-,i-ii>ii.
1

Treatise on the
"""> ec1dtod bj Engravings and

C.E.

High Holhom, London, W.C.

Digitized Dy

Google

On

n lnryn

full.,

shear, price 23. Od.

OFFICE

SEVILLE, JOHN,

C.E., J1.H.I.A.
IIYDItAL'lJiJ TAIil.Ki: !"']' Ih,Mi.;c r.f LilRineors engaged

i'

,i

1 i'i

?-n;-v--.

i' -:i

rz"i i'.l

v,-,ii.-v s,

i;,

,'

i:-.:!.-

.ly f.,r T.,-,i

'-dl

Cildnn.'iLl-RL: lln, Drainage,


Mill l\.wer.

m mill

Q UNA M E NTS? Ornaments


c.intainini;

CJF? 'JriS

Q'BBIEN'S, w.,

In J.-imv

ii

mtips nf

jiriii-

>
;

vn, cloth, wir.ii,

Ti.e-ji-y "f Ilia


ical formula?.

and

fricieu.

Essay on Canals
lii'.

COUNT DE.-STEAM

pAJiIHOUR,
T ENGINE; the
and

diap lnyed on

varii.li of ucl'it.iii; (.'\r>:rii'lrs of roSinsc

Proportion* of Steaui Engines,

In 8vo, cloth, beard*, wifh Platen, a second edition. 18a.

PRACTICAL TrJUT'ISE ON LOOOMO11'.

Iv

LVilMf

!,}<:

HML.IV? -

|.rwil'*l

I'AlllJtH/lt.

PARKER^

CHAlu'

Th8 Rural and


BCvcrnl very

and
land

l'^,'"'

Ardi'itoC FXB.A.

Villa Arc hi taut lire of Italy, portraying tho


examples in ttsnt country, B-itb Kstinmlcu
(' [lie same designs 111 KnjInpplh-.ii !
rli,

iiihT<<still),'

.-|n rhii:..tii.iis

i".,r

iVmi:i

l]'iiii];n.;,

and Florence, and arranged

for

:ul

n;-\ii

:.

in

Ili,>

vifiiiUv

nl"

Jit.nns

Rural and Domestic IJulldlng

IQ410.

pOL-E, WILI.IA1I. M

Inst..

C.

1'.

COR-

constructed a
the
ol

IIii-i

tn

Copper

!1<, i0

CAl'TAIN JKXKIXS;

Mdftes or

Mm:

Lnnguin,

olahiirai..

John

l .v

in Cornwall
erected mid

Iioprirliiicnt of
b

We:tl,

-;

now on

the T.olre

fluty at tho Uoal


Inftrleur, Nantes.
described.

iiicrnrifiilly iul scientifically

W, High Holborn, London, W.C.

Digitized By

Google

20

WORKS ON AUCIIITEO
MR. WEALU'S
Tuiie, EXC i'SKKitiM::, fixi; arts, &c.
'

BOl'lates, SSi.. boards.

PORTFOLIO OF GREEK ARCHITECTURE,


-in-. l>K:i:;^!i Dc.Y*Uii
ivi'.li

i!,...k;

Arcliitcrtr.i-.d

lining ml.ijiliol as i\i!ics


in tie Srecisn sty la of Architecture.
T,:il.

(l,-M'i-;>rii-,-

Classic Models

pORTEOLIO OR " DRAWIN G


of

Ui/hiIi,

:ui,i

i.-itii

c'-iLiirh'S.'

Engravings,
tlie beat

of

BOOK OF

L'-Viil

'

Awl; iter.! s

lo

Builders, and Stndflnts.

of. Technological COL-

Portfolio

LECTIONS. -Of
jeclsand patterns

cnriou*. insercstinjf,
for stained class irindon-s,

and ornamental sub.


from York.

OF GREA

iVOiiKS

Liverpool
tlia I.owloii and
Canal Turnip!*; Cmml lVuili
\V:i[!s, IJivsys,
Docks. I'Umi nml I'ililMi'.ini
l>-.fl; ::!',

foe

arbour and Fort


.Ma, hut; MooriiiR-Cluiins
t'lnn li the
of Lond, arid ollmr important Engineering Works, with Descriptions and Specifications.
tlieir

Rl

pUBLTC WORK^OF THe' UNITED STATES


mates

()[

I tii

Ml

(i

iti;i.iu

j it.1

3vo.

.10

f)"

wokks

of

i:tiuthvIi!-3.

.Ci

Tin-:

EST.1
l'-mted

'.

TIONS OF

AND PRACTICAL ILLUSTRAl'L"HI,IC

WORKS OF ItKCENT tiON'STlU'Cami a n; c A n. s.iniiii'ir.enhiiT to

TION no tii r.tUTisn


.\;
previous Fublications, and containing
Tliivi-.i

in m.ir.i.p, lin.'lv

BAWLJ NSON'S.

all the details of tlio

cn'n;nJ

Niagara

h,

l'!.,r,:s,

ROBERT, C.E.

3s.

Designs for

Factory, Furnace, and oilier Tall Chimney Shafts. Tall


nccii-isarj- for purposes of Trade am! Maim rehires.
:
>:,'
! Him!: icr;. lor tins Works, for

chimneys are

Tilt j- nrerrqnirn! in- l^ctorii

REID,

Mujcv-t^iiui-iil

ATT EM

'T

Sir

W-, F.R.S.,

AN

TO DEVELOP THE LAW O


CJF STOKMS
ith

a view'to practical

n Weale, C9, Higb Uolbom, London,. W.C.

Digitized By

Google

N ARCHLTEO,0s.,

tin' it.

;][]-<

w or ?T<iiiMrt
practical:-

lit'

with Charts and

EID^MSor-G! "sir" W
lllittlra: -1 v'i

'

IT 1'lates, 'i'liird EtUlic

li

RICHARDSON,
,;*:mU;;r>

II."

Ml

Circnl:i;i(.n.

\-'

metW

:t

[i..fk!j

ii

may now

to .fleet

i:

'i.ul

l>

m-.i

20

r->u.p

XE.

ji

uilt tope, price 18.

Work

on

Construction of British and


Tliis (rreat work
price 16.

'.r.sunnl-.

ivi.l

iris

ai.il

su^ikm-nt,

The

Progressive Screw
principal

,he
i

m ployed by

,110a

4to,

of

,-sifim

I7 U.S.,

.lOIIX,

Sir

Formation, and

tho Theory

FoiWn

Buildings;

of

Heated Water
the c imbination of large and
Hating hnilditiKS.

h\j|.|-.n-,-,l

-I" tin-

,.

EE N

A Popular

Archii;ect.

C. J.,

itilatiou

shM.it,"

'

"f.'r.S., &C.
.,
in:
Tin: i>;;vi::.!
Tin; vaiuahu: winds, with

and ok

i.Lin.Ur.]! (,! !l: :;ni>y,-t to ii.n U-.lion.

jii

ha

of the Theo-

~ith fm- Plmca, il X,.w IMitimi, cxt-nik-l, sewo-l, 5s.


C. II., Siiulptoi . .Report mid Investigaand Fituoss of Stone for Building

SMITH,

tion into tho QuBlificatii.nl

CMITH'S,

Colonel

Mad i

of the

Observaiion; on the Duties


or Minns.

nr,.I

Iivpotisiiii lilies

Involved in

tiie M.miL-fi:iiit

SOPWix
AWARD

ot

'T Ti/oha's^

ii'

Hi'

TUB

11

if

[:

KAN I'uKKST l."MMI:S;nM:i:i

AS TO TUT C OAT, AXD IRON MINES.


1

SEEIES
SO P WIT H, THO M A^T*l!S.a
THE COAL AND IKON

OF ENliK.WEI) ELANS
MIME a.

iH'

12 Platen, 4to,6s. in b wrapper.

QTAIRCASES, HANDRAILS, BALUSTj^ Ai> !.-,


|J

AM) newel* or

em /.a isi-rni an

Tin-:

,\i;k.

Staircase at Aualeyeud Old Manor House,


Consistioq oi
I.
Wilts; 2. ClmrlMi, :[.iu r..:, K-ist: U. LlrMt Ellin Ji.m Hull, Norfolk: i. DorC'lil.
C-lnr,.

II?.,],

iiv-hini;

iv,,,

7.

C:,nrl-rl,uiiM>i
C:-.>i:iw,.-u

i[.,:i.

ili

Oak

fi.

Staircase nt
H.

i,

nil:.-.;

!>.

CallK-riu.: iliil!, riuiiiiiiili;,-: lu. S: at reuse by liilgo Junes a.1 a


12. Ditto, ditto.
Iion.se in Eli.iii.lcs .Hum; 11. Ditto at East Sutton
Useful to thoso constructing edifices In the early English donieitio
style.
;

Larue etlas

folio Plates, price '1

QTALKARTT,
3

or,

M., N.A.
The Rudiments and Rules

In a Series of Draughts and Plans.

John Welle,

60,

High

lis.

Naval Architecture
of Ship Building:

No

exemplified

text.

Ilolborn, London,

W.C.

Digitized by

Google

QTEVENSON, DAVID,
.-n]>|.ti-mmt

Text in 4w, and

wood;
!

of Edinburgh,

C.E.,
C

Wra t

his

tii

large folio Alias of 7o Plates, half-cloth boards,


12b. 6d.

Vessels

OTEAjM NAVIGATION.
YY.U.

iiie

of Iron

and

Steam Engine; and on Sere* FroauWon.

By

AUa'-.MIi.N,

.MMisra.FOKlil-lrTLIi,

r.i;.S....f:,l:,iirl!.-/,-.:':

M.I. U.K., or l.ivfrrin.ul; .IUUN J.Ai LL1>. il.l.U.t:., of liirki-uitcnd


I.AXft, li.lni <! WimU-cii; SIessr.<.Si:AWAHD, Llmenn :1m
Ti.;:.'!>r Willi iii.-n'.ts o!" "T! i
^
Disturbance c.f thn Compass in Iron-built Snips. By G. B.AIRY,
M.A., Astronomer Royal.

OLIVKU

boilse, Ac. Ac. .to.

QT. PAUL'S

i-

1 1

-.

CATHEDRAL, LONDON, SEC-

Tf ON OP.
The Odglua.1 Splendid Engraving by J.
J.IVAM'. ,LiTnr i.!.l"f..j;-.-i.'.;: !j:v in tiits origin*] Intention
of Sir Chrlstopln-r Wnin a very line largo print, showing d is linctly
the ciiisrrin;tu!!i :>r' tli.it. m.i jiiilir.ont ctlitice.

GWYN,

!l

Mz.lof
.'.

PLAN.

J.

WALE

lil.-;lsi.ii-:l

and in

I'Utl; it .n"ii i]7l,^i t "!;t;"

CATHEDRAL, LONDON'. GREAT

QT. 1PAUL'S
rmrlnl)
figured

fnnii

and J.
n Llnl

t'j:n

II

OWYN'8 GREAT PLAN,


!

with

n;-.

detail, description of

Dimension*

Dip

ill)

Compartments by engraved

Second Edition, greatly enlarged, royal 6vo, with Plates, cloth


boards, pries 16e.

CTRENG Ti[ OF .M ATEIUAL3. FAIEBAIEN,


WILLIAM,

*J

C.F-, F.R.S.,

and of the Legion

New

Edition.

of

Honour of

The

HODGKINSTRENGTH OF MATERIALS.
TilOllAa
c.i:.

By

AM)
the

SON, EAT OS, I'.IL.M..


a I'iiAiTiriL fs=ay

IKON AND

<>

OTIIlCli Jll-iTAI.S;

EATON IIODGKINSON,
To be publlslied

TJIKUCOLD,

STi:i:sr.TH

ill U-iifU'cl

fn-

111"

of cast

it

P. U.S.

in 1861, In

crown 8vo. bound for nae.

CTRENGTH OF MATERIALS. POLE, WILO LIAM, C.E.. F.li.S. Tables and popular expianaiions of
the Strength of Materials, of Wrong)]!

me lull,

(:

m [no

iinil

imnrat

jtn.l

!>.-t

inn

iliivi-lfjiiii!?

iron with other


syste-sn ilic liinn.

the strengths, bearings, weights, and forms of these materials, whether used as Girders or arches fir the (von ruction of bridges and
viaducts, public buildings, domestic mansions, private buildings.
pillars, bressummters for warehouses, shops, working
iiiiuinfiirtiiiini; factories, &c. &c. &c.
The whole rendered
I

roi'oronco ! r i-'oiiitocis. lr.ii tiling, c;vi; mirl inoclninir-i


s, jniilivri'j'isis, iii.n^);i:ii!.T>, .t-. &i-. Ac, diul forming

Digitized 0/

Google

29

WORKS ON ARCHITECME. WEALE'S


tu:f.. i:v;iN;:i:iiisi;. fiM'!_Am, At.
drawn Engravings. In lirge eto, neatly
!.L
A few topiiS lit liuyr. im j^ilal

TO very elaborately
beunil u-

i!

li'Crrinl,

r.vt,.,h.T-!,i-!.:..-

bairsi?..;,

11.,

TEMPLE CHURCHThe Architectural History


->

Arc!ii'r>r<:iv,.l

iu:l

Glass, of

Pari

I.,

tlie

(Irii.witr's,

h)u

li

^- c nlu.

mil!

Tomple Chnren, London.

Wood and Copper,

with SO Engravings on

in cloth boards,

'THAMES TUNNEL. A^Memoir


Operation* an.

Papers by

ilia

Fourth Edition,

THOMAS
J-

Mm

Om.trno'i,,,!

of the several
Thames Tunnel, from

Hie

t.f

1SAMBAKD BKUNEL,

Civil

lalo

Sir

itli

a Snnplcmmitary Addition, large 8v, fa*. Od.


K'li.S.L.
Rilled Ordnance.

F.R.8.,

LYXALLi,

Practical Treatise on (lie Application of the Principle


It:* to Hi
i-i-i
.v,!tir-i r>; ,v,-v c i:iVo
to i.in:-h ii
added a New Theory of tho Initial Action and Force of Fixed
i:

tin:

(;:;!ip.-.i,'li.T

In

phi".

llo, complet", cVrili, Yul.

til:.).

'

1 IOb.; Vol. II.,


l.'i.Gd.

ivith Knfrravings,

T,.

Hi.,

S..;

ditto,

.uii

FRAN SACTION8 OF THE INSTITUTION


OF CIVIL ENGINEERS,

-1-

bound

In calf, price

per volume.

Is.

TRANSACTIONS
J-

THE NORTH

OF

ENGLAND INSTITUTE OF MINING

Commencing
in ,!.n:y

ami

in 1^7-',

|-

12:.

jiiici!

:
,

TITHUVIUy.

n.ily

H.,

' '

of tho

Chu rch

Cr>:it|ii

: i

OF

INKERS.

n:i li'din

i
j

11

Translated from the

Account

Architect.

at Stoke Golding.

Vol. VI.

"On

i-inV.
n

i.lio

tlta

(Parts

llio

1'tiiic:

On Water Wheels.

in 8vo, ciotli lic.ariK ami

S'hiii'B

WHITE'S, THOMAS,
*'

Ilea of

In 8vn.

and
of

12

completing

Water Power."

Fort and A
i)

Platen

11
pl-a

of

Plates,

ind.

Text

1 Wi.

a Medway and of
Pumping,

3 c

QUARTERLY PAPERS ON EN-

(ilSKKItlSt;.

tho work.)

The Architecture of Marcus

n 10 Rooks.

WALKER'S, THOMAS,

WE
ALE'S
*

KNli

to I860.

cov.ti.-a.:il

pro.f IVi.tuf aisd ViiincM

l:.i::n

l'J:n
srii:

i-

ii)

Analysis of lha

Combh

Plates.
alius

f,.ll>,

in cloth, 16B.

N.A., Theory and Pruts

Ship Building.

largo Sectional l'iitc, l9.6d.

JOHN, Architect.
I C H C 0 R D,
WIIOBSERVATIONS
ON KjJNTISH KAC. STONE ASA
nitli

.1

BUILDJXtl MATERIAL.
John

iVfiil.', in,

Ji;;li

it'lb or n, LoiLiloii,

W.C.

so

WtfAUTO WOKKS ON
Ito.colonrnl PUt-s.in

AECHITEC-

lialf-iiidvoeco, 7a.

6d.

irCOKD, J 0 ir N, Architcct.-HISWjriC
TOUY AND ANTIQUITIES OK THE COLLEGIATE
111
CHIIKH Ol
i'OXl
1

In

41o, 6j.

WICKSTEED, THOMAS,

C.E.

TIVE power
BY,

THE

EXUINEi,

AN

EXPE-

coNijj.;i;M.V!i Till". RELAnr, and usi'.rci, icfi- i-:ct it.oihtced


I'OLSXi^ll AMI ll'iL-l.TON ,(i
l'l'MI'ING
n:id tyiiiidi-iciiliiad \V K r,-aii- lloul Uailers.

IXoriliV

lilMENTAI.

WATT

D 8V0, 19

THOMAS,

w ICKSTEED,

O.E.

THE

ELAliUK.VIT.I.Y ENCUAVEI)
r.I.L'^T): ATiOXS Ol'
AM) [UHfl.TON .v WATT KNCSNU!? netted
idon Water Works, Old 1'oni. Eight large atlas
I

IE <Vl!IXl*:E

Wllh nnnwrous Woodcuts.


C. "WYE, Esq., M. Inst.
Tin; [D;ii!i:-vm;i of cuai. a:;!> the

WILLIAMS,

TION OF SMUKE,

clioiii;.'.!!;- li-.J

0.

"

I'KIZE

pe-ievi::*-

rrr.ciu"Uy ccnsidcred.

Imperial Svo, willi a Portrait. 2*. 03.

WILLIAM 8,

0. WYE. Ksq, M. Inst. C. E.


ESSAY ON THE PREVENTION OF THE

SMOKE NUISANCE, with


With

3 l'hiV

WILLIS,
"

a line portrait of the Author.

ci>:iMmi:l!r 31

:,

liljiiro--,

it-.

PKOFESSOE,

REV.

M.A.

system of Apparatus for tho use of Lecturers and Experimenters In Meelmnkrtl PliHosoj hy.
j

in 4 to, h.iun.1,

v.-i'.i,

Bii

].:.:<.:;

WILME'S MANUALS.
WltlYIXU

Maps and

AMI

Plan:!, in itn,

WILME'S

and 17 wir-iiwin.

I'.'j.

A MANUAL OF

E'tUXTlNG

CII A

It

ACTEilH,

liulli

half-honnd mnrocao. 3.

pliit.oi: c'i]<J-.iit;d,

MANUALS.

HANDBOOK

FOR MAPPING, ENGINEERING, AUR ARCHITECTURAL DRAWING.

Three

Vols., large 8vo,3.

- COURSE
WOOLWICH.
*
for the

Ob'

MA THEM A-

TICS. This course is essential lo nil Students destined


Royal Military Academy nt Woolwich.

7TJLE, MA JOE-GENERAL. ON BEEAKWATER* AND nn.iVS<.f YkEi'lECAI.ELOATS.


L

John Weale,

E3,

High Holbom, London, W.C.

TTOREIGN WORKS, KEPT IN STOCK AS


IV. LOWS :
Large

folio,

Si ulato,

coloured, ami 12 woodcuts, SO francs.

h...iua

ARCHITECTURE SUISSE. Ou
pons
fi\i;ii:i>

Hl..\

lliifi!JHii---

s'j'

ACuuji lUi <J.isit.n

il.'s

Ar.:,i:.-n.

::i.]'.r.

l-ITTKH

'/A:

l;

Chobc do MaiIWiip,

1c

G11A.F-

l'ar

ii,.,.,

Blil'UK

JJYZANTINISCIIE CA I* I TAEL E R. Mtinchen,

Lax is"

6a francs, 60 plates, and several vignettes, 2 6s.

f-.tiii,

fANETO,
v
p-rn

F.
ilu

[ill Lij ii.'

of Outline

Siiinte-Mfiri6Thed'Auch.
Cetle CstlieiWo.
of

iH;! -,i!.,:i

U:ii

r_i^,l

Atlas

Mono-

principally
in tiiiiiC.-.tiu-

4 i-<.:isist

I "I

(;:.iis V. iui-jv.-i

dral.
lL'i' p!i:1. :.

r!:V.l:il

luli-IHI);': OS.l

Hi Tft,

interleaved, 5 15s. 6d.

PARALKLLE

/-a^t;;,:
\N. A.
SONS no BHUXIiLIJSs

^ MvI.IM.

MAI-

; l

t-ltfTutioiiB,

Small

TJEGEN,

IS

dog

et dea PlSIKCII'ALES VH.LES da


iL i;..s jour';,
il^uis l.^i'i !ii--,
refill!'-coupoa ol de:nili inttriuiirs at

I'l.n.li-uili.'i

l:i

Bentea on plans,

L.

Full", 1-

plato

LES

i-H:-, i

of

Is.

CON STRUCT IONS EN

BRIQlES,MmpoEoesetpnblieea.

8 Hi-ralsniiH. 18B8.

tjege:

n AERTNE1VF. V. The* spllliTd^work^' of M.


*~*
(.1 A LI :'T N Lil
aistlnB of tl
i'iii...

of

a:i

M, m i,-!i

ol

firjivn

ornaments.

cnlcinvil

"if.

ii

v.-rv

l.i:-n

MZ:'.

r.i-

Tlio dunili, vUli uYini:*, Croa-

fta Mi Indian.

ts,

Small

folio,

tile

fine

])0i

pMes

.in,!

of the Architeclure, ornament, and detail


liviintv ilil'ii ; Ihn lliiildlii
;e,

li-iotli'S

li

viiiy

:W

liiilf-!i

mi.!

TT-ALLKNUAUir,
-aV

H li-l:iUiTlii:lmn

in n, .,:,,'

C. U.

.,

12s.

6(1.

Cht-oisol..

Baukunst. Milnclieri.

;it-

dor Uentaeh-

Fine Work.

ivorirs of ttio great masier KLENZIK nf Municli, in 5 parts


very lar^o folio,
plates of elevations, plans, sections, details
and ornaini nts of liis pnlillc and private Imildiin,'.- executed in
Munich ind St. Peteralmrir. 44i.

The

Samnilung Arthitec-

Digitized By

Google

FOLLOWS:
Upw.ird.-i 01 lUJ pl.itcs,

SCi7.-|'i:.,. S

>. a

Ijrjr,'

llfc.Gli.

VICTOR CHATEAUX DE FRANCE.

PETIT,

L.VI-S._ P.irs.

LivrainoiiB

very finely executed

1?,

il

platen, large imperial folio,

CHATEAUX DE LA VALLEE DE LA
Vlonie

SJECLl

i>i:s
Paris,

xv, xvi,

du xyii

183760.
It

Slf.iilali's,

.,

DE

J^IC Ul IL

cjo-iimlncljient

f.t

St

>

7-2W:

1,1

10'.

;u;'

<

OTiUQUE.

1 parts are published, price lis.

"DAMEE. HLSTOll'E G KX KKALE DE L'AR-"^

nill'K.

I/Ui^-TP:;-^

il'l:!'.

DANIEL KAMLi:,

i vol. giando

f-.>v:n>r

d* l'A"Cl.it.icl...rc, par
Svo, vuWidB en 8 fas-

iii

ciciilea.

ivnl.,1

m-.--. :i.!Hi(r.i-iH iiri-

SONNE,

KKcW.

dr.

'vtrT.

m:

Firtlf

DIUTIO.NXAIRE

TRIOLET- LE-DUC.

L'Arcbilccture Francs iee

r.

i-;o.

RAI-

du quinziiine au aeiziume.

1S54-S.

I'iiriK.

i ioK.i'XSra imperial

priic

trdiO,

U'i

(id!

L'iil.

"DADIA D'ALTACO.MLIA.

.SLoiiiv o JA'seriziuiiis
della Arilici Sc]i klui. dri l>nli dl i=avnl:i, fnndita da Amedio
III. r intm/;it.i d.i C:ir!
r. (i
LMnr'n Cli ri.it inn.

JJ

7D livrni-

in

in;

"I5ELLE ARTI.
J-*

n instral-

a v.,H.I,r;jv

Sou cnKi-nvitias. S

-.v-w. .Jin,

11

fl-.'

v,-rv naallv L-iiKl'H'-vd -mrim,.- 1'Ijm.,, ft Bh.

Tombe ed

if.

Monument

Illustri

Milium.

d'ltaHa.
i v >u.

(id.

Duailo di Venezia,
Vwili, IMG 1WS.

V-.xkvz.zo

I'm nr.-., Zi|,i"o._


.11..,

pANOVA.Le

to"r:i-,;

ii,

id-.

nAVALIEtU SAX-BEUTOLO (NICOLA).


iSTiTiiXioxi in Ai:fiirn;rrri:A
LIC A. ihnl nva.
2 v,d: i:..;
i:niv :v,.
nf \Yi,

.Ll

!i

,,l

[.,ii;,.:JI.

!;!

(-..isririsinr

i-1,im. ii'h-vnii.in-;,

-.

[aik-a

,'ivi.i,
V'.i:;,

cli

:,

i.(

[!:,'

1VL

ju;

i-l

1=

(.'-

!!::.

Li.'

sin] <:'. ill.;. -:s

.i,s,

I:

Idi

!l
;.<

lid.

f'lCOGNARA (COUNT;, L<; l<'iU.l,ndio eiMonumoiiti Ci.ii. i. mi di v.ni.v.ii, llu.-iruti du I,. Ci.'ojjnam, da A.
IV. li', f dn ti. A. tYlviL.
.ml pjjjiMS,: mdt: c.| a_-.:iiin:i- di
>:
i

FrAHce^D
Franceac.

ilH'lli

is in

C..[>ii'< (li!^aii:li

and

Zun.rr.i, ai-rii-rliiM di nn.ivi' tavi.tn f d.'lln Versions


Veiirr.Lu n-iio si.t,-. az. di t:. Ant'wlli a spesc dcRli

edit.
Allt
scriptivi-

!.r:i-i'-]l'(

I':.
li

ml.

'ti'-!^.-iri.i

iiiti-rljavul. (.:> >>:.

Tlie
)il!> R!slv d^printed.
UI IV5CO, extra gilt, library copy
1

!I:v;.Lj..|-,ri.i,

Ms,]

[rriliuu.
i:i

\e::cz

a,

-.").

-:i ri

i:".ilj

^.
r

iter!

'works at Venta!,

oic!,

1 4a.'

a r;i; i: icn e.-civiu ecclesiastiche


--

id

i.K-i-,1

M1I,ITA1!I HI MIC11KI.E SAN MIUilKLE dlsegnala


,!, I'.uN/.AM
I'.ANi lLSL'.i o I.
i

QIBOLAMO.
Jobn Weale,

59, lIi Kb

ilolbom, London, W.C.

Digitized

D/Copglc

FOREIGN WORKS KEPT IN STOCK AS


X

_KI.WWS._

].ar.L-jf..li.., ^.L^iU:.!,.;,-

Mtl'l

vill,H,

dl>

;i

nrus i

|.

,.:hiT

r!i'_Mu'ly

It.-.l;-.

rjAL.LEIUA

,,[

i'l.,n-..~i

T-Kli'Lt-

."i

Tn

1)1

177

ci.iii;.I,i,i.

M.l\,

i;jjli-k;]-;s

LNVimiSS.
VvUa,

I'insi.

:!

1 1

huiir,:;-^ n.

IH-.r

GAUTIIIEIi,
a

,!,

.11.

.1

1 ^

i'V

h-.uii.M.

i [

1 1 1

;,:.!!
l

Tu

ii:

PLUS

""^s

dn* sos

,MW,-r,-^r-

'ri ,,",l'

Ilr;tli

."-

,7,,"

-TJ

Vmvd,

KAMIN

iniikon.--.rt

imI:^

vi.^^.TT

.::. m

:l:i.1

i;,-.;

atmcljcd in end..

..I--ii;.[i.>M
(.

.,

;ii

,u

], 1L .,|

Elegant la

U'i.

.il

VENEZIA^MONOMENTALE PIT.

G.

in n

s~

1S15.

I'.ivi..

l.M.ln.ii.li'uI-

-i"

'.i:,:,:;;-;

r.ill,iO

iiiter-

r.'scniii!.

iUj^cuii^jriiii^

iv.\,)-.c.

litliwriflir, wiih

jgTER,

L-a^ n-i

IuIMi. u

ui.ni:s

,n.

IS3M-2.

l'nii.i,

In

&c

do MONTIII.W r i A.
AKi iirnx'TLKi-: th^'am:, f.i |n Liii,
il.!

and

M ! i pr.-of, 2.1 L>s.

Airliiluuto.

In!) I'lii'tps.if i,l,Liisi-li''-: ati..ii-.''sii l

fl.ntri>5 ciiilitili,

UWmi-

A.'nJ

3 vii.i.;:

nRANDJKAN
VJ

-i

fill

.];.i:aT,B>.

[ . j.

fii] ill,

MeFCinl nnita of

no {U\

i.
lllust rata da II. !>' AZJ :i
L m>. J t -ll'
Cr.pLi'i, In, linn ]>:.'.. 18 ise.
',* rinimil c..|nrH in c].- K an; Liilf-iiMiwi-n l,ii]r!iii

!]:]:;. ! -I

^T" 7-".i

eilm

liVVniinin, jirlee ta"vi.

vl

- volH.

in

lj'.iilcIiii^H

in rtJ Tmirecci',

]iilr-l;.>iiiid

:iG

folio,

Gl iiviMisMiia nr

-!

v.-ls., ,-.i;]i

;i

vula". of~te.il in

atp

18

T ETAROTJILLY,
1

P.

dome

UltJiiices

Rome Mo-

de

1'nris, iSi.V.W.

"ITICHEL A, IGNAZIO. DESCEIZIONE


11

TORIN0

di

"D EYNATJD,
-t"'

teriflJW

fu.H storm

Venn Kin,

V&hAZA0

L. Trattato

""^1 STRATI 8PPEEMI

d '' i

Tori

M
di Arcliitettura,

con-

n w.nini .;.-!, -i-ali s.il i>.;i 1C ipii ,],Jb I !u]n irn/,i .iriri
e
..in, <, unr.oi. per cm-a di I.miu, IJrbimi.

.lull

1S5T.

4 Imperial bulk;' fivi, v..Ii


ritv, and trws ..f 111,.
hllm-l," 31 il k., ^.,11,
Gi-';i! iTililiti'.;

-,:;l..>:.

yENEZIA. E

le

-s.
..;-!

v.

Il

|.m.r,j

y
11

'li'ri-1

i;.

;..

:,

mj

.-::!

h~t

fi.V.ri-

C .ii.iiitn.n

iv,-..-,!;:!

i:]i

sue Lagune.

,.f

"iiiu nr

nutlm-

aiK,

cstilTl .

t :,: ;ll ; [C ,.,

:[

ila

;:

Yenezia, 1847.

yENEZIA. Copies

CCADEMIA DI BELLI

ARTI,

Opere

GrsudiCoimdrni Premium dair I n. A:-- i.i .inH rl H 'i I


in MilRiin, n pulilinato, |.it rni-,i
\vii-icii.i t;
r
(Irnr.in. tn:r In Cini-i di \ c'lii.'l'Jr, 'liimr
In UlnisL
.

M nsr

per

Omito. Milano,

1825-29.
Jolin Weale, 59,

High Holboro, London, W.C.

Digitized 0/

Google

Alt*-*

fijli'i,

folio,

vi'rv

iVi.-ifii'itK-

AIM

li.iLf

i:i

in

boM:id

I)
|

t:-::r.i

moroc;!)

ana

LBERTOLLI,
Bale ed

criTiiji].-t(!

hi:[.

iiiif

1:1-:.

interleaved, 6 16s. 6d.

Alcune Deoorarioni di Nobili


G.
Oimmentl. Mllano, 1787, lS24,lS3a.

To be had

aparately. 1 8a.

Part

j^LBEETOLJ.1,

G.

very frequently

III.,

HOFFMAN, ET KELLEEHOVEN. Eecueil


da DBsslns relitlfi a 1'Art de la Decoration chei tons lea
rcnplc ot a-jx piui heLlcs I'-jj-iqurs do iciu- civi: Nation, ic, destines it liervir do ns.itifs rf di> m;Lt<i]i!ii;:; ai;.\ ii'imlu'a, rio; jrittmv,
peintres sur verre, et
dessinatenrs do rl>riqries.
1

HOrE, ALEXANDErTiiERESFORI).
iih^.
in

Esq.-

Abbilduiifffii
.-m;;idi;
der Halvator-Kiisho 2a
Kilndown in der GrslY-tcliaft Kent. Copies of pain tings on glass ia
Christ Chnreh, Kilnjmm, in tl n counry of Ki-nl, executed in the
Bnyal EHtsblisbmoiit
r i'.mi'in;; hti <jl!is<, Muii.:!!.
!,v
order ol
ALliXANDEli J. !!!;!: K-fl'tJUU HiU'L-l, Ivio.. published by P.
i:^]-!, I'aiiitor (in CiliLSj, Miincln'n. Tins work contains one sheet
with the dedication in A.J. IH. IIOPR, Esq., mid fourteen windows;
in tli.' wliuii! :k>m, lii iu.'.ifiii!; (n.^a.ed aiai ;avlidlv ook nrcd.
:

r".

folio, SO Plates, containing a profusion of rich Italian and


other ornaments. Elegant in li^i'mnn.cro. -it;, and interleaved,

In large

69.

comic I, lam pads, bronii.

e, iriBtslli, soffitti,

I 1861
ii50f., in folio, 3.

TEpotion
PATTX HE. Collection des
CI.UUA,
*->

.,

srjv.-i'S

par

METIVIEE, MONS.,
Skctclie^,

Ilri'ilill.!,-'.

ami

plus Indies

Com-

I,Unv::i ; c coil-

A:oi:te.

lJ i;

Architects. The original


'r'.ici::|^,

:1

['(-ll'.-il

ami

Ji!t

and

andfiltingsof them; his interiors are still much


lie built a mmisi^i] for Prince Charles, in a most
in admiration,
simple and elegant stvle (in l'rirnii'r Stis-r-r', which iH stilt now
Tile above
considered on'; of ibe p!::v5t Lnildin-:- or' JliiEjoli.

Twelve Parts,

in small ,,l,i ,n;; 4to, fin ri<.i,v^ I'liLi.

rately coloured and Kilt nranir.pil

AM EN TENBU C II Far b ge
..

f\ EN"

fllr

Fabrlkanten,

Zimmermnlor und

...

00 elabo-

Verzierrragen

andcra Baugewerkc.

MUnchen.

John Weil a,

69,

High Holbom, London, W.C.

Digitized By

Google

85

MM.

livi-i- ]

'i'llr-uM, du Wai'.iy,

QENEMENTS,Tirfs
Six
Of

l-,U.:

1:.

til'!

J.

--H-:

>i-

<

; .

TMANN,
lic'.i-

Very

L.

k":-,

Alii,

si-,-

lUraU.sis,
Blfil,:.!

.11

Kli;.>'.;i,

..

ri

3 ; l

L. 'l-Vici.eru

ill

inorrcf.

.iua
?1

.L!;:-lii-ii-:.

silt, urn!

i'.i..

Quatre

des
in

c.l'.iiri'ii,

-.;;i].^rtr:-..'llt.-,

<

(:-.

.i.,il.!:i

Villi!;-;,

;:!]:,

-1

ti,

Is.

deii vorziig'tiMl.

n t.-rl.-n v.-.l,

17i.

fid,

aimmitettonico

<;.-rtTtrim

ki n.

WIN AMENTA 1.1, dndica',1

JVHc

11

v>:i/iisr,

imit&j

'

Ornaments

Jiii'.v,.:

elejmnt. in half red mot-ncc

an

on

Ii-iiisrifiill)

S'l

ini.iui.'iifs.

AU->. dn.-imllv Imli-b^in.i

iM.

li>;.

nil'

J.

I:.

A.^iiinniri Vctirta dcllc

(mil a^.-uiiil'! ill-] i'r ['. !.. L'KliANl.


f./i.i, ill.
L'-O ..f ni v.! .l;il..";ati;ly ,].;-

ii.ii/.k.n.'

M-,!,-:i:il

A r.-l, it-ct

ml

IV Aiut !.n.-i:..i'

Utmimcy

l-'iiltiij:-;,

ili"ji;i>l

fjc

'!, li-.n VViirk i'.ir Inl.TLur.; nil


L.\!-:i.ir:. (.Sales ami
W'imh'ii Mil.--, limlm: Dec (.ratio us, &c, &c, including Uie
p.
!

Catalogue, of 40 pages/ to be had

gratis; printed in

demy

8vo.

Export Orders executed either


cipals abroad,

or

for Prin-

Merchants at home.

In Alias of Plata and

Text, 12uio,jirire

together,

IRON SHIP BUILDING.


PRACTICAL ILLUSTRATIONS.
JOHN GBANTHAM,

N.A.

DESCRIPTION OF PLATES.
1.

Hollow and Bar Keels, Stem and

Stem
2.

13.

Posts.

Pieces.

Keelsons,

3. Floorings continued

Gunwales

continued

Docks, and Orlop


6. Angle-Iron,

Iron,

Vtutl, showing details.


Hidihip Seclim, Hairbreadth.

15.

Uaetana

10.

Machines. Garfortb's Riveting


Machine, Drilling and Counter

17.

^ir^arnDcsforlleatlngPIaieaand
Angle-Iron various Toots tised

IS.

Gunwale,

for

Keel,

Moulding

(lie

Water

Ballast,

Frames

In Iron

hroadth Deck

and

Hans

Ilalf-

20.

nacle, with Adjusting Magnets.

Frame of
Wooden Ship Caulking Joints
|

of Plates.
22.

Great Bisrern-Lougltndlnal Sections and Half-breadth Plans.

2i.

Great Eaa tern Section in Engine


Room, and Paddle Boxes.

of different sizes.

Vent!, showing details.

Jlre End in Section, and Knd


View, with Stern Post, Crutches,
Deck Beams, &c.

This
Ilall

Grays Floating Compass and Bin-

21. Corroded Iron Bolt in

of large

Vessels, on a reduced scale.

Midship Sections of Three Vessels

Work may be had

Court,

for 11. 2s.

Flooring;

Magnetic Con-

dition of various Iron Ships.

Ship-building, Levelling Plates.


IB. Longitudinal Section,

and

Keel,

ID. Illustrations of the

Transverse Sections.

11.

Rivet

with Copper.

Iron Masts, with Longitudinal and

13. Largi

in Rivetlg and Plating.

Tetlng.
7. Plating, three plans, liulkheauj,

0. Sliding

An pie- Iron, anil


Plates

Sinking Machine.

In section, natnral

Fiush and Lapped Joints,


with Single and Double Risize,

8.

Punching and Shear-

Rending

Bulb

Iron,

lion, as rolled for Iron Ship-

shown

for

ing Plates and

Lower

Beam a.

Building.
6. Bivets,

Vetitl, showing details.


End in section, with End
View, Stem Frame for Soreit,
and ltudder.

H. Largt

Deck Beams, Cud wales, and


4.

Large

A/to-

Side Frames, Flooring*, and Bilge

and

6d,

also of

of Messrs.

Mr.

Lookwood k

Weale

Co., No. 7, Stationers'

either the Atlas

separately

or together with the Text price as above stated.

Bradbury and Evans, Printers, Whltefriara.

Digitized By

Google

Digitized By

GoogW

QigilizM By

Google

Dlgiuzoa Dy

Google

You might also like